You are on page 1of 285

ĐẠI HỌC QUỐC GIA THÀNH PHỐ HỒ CHÍ MINH

TRƯỜNG PHỔ THÔNG NĂNG KHIẾU

CHUYÊN ĐỀ TOÁN HỌC

SỐ 1
Đặc san kỉ niệm 20 năm trường Phổ thông Năng khiếu

TRẦN NAM DŨNG (Chủ biên)


LÊ PHÚC LỮ
NGUYỄN VĂN HUYỆN
LƯƠNG VĂN KHẢI
Hướng tới kỷ niệm 20 năm thành lập trường PTNK TPHCM
Chuyên đề Toán học Phổ thông năng khiếu TP.HCM số 10

CHUYÊN ĐỀ TOÁN HỌC


SỐ 10 CỦA PTNK

Thành phố Hồ Chí Minh, ngày 16/09/2016


Chuyên đề Toán học Phổ thông năng khiếu TP.HCM số 10
Mục lục
Chuyên đề Toán học Phổ thông năng khiếu TP.HCM số 10

1 Quá trình hình thành và phát triển khối Toán Tin trường PTNK 9

2 Câu chuyện về những bài toán 21

3 Bài toán đường và điểm cố định 27

4 Bài giảng về Phương trình đồng dư và Thặng dư chính phương 37

5 Các tính chất số học của dãy số nguyên 43

6 Nhìn lại đề thi vào lớp 10 chuyên Toán qua các năm 61

7 Một số bài toán chọn lọc về


phép biến hình 73

8 Sử dụng tính liên tục của chu kỳ để chứng minh hàm hằng 87

9 Ứng dụng của tỉ số đoạn thẳng và tỉ số lượng giác 95

10 Giới thiệu về Toán mô hình 127

11 Quy hoạch tuyến tính và Ứng dụng 143

12 Phép nghịch đảo 161

13 Định lý thặng dư Trung Hoa 191

14 Một số phương pháp tiếp cận bài toán hình học phẳng 201

15 Số phức trong hình học phẳng 215

16 Tứ giác ngoại tiếp 229

17 Giới thiệu kỳ thi Sharygin Olympic hình học toàn Nga mở rộng 241

18 Giới thiệu đề thi chọn đội tuyển PTNK qua các năm 255

3
4 Chuyên đề Toán học số 10
Chuyên đề Toán học Phổ thông năng khiếu TP.HCM số 10

Hướng tới kỷ niệm 20 năm thành lập trường PTNK, ĐHQG TP HCM
Lời nói đầu
Chuyên đề Toán học Phổ thông năng khiếu TP.HCM số 10

Kỷ niệm 20 năm dĩ nhiên là một câu chuyện dài, xuyên qua nhiều thế hệ
học sinh. Ngũ hổ đại tướng quân ngày xưa, vốn trẻ trung oanh liệt, nay
người đã về hưu, người cũng qua tuổi tri thiên mệnh. Nhiều lứa học trò
giờ đã trở thành đồng nghiệp của các thầy, nhiều em đã vượt các thầy trên
con đường khoa học.
Con đường PTNK là một con đường chông gai. Nếu chỉ xét thành tích của
5 năm cuối cùng, kể từ Trần Hoàng Bảo Linh đoạt huy chương đồng IMO
2012, đến gần đây là Phạm Nguyễn Mạnh đoạt huy chương bạc 2016, ta
có thể thấy dường như PTNK luôn ở đỉnh cao. Thực tình thì câu chuyện
gian khó hơn nhiều. Nếu không kể đến huy chương vàng APMO 1997 của
Lê Quang Nẫm, hay sau đó là các thành tích ở đấu trường APMO của Trần
Quang Vinh, Trần Vĩnh Hưng thì phải gần 10 năm sau khi thành lập, PTNK
mới có huy chương IMO đầu tiên của Nguyễn Đăng Khoa vào năm 2003.
Thành tích này được tiếp nối bởi Trần Chiêu Minh (2005, HCB), Đặng
Trần Tiến Vinh (2008, HCĐ), Phạm Hy Hiếu (2009, HCB) một cách vô
cùng chật vật. Và chiếc huy chương vàng toán quốc tế đầu tiên của PTNK
đã đến sau gần 20 năm, vào năm 2013, và hẳn 2 chiếc, được đem về bởi
Phạm Tuấn Huy và Cấn Trần Thành Trung. Phạm Tuấn Huy cũng là học
sinh duy nhất của miền Nam sau đó đã đoạt 2 huy chương vàng toán quốc
tế. Ngày nay PTNK đã thường xuyên được nhắc tới, được tính tới khi nói
về đội tuyển quốc gia. Nhưng ít ai biết nhiều năm trước, có học sinh ra thi
vòng 2 đã là niềm mơ ước của các thầy cô đội tuyển toán. Chông gai lắm,
khó khăn lắm nhưng đội ngũ các thầy cô giáo và các học trò đã không
chùn bước, luôn kiên gan, bền chí và luôn giữ vững tâm niệm sẽ giành
vinh quang bằng sức lực và trí tuệ của chính mình. Trung thực, công bằng,
nhiệt huyết, đó là bí quyết tạo nên sức mạnh và tinh thần của lớp lớp các
thế hệ đội tuyển toán PTNK.
Song hành với các thành công của đội tuyển toán PTNK, thật vui mừng là
không khí của phong trào chuyên toán các tỉnh phía Nam cũng được hâm
nóng và khích lệ. Giải quốc gia đã không còn phải là điều xa xỉ. Và những
thành tích quốc tế của Quảng Ngãi, Bình Định, Phú Yên, Khánh Hòa và
đặc biệt của Đà Nẵng thật đáng tự hào.
Vai trò dẫn dắt phong trào của PTNK, đầu tiên là về mặt tinh thần, rõ ràng
được khẳng định. Thêm vào đó, các ấn phẩm và các hoạt động của học
sinh PTNK như chuyên đề toán học, Seminar toán học, một mặt phục vụ

5
6 Chuyên đề Toán học số 10

phong trào học tập của các học sinh PTNK, mặt khác có tầm ảnh hưởng
Chuyên đề Toán học Phổ thông năng khiếu TP.HCM số 10

không nhỏ đến phong trào học toán ở Tp HCM và các tỉnh lân cận.
Chuyên đề toán học PTNK số 10 là một ấn phẩm đặc biệt nhằm kỷ niệm 20
năm thành lập trường PTNK. Ấn phẩm này được hình thành một mặt bởi
những bài viết rất mới của những học sinh hiện đang còn học tại trường,
mặt khác bởi những bài viết kinh điển của các tác giả đã từng học tại PTNK.
Và dĩ nhiên, không thể thiếu bài viết của tứ trụ "Trình-Dũng-Tuấn-Dũng".
Câu chuyện PTNK dĩ nhiên là còn nhiều, còn dài, khó lòng mà nói hết
được. Chỉ kể tên các tác giả đã viết cho chuyên đề PTNK đã không đủ thời
gian. Vì vậy tốt nhất là để chính các bài viết sẽ nói về các tác giả.
Vậy nhé, hãy đọc và cảm nhận tinh thần 20 năm PTNK.

Trần Nam Dũng


Chủ biên Chuyên đề Toán học PTNK

Hướng tới kỷ niệm 20 năm thành lập trường PTNK, ĐHQG TP HCM
Lời cảm ơn
Chuyên đề Toán học Phổ thông năng khiếu TP.HCM số 10

Tài liệu này sẽ không thể hoàn thành nếu thiếu đi những sự giúp đỡ nhiệt
tình, đáng trân trọng. Chúng tôi, Ban biên tập, xin gửi lời cảm ơn chân
thành đến quý thầy cô, các bạn sau đây

1. Thầy Nguyễn Tăng Vũ, Giáo viên PTNK thành phố Hồ Chí Minh.

2. Thầy Nguyễn Trọng Tuấn, Giáo viên PTNK thành phố Hồ Chí Minh.

3. Anh Võ Quốc Bá Cẩn, Giáo viên trường Archimedes Academy.

4. Anh Đặng Nguyễn Đức Tiến, Trento University, Italia.

5. Bạn Tống Hữu Nhân, Sinh viên trường Đại học Y Dược thành phố
Hồ Chí Minh.

6. Bạn Phạm Tiến Kha, Sinh viên trường Đại học Sư Phạm thành phố
Hồ Chí Minh.

7. Bạn Huỳnh Phước Trường, Sinh viên trường Đại học Sư Phạm thành
phố Hồ Chí Minh.

8. Bạn Bùi Mạnh Khang, Học sinh THPT Chuyên Lê Hồng Phong.

9. Bạn Cấn Trần Thành Trung, Duke University, USA.

10. Bạn Lê Việt Hải, Pennsylvania State University, USA.

11. Bạn Phạm Hy Hiếu, Stanford University, USA.

12. Bạn Hồ Quốc Đăng Hưng, University of Chicago, USA.

13. Bạn Phạm Tuấn Huy, Stanford University, USA.

14. Bạn Nguyễn Huy Hoàng, Minerva Schools at KGI, USA.

15. Bạn Nguyễn Minh Trường Giang, Học sinh PTNK TP. Hồ Chí Minh.

16. Bạn Nguyễn Minh Châu, Học sinh PTNK thành phố Hồ Chí Minh.

7
8 Chuyên đề Toán học số 10

Trong chuyên đề này, chúng tôi đã rất cố gắng để có thể có đầy đủ các bài
Chuyên đề Toán học Phổ thông năng khiếu TP.HCM số 10

viết của những tên tuối gắn liền với PTNK như thầy Trần Nam Dũng, thầy
Lê Bá Khánh Trình, thầy Nguyễn Thanh Dũng, thầy Nguyễn Trọng Tuấn.
Bên cạnh đó, chuyên đề còn có sự góp mặt của các bài viết mà tác giả là
các IMOer những năm gần đây của PTNK như Phạm Hy Hiếu, Trần Hoàng
Bảo Linh, Cấn Trần Thành Trung, Phạm Tuấn Huy, Hồ Quốc Đăng Hưng,
Nguyễn Huy Hoàng và Phạm Nguyễn Mạnh. Để tiếp nối truyền thống, mục
đề thi cũng không thể thiếu trong chuyên đề này với hai bài viết về Đề thi
tuyển sinh vào lớp Chuyên và đề thi chọn đội tuyển của PTNK.
Ngoài ra còn có các bài viết của các bạn học sinh, cựu học sinh PTNK với
nhiều chủ đề phong phú. Nhưng bạn đọc có thể dễ phát hiện ra rằng có
rất nhiều chuyên đề viết về hình phẳng, một đặc điểm rất đặc trưng của
học sinh Năng khiếu.
Chúng tôi có nói vui là trong lời nói đầu của Chuyên đề Toán học số 9
ra vào tháng 10/2010, có đoạn viết: "Chuyên đề Toán học số 9 của trường
PTNK mà các bạn đang cầm trên tay là một ấn phẩm có nhiều điều đặc biệt.
Thứ 1, nó được thai nghén trong một khoảng thời gian dài kỷ lục: ít nhất là
1 năm. Thứ 2, nó được ra đời cách chuyên đề trước đó 5 năm."
Chuyên đề lần này cũng đã tạo một kỷ lục mới: thai nghén trong 3 tháng
và ra đời cách chuyên đề trước đó gần 6 năm. Và hơn thế nữa, nó còn ra
đời vì mục tiêu hướng đến kỷ niệm 20 năm ngày thành lập lớp chuyên
Toán của PTNK.
Mong rằng những sự cố gắng này của Ban biên tập sẽ đem lại một tài liệu
có ích, có ý nghĩa cho các bạn học sinh, các thầy cô chuyên Toán. Hy vọng
rằng nó sẽ là một món quà từ các thầy cô, của các thế hệ đàn anh đi trước
dành tặng cho những lớp chuyên Toán của trường Phổ thông năng khiếu
hiện tại và tương lai.

Ban Biên tập Chuyên đề Toán học số 10 PTNK.

Hướng tới kỷ niệm 20 năm thành lập trường PTNK, ĐHQG TP HCM
Chuyên đề Toán học Phổ thông năng khiếu TP.HCM số 10

Quá trình hình thành và phát triển


khối Toán Tin trường PTNK

Ban biên tập

Lời Ban biên tập: Trong khi đang cố gắng viết một bài giới thiệu tổng quát về tổ
Toán trường Phổ thông Năng khiếu, ban biên tập nhận được bài viết đóng góp của
một nhóm học sinh lớp 11 Toán năm nay. Hỏi ra mới biết, đây là một bài tập của
thầy Nguyễn Tăng Vũ, vừa giúp các bạn học sinh hiểu thêm về lịch sử thành lập tổ
Toán PTNK, vừa rèn kĩ năng làm việc nhóm cho các bạn. Sau khi chỉnh sửa, biên
tập lại, ban biên tập xin giới thiệu cùng bạn đọc bài viết này. Trân trọng cảm ơn
thầy Nguyễn Tăng Vũ cùng các bạn học sinh Tiêu Phát Đạt, Thái Thành Trung, Đào
Minh Khôi, Bùi Minh Kiệt, Nguyễn Tuấn Khôi, Dương Minh Nhất đã đóng góp.

1. Sự hình thành khối Toán trường Phổ Thông


Năng Khiếu

1.1. Lịch sử thành lập trường


Lịch sử khối Toán – Tin

Sau khoảng thời gian tìm hiểu công tác đào tạo học sinh năng khiếu Toán
- Tin tại những trường Đại học phía Bắc, cụ thể là các mô hình các Khối
Chuyên Toán-Tin Đại Học Khoa Học Tự Nhiên Hà Nội và Khối Chuyên
Toán Đại Học Sư Phạm Hà Nội, khoa Toán trường Đại học Tổng Hợp đã
lập tờ trình xin được mở khối chuyên nhằm phát hiện và bồi dưỡng học
sinh năng khiếu về toán học, tin học và tạo nguồn đào tạo tốt cho khoa.
Ngày 14/07/1993, Thứ trưởng Bộ Giáo dục và Đào tạo Trần Chí Đáo ký
quyết định thành lập "Hệ phổ thông trung học chuyên Toán – Tin, trường
Đại Học Tổng Hợp Tp. HCM".
Hơn 2 năm hoạt động, Hệ phổ thông trung học chuyên Toán – Tin Học đã
gặt hái được nhiều thành tích tốt đẹp trong các kỳ thi năm 1995, 1996;

9
10 Chuyên đề Toán học số 10

mô hình giảng dạy học sinh năng khiếu ngay trong lòng trường Đại học
Chuyên đề Toán học Phổ thông năng khiếu TP.HCM số 10

đã chứng tỏ được tính ưu việt của mình. Do đó, Ban Đào Tạo Đại Học
Quốc Gia TpHCM đã lập dự án thành lập trường Phổ thông trung học
chuyên thuộc Đại học. Và ngày 4/7/1996 Bộ Trưởng Bộ GD ĐT Trần Hồng
Quân đã ký quyết định số 2693/GD-ĐT thành lập trường Phổ Thông Năng
Khiếu.

Lịch sử thành lập

Ngày 8/8/1996, GS. Trần Chí Đáo, Giám Đốc Đại học Quốc gia Thành phố
Hồ Chí Minh (ĐHQG - TP. HCM) ký quyết định số 92/ĐHQG – TCCB về
việc bổ nhiệm Ban Giám Hiệu trường Phổ Thông Năng Khiếu bao gồm:

• GS.TS Nguyễn Văn Đến, Hiệu Trưởng trường ĐH KHTN, kiêm Hiệu
Trưởng trường Phổ Thông Năng Khiếu

• PGS.TS Hoàng Văn Kiếm, Trưởng khoa Công Nghệ Thông Tin trường
ĐH KHTN, kiêm Phó Hiệu Trưởng trường Phổ Thông Năng Khiếu

• GVC. Đặng Thị Hưởng, Trưởng khoa Ngữ Văn Anh trường Đại Học
Khoa Học Xã Hội và Nhân Văn (ĐH KHXHNV), kiêm Phó Hiệu Trưởng
trường Phổ Thông Năng Khiếu.

Ngày 8/8/1996, Phó giám đốc ĐHQG, GS. Nguyễn Văn Hanh ký quyết
định số 90/ĐHQG – TCC, quyết định trường Phổ thông Năng khiếu có cơ
sở chính đặt tại trường Đại học Khoa Học Tự Nhiên (số 227 Nguyễn Văn
Cừ, Q5, TPHCM) cho đến khi có đủ điều kiện hình thành một cơ sở độc
lập.
Ngày 6/12/1996, Phó giám đốc ĐHQG TP. HCM, GS. Nguyễn Văn Hanh
ký quyết định số 206/ĐHQG/KHTC về việc giao cơ sở 135B (nay là 153
Nguyễn Chí Thanh, Phường 9, Quận 5, TP.HCM) do trường ĐHKHTN tạm
thời quản lý cho trường Phổ thông Năng khiếu kể từ ngày 1/1/1997. Đây
cũng là cơ sở của trường Phổ Thông Năng Khiếu cho đến nay. Hiện nay,
trường cũng đã bắt đầu mở một cơ sở nữa tại Linh Trung, Thủ Đức, và
tăng chỉ tiêu tuyển sinh để có thể chọn lọc thêm nhiều nhân tài.

1.2. Thành lập tổ Toán


Tổ Toán là tổ chuyên môn có lịch sử lâu đời nhất trong các tổ chuyên môn
của Trường Phổ Thông Năng Khiếu. Điều này cũng dễ hiểu vì Trường Phổ
Thông Năng Khiếu được hình thành dựa trên cơ sở các lớp chuyên Toán –
Tin của trường ĐHTH TPHCM, nay là ĐHKHTN TPHCM.

Hướng tới kỷ niệm 20 năm thành lập trường PTNK, ĐHQG TP HCM
Chuyên đề Toán học số 10 11

Những thành viên đầu tiên của tổ Toán là cô Trần Thị Lệ, các thầy Hoàng
Chuyên đề Toán học Phổ thông năng khiếu TP.HCM số 10

Lê Minh, Lê Bá Khánh Trình, Bùi Xuân Hải, Thái Minh Đường, Nguyễn Hội
Nghĩa, Lê Văn Hợp, Trần Ngọc Hội. Thời gian đầu, nhiệm vụ của tổ Toán
là dạy phần Toán cơ bản cho các lớp chuyên Toán và chuyên Tin học, cũng
như xây dựng chương trình để dạy cho đội tuyển Tóan. Về mặt khối lượng
công việc cụ thể thì không nhiều vì đầu tiên chỉ có 1 lớp chuyên Toán –
Tin, sau đó là 3 lớp rồi 5 lớp, tuy nhiên, vì chưa có kinh nghiệm về chương
trình dạy chuyên (lúc này Bộ chưa ban hành nội dung chương trình cho
các lớp chuyên) nên các thành viên của tổ đã phải vừa làm, vừa mày mò
học hỏi kinh nghiệm, dần dần lên được chương trình khung cho cấp độ
đội tuyển.
Từ năm học 1996-1997, nhiệm vụ của tổ Toán trở nặng nề hơn vì khối
chuyên Toán nay đã trở thành trường Phổ thông Năng Khiếu với các lớp
Toán – Tin – Lý – Hoá – Sinh – Anh – Văn. Để đảm bảo cho hai mặt trận –
đào tạo mũi nhọn thi HSG và đào tạo Toán phổ thông cho tất cả các lớp,
lực lượng của tổ Toán đã được bổ sung thêm rất nhiều các thầy cô giỏi từ
các trường PTTH và ĐHKHTN: cô Lợi, các thầy Danh, thầy Viết Đông, thầy
Nam Dũng, thầy Thanh Dũng, thầy Thùy, ... và sau này là các sinh viên trẻ
mới ra trường như Tuấn, Đức, Khanh (Tuấn và Đức là cựu học sinh PTNK).

Thầy Thái Minh Đường (đứng bên trái), nguyên Phó Hiệu trưởng trường
Phổ thông Năng khiếu

Và các thành viên tổ Toán có thể tự hào là đã hoàn thành tốt hai nhiệm vụ.
Cùng với các tổ chuyên môn khác, tổ Toán đã góp phần không nhỏ trong

Hướng tới kỷ niệm 20 năm thành lập trường PTNK, ĐHQG TP HCM
12 Chuyên đề Toán học số 10

việc đào tạo kiến thức phổ thông cho tất cả các khối lớp, giúp tỷ lệ đậu tốt
Chuyên đề Toán học Phổ thông năng khiếu TP.HCM số 10

nghiệp và đậu đại học của học sinh PTNK luôn ở mức cao.
Niềm tự hào của các thành viên tổ Toán không chỉ nằm ở những thành
tích bề nổi, những tỷ lệ đậu tốt nghiệp, đậu đại học hay số các huy chương
quốc gia, quốc tế. Bằng những bài giảng đầy tâm huyết, họ đã truyền cho
các thế hệ học sinh tình yêu môn toán, những kiến thức cơ bản và nền
tảng của toán học phổ thông và cả toán hiện đại. Qua phong cách dạy rất
khác nhau của các thầy cô, các học sinh đã học được lối học tập chủ động,
biết tự tìm tòi và đào sâu vấn đề. Chính điều này đã giúp cho các học sinh
PTNK tiếp tục thành công ở các bậc học cao hơn. Một điều rất quan trọng
mà các thầy muốn gửi đến các em học sinh là: động cơ học tập không chỉ
là đạt giải quốc gia, quốc tế mà quan trọng hơn cả là một kiến thức nền
tảng tốt, một phong các làm việc khoa học dành cho những phát triển về
sau. Sự trung thực trong thi cử, sự chia sẻ, hỗ trợ lẫn nhau trong học tập,
vì thế luôn là kim chỉ nam của các thầy cô và các em học sinh.
Bên cạnh đó, trong quá khứ các thầy cô tổ Toán cũng đã tổ chức nhiều đợt
tập huấn đội tuyển ở Hóc Môn, Vũng Tàu để cùng sinh hoạt, giao lưu. Đấy
cũng là một trong những ấn tượng rất sâu đậm của các cựu học sinh với
trường, là những điều cần được phát huy để nâng cao tình thầy trò trong
trường lớp. Sau nhiều năm giảng dạy, tổ toán cũng đã có nhiều thay đổi
trong danh sách thành viên. Dưới đây là một số thầy cô đã và đang công
tác tại tổ Toán:

Thầy Lê Bá Khánh Trình, người thầy có phong cách giảng bài gây ấn tượng
với những thế hệ học sinh Năng khiếu

Hướng tới kỷ niệm 20 năm thành lập trường PTNK, ĐHQG TP HCM
Chuyên đề Toán học số 10 13
Chuyên đề Toán học Phổ thông năng khiếu TP.HCM số 10

Thầy Trần Nam Dũng, người thầy quen thuộc của các học sinh chuyên Toán

Thầy Trịnh Thanh Đèo và thầy Nguyễn Văn Thùy

Thầy Nguyễn Trọng Tuấn với những bài giảng về Phương trình hàm sâu sắc

Hướng tới kỷ niệm 20 năm thành lập trường PTNK, ĐHQG TP HCM
14 Chuyên đề Toán học số 10
Chuyên đề Toán học Phổ thông năng khiếu TP.HCM số 10

Cô Nguyễn Thị Duyên, cô Nguyễn Thị Tuân, thầy Nguyễn Thanh Dũng, thầy
Tạ Hoàng Thông, thầy Nguyễn Trọng Tuấn, thầy Nguyễn Tăng Vũ, thầy
Vương Trung Dũng, thầy Võ Tiến Trình, thầy Nguyễn Ngọc Duy

2. Quá trình phát triển – Thành tích của tổ chuyên


môn
Đội tuyển khối chuyên tham gia thi HSG Quốc gia lần đầu tiên vào năm
1995 và kết quả là con số 0. Không nản chí, thầy trò lại tiếp tục miệt mài
dùi mài kinh sử, các thầy đều đem hết tâm huyết ra truyền lại cho thế hệ
học sinh và đàn em giáo viên.
Sau một năm rèn luyện vất vả, kết quả đầu tiên cũng đã đến! Trong kỳ thi
HSG Quốc gia năm học 1995-1996, học sinh Lê Long Triều đã đạt giải nhì
môn Toán, làm nức lòng các thầy cô và bạn bè. Năm đó, Triều được tham
dự kỳ thi chọn đội tuyển Việt Nam thi Toán quốc tế. Tuy chưa lọt vào được
đội tuyển nhưng Triều và Lê Quang Nẫm (mới học lớp 10) cũng đã học
hỏi được thêm rất nhiều qua hai tuần tập huấn tại Hà Nội. Quan hệ giữa
tổ Toán với các thầy ở Hà Nội cũng được thiết lập và mở rộng.
Trong lĩnh vực đào tạo mũi nhọn, những thành tích bắt đầu đến một cách
từ từ nhưng chắc chắn: sau Lê Long Triều đạt giải nhì Toán quốc gia vào

Hướng tới kỷ niệm 20 năm thành lập trường PTNK, ĐHQG TP HCM
Chuyên đề Toán học số 10 15

năm 1996 là Lê Quang Nẫm đọat HCV Olympic châu Á – Thái Bình Dương
Chuyên đề Toán học Phổ thông năng khiếu TP.HCM số 10

năm 1997 và tiếp theo là những thành tích ấn tượng vào những năm 1998,
2000, 2002, 2005 và đặc biệt là hai chiếc HCB Toán Quốc tế của Nguyễn
Đăng Khoa (2003) và Trần Chiêu Minh (2005).

Đặc biệt hơn, từ năm 2012 đến nay (tức 5 năm liên tiếp), trường Phổ
Thông Năng Khiếu luôn có đại diện trong đội tuyển Việt Nam thi Toán
Quốc tế. Việc cạnh tranh một trong sáu suất dự IMO rõ ràng là rất khó, và
việc trường Phổ Thông Năng Khiếu liên tục có học sinh trong đội tuyển dự
thi IMO là một điều rất vinh dự cho trường, cho thành phố Hồ Chí Minh,
cho toàn miền Nam.

Buổi lễ xuất quân thi HSG Quốc gia 2016

2.1. Giải quốc gia

Thành tích của PTNK qua các kì thi học sinh giỏi Toán quốc gia là vô cùng
đáng nể. Có nhiều năm, cả đội tuyển PTNK gồm 10 bạn đều có giải quốc
gia. Bảng sau đây thống kê thành tích thi quốc gia của trường trong giai
đoạn gần đây (từ 2008 đến 2016).

Hướng tới kỷ niệm 20 năm thành lập trường PTNK, ĐHQG TP HCM
16 Chuyên đề Toán học số 10

Năm Giải nhất Giải nhì Giải ba Giải KK Tổng


Chuyên đề Toán học Phổ thông năng khiếu TP.HCM số 10

2008 0 0 0 1 1
2009 0 0 1 2 3
2010 0 0 2 2 4
2011 0 1 1 2 4
2012 1 2 3 0 6
2013 1 2 1 1 5
2014 0 2 4 1 7
2015 0 2 3 2 7
2016 0 2 6 2 10

Trong đó hai học sinh đạt giải nhất, cũng là thủ khoa của kỳ thi năm đó
là: Trần Hoàng Bảo Linh (35/42 điểm) - VMO 2012 và Phạm Tuấn Huy
(33/42 điểm) - VMO 2013.

2.2. Giải quốc tế


Bắt đầu từ chiếc HCB Toán Quốc tế đầu tiên năm 2003, đến bây giờ các
học sinh ưu tú của trường Phổ Thông Năng Khiếu đã giành được tổng cộng
là 11 chiếc huy chương trong kỳ thi Toán Quốc tế (gồm 3 HCV, 6 HCB và
2 HCĐ). Sau đây là danh sách chi tiết các giải IMO của trường PTNK.

Năm Thí sinh Giải thưởng Điểm số


2003 Nguyễn Đăng Khoa 26/ 42
2005 Trần Chiêu Minh 31/42
2008 Đặng Trần Tiến Vinh 21/ 42
2009 Phạm Hy Hiếu 29/ 42
2012 Trần Hoàng Bảo Linh 20/ 42
2013 Phạm Tuấn Huy 33/ 42
2013 Cấn Trần Thành Trung 31/ 42
2014 Phạm Tuấn Huy 32/ 42
2014 Hồ Quốc Đăng Hưng 22/ 42
2015 Nguyễn Huy Hoàng 23/ 42
2016 Phạm Nguyễn Mạnh 26/ 42

Hướng tới kỷ niệm 20 năm thành lập trường PTNK, ĐHQG TP HCM
Chuyên đề Toán học số 10 17
Chuyên đề Toán học Phổ thông năng khiếu TP.HCM số 10

Đặng Trần Tiến Vinh, IMO 2008 và Phạm Hy Hiếu, IMO 2009

Trần Hoàng Bảo Linh, HCĐ IMO 2013

Hướng tới kỷ niệm 20 năm thành lập trường PTNK, ĐHQG TP HCM
18 Chuyên đề Toán học số 10
Chuyên đề Toán học Phổ thông năng khiếu TP.HCM số 10

Phạm Tuấn Huy và Cấn Trần Thành Trung - những chàng trai vàng PTNK

Học sinh Phạm Tuấn Huy đã xuất giành được hai Huy chương vàng tại
IMO 2013 (Colombia) và IMO 2014 (Nam Phi).

Hồ Quốc Đăng Hưng, HCB IMO 2014

Hướng tới kỷ niệm 20 năm thành lập trường PTNK, ĐHQG TP HCM
Chuyên đề Toán học số 10 19
Chuyên đề Toán học Phổ thông năng khiếu TP.HCM số 10

Nguyễn Huy Hoàng - HCB IMO 2015


Phạm Nguyễn Mạnh - HCB IMO 2016

2.3. Các thành tích khác

Mặc dù không phải là ưu tiên trọng điểm trong dạy và học, thành tích của
khối Toán trường Phổ Thông Năng Khiếu tại kỳ thi Olympic 30/4 truyền
thống toàn miền Nam vẫn luôn rất ấn tượng. Năm ngoái (2016), đội tuyển
Toán của trường (gồm 6 người) đã giành tổng cộng 4 Huy chương vàng
và 2 Huy chương bạc. Trong đó, khối 10 dù là lần đầu dự thi đã xuất sắc
giành trọn vẹn cả 3/3 Huy chương vàng.

Ngoài các hoạt động học tập chuyên sâu, học sinh khối Toán trường Phổ
Thông Năng Khiếu cũng có rất nhiều thành tích khác trong các trò chơi trí
tuệ, trong các hoạt động văn nghệ, thể thao,... Tiêu biểu nhất có lẽ chính
là Thân Ngọc Tĩnh (Toán 0912) và Nguyễn Huy Hoàng (Toán 1215) khi
đã xuất sắc vô địch cuộc thi học thuật thường niên của Phổ Thông Năng
Khiếu (Thách Thức Entropy), sau đó đem cầu truyền hình đường lên đỉnh
Olympia về cho trường.

Hướng tới kỷ niệm 20 năm thành lập trường PTNK, ĐHQG TP HCM
20 Chuyên đề Toán học số 10

Dù chưa thể vô địch Đường lên đỉnh Olympia, những thành tích mà Ngọc
Chuyên đề Toán học Phổ thông năng khiếu TP.HCM số 10

Tĩnh và Huy Hoàng đem lại đã góp phần làm đẹp thêm hình ảnh của không
chỉ tổ Toán, mà còn là hình ảnh chung của ngôi trường Năng Khiếu.

Nguyễn Huy Hoàng - Á quân Đường lên đỉnh Olympia 2015


Thân Ngọc Tĩnh - Á quân Đường lên đỉnh Olympia 2012

3. Lời kết
Đã gần 20 năm trôi qua kể từ khi PTNK ra đời, và cũng từng ấy thời gian,
khối Toán gồm thầy và trò đã đồng lực chung sức phấn đấu, đưa PTNK trở
thành một trong những trường chuyên lớn mạnh nhất quốc gia. Giờ đây,
cơ sở 2 của trường cũng đã được thành lập, nhiệm vụ của các giáo viên tổ
Toán cũng ngày càng trở nên vất vả hơn.
Với định hướng mục tiêu phát hiện, bồi dưỡng năng khiếu học sinh Việt
Nam trở thành những chủ nhân tương lai của đất nước, chắc chắn trong
tương lai tới, sẽ còn thêm thật nhiều học sinh của PTNK đem về vinh quang
cho bản thân, cho gia đình, nhà trường và cho đất nước.

Hướng tới kỷ niệm 20 năm thành lập trường PTNK, ĐHQG TP HCM
Chuyên đề Toán học Phổ thông năng khiếu TP.HCM số 10

Câu chuyện về những bài toán

Thầy Trần Nam Dũng


(Đại học Khoa học tự nhiên Thành phố Hồ Chí Minh)

Trong suốt hơn 20 năm dạy học, tôi đã sáng tác rất nhiều những bài toán
cho các đề kiểm tra và đề thi ở các cấp. Tuy nhiên, đa số những bài toán
rồi sẽ bị lãng quên, chỉ còn một số ít những bài toán ở lại, vượt qua thời
gian. Tôi muốn kể về những bài toán như vậy.
Bài toán đầu tiên là bài toán được chọn trong đề thi Olympic 30/4/1996.
Tôi rất thích thú với bài toán này vì để chọn được các hằng số đẹp như mơ
như thế, tôi đã rất tốn công sức, phải giải một hệ phương trình nghiệm
nguyên. Và thú vị là lời giải cũng ngắn gọn, chỉ trong 2 dòng. Bài toán đó
như sau

Bài 1. (Olympic 30/4/1996 khối lớp 10) Cho 0 ≤ x ≤ 1. Chứng minh rằng
p p
x(13 1 − x 2 + 9 1 + x 2 ) ≤ 16.

Lời giải thì vô cùng ngắn gọn. Ta có


p p 13 p 3 p
13x 1 − x 2 + 9x 1 + x2 = · x · 2 1 − x 2 + · 3x · 2 1 + x 2
2 2
13  2  3 2 
≤ x + 4(1 − x ) +
2
9x + 4(1 + x 2 )
4 4
= 16.

Có lẽ nhờ lời giải ngắn gọn đó mà bài toán đã được chọn và còn được đưa
vào vị trí bài toán dễ. Ban giám khảo không ngờ rằng đây là bài toán khó
nhất của kỳ thi. Chỉ có 1 học sinh duy nhất giải được bài toán này, đó là
bạn Vũ Đức Phú. Ba học trò của tôi là Lê Quang Nẫm, Nguyễn Lê Lực và
Lưu Minh Đức đã bó tay. “Em đã thử mọi cách mà không ra, kể cả dùng đạo
hàm” - Nẫm nói. Chắc là Nẫm tính đạo hàm sai, chứ không cậu ấy đã đi
đến cái phương trình này.

21
22 Chuyên đề Toán học số 10

Bài 2. (Olympic Trại hè Phương Nam 2016) Giải phương trình


Chuyên đề Toán học Phổ thông năng khiếu TP.HCM số 10

13(1 − 2x 2 ) 9(1 + 2x 2 )
p + p = 0.
1 − x2 1 + x2
2
Và sẽ tìm được điểm rơi x = p .
5
Liên quan đến nhóm 3 bạn Nẫm, Lực, Đức (bây giờ đều là giáo sư Toán)
còn có 1 bài toán thú vị khác, đó là bài toán được tôi sáng tác trên cơ sở
phát triển bài toán sau:
Cho các số dương a, b, x, y thỏa mãn điều kiện x y = ax + b y. Chứng minh
rằng
p p 2
x + y ≥ ( a + b) ,
mà tôi cho lớp 10 chuyên toán làm ngay trong buổi học đầu tiên.

Bài 3. Cho các số dương a, b, c, x, y, z thỏa mãn điều kiện x yz = ax + b y +


cz. Chứng minh rằng
p p p
x + y + z > a + b + b + c + c + a.

Sau này, tôi đã khai thác đề tài này theo. Hướng tìm giá trị nhỏ nhất của
biểu thức x + y + z. Kết quả liên quan đến nghiệm của một phương trình
bậc 3, do đó tôi đã chọn các hằng số a, b, c để phương trình đó có nghiệm
đẹp, từ đó dẫn đến bài toán sau:

Bài 4. Cho ba số dương x, y, z thỏa mãn điều kiện 2x yz = 2x + 4 y + 7z.


Tìm giá trị nhỏ nhất của P = x + y + z.

Bài toán này được thầy Nguyễn Khắc Minh chỉnh trang lại một chút và
chọn vào đề thi chọn đội tuyển IMO năm 2001.
Mùa chọn đội tuyển trường năm 2001 xảy ra tình huống là có một số bạn
xếp cuối cùng điểm. Vậy là phải thi lại. Tôi phụ trách ra đề phần đại số đã
lấy ý tưởng phép co để xét một tam giác nội tiếp trong ellip x 2 + 3 y 2 = 4
rồi tìm diện tích lớn nhất. Đại loại là cho x 2 + 3 y 2 = 4 và z 2 + 3t 2 = 4, tìm
giá trị lớn nhất của x t − yz − x − t + y + z. Tuy nhiên tôi đã nhầm lẫn thế
nào mà cuối cùng giải không ra, tính y theo x cũng không xong mà tính x
theo y cũng không được, đều dẫn đến các phương trình bậc 3 không giải
được. Thú vị là từ bài toán sai này sau này tôi đã phát biểu thành một bài
toán đẹp, được sử dụng làm đề thi học sinh giỏi quốc gia năm 2002 :

Bài 5. (VMO 2002) Cho hai đa thức

P(x) = 4x 3 − 2x 2 − 15x + 9, Q(x) = 12x 3 + 6x 2 − 7x + 1.

Hướng tới kỷ niệm 20 năm thành lập trường PTNK, ĐHQG TP HCM
Chuyên đề Toán học số 10 23

a) Chứng minh rằng mỗi một đa thức đều có 3 nghiệm thực phân biệt
Chuyên đề Toán học Phổ thông năng khiếu TP.HCM số 10

b) Gọi α và β lần lượt là các nghiệm thực lớn nhất của P và Q tương
ứng. Chứng minh rằng α2 + 3β 2 = 4.
Năm 2003, Nguyễn Đăng Khoa lọt vào đội tuyển quốc gia dự IMO tại Nhật
Bản còn tôi vinh dự được tháp tùng anh Nguyễn Khắc Minh. Năm đó lần
đầu tiên được nhìn thấy mấy cái booklet nguyên bản của các trưởng đoàn
cho. Đề thi quốc gia của Ấn Độ năm đó có bài:
Cho x, y là các số thực dương thỏa mãn điều kiện x + y = 2. Chứng minh
rằng
x 3 y 3 (x 3 + y 3 ) ≤ 2.
Từ bài toán này, tôi đã đặt ra bài toán tổng quát:
Cho số nguyên dương n, tìm số nguyên dương k nhỏ nhất sao cho bất đẳng
thức sau đúng với mọi số dương x, y thỏa mãn x + y = 2

x k y k (x n + y n ) ≤ 2.

Bằng cách chọn x = 1 − ", y = 1 + " rồi dùng khai triển vô cùng bé, tôi
n(n−1)
tìm được điều kiện k ≥ 2 . Việc chứng minh bất đẳng thức

x n(n−1)/2 y n(n−1)/2 (x n + y n ) ≤ 2, (∗)

đúng với mọi số dương x, y thỏa mãn x + y = 2, tiến hành theo phương
pháp quy nạp toán học, được quy về việc chứng minh

x n y n (x n+1 + y n+1 ) ≤ x n + y n .

với điều kiện x + y = 2. Tiến hành thuần nhất hóa, ta được bất đẳng thức
tương đương
 x + y 2n+1
x n y n (x n+1 + y n+1 ) ≤ (x n + y n ) .
2
Bất đẳng thức này nếu cho y = 1 thì thành bài toán sau:
Bài 6. (VMO 2011) Cho số nguyên dương n. Chứng minh rằng với mọi số
thực dương x ta có bất đẳng thức

x n (x n+1 + 1) x + 1 2n+1
 ‹
≤ .
xn + 1 2
Sau này, trong quá trình dạy một nhóm học sinh lớp 8, tôi đã phát hiện
ra một cách chứng minh khác cho bất đẳng thức (∗) ở trên, một dạng quy
nạp khá độc đáo: Từ A(n) chứng minh A(2n) và từ A(n), A(n + 1) chứng
minh A(2n + 1). Bạn đọc hãy thử xem nhé.

Hướng tới kỷ niệm 20 năm thành lập trường PTNK, ĐHQG TP HCM
24 Chuyên đề Toán học số 10

Bài 7. (Hello IMO 2007) Cho k là một số thực thuộc khoảng (−1, 2) và
Chuyên đề Toán học Phổ thông năng khiếu TP.HCM số 10

a, b, c là ba số thực phân biệt. Chứng minh rằng ta có bất đẳng thức


 
 2  1 1 1 9(2 − k)
a + b + c + k(ab + bc + ca)
2 2
+ + ≥ .
(a − b)2 (b − c)2 (c − a)2 4

Dấu bằng xảy ra khi và chỉ khi nào?

Trong phần bình luận, tôi có đặt ra một bài toán như sau:

Bài 8. Cho a, b, c là các số thực không âm đôi một khác nhau. Chứng minh
rằng  
1 1 1
(ab + bc + ca) + + ≥ 4.
(a − b)2 (b − c)2 (c − a)2
Vì năm đó bài toán chưa được sử dụng nên năm sau tôi đã gửi bài toán
này vào danh sách đề đề xuất VMO 2008 và đã được chọn.
Năm 2011, làm đề chọn đội tuyển ở Côn Sơn. Các nguồn đề đề xuất rất
hạn chế và thầy Nguyễn Khắc Minh yêu cầu phải tìm một bài đại số. Vẩn
vơ chưa tìm được ý nào hay, tự dưng vớ cuốn Short list 2010, đọc qua có
bài

Bài 9. Four real number p, q, r, s satisfy p + q + r + s = 9 and p2 + q2 + r 2 +


s2 = 21. Prove that there is a permutation (a, b, c, d) of (p, q, r, s) such that
a b − cd ¾ 2.

Đọc kỹ lời giải thì thấy ý chính của bài toán là nếu giả sử p ¾ q ¾ r ¾ s thì
ta sẽ có p + q ¾ 5. Từ ý tưởng này, tôi nghĩ đến bài toán tổng quát là với
hệ 2 điều kiện

a1 + a2 + · · · + an = A, a12 + a22 + · · · + an2 = B,

và a1 , a2 là hai số lớn nhất, tìm giá trị nhỏ nhất của biểu thức a1 + a2 . Sau
khi chuẩn hóa tôi đi đến bài toán.

Bài 10. (Việt Nam TST 2011) Cho số nguyên dương n ¾ 3. Các số thực
x 1 , x 2 , . . . , x n thỏa mãn điều kiện:

a) x 1 ¾ x 2 ¾ . . . ≥ x n .

b) x 1 + x 2 + · · · + x n = 0.

c) x 12 + x 22 + · · · + x n2 = n(n − 1).

Tìm giá trị lớn nhất và giá trị nhỏ nhất của S = x 1 + x 2 .

Hướng tới kỷ niệm 20 năm thành lập trường PTNK, ĐHQG TP HCM
Chuyên đề Toán học số 10 25

Bài toán này đầu tiên thầy Nguyễn Chu Gia Vượng giải được bằng phương
Chuyên đề Toán học Phổ thông năng khiếu TP.HCM số 10

pháp phản chứng (khá khó khăn). Sau đó tôi đã tìm ra cách giải bằng quy
nạp và định lý Lagrange rất đẹp mắt.
Những bài toán hay, đẹp, đáng nhớ có lẽ vẫn còn nhiều (trong đó có nhiều
bài toán của tác giả khác nhưng vì mình tìm được lời giải đẹp nên vẫn
nhớ), nhưng tôi chỉ xin tạm dựng ở 10 bài toán cùng chủ đề bất đẳng thức
và cực trị.

Hướng tới kỷ niệm 20 năm thành lập trường PTNK, ĐHQG TP HCM
26 Chuyên đề Toán học số 10
Chuyên đề Toán học Phổ thông năng khiếu TP.HCM số 10

Hướng tới kỷ niệm 20 năm thành lập trường PTNK, ĐHQG TP HCM
Chuyên đề Toán học Phổ thông năng khiếu TP.HCM số 10

Bài toán đường và điểm cố định

Thầy Lê Bá Khánh Trình


(Đại học Khoa học tự nhiên Thành phố Hồ Chí Minh)

Lời Ban biên tập: Bài viết này được tổng hợp từ các bài giảng của thầy Trình cho
đội tuyển Phổ thông năng khiếu và đội tuyển TPHCM năm 2013. Thầy giảng nhiều
bài nhưng để chủ đề được thống nhất, chúng tôi xin giới thiệu các bài liên quan đến
đường và điểm cố định.

1. Một số bài tập có hướng dẫn giải.


Bài 1. Cho tam giác ABCcó D, E, F theo thứ tự là trung điểm các cạnh
BC, CA, AB. Gọi K là một điểm cố định thuộc đoạn E F và giả sử đường tròn
đường kính AD cắt một đường thẳng bất kì qua K tại M , N . Các đường
thẳng M F, N F cắt đường tròn đường kính AD lần lượt ở P, Q. Chứng minh
rằng trung điểm PQ thuộc một đường tròn cố định.

Gợi ý.

27
28 Chuyên đề Toán học số 10

Gọi O là trung điểm AD thì O cố định và cũng là tâm của đường tròn đang
Chuyên đề Toán học Phổ thông năng khiếu TP.HCM số 10

xét. Gọi H là giao điểm của PQ, E F . Dễ dàng chứng minh được H cố định
(có thể dùng định lý Pascal hoặc bổ đề Haruki). Suy ra đường tròn cần tìm
là đường tròn đường kính OH.
Bài 2. Cho tam giác ABC nội tiếp đường tròn (O; R) và P thay đổi sao cho
OP = R3 . Các đường thẳng AP, BP, C P cắt (O) tại K, M , N . Gọi α1 là đường
tròn qua A, K và tiếp xúc với OA tại A. Định nghĩa tương tự với α2 , α3 . Chứng
minh rằng các đường tròn α1 , α2 , α3 có đúng 2 điểm chung là E, F và trung
điểm I của E F thuộc đường cố định.
Gợi ý.

Ta thấy O có cùng phương tích đến 3 đường tròn, ngoài ra điểm P cũng
thế. Do đó, ba đường tròn α1 , α2 , α3 có nhiều hơn 1 tâm đẳng phương nên
có vô số tâm đẳng phương. Điều này cho thấy 3 đường tròn này phải có 2
điểm chung. Bằng cách tính phương tích, dễ dàng chứng minh được trung
điểm I thuộc đường tròn (O; 3R).
Bài 3. Cho tam giác nhọn ABC có BC cố định và A di động. Xét các đường
cao AD, BE, C F Gọi P là điểm thuộc AD sao cho ∠ABP + ∠AC P = ∠E P F .
Đường tròn (A, AP) tiếp xúc với một đường tròn qua B, C tại điểm T .

Hướng tới kỷ niệm 20 năm thành lập trường PTNK, ĐHQG TP HCM
Chuyên đề Toán học số 10 29

1. Chứng minh rằng P tồn tại và duy nhất.


Chuyên đề Toán học Phổ thông năng khiếu TP.HCM số 10

2. Chứng minh rằng H T đi qua điểm cố định.

Gợi ý.

Vẽ đường tròn qua B, F tiếp xúc với AH tại P. Dễ thấy AP 2 = AB · AF =


AC · AE nên đường tròn (C E P) cũng tiếp xúc với AH tại P. Suy ra P chính
là điểm cần dựng. Dễ dàng chứng minh đây là điểm duy nhất.

Gọi K là giao điểm của E F với BC thì (K, D, B, C) = −1 nên K E · K F = K D ·


2
K M . Gọi T 0 là tiếp tuyến kẻ từ K đến (A, AP) thì K T 0 = K E · K F = K B · KC
nên K T 0 cũng chính là tiếp tuyến của (BC T 0 ) nên T ≡ T 0 . Phần còn lại
tương đối rõ ràng.

Bài 4. Cho tam giác ABC có P là điểm thuộc đường cao AH. Đường kính
qua B của đường tròn (PAB) cắt đường kính qua C của đường tròn (PAC) ở
Q. Chứng minh rằng Q thuộc một đường cố định.

Gợi ý.

Hướng tới kỷ niệm 20 năm thành lập trường PTNK, ĐHQG TP HCM
30 Chuyên đề Toán học số 10
Chuyên đề Toán học Phổ thông năng khiếu TP.HCM số 10

Đường cố định cần tìm chính là OA. Chú ý rằng nếu gọi O1 , O2 lần lượt là
tâm của các đường tròn (ABP), (AC P) thì O1 O2 song song với BC.

Bài 5. Cho tam giác ABC, A1 B1 C1 thỏa mãn ∆ABC ∼ ∆A1 B1 C1 và A ∈


B1 C1 , B ∈ C1 A1 , C ∈ A1 B1 . Biết rằng các điểm A, B, C cố định còn A1 , B1 , C1
thay đổi.

1. Chứng minh rằng SA1 B1 C1 ≤ 4SABC .

2. Chứng minh rằng tâm đường tròn nội tiếp tam giác A1 B1 C1 thuộc một
đường cố định.

Gợi ý.
Gọi H là trực tâm tam giác ABC. Bằng biến đổi góc, dễ dàng chứng minh
được các tứ giác A1 BH C, B1 C HA, C1 AH B nội tiếp (theo thứ tự thuộc các
đường tròn (O1 ), (O2 ), (O3 )) nên suy ra H cũng chính là tâm đường tròn
ngoại tiếp tam giác ABC.
Gọi M , N , P là trung điểm các cung nhỏ BC, CA, AB không chứa A1 , B1 , C1
của các đường tròn (O1 ), (O2 ), (O3 ) thì ta có được I, H cùng thuộc đường
tròn cố định M N P.

Hướng tới kỷ niệm 20 năm thành lập trường PTNK, ĐHQG TP HCM
Chuyên đề Toán học số 10 31
Chuyên đề Toán học Phổ thông năng khiếu TP.HCM số 10

Bài 6. Cho tam giác ABC nhọn có D di động trên BC. Gọi I, J lần lượt là
trung điểm BD, C D. Đường thẳng qua I, J vuông góc với BC cắt AB, AC lần
lượt tại M , N . Chứng minh rằng tâm đường tròn ngoại tiếp AM N luôn thuộc
một đường cố định.

Gợi ý.

Gọi O là tâm đường tròn ngoại tiếp tam giác ABC và E là điểm đối xứng
với D qua M N .

Dễ thấy rằng M , N chính là tâm đường tròn ngoại tiếp (BDE), (C DE) nên
biến đổi góc được ∠BAC = ∠BEC nên E ∈ (O).

Từ đó chứng minh 5 điểm A, M , O, N , E cùng thuộc một đường tròn hay


tâm đường tròn ngoại tiếp AM N thuộc trung trực của AO.

Hướng tới kỷ niệm 20 năm thành lập trường PTNK, ĐHQG TP HCM
32 Chuyên đề Toán học số 10
Chuyên đề Toán học Phổ thông năng khiếu TP.HCM số 10

Bài 7. Cho tam giác ABC cố định có BC cố định và A di động sao cho SABC
không đổi. Gọi O, H lần lượt là tâm đường tròn ngoại tiếp và trực tâm tam
giác, E, F lần lượt là chân đường cao kẻ từ B, C. Đường thẳng qua A vuông
góc với OH cắt E F ở K. Chứng minh rằng K thuộc một đường cố định.

Gợi ý.

Hướng tới kỷ niệm 20 năm thành lập trường PTNK, ĐHQG TP HCM
Chuyên đề Toán học số 10 33

Đường thẳng cố định cần tìm chính là đường trung bình song song với BC
Chuyên đề Toán học Phổ thông năng khiếu TP.HCM số 10

của tam giác ABC.

Bài 8. Cho góc xO y có đường tròn (I) tiếp xúc với Ox, O y ở B, C và một
đường thẳng qua I cắt Ox, O y ở M , N và cắt (I) ở P, Q trong đó P ∈ I M , Q ∈
I N . Gọi K là giao điểm của QA, P B và H là giao điểm của N A, M B. Chứng
minh rằng H K đi qua điểm cố định.

Gợi ý.

Lần lượt chứng minh OH, OK vuông góc với đường thẳng M N để suy ra
rằng H K luôn đi qua O cố định.

2. Các bài toán tự luyện


Bài 9. Cho tam giác ABC nhọn có D di động trên BC. Lấy các điểm M , N lần
lượt trên AB, AC sao cho ∠BM D = ∠C, ∠C N D = ∠B. Gọi P là giao điểm của
BN , C M . Dựng điểm Q cùng phía với P so với BC sao cho ∆QM N ∼ ∆P BC.
Chứng minh rằng đường thẳng DQ luôn đi qua một điểm cố định.

Bài 10. Cho tam giác ABC cân tại A ngoại tiếp đường tròn (I). Một tiếp
tuyến thay đổi của (I) cắt AB, AB, BC ở M , N , P. Gọi Q là giao điểm của
BN , C M và IQ cắt AP ở K. Chứng minh rằng tâm đường tròn (PQK) thuộc
một đường cố định.

Hướng tới kỷ niệm 20 năm thành lập trường PTNK, ĐHQG TP HCM
34 Chuyên đề Toán học số 10

Bài 11. Cho góc nhọn xO y và (I) tiếp xúc với Ox, O y lần lượt tại A, B. Biết
Chuyên đề Toán học Phổ thông năng khiếu TP.HCM số 10

rằng I B cắt Ox tại C. Điểm M tùy ý di động trên AC. Tiếp tuyến kẻ từ M
của (I) cắt O y tại N . Giả sử I K⊥C N . Chứng minh rằng đường thẳng đối
xứng với M K qua I K luôn đi qua điểm cố định.
Bài 12. Cho tam giác nhọn ABC nội tiếp (O) có B, C cố định và A di động
trên (O). Gọi I, M lần lượt là trung điểm đường cao AH, phân giác AK. Gọi
W là điểm nằm trên AK sao cho ∠AC M = ∠BC N . Giả sử I N cắt (O) tại
P, Q. Chứng minh rằng trung điểm J của PQ thuộc một đường tròn cố định.
Bài 13. Cho tam giác ABC có D nằm trên tia C B sao cho BD = 12 BC. Đường
thẳng bất kì song song với BC cắt AB, AC ở M , N . DM cắt (DAC) ở P và P N
cắt (DAC) ở Q. Chứng minh rằng trung điểm I của PQ thuộc một đường cố
định.
Bài 14. Cho tam giác ABC có D, E, F là chân các đường cao. Đường thẳng
bất kì qua trực tâm H của tam giác ABC cắt BC, CA, AB theo thứ tự tại
P, Q, R. Gọi K, M , N lần lượt là trung điểm của H P, HQ, HR. Chứng minh
rằng DK, E M , F N đồng quy tại S và S di chuyển trên một đường cố định.
Bài 15. Cho tam giác ABC nội tiếp (O). Xét điểm D thuộc cung nhỏ BC
không chứa A và I là trung điểm BC. Đường thẳng qua I song song với AD
cắt nửa đường tròn đường kính BC (phần nằm cùng phía với A) tại điểm J.
Đường thẳng đối xứng với AB qua BJ và đường thẳng đối xứng với CA qua
C J cắt nhau tại K. Chứng minh rằng K D đi qua điểm cố định.
Bài 16. Cho tam giác nhọn ABC nội tiếp (O) có B, C cố định và A di động
trên (O). Gọi BD, BE là phân giác trong, ngoài tại đỉnh B và C F, C G là phân
giác trong, ngoài tại đỉnh C của tam giác ABC. Biết rằng E F cắt BD ở M ,
G D cắt C F ở N . Chứng minh rằng phân giác trong góc ∠M AN đi qua điểm
cố định.
Bài 17. Cho tam giác ABC cân tại A và đường tròn nội tiếp (I) tiếp xúc
với BC, CA, AB lần lượt tại D, E, F . Đường thẳng qua D cắt (I) tại P và cắt
AB, AC tại M , N . Biết rằng Q là giao điểm của M E, N F . Chứng minh rằng
DQ đi qua điểm cố định.
Bài 18. Cho tam giác ABC nội tiếp (O) có trung tuyến AD, trực tâm H. Giả
sử P di động trên AD. Đường thẳng BD, C D cắt (O) lần lượt tại M , N . Gọi
R, S lần lượt là điểm đối xứng với M , N qua trung điểm các cạnh AC, AB.
Chứng minh rằng tâm đường tròn ngoại tiếp tam giác HRS luôn thuộc một
đường cố định.
Bài 19. Cho tam giác ABC nhọn có M , N thay đổi trên AB, AC sao cho
AM ON nội tiếp.

Hướng tới kỷ niệm 20 năm thành lập trường PTNK, ĐHQG TP HCM
Chuyên đề Toán học số 10 35

1. Chứng minh rằng đường tròn (M , M B) và (N , N C) có hai giao điểm


phân biệt và có một điểm nằm trong (O), đặt là K.
Chuyên đề Toán học Phổ thông năng khiếu TP.HCM số 10

2. Chứng minh đường thẳng qua hai giao điểm này qua một điểm cố định
và K di chuyển trên một đường cố định.

Bài 20. Cho góc xAy và (I) tiếp xúc với Ax, Ay tại B, C. Điểm P thay đổi
trên cung nhỏ BC và I là điểm thay đổi trên đường thẳng qua O, vuông góc
với AP. Gọi M , N là giao điểm của BP, C P với đường tròn (I, I P). Chứng
minh rằng trung điểm M N thuộc một đường cố định.

Bài 21. Cho tam giác nhọn ABC nội tiếp (O) có B, C cố định và A di động
trên (O). Gọi D là trung điểm BC, trực tâm H và đường cao BE, C F. Gọi
M , N là các điểm thuộc AB, AC sao cho M , D, N thẳng hàng và AM = AN .
Giả sử M N cắt (DE F ) tại P. Chứng minh H P luôn đi qua điểm cố định.

Bài 22. Cho tam giác nhọn ABC nội tiếp (O) có B, C cố định và A di động
trên (O). Gọi E, F là chân đường cao kẻ từ B, C và I là trung điểm BC.
Đường thẳng qua I song song với C F cắt (BF I) tại M . Đường thẳng qua I
song song với BE cắt (C E I) tại N . Gọi d1 , d2 là các đường thẳng đối xứng
với M N qua AB, AC. Chứng minh giao điểm d1 , d2 thuộc đường cố định.

Bài 23. Cho tam giác ABC có tâm đường tròn nội tiếp I và P là điểm cố
định thuộc BC. Gọi M , N lần lượt là các điểm thuộc BI, C I sao cho M I N P
là hình bình hành. Gọi J, K lần lượt là tâm của (IAB), (IAC). Chứng minh
rằng giao điểm Q của I M , K N luôn thuộc một đường tròn cố định.

Bài 24. Cho tam giác ABC cân tại A có I là trung điểm BC. Một đường thẳng
qua I cắt AB, AC ở M , N . Giả sử BC cắt các đường tròn (M , M A) và (N , N A)
lần lượt tại P, Q. Giả sử D đối xứng với A qua M N (P, I nằm khác phía so với
AB, Q, I khác phía so với AC). Chứng minh rằng đường tròn (PQD) luôn đi
qua điểm cố định.

Bài 25. Cho tam giác nhọn ABC nội tiếp (O) có B, C cố định và A di động
trên (O), I là tâm đường tròn nội tiếp. Biết rằng AI cắt BC tại D và J là
trung điểm AD. Biết rằng BJ, C J cắt đường tròn đường kính AC, AB lần lượt
tại E, F (E, J nằm cùng phía so với AC, F, J nằm cùng phía với AB). Đường
thẳng đối xứng với BI, C I qua các đường thẳng BE, C F cắt nhau ở K. Chứng
minh rằng AK đi qua điểm cố định.

Bài 26. Cho tam giác ABC có trung điểm D của BC và H là trực tâm.

1. Chứng minh tồn tại I thuộc AD sao cho ∠IAB = ∠IAC.

Hướng tới kỷ niệm 20 năm thành lập trường PTNK, ĐHQG TP HCM
36 Chuyên đề Toán học số 10

2. Đường tròn qua I và A cắt AB, AC lần lượt tại M , N . Chứng minh rằng
Chuyên đề Toán học Phổ thông năng khiếu TP.HCM số 10

trực tâm tam giác AM N luôn thuộc một đường thẳng cố định d và d
đi qua điểm J đối xứng với I qua H.
Bài 27. Cho tam giác nhọn ABC nội tiếp (O) có B, C cố định và A di động
trên (O). Gọi AD là đường phân giác trong và đường tròn đường kính AD cắt
BC, CA, AB lần lượt tại K, M , N . Giả sử K M , K N cắt đường tròn đường kính
AD tại P, Q. Chứng minh rằng đường tròn (K PQ) luôn đi qua điểm cố định.
Bài 28. Cho tam giác nhọn ABC nội tiếp (O) có B, C cố định và A di động
trên (O). Điểm D đối xứng với A qua O. Giả sử C D cắt AB tại E, BD cắt
AC tại F . Đường thẳng EO cắt đường thẳng đối xứng với BC qua phân giác
∠AC D tại M , đường thẳng F O cắt đường thẳng đối xứng với BCqua phân
giác ∠ABD tại N . Chứng minh M N luôn qua một điểm cố định.
Bài 29. Cho tam giác nhọn ABC nội tiếp (O) có B, C cố định và A di động
trên (O). Dựng hình vuông ABDE, AC GF nằm phía ngoài tam giác. Giả sử
AF, BD cắt nhau tại M , AE, C G cắt nhau tại N . Giả sử (DM F ), (GN E) cắt
nhau tại P, Q. Chứng minh rằng đường thẳng PQ đi qua điểm cố định.
Bài 30. Cho tam giác ABC có BC cố định. Kí hiệu β, γ lần lượt là các đường
tròn tiếp xúc với BC ở B, C và cùng bán kính r bé hơn BC. Điểm A thay đổi
sao cho đường tròn α qua A và tiếp xúc với BC tại trung điểm M của BC có
bán kính lớn hơn r. Gọi t 1 là tiếp tuyến chung trong của α, β và t 2 là tiếp
tuyến chung trong của γ, α. Giả sử t 1 , t 2 lần lượt cắt BC tại E, F và t 1 , t 2 cắt
nhau tại D. Giả sử tam giác DE F không cân tại D. Chứng minh rằng tâm
đường tròn nội tiếp của tam giác DE F thuộc một đường cố định.
Bài 31. Cho tam giác nhọn ABC nội tiếp (O) có B, C cố định và A di động
trên (O). Gọi M , N lần lượt là trung điểm các cung AB, AC. Xét các đường
tròn tâm M , N tiếp xúc với AB, AC và t là tiếp tuyến chung ngoài của chúng
sao cho A và B, C khác phía với t. Giả sử t cắt AB, AC lần lượt tại P, Q. Gọi
K là giao điểm của M P, NQ. Chứng minh K thuộc một đường cố định.
Bài 32. Cho tam giác ABC có tâm đường tròn nội tiếp I và E, F là tiếp điểm
của (I) với AB, AC. Điểm P thay đổi trên cung E F sao cho tiếp tuyến tại P
của (I) cắt đường thẳng E F ở M . Đường thẳng qua M song song với AB
cắt AP tại N . Chứng minh rằng tâm đường tròn ngoại tiếp tam giác (E M N )
thuộc một đường cố định.
Bài 33. Cho tứ giác ABC D nội tiếp có ∠A = ∠C = 90◦ . Các điểm M , N , P, Q
lần lượt thuộc các đoạn BA, BC, DA, DC kéo dài sao cho ∠M DA = ∠N DC =
∠P BA = ∠QBC. Gọi H là giao điểm của AQ, C P, K là giao điểm của AN , C M .
Chứng minh rằng trung điểm I của H K thuộc một đường cố định.

Hướng tới kỷ niệm 20 năm thành lập trường PTNK, ĐHQG TP HCM
Chuyên đề Toán học Phổ thông năng khiếu TP.HCM số 10

Bài giảng về Phương trình đồng dư và


Thặng dư chính phương

Thầy Nguyễn Thanh Dũng 1


(Giáo viên trường PTNK Thành phố Hồ Chí Minh)

1. Kiến thức cần nhớ


Cho n là số nguyên dương. Khi đó, số nguyên x 0 gọi là nghiệm của phương
trình đồng dư f (x) ≡ 0 (mod n) khi f (x 0 ) chia hết cho n. Ta quy ước hai
nghiệm của phương trình là phân biệt nếu như chúng không đồng dư với
nhau theo modulo n.
Định lý Lagrange: Cho số nguyên tố p và n ∈ Z+ . Nếu

f (x) = an x n + an−1 x n−1 + . . . + a1 x + a0

là đa thức hệ số nguyên và (an , p) = 1 thì phương trình f (x) ≡ 0 (mod p)


có không quá n nghiệm phân biệt.
Hệ quả: nếu p là số nguyên tố lẻ và d là ước nguyên dương của p − 1 thì
x d ≡ 1 (mod p) có đúng d nghiệm phân biệt theo modulo p.

2. Bài tập rèn luyện


Bài 1. Cho n, k là các số nguyên dương.

1. Tìm Cnk , Cn+1
k k
, . . . , Cn+k .
1 3 2n−1

2. Tìm C2n , C2n , . . . , C2n .

Bài 2. Tìm n ≥ 2 sao cho Cnk là số lẻ với mọi k ∈ {0, 1, 2, . . . , n} .


1
Ban biên tập trích từ Bài giảng tại Gặp gỡ Toán học 2014

37
38 Chuyên đề Toán học số 10

Bài 3. Tìm n ≥ 2 sao cho i + j và Cni + Cnj có cùng tính chẵn lẻ với mọi
i, j ∈ [0, n].
Chuyên đề Toán học Phổ thông năng khiếu TP.HCM số 10

Bài 4. Cho a là số nguyên và p nguyên tố lẻ với (a, p) = 1. Đặt A =


{1, 2, 3, . . . , p − 1} . Xét phương trình x 2 ≡ a (mod p) (*).

1. Giả sử p = 4k + 1 và a = −1. Chứng minh rằng x = (2p)! là nghiệm


của phương trình.

2. Chứng minh rằng nếu (∗) có nghiệm với a = −1 thì p = 4k + 1.

3. Chứng minh rằng nếu (∗) có nghiệm thì có đúng 2 nghiệm thuộc A.

4. Gọi P là tập con của A sao cho (∗) có nghiệm. Chứng minh rằng |P| =
p−1
2 .

p−1
5. Chứng minh rằng (∗) có nghiệm khi và chỉ khi a 2 ≡ 1 (mod p).

6. Gọi S là tổng tất cả các phần tử của P khi p = 4k + 1. Tính S theo k.

Bài 5. 1. Cho p là số nguyên tố và xét đa thức f (x) = (x 2 − 2)(x 2 +


7)(x 2 + 14). Chứng minh rằng tồn tại x ∈ N sao cho p| f (x).

2. Tìm tất cả các số nguyên dương n sao cho f (x) ≡ 0 (mod n) có


nghiệm.

Bài 6. Cho a > 1 là số nguyên dương và xét hàm số f (x) = x n − a với n > 1.
Với mỗi số nguyên dương m, giả sử rằng tồn tại k nguyên sao cho m| f (k).
Chứng minh rằng a = b n , b ∈ Z.

Bài 7. Tìm số nghiệm của phương trình x 2 ≡ 1 (mod n) trong các trường
hợp sau đây:

1. n = p m với p là số nguyên tố lẻ.

2. n = 2k với k ∈ Z, k ≥ 2.

Bài 8. Với n nguyên dương lớn hơn 2, hỏi có bao nhiêu số chính phương
modulo 2n thuộc tập hợp {1, 2, 3, . . . , 2n − 1}?
α α α
Bài 9. Cho n > 1 nguyên dương và đặt n = 2α p1 1 p2 2 . . . pk k với p1 , p2 , . . . , pk
là các số nguyên tố lẻ và α, α1 , α2 , . . . , αk là các số nguyên dương. Chứng
minh rằng x 2 ≡ 1 (mod n) có 2k+" nghiệm với

1. " = 0 nếu α2 = α.

Hướng tới kỷ niệm 20 năm thành lập trường PTNK, ĐHQG TP HCM
Chuyên đề Toán học số 10 39

2. " = 1 nếu α = 2.
Chuyên đề Toán học Phổ thông năng khiếu TP.HCM số 10

3. " = 2 nếu α > 2.

Bài 10. Cho số nguyên tố p > 3. Xét đa thức hệ số nguyên f (x) = ax 2 +


b x + c và (a, p) = 1. Chứng minh rằng:

1. Nếu có p số nguyên x 0 , x 1 , . . . , x p−1 phân biệt sao cho f (x i ) là số chính


phương (mod p) với i = 0, 1, . . . , p − 1 thì phương trình f (x) ≡ 0
(mod p) có nghiệm.

2. Nếu ∆ = b2 − 4ac 6= 0 (mod p) thì f (x) ≡ 0 (mod p) hoặc có 2


nghiệm hoặc vô nghiệm.

Bài 11. Cho p là số nguyên tố. Chứng minh rằng tồn tại các số tự nhiên
x, y, z, t thỏa mãn đẳng thức

x 2 + y 2 + z 2 = pt với 0<t<p .

Bài 12. Cho số nguyên tố p > 3 và p chia 3 dư 2. Đặt A = x 4 + x 2 y 2 + y 4


với x, y ∈ Z. Chứng minh rằng nếu A chia hết cho p thì cũng chia hết cho
p4 .

Bài 13. Cho số nguyên tố p > 3 và a ∈ A = {1, 2, 3, . . . , p − 1} . Xét phương


trình x 3 ≡ a (mod p). Gọi Q là tập con của A sao cho phương trình trên có
nghiệm.

1. Tìm |Q| khi a = 1.

2. Chứng minh rằng nếu p = 3k + 1 thì |Q| = k.

3. Chứng minh rằng nếu p = 3k + 2 thì |Q| = p − 1.


p 
Bài 14. Cho p = 4k + 1 là số nguyên tố và a = (2k)!. Đặt q = p . Chứng
minh rằng:

1. Tồn tại x 1 , x 2 , y1 , y2 ∈ {0, 1, 2, . . . , q} sao cho (x 1 , y1 ) 6= (x 2 , y2 ) và


ax 1 + y1 ≡ ax 2 + y2 (mod p).

2. Tồn tại các số tự nhiên x, y sao cho x 2 + y 2 = p.

Bài 15. Cho số nguyên tố p và số nguyên a thỏa mãn (a, p) = 1. Chứng


minh rằng a là số chính phương modulo p khi và chỉ khi a là số chính
phương modulo p k .

Hướng tới kỷ niệm 20 năm thành lập trường PTNK, ĐHQG TP HCM
40 Chuyên đề Toán học số 10

Bài 16. Cho các số nguyên dương a, b, c lớn hơn 1. Chứng minh rằng nếu
với mỗi số nguyên dương n, tồn tại k nguyên dương sao cho a k + b k = 2c n
Chuyên đề Toán học Phổ thông năng khiếu TP.HCM số 10

thì a = b.

Bài 17. Cho a, b, c là các số nguyên và a 6= 0 sao cho an2 + bn + c là số


chính phương với mọi n > 20132014 . Chứng minh rằng tồn tại x, y nguyên
sao cho a = x 2 , b = 2x y, c = y 2 .

Bài 18. Cặp số nguyên (x, y) được gọi là giả nếu x 6= y và


2
(4x y − 1)| (4x 2 − 1) .

1. Chứng minh rằng nếu (x, y) giả thì tồn tại số tự nhiên z < x mà (x, z)
giả.

2. Chứng minh rằng nếu (x, y) giả thì ( y, x) cũng giả.

3. Chứng minh rằng nếu (x, y) giả thì x = y.

Bài 19. Chứng minh rằng phương trình x 2 + y 2 + 4 = z 4 không có nghiệm


nguyên dương.

Bài 20. Tìm tất cả các số nguyên dương a, b, c sao cho


 2
 a + b = (a, b)

b + c = (b, c)2

c + a = (c, a)2

Bài 21. Cho số nguyên tố p = 4k + 1. Tính tổng


p−1 •
2i 2
˜ • 2 ˜‹
X i
−2 .
i=1
p p

Bài 22. Cho số nguyên tố p = 3k + 2. Xét đa thức hệ số nguyên

f (x) = x 3 + ax 2 + bx + c.

1. Chứng minh rằng nếu a1 , a2 , . . . , a p là hệ thặng dư đầy đủ modulo p


thì f (a1 ), f (a2 ), . . . , f (a p ) cũng thế.

2. Tìm m ∈ {0, 1, 2, . . . , p − 1} sao cho a3 − 3b ≡ m (mod p) có nghiệm.



Bài 23. Cho p > 5 là số nguyên tố. Đặt S = p − n2 n ∈ N, n2 < p . Chứng
minh rằng S chứa hai phần tử a, b sao cho 1 < a < b và b chia hết cho a.

Hướng tới kỷ niệm 20 năm thành lập trường PTNK, ĐHQG TP HCM
Chuyên đề Toán học số 10 41

Bài 24. Tìm n ≥ 2 sao cho tồn tại hoán vị (a1 , a2 , . . . , an ) của các phần tử
Chuyên đề Toán học Phổ thông năng khiếu TP.HCM số 10

của tập hợp {1, 2, 3, . . . , n} sao cho:


1. Các số a1 , a1 a2 , a1 a2 a3 , . . . , a1 a2 a3 . . . an lập thành một hệ thặng dư
đầy đủ (mod n).

2. Các số a1 , a1 + a2 , a1 + a2 + a3 , . . . , a1 + a2 + a3 + . . . + an lập thành hệ


thặng dư đầy đủ (mod n).
Bài 25. Hỏi có tồn tại hay không các hoán vị (ai ), (bi ), (ci ), (di ) của tập hợp
50 50
ai bi = 2 ci di ?
P P
{1, 2, 3, . . . , 50} sao cho
i=1 i=1

Bài 26. Cho n lẻ và các số nguyên dương c1 , c2 , c3 , . . . , cn . Gọi S là tập hợp


các hoán vị của n số nguyên dương đầu tiên. Với mỗi a = (ai ) ∈ S, ta ký hiệu
n
H(a) =
P
ci ai . Chứng minh rằng tồn tại hai hoán vị khác nhau a, b ∈ S sao
i=1
cho H(a) − H(b) chia hết cho n!.
Bài 27. Cho số nguyên tố p có dạng 4k + 3. Tính tổng các nghiệm của hệ

x ≡ 4 (mod p)
 4

x ∈ {1, 2, 3, . . . , p − 1}

Bài 28. Tìm số nguyên dương n > 2 sao cho tồn tại hoán vị (a1 , a2 , . . . , an )
của n số nguyên dương đầu tiên mà {ai + i} và {ai − i} cũng là các hệ thặng
dư đầy đủ modulo n.
Bài 29. Cho p là số nguyên tố lẻ. Hỏi có bao nhiêu tập con của tập hợp A có
tổng các phần tử chia hết cho p nếu:
1. A = {1, 2, 3, . . . , 2p} và tập con có đúng p phần tử?

2. A = {1, 2, 3, . . . , p − 1}?
Bài 30. 1. Cho tập hợp X gồm n điểm đồng phẳng sao cho với mỗi A ∈ X
thì tồn tại B, C ∈ X mà AB = AC = 1. Ngoài ra với mỗi D, E ∈ X thì
DE ≤ 1. Chứng minh rằng n lẻ.

2. Với n ≥ 3, trong mặt phẳng tọa độ Ox y, cho n điểm Ai (x i ; yi ) sao cho


A1 A2 = A2 A3 = . . . = An A1 và x i , yi ∈ Q, i = 1, n. Chứng minh n chẵn.

3. Cho n > 2 người sao cho 2 người quen nhau thì không có người quen
chung, còn 2 người không quen nhau thì có đúng 2 người quen chung.
Chứng minh rằng 8n − 7 là số chính phương.

Hướng tới kỷ niệm 20 năm thành lập trường PTNK, ĐHQG TP HCM
42 Chuyên đề Toán học số 10
Chuyên đề Toán học Phổ thông năng khiếu TP.HCM số 10

Hướng tới kỷ niệm 20 năm thành lập trường PTNK, ĐHQG TP HCM
Chuyên đề Toán học Phổ thông năng khiếu TP.HCM số 10

Các tính chất số học của


dãy số nguyên

Thầy Nguyễn Trọng Tuấn 1


(Giáo viên trường PTNK Thành phố Hồ Chí Minh)

1. Các tính chất số học của dãy truy hồi


Bài 1. Cho dãy số Fibonacci (Fn ): F0 = F1 = 1, Fn = Fn−1 + Fn−2 (n ≥ 2).
3
Chứng minh rằng 7Fn+2 − Fn3 − Fn+1
3
chia hết cho Fn+3 .

Lời giải.
Ta có a = 7Fn+2
3
+ 7Fn+1
3
chia hết cho Fn+2 + Fn+1 = Fn+3 . Mặt khác, b =
8Fn+1 + Fn chia hết cho 2Fn+1 + Fn = Fn+1 + Fn+1 + Fn = Fn+1 + Fn+2 = Fn+3 .
3 3

3
Từ đó suy ra 7Fn+2 − Fn3 − Fn+1
3
= a − b chia hết cho Fn+3 .

Bài 2. Cho dãy số (an ) được xác định bởi

a1 = 1, a2 = 12, a3 = 20, an+3 = 2an+2 + 2an+1 − an (n ≥ 0).

Chứng minh rằng 1 + 4an an+1 là số chính phương.

Lời giải.
Bằng tính toán đơn giản ta được

4a1 a2 + 1 = 72 , 4a2 a3 + 1 = 312 , 4a3 a4 + 1 = 712 .

Nếu 1 + 4an an+1 là số chính phương thì nó phải là số chính phương lẻ. Do
đó nếu như 1+4an an+1 = (2k − 1)2 thì an an+1 = k(k−1), nghĩa là bằng tích
của hai số tự nhiên liên tiếp. Quan sát a1 a2 = 3 · 4; a2 a3 = 15 · 16, a3 a4 =
35 · 36, ta có thể dự đoán
2
an an+1 = 4n2 (4n2 − 1) ⇒ 1 + 4an an+1 = (8n2 − 1) .
1
Ban biên tập trích từ bài giảng của thầy trong file bài viết cùng tên

43
44 Chuyên đề Toán học số 10

Bằng quy nạp ta chứng minh được rằng


Chuyên đề Toán học Phổ thông năng khiếu TP.HCM số 10

2
1 + 4an an+1 = (8n2 − 1) = (an+1 + an − an−1 )2

với mọi n ¾ 1.

Bài 3. Cho dãy số (an ) thỏa mãn a0 = 1, a1 = 45, an+2 = 45an+1 − 7an với
n = 0, 1, 2, . . .
2
a) Tính số các ước dương của an+1 − an an+2 theo n.

b) Chứng minh rằng 1997an2 + 4 · 7n là số chính phương với mọi n.

Lời giải.
a) Ta có a1 = 2018. Ta chứng minh an+1
2
− an an+2 = 7n với mọi n.
Hiển nhiên đẳng thức đúng với n = 1. Giả sử đẳng thức đúng với n. Khi

2
an+2 − an+1 an+3 = (45an+1 − 7an )2 − an+1 (45an+2 − 7an+1 )
= 2025an+1
2
− 630an an+1 + 49an2 − 45an+1 an+2 + 7an+1
2
.
2
b) Từ câu a) ta có an+1 − an (45an+1 − 7an ) − 7n = 0, hay an+1
2
− 45an an+1 +
7an − 7 = 0.
2 n

Như vậy an+1 là nghiệm nguyên của phương trình x 2 −45an x +7an2 −7n = 0.
Suy ra rằng ∆ = 1997an2 + 4 · 7n phải là số chính phương.

Bài 4. (VMO 1999B) Cho hai dãy số (x n ) và ( yn ) được xác định như sau:
x 1 = 1, y1 = 2

x n+1 = 22 yn − 15x n , yn+1 = 17 yn − 12x n , n = 1, 2, 3, . . .

1. Chứng minh rằng dãy (x n ) cũng như dãy ( yn ) đều

a) Không có số hạng nào bằng 0.


b) Có vô số số hạng là số âm.
c) Có vô số số hạng là số dương.

2. Hỏi số hạng thứ 19991945 của hai dãy (x n ) và dãy ( yn ) có chia hết cho
7 hay không ? Vì sao ?

Lời giải.
Từ giả thiết ta có

17x n+1 − 22 yn+1 = 17(22 yn − 15x n ) − 22(17 yn − 12x n ).

Hướng tới kỷ niệm 20 năm thành lập trường PTNK, ĐHQG TP HCM
Chuyên đề Toán học số 10 45

1
Suy ra yn+1 = 22 (17x n+1
− 9x n ). Từ đó ta có
Chuyên đề Toán học Phổ thông năng khiếu TP.HCM số 10

1 1
• ˜
yn+1 = (17x n+1 − 9x n ) = 17 (17x n − 9x n−1 ) − 12x n .
22 22
Do đó x n+1 = 2x n − 9x n−1 và từ đó yn+1 = 2 yn − 9 yn−1 .
Như thế hai dãy số (x n ) và ( yn ) được xác định một cách độc lập như sau

x 1 = 1, x 2 = 29


x n+1 = 2x n − 9x n−1 (n ≥ 2)


y1 = 2, y2 = 22


yn+1 = 2 yn − 9 yn−1 (n ≥ 2)
Giả sử có n mà yn = 0. Khi đó

2 yn−1 − 9 yn−2 = 0 ⇒ yn−1 ≡ 0 (mod 3).

Lại có yn−1 = 2 yn−2 − 9 yn−3 ⇒ yn−2 ≡ 0 (mod 3). Cứ tiếp tục như thế ta
được y1 = 2 chia hết cho 3 (vô lí).
Chứng minh tương tự ta có kết quả đối với dãy (x n ).
Bây giờ ta có

x n+1 = 2x n − 9x n−1 = 2(2x n−1 − 9x n−2 ) − 9x n−1 ,

suy ra
x n+1 + 5x n−1 + 18x n−2 = 0 ∀n ≥ 3.
Vì dãy (x n ) không chứa số 0 nên đẳng thức trên cho ta kết quả dãy (x n )
có chứa vô số số âm và vô số số dương.
Chứng minh tương tự đối với dãy ( yn ).
2) Ta có x n+1 = −5(2x n−2 − 9x n−3 ) − 18x n−2 = −28x n−2 + 45x n−3 với n ¾ 4.
Như thế ta suy ra rằng x n−3 ≡ 0 (mod 7) khi và chỉ khi x n+1 ≡ 0 (mod 7).
Mà x 3 = 49 nên x 4k+3 ≡ 0 (mod 7) với mọi k.
Chú ý rằng 19991945 ≡ 3 (mod 4) nên số hạng thứ 19991945 của dãy (x n )
chia hết cho 7. Xét tương tự đối với dãy ( yn ) ta thấy rằng số hạng thứ
19991945 không chia hết cho 7.
Bài 5. Cho dãy số ( yn ) được xác định bởi

y1 = y2 = 1, yn+2 = (4k − 5) yn+1 − yn + 4 − 2k (n ≥ 1).

Tìm tất cả các giá trị nguyên của k sao cho mọi số hạng của dãy đều là số
chính phương.

Hướng tới kỷ niệm 20 năm thành lập trường PTNK, ĐHQG TP HCM
46 Chuyên đề Toán học số 10

Lời giải.
Ta có y3 = 2k − 2 = 4a2 ⇒ k = 2a2 + 1. Từ đó y4 = 8k2 − 20k + 13, và
Chuyên đề Toán học Phổ thông năng khiếu TP.HCM số 10

y5 = 32k3 − 120k2 + 148k − 59 = 256a6 − 96a4 + 8a2 + 1.


Dễ thấy với a = 0 ⇒ k = 1 (dãy số thỏa mãn yêu cầu bài toán).
Xét a > 0. Chú ý rằng
2 2
(16a3 − 3a) ≥ y5 = 256a6 − 96a4 + 8a2 + 1 > (16a3 − 3a − 1) .
2
Do đó ta phải có y5 = (16a3 − 3a) ⇒ a = 1 ⇒ k = 3. Dãy số đầu bài trở
thành
y1 = y2 = 1, yn+2 = 7 yn+1 − yn − 2 (n ≥ 1).
Ta có y3 = 22 , y4 = 52 , y5 = 133 . Để ý đến dãy Fibonaci
(un ) : u1 = u2 = 1, un+2 = un+1 + un (n ≥ 1).
Ta dự đoán rằng và chứng minh rằng yn = u22n−3 . Trước hết, ta có
un+2 = un+1 + un = 2un + un−1 = 3un − un + un−1 = 3un − un−2 .
Và un+2 un−2 − u2n = 1 với mọi n ¾ 3. Khi đó ta có(un+2 + un−2 )2 = 9u2n .
Suy ra
u2n+2 = 9u2n − u2n−2 − 2un−2 un+2 = 9u2n − 2(1 + u2n ) − u2n−2 = 7u2n − u2n−2 − 2.

Từ đó từ giả thiết quy nạp ta có


yn = 7 yn−1 − yn−2 − 2 = 7u22m−5 − u22n−7 − 2 = u22n−3 ,
với mọi n ¾ 2.

Bài tập tương tự


Bài 1. Tìm số nguyên dương k sao cho mọi số hạng của dãy số sau đây đều
là số nguyên
Æ
a1 = 1, an+1 = 5an + kan2 − 8, n = 1, 2, 3, . . .

Bài 2. Cho dãy số (an ) được xác định như sau a1 = 5, a2 = 11 và


an+1 = 2an − 3an−1 , n = 2, 3, . . .
Chứng minh rằng
a) Dãy số trên có vô số số dương và vô số số âm.

b) a2002 chia hết cho 11.

Hướng tới kỷ niệm 20 năm thành lập trường PTNK, ĐHQG TP HCM
Chuyên đề Toán học số 10 47

2. Tính tuần hoàn của dãy số theo modulo m


Chuyên đề Toán học Phổ thông năng khiếu TP.HCM số 10

Bài 6. Cho dãy số (an ) xác định bởi

a1 = 43, a2 = 14


an+1 = 3an + an−1 , n ≥ 2

a) Chứng minh rằng với mọi an+1 và an nguyên tố cùng nhau.

b) Chứng minh rằng với mọi số tự nhiên m, tồn tại vô hạn số tự nhiên n
sao cho cả hai số an − 1 và an+1 − 1 dều chia hết cho m.

Lời giải.
a) Gọi d = gcd(an+1 , an ). Từ hệ thức truy hồi ở đầu bài ta có d | an−1 .
Lại có an = 3an−1 + an−2 nên d | an−2 . Áp dụng liên tiếp lập luận trên ta
suy ra d | a1 và d | a2 . Từ đó d = 1 và ta có điều phải chứng minh.
b) Với mọi số tự nhiên m, có vô số cặp (r, s) thỏa mãn r < s và a r ≡ as
(mod m). Trong tập hợp vô hạn các cặp (r, s) như thế có ít nhất một cặp
(r, s) mà a r+1 ≡ as+1 (mod m).
Khi đó với mọi n ta có a r+n ≡ as+n (mod m). Như vậy dãy số đã cho tuần
hoàn theo mod m. Xét dãy số đã cho với chỉ số nguyên. Ta có a−3 ≡ a−2 ≡ 1
(mod m).
Từ đó suy ra có vô hạn n sao cho an − 1 và an+1 − 1 đều chia hết cho m.

Bài 7. Cho dãy số (an ) được xác định bởi a1 = 39, a2 = 45, an+1 = an2 − an−1 .
Chứng minh rằng tồn tại vô hạn số hạng của dãy chia hết cho 1986.

Lời giải.
Nhận xét rằng dãy số được hoàn toàn xác định nếu biết hai số hạng liên
tiếp bất kì của nó.
Xét tập hợp các cặp (an , an+1 ) theo modulo 1986, ta thấy rằng tồn tại k
sao cho cặp ak = a, ak+1 = b xuất hiện vô hạn lần. Từ đó suy ra nếu
ak = a, ak+1 = b và am = a, am+1 = b với m > k là số nhỏ nhất thì dãy số
tuần hoàn. Từ đó bài toán được chứng minh nếu trong dãy số có một số
hạng chia hết cho 1986. Điều này là rõ ràng vì a3 = 1986.

Bài 8. Cho dãy số (an ) được xác định bởi a1 = 1, an+1 = 5an + 24an2 + 25.
Æ

Chứng minh rằng tồn tại vô hạn sao cho an và an+4 đều chia hết cho 7.

Hướng tới kỷ niệm 20 năm thành lập trường PTNK, ĐHQG TP HCM
48 Chuyên đề Toán học số 10

Lời giải.
Chuyên đề Toán học Phổ thông năng khiếu TP.HCM số 10

Từ giả thiết ta có

(an+1 − 5an )2 = 24an2 + 25 ⇔ an+1


2
− 10an+1 an + an2 − 25 = 0.

2
Từ đó ta cũng có an+2 − 10an+2 an+1 + an+1
2
− 25 = 0. Suy ra rằng an , an+2 là
các nghiệm của phương trình t 2 − 10an+1 t + an+1
2
− 25 = 0.
Như thế theo định lý Vi-et ta có an+2 + an = 10an+1 ⇒ an+2 = 10an+1 − an .
Do đó
an+2 = 10an+1 − an = 10(10an − an−1 ) − an
= 99an − 10an−1
= 99(10an−1 − an−2 ) − 10an−1
= 980an−1 − 99an−2 .

Vì a3 = 119 chia hết cho 7 nên bằng lập luận như các bài toán trên ta suy
ra dãy số có vô hạn các số hạng chia hết cho 7. Lại do 980 chia hết cho 7
nên nếu an chia hết cho 7 thì an+4 cũng chia hết cho 7.
Từ đó ta có điều phải chứng minh.

Bài tập tương tự


Bài 3. (VMO 1995) Cho dãy số (an ) được xác định như sau a0 = 1, a1 = 3
và với mọi n = 0, 1, 2, 3, . . . thì
¨
an+1 + 9an nếu n ≡ 0 (mod 2)
an+2 =
9an+1 + 5an nếu n ≡ 1 (mod 2)

Chứng minh rằng


2000
X
a) ak2 chia hết cho 20.
k=1995

b) a2n+1 không thể là số chính phương với mọi n ∈ N.

Bài 4. Dãy số (an ) được xác định bởi a0 = 0, a1 = 1 và an = an−1 + an−2 +


1 (n ≥ 2). Chứng minh rằng với mọi số nguyên dương m, tồn tại hai số hạng
liên tiếp của dãy số chia hết cho m.

Hướng tới kỷ niệm 20 năm thành lập trường PTNK, ĐHQG TP HCM
Chuyên đề Toán học số 10 49

3. Tính bị chặn dưới của dãy số nguyên


Chuyên đề Toán học Phổ thông năng khiếu TP.HCM số 10

Bài 9. Từ dãy các số chính phương có thể chọn được một cấp số cộng vô hạn
hay không ?

Lời giải.
Câu trả lời là "không". Thật vậy, nếu tà dãy các số chính phương ta có thể
chọn được cấp số cộng vô hạn là an2 . Khi đó

a32 − a22 = a22 − a12 ⇒ (a3 − a2 )(a3 + a2 ) = (a2 − a1 )(a2 + a1 ).

Vì a2 + a1 < a2 + a3 nên a2 − a1 > a3 − a2 . Tiếp tục như thế, ta có

a2 − a1 > a3 − a2 > a4 − a3 > · · ·

Điều này vô lí.

Bài 10. Cho dãy số (x n ) được xác định bởi x 1 = 2 và nx n = 2(2n − 1)x n−1
với n = 2, 3, . . . Chứng minh rằng x n nguyên với mọi n.

Lời giải.
Bằng tính toán đơn giản ta có

(2n − 1)!
n!x n = 2n · 1 · 3 · 5 · · · (2n − 3)(2n − 1) = = 2n(n + 1) · · · (2n − 1).
(n − 1)!

Từ đó suy ra điều phải chứng minh vì tích của n số tự nhiên liên tiếp chia
hết cho n!.

Bài 11. (Shortlist IMO 1988) Cho dãy số nguyên (an ) được xác định bởi
a1 = 2, a2 = 7 và
1 an2 1
− < an+1 − ≤ , n ¾ 2.
2 an−1 2
Chứng minh rằng an lẻ với mọi n ¾ 2.

Lời giải.
Từ giả thiết ta có −an−1 < 2an−1 an+1 − an2 ≤ an−1 . Đặt k = 2an−1 an+1 − 2an2
thì k là số nguyên và
2an2 + k
an+1 = .
2an−1

Như vậy k là số nguyên thỏa các điều kiện −an−1 < k ≤ an−1 và 2an−1 2an2 + k .

Hướng tới kỷ niệm 20 năm thành lập trường PTNK, ĐHQG TP HCM
50 Chuyên đề Toán học số 10

0 duy nhất vì nếu có −an−1 < k < k ≤


0
Ta thấy rằng số
k như 0thế là tồn tại
an−1 và 2an−1 2an2 + k thì 2an−1 k − k , vô lí.
Chuyên đề Toán học Phổ thông năng khiếu TP.HCM số 10

Ta có
2a2 + k 98 + k
a3 = 2 = .
2a1 4
Vì −2 < k ≤ 2 nên k = 2 và a3 = 25. Lại có
2a32 + k 1250 + k
a4 = = .
2a2 14
Vì −7 < k ≤ 7 nên số k duy nhất là k = −4 và a4 = 1250−4 14 = 89. Tiếp tục
như thế ta có a5 = 137.
Từ đó bằng qui nạp ta sẽ chứng minh rằng an2 − an−1 an+1 = (−2)n−2 với mọi
n ¾ 2.
Thât vậy, theo giả thiết qui nạp ta có an2 − an−1 an+1 = (−2)n−2 , và an−1
2

n−3
an−2 an = (−2) .
Mặt khác
an−1 an+1 ≡ an2 − (−2)n−2 ≡ −(−2)n−2 (mod an )
= 2(−2)n−3 = 2(an−1
2
− an−2 an )
2
≡ 2an−1 (mod an ).
Suy ra an+1 ≡ 2an−1 (mod an ). Vậy
2
an+1 2
≡ 4an−1 2
⇒ 2an+1 2
≡ 8an−1 (mod an )).
Do đó
2
2an+1 2
≡ 8an−1 ≡ 8an−1 an + 8(−2)n−3 ≡ −(−2)n (mod an ).

Như vậy 2an+12


+ (−2)n ≡ 0 (mod an ). Từ tính duy nhất của số k ta được
k = (−2)n và có điều phải chứng minh.
Bây giờ với mọi n ¾ 3 ta có an2 − an−1 an+1 = (−2)n−2 . Xét tính chia hết cho
2 ta suy ra điều phải chứng minh.

Bài tập tương tự


Bài 5. Cho (an ) là dãy số nguyên sao cho với mọi n ¾ 1 thì
(n − 1)an+1 = (n + 1)an − 2(n − 1).
Biết 2000 chia hết a1999 . Hãy tìm giá trị n nguyên dương nhỏ nhất sao cho
2000 chia hết an .

Hướng tới kỷ niệm 20 năm thành lập trường PTNK, ĐHQG TP HCM
Chuyên đề Toán học số 10 51

4. Hệ thức bất biến đối với chỉ số


Chuyên đề Toán học Phổ thông năng khiếu TP.HCM số 10

Bài 12. Cho dãy số

 a1 = a2 = 1

(an ) : 2
an−1 +2
 an = ,n ≥ 3
an−2

Chứng minh rằng an nguyên với mọi n ∈ N∗ .

Lời giải.
2
an−1 +2
Từ giả thiết an = an−2 suy ra

an an−2 = an−1
2
+ 2.

Thay n bởi n + 1 ta có
an+1 an−1 = an2 + 2.
Trừ hai đẳng thức trên ta được
an+1 + an−1 an + an−2 a3 + a1
= = ... = = 4.
an an−1 a2

Suy ra
an+1 = 4an − an−1 .
Vậy an nguyên với mọi n ∈ N∗ .

Bài 13. Cho dãy số



 a1 = 2, a2 = 500, a3 = 2000
(an ) : an+2 + an+1 an+1
= , n≥2
an+1 + an−1 an−1

Chứng minh rằng mọi số hạng của dãy số đều là số nguyên dương và a2000
chia hết cho 22000 .

Lời giải.
Từ giả thiết suy ra an+2 an−1 = an+1
2
, nên mọi số hạng của dãy số đều khác
an+2 an+1
0. Từ đó, do an+1 = an−1 nên

an+2 an+1 a3
= = ··· = = 2.
an+1 an an an−1 a2 a1

Hướng tới kỷ niệm 20 năm thành lập trường PTNK, ĐHQG TP HCM
52 Chuyên đề Toán học số 10

Suy ra an+2 = 2an an+1 , nên mọi số hạng của dãy đều là số nguyên dương.
Chuyên đề Toán học Phổ thông năng khiếu TP.HCM số 10

Mặt khác, ta có
a2000 a1999 a
a2000 = · · · · 2 · a1 ,
a1999 a1998 a1
nên a2000 chia hết cho 22000 .

Bài 14. Cho dãy số

 a1 = 1, a2 = 2, a3 = 3

(an ) : a2
 an+1 = an − an−1 + n , n ≥ 3
an−2

Chứng minh rằng với mọi n ∈ N∗ , an là số tự nhiên và không chia hết cho 4.

Lời giải.
Bằng qui nạp, dễ thấy rằng an > 0 với mọi n ∈ N∗
a +a a +a
Từ giả thiết, ta có n+1an n−1 = nan−2n−2 . Từ đó, suy ra

an+1 + an−1 an + an−2 a3 + a1


= = ··· = = 2.
an an−1 an−1 an−2 a2 a1
Hay
an+1 = 2an an−1 − an−1 = an−1 (2an − 1) .
Bằng qui nạp chứng minh được với mọi n ∈ N∗ , an là số tự nhiên và không
chia hết cho 4.

Bài tập tương tự


Bài 6. Cho dãy số

an2
(an ) : an+1 = kan − kan−1 + .
an−2

Chứng minh rằng nếu k, a1 , a2 , a3 là các số nguyên thì an nguyên.

Bài 7. Cho dãy số



 a1 = a2 = 1
(an ) : an+1 an+2 + 5
 an+3 = , n≥0
an

Chứng minh rằng mọi số hạng của dãy số đều là số nguyên.

Hướng tới kỷ niệm 20 năm thành lập trường PTNK, ĐHQG TP HCM
Chuyên đề Toán học số 10 53

Bài 8. Cho dãy số


Chuyên đề Toán học Phổ thông năng khiếu TP.HCM số 10

a1 = a ∈ N ∗
¨
(an ) : Æ
an+1 = (an + 1) a + (a + 1) an + 2 a (a + 1) an (an + 1), n ≥ 1

Chứng minh rằng mọi số hạng của dãy số đều là số tự nhiên.

5. Dãy số và định lí Fermat nhỏ (đang cập nhật)


Bài 15. (IMO Shortlist) Cho dãy số (an ) được xác định bởi a1 = 1, a2 = 2,
a3 = 24 và
2 2
6an−1 an−3 − 8an−1 an−2
an = , n ¾ 4.
an−2 an−3
Chứng minh rằng với mọi n thì an chia hết cho n.

Lời giải.
Xét tỷ số
2
an 6an−1 an−3 − 8an−2
vn = = = 6vn−1 − 8vn−2 .
an−1 an−2 an−3

Từ đó tìm được vn = 4n−1 − 2n−1 với mọi n ≥ 2. Do đó


an an−1 a3 a
an = · ··· · 2
an−1 an−2 a2 a1
= vn vn−1 · · · v1
= (4n−1 − 2n−1 )(4n−2 − 2n−2 ) · · · (42 − 22 )(4 − 2).

Với mọi số nguyên tố p, theo định lí Fermat nhỏ ta có 4 p−1 ≡ 1 (mod p) và


2 p−1 ≡ 1 (mod p). Từ đó 4 p−1 − 2 p−1 ≡ 0 (mod p).
k k
Bây giờ giả sử n = p11 p22 · · · pm
km
là phân tích tiêu chuẩn của n. Ta có
n = mi p1 cho nên n − mi = mi (p1 − 1) với mỗi i = 1, 2, . . . , k1 . Chú ý rằng
i i

m1 > m2 > · · · > mk1 .


Chú ý rằng do n − mi < n nên trong phân tích của an có chứa k1 thừa số

4n−mi − 2n−mi = 4mi (p1 −1) − 2mi (p1 −1) ≡ 0 (mod p1 ).


i i

. k . k
Suy ra an .. p11 . Chứng minh tương tự ta có an .. pi i với mọi i = 1, 2, . . . , r. Từ
đó có điều phải chứng minh.

Hướng tới kỷ niệm 20 năm thành lập trường PTNK, ĐHQG TP HCM
54 Chuyên đề Toán học số 10

Bài 16. Cho dãy số (x n ) xác định như sau: x 1 = 7, x 2 = 50 và


Chuyên đề Toán học Phổ thông năng khiếu TP.HCM số 10

x n+1 = 4x n + 5x n−1 − 1975.


.
Chứng minh rằng x 1996 .. 1997.

Lời giải.
Xét dãy số ( yn ) xác định bởi y1 = 7, y2 = 50, x n+1 = 4x n + 5x n−1 + 22.
Dễ thấy yn ≡ x n (mod 1997). Do đó ta chỉ cần chứng minh y1996 ≡ 0
(mod 1997). Chú ý rằng 4 yn+1 + 11 = 4( yn + 11) + 5(4 yn−1 + 11) nên bằng
cách đặt zn = 4 yn + 11 ta được dãy số (zn ) xác định bởi

z1 = 39, z2 = 211, zn+1 = 4zn + 5zn−1 .

Sử dụng phương trình đặc trưng ta có

8(−1)n + 25 · 5n
zn = .
3
Từ đó bằng cách sử dụng định lí Fermat nhỏ ta dễ dàng chứng minh được
..
x1996 . 1997.

Bài 17. Cho (x n ), ( yn ) được xác định như sau x 1 = −1, y1 = 1 và

x n+1 = −3x n2 − 2x n yn + 8 yn2 , yn+1 = 2x n2 + 3x n yn − 2 yn2 , n = 1, 2, 3, . . .

Tìm số p nguyên tố sao cho x p + y p không chia hết cho p.

Lời giải.
Ta có
x k+1 + 2 yk+1 = −3x k2 − 2x k yk + 8 yk2 + 2(2x k2 + 3x k yk − 2 yk2 )
= x k2 + 4x k yk + 4 yk2
= (x k + 2 yk )2 .

Từ đó bằng quy nạp ta chứng minh với mọi n thì x n + 2 yn = 1. Ta có

x n+1 = −(x n + yn )(3x n − 4 yn ) = −3x n + 4 yn ,


yn+1 = (x n + 2 yn )(2x n − yn ) = 2x n − yn .
Suy ra

x n+2 = −3x n+1 + 4 yn+1 = −3x n+1 + 4(2x n − yn ) = −4x n+1 + 5x n .

Hướng tới kỷ niệm 20 năm thành lập trường PTNK, ĐHQG TP HCM
Chuyên đề Toán học số 10 55

Như vậy dãy (x n ) được xác định một cách độc lập như sau
Chuyên đề Toán học Phổ thông năng khiếu TP.HCM số 10

x 1 = −1, x 2 = 7, x n+2 = −4x n+1 + 5x n .

Công thức tổng quát của dãy (x n ) và ( yn ) là

1 − 4(−5)n−1 1 + 2(−5)n−1
xn = , yn = .
3 3

2−2(−5)n−1
Thành thử x n + yn = 3 . Xét p là số nguyên tố lẻ khác 3 và 5. Theo
định lí Fermat nhỏ ta có:

5 p−1 ≡ 1 (mod p) ⇒ 1 − (−5) p−1 ≡ 0 (mod p).

Nghĩa là x p + y p chia hết cho p. Xét p = 3 và p = 5 ta thấy thỏa mãn.

Bài 18. Tồn tại hay không dãy vô hạn tăng các số nguyên tố (pn ) thỏa mãn

|pn+1 − 2pn | = 1, n ¾ 1.

Lời giải.
Rõ ràng ta chỉ cần xét p1 > 3. Dễ dàng chứng minh được rằng nếu pk ≡ 1
(mod 3) thì pk+1 ≡ 2pk + 1.
Ngược lại nếu pk ≡ −1 (mod 3) thì pk+1 ≡ 2pk − 1. Từ đó pk ≡ 1 mod 3,
tương đương với pk+1 ≡ 1 (mod 3). Có hai trường hợp sau đây:

1. p1 ≡ −1 (mod 3). Khi đó pk+1 ≡ 2pk + 1 và suy ra pk ≡ 2k−1 p1 +


(2k−1 − 1). Như vậy pk ≡ 2k−1 − 1 (mod p1 ). Thành thử theo định lý
Fermat nhỏ

p p1 ≡ 2 p1 −1 − 1 (mod p1 )) ≡ 0 (mod p1 ).

Ta có điều mâu thuẫn là p p1 chia hết cho p1 .

2. p1 ≡ 1 (mod 3). Khi đó pk+1 ≡ 2pk −1 và suy ra pk ≡ 2k−1 p1 −(2k−1 −


1).
Lập luận như trên ta cũng có điều mâu thuẫn.

Vậy không tồn tại dãy số thỏa mãn yêu cầu bài toán.

Hướng tới kỷ niệm 20 năm thành lập trường PTNK, ĐHQG TP HCM
56 Chuyên đề Toán học số 10

Bài tập tương tự


Chuyên đề Toán học Phổ thông năng khiếu TP.HCM số 10

Bài 9. Cho dãy số (an ) được xác định bởi a1 = 2, an+1 = 4an + 15an2 − 60
Æ
a +8
với mọi n ≥ 1. Chứng minh rằng 2n+1 5 có thể biểu diễn thành tổng bình
phương của ba số nguyên liên tiếp.

Bài 10. (VMO 2011) Cho dãy số nguyên (an ) xác định bởi a0 = 1, a1 = −1

an = 6an−1 + 5an−2 , n ≥ 2.
Chứng minh rằng a2012 − 2010 chia hết cho 2011.

Bài 11. Cho dãy số (an ) được xác định bởi a1 = 0, a2 = 14, a3 = −18 và

an+1 = 7an−1 − 6an−2 , n ≥ 3.

Chứng minh rằng với mọi số nguyên tố p ta luôn có a p chia hết cho p.

Bài 12. Cho dãy số (un ) được xác định bởi u1 = 2 và

un = 3un−1 + 2n3 − 9n2 + 9n − 3 , n = 2, 3, 4, . . .


p−1
X
Chứng minh rằng với mọi số nguyên tố p thì 2000 ui chia hết cho p.
i=1

6. Hữu tỉ và vô tỉ
2+x n
Bài 19. Cho dãy số (x n ) được xác định bởi x 1 = 2, x n+1 = 1−2x n . Chứng
minh rằng:

a) x n 6= 0 , ∀n.

b) Dãy (x n ) không tuần hoàn.

Lời giải.
Đặt 2 = tan α 0 < α < π2 . Bằng quy nạp dễ dàng chứng minh được x n =


tan(nα).
a) Giả sử có n mà x n = 0. Viết n dưới dạng duy nhất n = 2m (2k + 1) với
m, k là các số tự nhiên.
2 tan 2n α
Nếu m > 0 thì 0 = tan(nα) = 1−tan2 2n α
⇒ tan 2n α = 0.

Hướng tới kỷ niệm 20 năm thành lập trường PTNK, ĐHQG TP HCM
Chuyên đề Toán học số 10 57

Tiếp tục như thế sẽ dẫn đến x 2k+1 = 0 ⇒ x 2k = −2 ⇒ tan(2kα) = −2. Từ


Chuyên đề Toán học Phổ thông năng khiếu TP.HCM số 10

đó
2 tan(kα) 2x k
= −2 ⇒ = −2,
1 − tan (kα)
2 1 − x k2

suy ra x k là số vô tỉ (vô lí).


b) Suy trực tiếp từ câu a).

Bài 20. Cho dãy số dương (an ) thỏa mãn an+1


2
= an +1 (n ≥ 1). Chứng minh
rằng dãy số chứa số hạng vô tỉ.

Lời giải.
bn
Giả sử trái lại rằng mọi số hạng của dãy là số hữu tỉ. Đặt an = cn với
gcd(bn , cn ) = 1.
2
Từ giả thiết an+1 = an + 1 ta có

2 2
bn+1 bn bn+1 bn + cn
2
= +1⇔ 2 = .
cn+1 cn cn+1 cn

Do gcd(bn , cn ) = 1 nên gcd(bn + cn , cn ) = 1. Từ đó ta được hai dãy số


nguyên (bn ) và (cn ) thỏa mãn
2
bn+1 = bn + cn , cn+1
2
= cn , n = 1, 2, . . .

Suy ra

bn = bn+1
2
− cn = bn+1
2 2
− cn+1 = (bn+1 − cn+1 )(bn+1 + cn+1 ).

Chú ý rằng bn+1 > cn+1 ⇒ bn+1 − cn+1 ≥ 1. Từ đó bn > bn+1 + cn+1 > bn+1
với mọi n. Như thế dãy (bn ) là dãy giảm các số nguyên dương. Điều này
vô lí.

Bài 21. (Việt Nam TST 2006) Cho dãy số (an ), n = 0, 1, 2, . . . xác định bởi
a0 = 1 và
1 1
 ‹
an+1 = an + , n = 0, 1, 2, . . .
2 3an
3
Chứng minh rằng với mỗi số nguyên dương n số An = 3an2 −1 là một số chính
phương và có ít nhất n ước nguyên tố phân biệt.

Lời giải.
bn
Đặt an = cn với bn , cn là những số nguyên dương và gcd(bn , cn ) = 1.

Hướng tới kỷ niệm 20 năm thành lập trường PTNK, ĐHQG TP HCM
58 Chuyên đề Toán học số 10

Từ hệ thức đầu bài ta có


Chuyên đề Toán học Phổ thông năng khiếu TP.HCM số 10

bn+1 3bn2 + cn2


= .
cn+1 6bn cn
7
Lại có a2 = 23 , a3 = 12 17
, a4 = 168 . Bằng qui nạp đơn giản ta thấy rằng với
mọi n ≥ 1 thì bn và cn luôn khác tính chẵn lẻ.
Gọi p là ước nguyên tố chung của 3bn2 + cn2 và 6bn cn . Nếu p là ước của
bn hoặc cn thì p là ước chung của bn và cn . Điều này mâu thuẫn với
gcd(bn , cn ) = 1.
Do đó nếu p là ước nguyên tố chung của 3bn2 + cn2 và 6bn cn thì p = 3. Ta sẽ
3bn2 +cn2
chứng minh rằng phân số 6bn cn luôn được rút gọn cho 3.

Thật vậy, các số c1 = 3, c2 = 12, c3 = 168 đều chia hết cho 3 và không chia
hết cho 9. Giả sử cn = 3c chia hết cho 3 và (c, 3) = 1. Khi đó

3bn2 + cn2 3bn2 + 9c 2 bn2 + 3c 2


= = .
6bn cn 18bn c 6bn c

Chú ý rằng do gcd(bn , cn ) = 1 nên bn không chia hết cho 3. Như thế phân
số trên tối giản và do đó bn+1 = bn2 + 3c, cn+1 = 6bn c. Nghĩa là cn+1 cũng
chia hết cho 3 mà không chia hết cho 9.
Tóm lại, các dãy số (bn ) và (cn ) được xác định như sau

3bn2 + cn2
b1 = 2, c1 = 3, bn+1 = , cn+1 = 2bn cn .
3
Bằng quy nạp ta sẽ chứng minh rằng 3bn2 − cn2 = 3 với mọi n = 1, 2, 3, . . .
Thật vậy, hiển nhiên khẳng định đúng với n = 1. Ta có
2
3bn2 + cn2 9bn4 − 6bn2 cn2 + cn4 (3bn2 − cn2 )
 
2
3bn+1 2
−cn+1 = 3. −4bn2 cn2 = = = 3,
3 3 3

theo giả thiết qui nạp.


Bây giờ
3 3 3cn2 3cn2
An = 2 = = = = cn2 ,
3an − 1 3 · b2n2 − 1 3bn2 − cn2 3
c n

là số chính phương.
Mặt khác do cn = 2bn−1 cn−1 và gcd(bn−1 , cn−1 ) = 1 nên dễ thấy rằng cn có
ít nhất n ước số nguyên tố phân biệt.

Hướng tới kỷ niệm 20 năm thành lập trường PTNK, ĐHQG TP HCM
Chuyên đề Toán học số 10 59

Bài 22. Cho các số thực α, β. Xét dãy số (an ) xác định bởi an = αn − β n với
n ∈ N. Chứng minh rằng nếu an ∈ N∗ với mọi n thì α, β là các số nguyên.
Chuyên đề Toán học Phổ thông năng khiếu TP.HCM số 10

Lời giải.
Ta có
a1 = α − β a1 = α − β
 

a2 = α − β 2
a2 = a1 (α + β)
2

€ Š € Š
a a
Từ đó ta có α = 12 a1 + a21 ∈ Q và β = 12 −a1 + a21 ∈ Q.
Đặt α = kl , β = mn với gcd(k, l) = gcd(m, n) = 1. Khi đó a1 = kl − mn = kn−ml
nl .
Suy ra kn − ml = a1 nl.
Từ đẳng thức trên ta có n | ml , suy ra n | l . Tương tự l | n . Vậy n = l và ta
có α = nk , β = mn . Phần còn lại là chứng minh n = 1.

Giả sử n 6= 1. Khi đó có số nguyên tố p là ước của n. Hiển nhiên p không


phải là ước của k và m. Với mỗi i ∈ N ta có ai = αi − β i ∈ N∗ , hay

k i − mi ∗

i i
∈ N ⇒ n k − mi
ni

Từ đó suy ra điều phải chứng minh.

Bài 23. Cho dãy số (bn ) thỏa mãn hệ thức bn+1 = bn2 − 6. Tìm tất cả b1 ∈ Q
sao cho dãy số đã cho tuần hòan.

Lời giải.
Xét hiệu bn+1 − bn = (bn − 3)(bn + 2). Suy ra rằng nếu |b1 | > 3 thì |b1 | <
b2 < · · · < bn < bn+1 và yêu cầu bài toán không được thỏa mãn.
pn
Xét |b1 | ≤ 3. Viết bn = qn với pn , qn ∈ Z, qn ≥ 1 và gcd(pn , qn ) = 1.

Ta có từ giả thiết
pn+1 pn2 pn2 − 6qn
= 2 −6= .
qn+1 qn qn2

Dễ thấy rằng từ gcd(pn , qn ) = 1 suy ra gcd(pn2 − 6qn , qn2 ) = 1. Kết hợp với
gcd(pn+1 , qn+1 ) = 1 ta được pn+1 = pn2 − 6qn , qn+1 = qn2 .
n
Do đó qn = q12 . Như thế dãy số đã cho tuần hòan nếu như có m 6= n sao
cho
bm = bn ⇒ qm = qn ⇒ q1 = 1 ⇒ b1 ∈ Z.
Từ đó b1 ∈ {±1, ±2, ±3} .

Hướng tới kỷ niệm 20 năm thành lập trường PTNK, ĐHQG TP HCM
60 Chuyên đề Toán học số 10

Bài tập tương tự


Chuyên đề Toán học Phổ thông năng khiếu TP.HCM số 10

a
Bài 13. Cho dãy số (an ) được định nghĩa bởi a1 = 1 và an+1 = 2n + 4a1n với
Ç
n = 1, 2, 3, . . . Chứng minh rằng 2a22−1 là số nguyên dương với mọi n > 1.
n

an 1
Bài 14. Cho dãy số (an ) được xác định bởi a0 = 1, an+1 = 2 + an . Chứng
minh rằng với mọi n > 1 thì p 22 luôn là số nguyên.
an −2

Bài 15. Cho ba dãy số (x n ), ( yn ), (zn ) được xác định như sau:

6 2x n 2 yn 2zn
x 1 = 2, y1 = 4, z1 = , x n+1 = 2 , yn+1 = 2 , zn+1 = 2 .
7 xn − 1 yn − 1 zn − 1

a) Chứng minh rằng các dãy số trên được xác định.

b) Tồn tại hay không n sao cho x n + yn + zn = 0.

Hướng tới kỷ niệm 20 năm thành lập trường PTNK, ĐHQG TP HCM
Chuyên đề Toán học Phổ thông năng khiếu TP.HCM số 10

Nhìn lại đề thi


vào lớp 10 chuyên Toán qua các năm

Nguyễn Tăng Vũ
(Giáo viên trường PTNK Thành phố Hồ Chí Minh)

Trường Phổ thông Năng khiếu chính thức được thành lập năm 1996, tiền thân là
khối chuyên toán tin thuộc Đại học Tổng hợp TPHCM. Qua 20 năm hình thành và
phát triển, bộ môn toán đã đạt nhiều kết quả tốt đẹp. Ngoài công tác giảng dạy tại
trường, khâu tuyển chọn cũng rất quan trọng để tìm ra những em có năng khiếu
toán thực sự. Và đề thi vào lớp 10 chuyên toán luôn được đón nhận một cách rất
hào hứng từ giáo viên và học sinh. Nay nhân dịp 20 năm thành lập trường, chúng
ta cùng nhìn lại một số đề thi, những bài toán dã là mục tiêu phấn đấu của nhiều
học sinh trong suốt quá trình học THCS.

Những năm đầu thành lập, đề chuyên toán được sử dụng cho tuyển sinh đầu vào
lớp chuyên Toán và chuyên Tin, ngoài ra để thi vào chuyên Toán thì học sinh phải
làm một đề thi chung cho các bạn thi các môn KHTN, được gọi là đề toán AB.
Trong vài năm gần đây thì đề chuyên Toán còn dùng để tuyển sinh đầu vào cho
các lớp chuyên Tin, chuyên Lý và chuyên Sinh và thay vì các đề toán AB, toán CD
chỉ còn lại một đề toán chung cho các khối lớp chuyên.

Ngoài ra, trường Phổ thông Năng khiếu còn tuyển sinh khắp miền nam chứ không
riêng gì khu vực TPHCM, các bạn nơi có điều kiện học tập tốt và các bạn nơi khó
khăn hơn đều có cơ hội đỗ vào trường như nhau, không cộng điểm ưu tiên vì bất
cứ lí do gì, điều đó cũng ảnh hưởng đến cách ra đề.
Trước tiên ta hãy xem lại đề thi vào chuyên toán của Phổ thông năng khiếu
năm 1996, năm học đầu tiên:
Bài 1. Gọi a, b là hai nghiệm của phương trình x 2 + px + 1 = 0; c, d là hai
nghiệm của phương trình y 2 + q y + 1 = 0. Chứng minh rằng

(a − c)(a − d)(b − c)(b − d) = (p − q)2

61
62 Chuyên đề Toán học số 10

Bài 2. Cho các số x, y, z thỏa x + y + z = 5, x 2 + y 2 + z 2 = 9. Chứng minh


7
Chuyên đề Toán học Phổ thông năng khiếu TP.HCM số 10

rằng 1 ≤ x, y, z ≤ .
3
Bài 3.
a) Cho tứ giác lồi ABC D. Hãy dựng đường thẳng qua A chia đôi diện tích tứ
giác ABC D.

b) Cho tam giác ABC và đường thẳng d||BC và nằm khác phía của A đối
với BC. Lấy điểm M di động trên d sao cho ABM C là tứ giác lồi. Đường
thẳng qua A chia đôi diện tích tứ giác cắt BM hoặc C M tại N . Tìm quỹ
tích điểm N .
p p
Bài 4. Chứng minh không tồn tại số tự nhiên n sao cho n − 1 + n + 1 là
số hữu tỷ.
Bài 5.
a) Chứng minh với N ≥ 3, luôn luôn có N số chính phương đôi một khác
nhau sao cho tổng của chúng là một số chính phương.

b) Chứng minh rằng với mọi số nguyên mn ≥ 3 bao giờ cũng xây được một
bảng chữ nhật gồm m × n số chính phương đôi một khác nhau cho tổng
của mỗi dòng là một số chính phương và tổng của mỗi cột là một số chính
phương.
Chưa bàn đến độ khó của đề, ta thấy rằng về cấu trúc đề thi có đầy đủ các
phần: Đại số, số học, hình học và tổ hợp. Đó cũng là cấu trúc chung của
các đề thi chuyên toán PTNK, định hình từ ngày thành lập trường đến hiện
nay. Đến đây, ta có thể tách riêng từng phần để nhận xét kĩ hơn. Trước tiên
ta xem qua phần đại số.

1. Đại số
Đại số là các bài toán liên quan đến biến đổi biểu thức, đa thức, áp dụng
định lý Viete, phương trình, hệ phương trình, các bài toán lập và giải
phương trình,...Trong đó phần bất đẳng thức, cực trị luôn chiếm một vai
trò lớn trong toán chuyên, nhiều học sinh rất giỏi đại số và dễ dàng đạt
điểm cao trong phần này. Mặc dù có độ khó ngày càng tăng theo năm
nhưng những bài toán đại số trong đề tuyển sinh PTNK vẫn không quá
mẹo mực, đòi hỏi kĩ thuật nhiều. Một số bài biến đổi biểu thức khá hay:
Bài 5 - 1999
a) Chứng minh đẳng thức x + y + |x − y| = 2 max{x, y}, ∀x, y ∈ R.

Hướng tới kỷ niệm 20 năm thành lập trường PTNK, ĐHQG TP HCM
Chuyên đề Toán học số 10 63

b) Chứng minh đẳng thức:


Chuyên đề Toán học Phổ thông năng khiếu TP.HCM số 10

a + b a − b 2 a + b a − b 2

1 1 1
§ ª
ab + ab − c + ab + ab + c = 4 max a , b , c , ∀a, b, c 6= 0

trong đó max là kí hiệu số lớn nhất trong các số đi kèm.


1 1 1
Bài 4-2002 Cho các số thực a, b, c thỏa mãn điều kiện a+ = b+ = c + .
b c a
a) Cho a = 1, tìm b, c.

b) Chứng minh rằng nếu a, b, c đôi một khác nhau thì a2 b2 c 2 = 1.

c) Chứng minh rằng nếu a, b, c đều dương thì a = b = c.


Cùng với thời gian thì phần biến đổi này cũng trở nên dễ hơn và ít được
xuất hiện trong các đề thi, cũng có thể tập trung cho các phần khác. Ta
xem một số bài phương trình, hệ phương trình:
Bài 1-2006
2x 2 + x y = 1
§
a) Giải hệ phương trình
2 y2 + x y = 1
p
b) Giải bất phương trình 3x − 5x 2 ≤ 5x − 2
Bài 1-2008
1) Cho phương trình x 2 − mx + 2m − 2 = 0 (1)

a) Chứng minh rằng (1) không thể có hai nghiệm đều âm.
b) Giả sử x 1 , x 2 là hainghiệm phân biệt
 của (1). Chứng minh rằng biểu
x 12 − 2x 1 + 2 x 22 − 2x 2 + 2
thức không phụ thuộc vào giá trị của
x 12 + x 22
m

2) Giải hệ phương trình



 x = y 2 + z2
y = z2 + x 2
 z = x2 + y2

Bài 1-2016
(x − 2 y)(x + m y) = m2 − 2m − 3
§
a) Giải hệ khi m = −3 và tìm m để
( y − 2x)( y + mx) = m2 − 2m − 3
hệ co ít nhất một nghiệm (x o , yo ) thỏa x o > 0, yo > 0.

Hướng tới kỷ niệm 20 năm thành lập trường PTNK, ĐHQG TP HCM
64 Chuyên đề Toán học số 10

b) Tìm a ≥ 1 để phương trình ax 2 + (1 − 2a)x + 1 − a = 0 có hai nghiệm


phân biệt x 1 , x 2 thỏa x 22 − ax 1 = a2 − a − 1.
Chuyên đề Toán học Phổ thông năng khiếu TP.HCM số 10

Nhìn chung các bài phương trình, hệ phương trình khác dễ lấy điểm. Tuy
có năm 2016, bài hệ phương trình có tham số nên trở thành một bài toán
quá phức tạp, rất ít học sinh giải được trọn vẹn bài toán này.
Tiếp theo ta cùng nhìn qua một số bài bất đẳng thức. Bất đẳng thức trong
kì thi đầu vào của PTNK chỉ là các bài toán biến đổi tương đương hoặc chỉ
áp dụng bất đẳng thức Cauchy cho 2 số. Có thể đây là mục tiêu của ban
ra đề nhằm tránh cho các bạn quá sa đà vào các kĩ thuật chứng minh bđt
mà bỏ quên các phần khác.
Bài 4-1998 Cho x, y, z, p, q, r là các số thực dương thỏa mãn điều kiện
1
x + y + z = p + q + r = 1 và pqr ≤ .
2
x+y
a) Chứng minh rằng nếu x ≤ y ≤ z thì px + q y + rz ≥
2
b) Chứng minh rằng px + q y + rz ≥ 8x yz

Bài 5-2000

a) Cho ab, c là 3 số không âm thỏa điều kiện a2 + b2 + c 2 ≤ 2(ab + bc + ac)


(1) p p p
Chứng minh rằng a + b + c ≤ 2( ab + bc + ac) (2)
Hỏi từ (2) có thể suy ra (1) được không? Vì sao?

b) Cho a, b, c là 3 số không âm thỏa điều kiện (1) và p, q, r là các số thỏa


điều kiện p + q + r = 0. Chứng minh apq + bqr + cr p ≤ 0.

Bài 3-2016 Biết x ≥ y ≥ z, x + y + z = 0 và x 2 + y 2 + z 2 = 6.

a) Tính S = (x − y)2 + (x − y)( y − z) + ( y − z)2 .

b) Tìm giá trị lớn nhất của P = |(x − y)( y − z)(z − x)|.

2. Hình học
Hình học là một trong những bài toán hay nhất của đề thi PTNK, trong
những năm học vừa qua PTNK luôn có những học sinh rất giỏi hình học
như Hồ Quốc Đăng Hưng, Nguyễn Huy Hoàng, cũng như tinh thần chung
của đề thi PTNK, hình học cũng là những bài toán có thể là quen thuộc

Hướng tới kỷ niệm 20 năm thành lập trường PTNK, ĐHQG TP HCM
Chuyên đề Toán học số 10 65

và được phát biểu với dạng khác. Ta cùng xem qua một số bài toán mà tôi
Chuyên đề Toán học Phổ thông năng khiếu TP.HCM số 10

thấy khá hay:


Bài 3-1999 Cho tam giác ABC có diện tích S và một điểm P nằm trong tam
giác.

a) Gọi S1 , S2 , S3 lần lượt là diện tích của tam giác P BC, P CA, PAB. Hãy tìm
giá trị nhỏ nhất của S12 + S22 + S32 .

b) Gọi P1 , P2 , P3 lần lượt là các điểm đối xứng của P qua BC, CA và AB.
Đường thẳng qua P1 song song với BC cắt AB và AC tại B1 và C1 . Đường
thẳng qua P2 song song với AC cắt BC, BA tại C2 , A2 , đường thẳng qua P3
và song song với AB cắt CA, C B tại A3 , B3 . Hãy xác định vị trí của điểm P
để tổng diện tích ba hình thang BC C1 B1 , CAA2 C2 và ABB3 A3 đạt giá trị
nhỏ nhất và tính giá trị đó.

Bài 3-2006 Cho tam giác đều ABC. P là một điểm nằm trong tam giác. Gọi
x, y, z lần lượt là kkhoảng cách từ P đến các cạnh BC, CA, AB tương ứng.

a) Biết rằng x = 1, y = 2, z = 3. Hãy tính diện tích của tam giác ABC.

b) Tìm quỹ tích điểm P trong tam giác sao cho x + y = z. Từ đó suy ra tập
hợp những điểm P trong tam giác sao cho x, y, z lập thành 3 cạnh của
một tam giác.

Bài 4-2010
p Cho đường tròn tâm O, bán kính R, dây cung BC cố định có độ
dài R 3. A là một điểm thay đổi trên cung lớn BC. Gọi E là điểm đối xứng
của C qua AB; F là điểm đối xứng của B qua AC. Các đường tròn ngoại tiếp
các tam giác ABE và AC F cắt nhau tại K (K 6= A).

a) Chứng minh K luôn thuộc một đường tròn cố định.

b) Xác định vị trí của K để tam giác K BC có diện tích lớn nhất và tính diện
tích đó theo R.

c) Gọi H là giao điểm của BE và C F . Chứng minh rằng tam giác ABH đổng
dạng với tam giác AC K và AK đi qua điểm cố định.

Bài 4Cho điểm C thay đổi trên nửa đường tròn đường kính AB = 2R (C 6=
A, C 6= B). Gọi H là hình chiếu vuông góc của C lên AB; I và J lần lượt là
tâm đường tròn nội tiếp các tam giác AC H và BC H. Các đường thẳng C I, C J
cắt AB tại M , N .

a) Chứng minh AN = AC, BM = BC.

Hướng tới kỷ niệm 20 năm thành lập trường PTNK, ĐHQG TP HCM
66 Chuyên đề Toán học số 10

b) Chứng minh 4 điểm M , N , I, J cùng nằm trên một đường tròn và các
Chuyên đề Toán học Phổ thông năng khiếu TP.HCM số 10

đường thẳng M J, N I và C H đồng quy.

c) Tìm giá trị lớn nhất của MN và giá trị lớn nhất của diện tích tam giác
CMN theo R.

3. Số học
Một trong những điểm khác biệt của đề thi vào PTNK là bài toán số học.
Bài toán số học luôn chiếm một vị trí trong đề thi và ngày càng được chú
trọng, do đó độ khó tăng rõ rệt. Những năm đầu tiên, do dùng chung với
đề tuyển sinh chuyên tin nên bài toán số học đôi khi được phát biểu dưới
dạng mệnh đề đúng, mệnh đề sai:
Bài 2-1998 Cho hai số nguyên dương a và b. Biết rằng trong bốn mệnh đề
P, Q, R, S dưới đây chỉ có duy nhất một mệnh đề sai: P = "a = 2b + 5"
Q = "(a + 1) chia hết cho b"
R = "(a + b) chia hết cho 3"
S = "(a + 7b) là số nguyên tố"

a) Hãy chỉ ra mệnh đề nào sai trong bốn mệnh đề trên (có giải thích).

b) Hãy tìm tất cả các cặp số nguyên dương a, b thỏa ba mệnh đề đúng còn
lại.

Bài toán số học chung quy cũng liên quan đến chia hết, số nguyên tố,
phương trình nghiệm nguyên... nhưng được phát biểu một cách nhẹ nhàng
và có những ý khá đơn giản cho các em học sinh có thể làm được. Và ngày
được chú trọng nên độ khó cũng ngày được nâng lên.
Bài 3-2010

a) Hãy chỉ ra một bộ 4 số nguyên dương phân biệt mà tổng bộ ba số bất kỳ


trong chúng là một số nguyên tố.

b) Chứng minh rằng không tồn tại 5 số nguyên phân biệt mà tổng 3 số bầt
kỳ trong chúng là một số nguyên tố.

Câu b cũng chỉ là dạng phát biểu mới của bài toán: Chứng minh rằng trong
5 số nguyên dương phân biệt bất kì luôn có 3 số có tổng chia hết cho 3. Và
đây cũng là một bài toán dễ. Tuy nhiên, vài năm gần đây độ khó tăng lên,
ta xem bài toán số học của 3 năm gần đây.
1 1 1
Bài 3-2014 Cho các số nguyên dương a, b thỏa + = .
a b c

Hướng tới kỷ niệm 20 năm thành lập trường PTNK, ĐHQG TP HCM
Chuyên đề Toán học số 10 67

a) Chứng minh rằng a + b không thể là số nguyên tố.


Chuyên đề Toán học Phổ thông năng khiếu TP.HCM số 10

b) Chứng minh rằng nếu c > 1 thì a + c và b + c không thể đồng thời là số
nguyên tố.

Bài 2-2015

a) Tìm các số nguyên a, b, c sao cho a + b + c = 0 và ab + bc + ac + 3 = 0.

b) Cho m là số nguyên. Chứng minh rằng nếu tồn tại các số nguyên a, b, c
khác 0 sao cho a + b + c = 0 và ab + bc + ac +4m = 0 thì cũng tồn tại các
số nguyên a0 , b0 , c 0 sao cho a0 + b0 + c 0 = 0 và a0 b0 + b0 c 0 + a0 c 0 + m = 0.

c) Với k là số nguyên dương, chứng minh rằng không tồn tại các số nguyên
a, b, c khác 0 sao cho a + b + c = 0 và ab + bc + ac + 2k = 0.

Bài 2-2016 Cho x, y là hai số nguyên dương mà x 2 + y 2 + 10 chia hết cho


x y.

a) Chứng minh rằng x, y là hai số lẻ và nguyên tố cùng nhau.


x 2 + y 2 + 10
b) Chứng minh k = chia hết cho 4 và k ≥ 12.
xy
Tôi rất thích các bài toán số học này, từ cách phát biểu đến kiến thức cần
sử dụng. Tuy khó hơn khá nhiều so với những năm đầu, tuy nhiên các bài
toán này vẫn có ý để cho học sinh làm và phân loại tốt.

4. Tổ hợp
Tổ hợp có lẽ là phần quan trọng nhất trong đề tuyển sinh vào PTNK, các
thầy ra đề luôn chú trọng tới phần này. Tổ hợp là dạng bài tập khó dùng
để phân loại học sinh có năng khiếu. Nhìn chung các bài toán liên quan
đến các phương pháp chứng minh:
Phương pháp phản chứng
Bài 5- 2006 Cho 13 số thực thỏa mãn điều kiện: tổng của 6 số bất kỳ trong
chúng nhỏ hơn tổng của 7 số còn lại. Chứng minh rằng tất cả các số đã cho
đều dương.
Quy nạp
Các năm đầu, phương pháp quy nạp được sử dụng nhiều trong các bài
toán tổ hợp. Và các bài toán tổ hợp cũng rất khó, tôi luôn nhớ bài toán tổ
hợp năm tôi thi vào PTNK:

Hướng tới kỷ niệm 20 năm thành lập trường PTNK, ĐHQG TP HCM
68 Chuyên đề Toán học số 10

Bài 4-1999 Người ta lát nền nhà hình vuông kích thước n × n ô bằng các
Chuyên đề Toán học Phổ thông năng khiếu TP.HCM số 10

viên gạch như hình vẽ dưới sao cho còn chừa lại một ô không lát.

a) Hãy chỉ ra một cách lát như trên với nền nhà kích thước 4 × 4 và 8 × 8.

b) Hãy chứng minh rằng luôn tồn tại một cách lát nền nhà có kích thước
2k × 2k (k nguyên dương) với ô trống còn lại nằm ở vị trí (i, j) bất kì.

Bài 5-1998

a) Hãy chỉ ra cách sắp 8 số nguyên dương đầu tiên 1, 2, . . . , 8 thành một
dãy a1 , a2 , . . . , a8 sao cho 2 số ai , a j bất kì (i < j) thì mọi số trong dãy
ai + a j
nằm giữa ai và a j đều khác .
2
b) Chứng minh rằng với N số nguyên dương đầu tiên 1, 2, . . . , N luôn tìm
được cách sắp thành dãy a1 , a2 , . . . , aN sao cho dãy thỏa mãn điều kiện
như câu a).

Cùng với quy nạp, bài toán bất biến cũng xuất hiện nhiều trong các bài
toán tổ hợp:
Bài 5-2003

a) Cho một bảng vuông 4×4. Trên các ô của hình vuông này, ban đầu người
ta ghi 9 số 1 và 7 số 0 một cách tùy ý (mỗi ô một số). Với mỗi phép biến
đổi bảng, cho phép một hàng hoặc một cột bất kỳ trên hàng hoặc cột được
chọn đổi đồng thời các số 0 thành số 1, các số 1 thành số 0. Chứng minh
rằng sau một số hữu hạn các phép biến đổi như vậy, ta không thể đưa
bảng ban đầu về bảng toàn các số 0.

b) Ở vương quốc "Sắc màu kỳ ảo" có 45 hiệp sĩ: 13 hiệp sĩ tóc đỏ, 15 hiệp
sĩ tóc vàng và 17 hiệp sĩ tóc xanh. Khi hai hiệp sĩ có màu tóc khác nhau
gặp nhau thì tóc của họ lập tức đổi sang màu tóc thứ ba (ví dụ khi hiệp
sĩ tóc đỏ gặp hiệp sĩ tóc vàng thì cả hai đổi sang tóc xanh). Hỏi có thể xảy
ra trường hợp sau một số hữu hạn lần gặp nhau như vậy ở "Sắc màu kỳ
ảo" tất cả các hiệp sĩ đều có cùng màu tóc không? Vì sao?

Và nguyên lí Dirichlet xuất hiện cũng khá nhiều, một số bài có sử dụng
nguyên lí này:
Bài 5-2005 Xét 81 chữ số trongg đó có 9 chữ số , 9 chữ số 2, ..., 9 chữ số 9.
Hỏi có thể xếp 81 chữ số này thành một dãy số cho với mỗi k = 1, 2, ..., 9 thì
giữa hai chữ số k liên tiếp có đúng k chữ số.
Bài 5-2011 Cho hình chữ nhật ABC D có AB = 3, AD = 4.

Hướng tới kỷ niệm 20 năm thành lập trường PTNK, ĐHQG TP HCM
Chuyên đề Toán học số 10 69

a) Chứng minh rằng từ 7 điểm bất kì trong hình chữ nhật ABC
p D luôn tìm
Chuyên đề Toán học Phổ thông năng khiếu TP.HCM số 10

được hai điểm mà khoảng cách giữa chúng không lớn hơn 5

b) Chứng minh khẳng định ở câu a) vẫn còn đúng với 6 điểm bất kì nằm
trong hình chữ nhật ABC D.

Bài 5-2015 Để khuyến khích phong trào học tập, một trường THCS đã tổ
chứng 8 đợt thi cho các học sinh. Ở mỗi đợt thi, có đúng 3 học sinh được
chọn để trao giải. Sau khi tổ chứng xong 8 đợt thi, người ta nhận thấy rằng
với hai đợt thi bất kì thì có đúng 1 học sinh được trao giải ở cả hai đợt thi
đó. Chứng minh rằng:

a) Có ít nhất một học sinh được trao giải ít nhất bốn lần.

b) Có đúng một học sinh được trao giải ở 8 đợt thi.

Ngoài các phương pháp chứng minh trên, các bài toán tổ hợp cũng rất
phong phú về nội dung, đôi khi mang màu sắc tin học như bài năm 1999
và một số bài sau:
Bài 5-1996

a) Chứng minh với N ≥ 3, luôn luôn có N số chính phương đôi một khác
nhau sao cho tổng của chúng là một số chính phương.

b) Chứng minh rằng với mọi số nguyên mn ≥ 3 bao giờ cũng xây được một
bảng chữ nhật gồm m × n số chính phương đôi một khác nhau cho tổng
của mỗi dòng là một số chính phương và tổng của mỗi cột là một số chính
phương.

Bài 5-2012 Cho đa giác đều n cạnh . Dùng 3 màu xanh , đỏ, vàng tô màu
các đỉnh đa giác một cách tùy ý ( mỗi đỉnh được tô bởi một màu và tất cả
các đỉnh đều được tô màu). Cho phép thực hiện thao tác sau đây : chọn hai
đỉnh kề nhau bất kì ( nghĩa là hai đỉnh liên tiếp) khác màu và thay màu của
hai đỉnh đó bằng màu còn lại.

a) Chứng minh rằng bằng cách thực hiện thao tác trên một số lần ta luôn
luôn làm cho các đỉnh của đa giác chỉ còn được tô bởi hai màu.

b) Chứng minh rằng với n = 4 và n = 8, bằng cách thực hiện thao tác trên
một số lần ta có thể làm cho các đỉnh của đa giác chỉ còn được tô bởi một
màu.

Hướng tới kỷ niệm 20 năm thành lập trường PTNK, ĐHQG TP HCM
70 Chuyên đề Toán học số 10

Bài số 5 - 2012 có lẽ là bài toán tổ hợp khó nhất trong những năm gần
Chuyên đề Toán học Phổ thông năng khiếu TP.HCM số 10

đây, không có học sinh nào làm được trọn vẹn bài nay. Tôi nhớ rằng năm
đó điểm cao nhất là 7.75, một năm mà đề thi rất khó. Tổ hợp luôn là bài
toán được quan tâm từ lúc ra đề đến khi vào học, có thể đó là một trong
những lí do học sinh PTNK khá mạnh phần này, điển hình là Phạm Tuấn
Huy - 2 HCV IMO liên tiếp vào các năm 2013, 2014.

5. Các bài toán bóng đá


Có lẽ đây là một đề tài rất thú vị của đề thi vào PTNK. Các bài toán bóng
đá xuất hiện thường xuyên và cách năm, cứ mỗi năm xảy ra Worldcup hay
Euro thì sẽ có một bài liên quan đến bóng đá, mặc dù chỉ bó hẹp trong
phạm vi bóng đá, tuy nhiên đề bài lại rất phong phú đa dạng, đòi hỏi suy
luận chặc chẽ và cẩn thận. Một số bài liên quan đến lập và giải phương
trình:
Bài 5-2002 Trong một giải bóng đá có N đội tham gia thi đấu theo thể thức
vòng tròn một lượt (hai đội bất kỳ đều gặp nhau đúng một lần). Sau mỗi
trận đấu, thắng được 3 điểm, đội thua không được điểm nào, còn nếu trận
đấu có kết quả hòa thì mỗi đội được 1 điểm. Các đội xếp hạng dựa theo tổng
điểm. Trong trường hợp một số đội có tổng điểm bằng nhau thì các đội này
được xếp hạng theo chỉ số phụ. Kết hức giải người ta nhận thấy rằng không
có trận đấu nào kết thúc với tỉ số hòa; các đội xếp nhất, nhì, ba có tổng điểm
lần lượt là 15, 12, 12 và tất cả các đội xếp nhau có tổng điểm đôi một khác
nhau.

a) Chứng minh rằng N ≥ 7.

b) Tìm N và tổng điểm của mỗi đội tham gia giải.

Bài 5-2008 Trong một giải vô địch bóng đá có 6 đội tham gia. Theo điều
lệ của giải, hai đội bất kỳ thi đấu với nhau đúng một trận, đội thắng được
3 điểm, đội hòa được 1 điểm và đội thua 0 điểm. Kết thúc giải, số điểm 
của các đội lần lượt là D1 , D2 , D3 , D4 , D5 , D6 D1 ≥ D2 ≥ D3 ≥ D4 ≥ D5 ≥ D6 .
Biết rằng đội bóng với số điểm D1 thua đúng một trận và D1 = D2 + D3 =
D4 + D5 + D6 . Hãy tìm D1 và D6 . Đây là hai bài toán khó nhất về đề tài bóng
đá, trong đó bài năm 2008 là rất khó đòi hỏi nhiều suy luận và xét các
trường hợp một cách cẩn thận. Một số bài toán khác không liên quan đến
điểm thì liên quan đồ thị:
Bài 5-2010 Trong một giải bóng đá có 12 đấu vòng tròn một lượt (2 đội bất
kỳ đấu với nhau một trận).

Hướng tới kỷ niệm 20 năm thành lập trường PTNK, ĐHQG TP HCM
Chuyên đề Toán học số 10 71

a) Chứng minh rằng sau 4 vòng đấu (mỗi đội đấu 4 trận) luôn tìm được 3
Chuyên đề Toán học Phổ thông năng khiếu TP.HCM số 10

đội bóng đôi một chưa đấu với nhau.

b) Khẳng định còn đúng không khi mỗi đội đã thi đấu đúng 5 trận.

Đây là bài toán về bóng đá cuối cùng xuất hiện trong các đề thi PTNK, có
lẽ đem lại một chút tiếc nuối cho các học sinh, tuy nhiên vì bóng đá luôn
là đề tài hấp dẫn, biết đâu nó sẽ trở lại vào một ngày không xa.
Lời kết Trên đây tôi đã ngược dòng lịch sử, điểm lại một số đặc điểm về
đề thi tuyển sinh vào PTNK trong 20 năm qua để các bạn có một cái nhìn
bao quát nhất, và từ đó ta cũng thấy được cách tuyển chọn học sinh năng
khiếu toán của trường. Một trong những lí do mang lại thành công cho học
sinh và cựu học sinh của trường trên con đường học thuật sau này.

Hướng tới kỷ niệm 20 năm thành lập trường PTNK, ĐHQG TP HCM
72 Chuyên đề Toán học số 10
Chuyên đề Toán học Phổ thông năng khiếu TP.HCM số 10

Hướng tới kỷ niệm 20 năm thành lập trường PTNK, ĐHQG TP HCM
Chuyên đề Toán học Phổ thông năng khiếu TP.HCM số 10

Một số bài toán chọn lọc về


phép biến hình

Phạm Hy Hiếu 1
(Lớp Chuyên Toán khóa 2007 − 2010)
Lời Ban biên tập: Phép biến hình là một công cụ mạnh, hỗ trợ đắc lực cho các
bài toán hình học. Nhờ có nó, nhiều bước lập luận, chứng minh có thể được giảm
bớt và ý tưởng rõ ràng, mạch lạc hơn. Chúng ta xin giới thiệu một số ứng dụng
của công cụ này thông qua các bài toán cụ thể, trong đó chúng ta có thể thấy
được phép tịnh tiến, đối xứng trục - tâm, vị tự, nghịch đảo, phép quay, ...
Bài 1. Cho tam giác ABC có AA0 , BB 0 , C C 0 là các đường cao. Chứng minh
rằng đường thẳng Euler của các tam giác AB 0 C 0 , BC 0 A0 , CA0 B 0 đồng qui.
Lời giải.

Ha
Oa
B0
S

C0
I O
H Hc

Ob
Oc

Hb
C
B A0

1
Trích từ một bài viết "Geometry Note" của chính tác giả

73
74 Chuyên đề Toán học số 10

Gọi Oa , Ob , Oc là tâm đường tròn ngoại tiếp các tam giác AB 0 C 0 , BC 0 A0 , CA0 B 0
Chuyên đề Toán học Phổ thông năng khiếu TP.HCM số 10

còn H a , H b , H c là trực tâm của các tam giác ấy theo thứ tự.
Xét phép vị tự đối xứng Sa có trục đối xứng là phân giác l a của góc ∠BAC,
0
tâm vị tự A và tỉ số k = AB
AB = cos A. Ta có

Sa : B 7→ B 0 , C 7→ C 0 , O 7→ Oa , H 7→ H a ,

nên Oa H a và OH đối song với nhau trong góc ∠BAC.


Gọi I là tâm đường tròn nội tiếp tam giác ABC, ta biết rằng tích của hai
phép đối xứng trục IA, I B là phép quay tâm I, góc quay 2 (IA, I B) (mod π)
nên ta có
(Oa H a , Ob H b ) ≡ 2 (IA, I B) ≡ 2 ((IA, AB) + (AB, I B))
≡ (CA, AB) + (AB, BC) ≡ (CA, C B) (mod π).

Mặt khác, Oc Oa và Oc Ob là đường trung bình của tam giác HAC và tam
giác HAB nên

(Oc Oa , Oc Ob ) ≡ (CA, C B) ≡ (Oa H a , Ob H b ) (mod π).

Suy ra giao điểm Sc của Oa H a với Ob H b nằm trên đường tròn (Oa Ob Oc ),
tức là đường tròn Euler của tam giác ABC.
Lý luận tương tự thì ta có Sa ≡ S b ≡ Sc ≡ S ∈ (Σ), hay Oa H a , Ob H b , Oc H c
đồng qui trên đường tròn (Σ), trong đó ( Σ ) là đường tròn Euler của tam
giác ABC.
Bài 2. Cho tam giác ABC nội tiếp trong đường tròn (O). S1 , S2 là hai điểm
di động trên đường tròn (O) và đối xứng nhau của O. Giả sử ∆1 , ∆2 là các
đường thẳng Simson ứng với S1 , S2 . Chứng minh rằng ∆1 ⊥∆2 cũng như giao
điểm của ∆1 và ∆2 luôn thuộc một đường tròn cố định.
Lời giải.
Gọi Mai , M bi , Mci (i = 1, 2) là hình chiếu của Si tương lên BC, CA, AB.
Ta có
(∆1 , ∆2 ) ≡ (∆1 , AB) + (AB, ∆2 ) ≡ (S1 Ma1 , S1 B) + (S2 A, S2 M b2 )
π π
≡ (BC, S1 B) + + (S2 A, AC) + ≡ (AC, AS1 ) + (S2 A, AC)
2 2
π
≡ (S2 A, AS1 ) ≡ (mod π).
2
Suy ra ∆1 ⊥∆2 tại S. Gọi H là trực tâm của tam giác ABC và (E) là đường
tròn Euler của tam giác ABC. Xét phép vị tự H có tâm H, tỉ số k = 2, ta có
0 0
H : ∆1 7→ ∆1 , ∆2 7→ ∆2 , S 7→ S 0 , E 7→ O, (E) 7→ (O) .

Hướng tới kỷ niệm 20 năm thành lập trường PTNK, ĐHQG TP HCM
Chuyên đề Toán học số 10 75

Mc2
Chuyên đề Toán học Phổ thông năng khiếu TP.HCM số 10

S2
A

M b2

O
E
H
M b1

S
Ma1

B C
Ma2

Mc1

S1

0 0
Thế thì theo tính chất của đường thẳng Steiner, S1 ∈ ∆1 và S2 ∈ ∆2 . Mặt
0 0 0 0
khác do S1 S2 là đường kính của (O) và ∆1 ⊥∆2 nên S 0 ≡ ∆1 ∩ ∆2 ∈ (O),
mà H : (E) 7→ (O) nên S ∈ (E).
Vậy giao điểm S của ∆1 , ∆2 thuộc đường tròn Euler của tam giác ABC cố
định.

Bài 3. Cho tam giác ABC nội tiếp trong đường tròn (O). Gọi A1 ∈ BC là
chân đường đối trung xuất phát từ A của tam giác ABC. Ba ∈ AC và Ca ∈ AB
sao cho A1 Ba ACa là hình bình hành.

a) Chứng minh rằng B, C, Ba , Ca đồng viên.

b) Gọi Oa là tâm đường tròn (BC Ba Ca ). Chứng minh rằng AOa đi qua
trung điểm của OL với L là điểm Lemoine của tam giác ABC.

Lời giải.
a) Ta có BC Ba Ca là hình bình hành nên AA1 đi qua trung điểm của Ba Ca ,
suy ra B, C, Ba , Ca đồng viên.
b) Gọi I là trung điểm của OL. Xét phép vị tự
AA1
k=
HA AL
: L 7→ A1 , B 7→ B 0 , C 7→ C 0 , O 7→ O0 .

Hướng tới kỷ niệm 20 năm thành lập trường PTNK, ĐHQG TP HCM
76 Chuyên đề Toán học số 10
Chuyên đề Toán học Phổ thông năng khiếu TP.HCM số 10

Ba

Ca
L
I O

B C
A1
Oa
O0

B0 C0

Ta có A1 là điểm Lemoine của tam giác AB 0 C 0 hơn nữa


BA1 C k B 0 C 0 , A1 Ba k AB 0 , A1 Ca k AC 0
nên (BC Ba Ca ) chính là đường tròn Lemoine thứ nhất của tam giác AB 0 C 0 ,
từ đó vì Oa là tâm của đường tròn này nên Oa là trung điểm của O0 A1 , mặt
khác
H : L 7→ A1 , O 7→ O0 nên H : I 7→ Oa .
Suy ra A, I, Oa thẳng hàng.
Bài 4. Cho sáu điểm A, B, C, D, E, F cùng thuộc đường tròn (O, R) thỏa
mãn điều kiện AB = C D = E F = R. Gọi M , N , P lần lượt là trung điểm
BC, DE, FA. Chứng minh rằng tam giác M N P đều.
Lời giải.
Giả sử tam giác OAB có hướng dương. Xét phép quay vector
◦ −→ −→ −→ −→ −→ −→
Q60 : OA 7→ OB, OC 7→ OD, OE 7→ OF .

Hướng tới kỷ niệm 20 năm thành lập trường PTNK, ĐHQG TP HCM
Chuyên đề Toán học số 10 77

B
Chuyên đề Toán học Phổ thông năng khiếu TP.HCM số 10

A M

P
O

D
N

F
E

Ta có
−→ −−→ 1 −→ −→ 1 −→ −→
• ˜
−−→
Q( M N ) = Q(ON − OM ) = Q (OD + OE) − (OB + OC)
2 2
1 €−→ −→ −→ −→Š
= Q OD + OE − OB − OC
2
1 €−→ −→ − → −→Š
= C D + OF − AB − OD
2
1 ”−
→ −→ −→ −→ —
= BA + (C D + DO + OF )
2
1 −→ −→ −−→
= (BA + C F ) = M P.
2
€−−→ −−→Š
Suy ra M N , M P = 60◦ và M N = M P hay tam tam giác M N P đều.

Bài 5. Hai tứ giác ABPQ và QP C D nội tiếp cùng hướng. Biết rằng tồn tại
điểm E ∈ PQ sao cho ∠EAP = ∠EBQ và ∠ECQ = ∠E DP. Chứng minh rằng
ABC D nội tiếp.

Lời giải.
Ta đặt các giao điểm

X ≡ AE∩(ABPQ) , Y ≡ BE∩(ABPQ) , Z ≡ C E∩(QP C D) , T ≡ DE∩(QP C D) .

Hướng tới kỷ niệm 20 năm thành lập trường PTNK, ĐHQG TP HCM
78 Chuyên đề Toán học số 10
Chuyên đề Toán học Phổ thông năng khiếu TP.HCM số 10

Từ đề bài ta có ∠EAP = ∠EBQ nên X Y PQ là hình thang cân, hơn nữa


∠ECQ = ∠E DP nên Z TQP cũng là hình thang cân. Vậy phép đối xứng
qua đường trung trực của PQ biến Z 7→ T, X 7→ Y , suy ra X Y Z T cũng là
hình thang cân, tức X , Y, Z, T đồng viên.
Mặt khác xét phép nghịch đảo I có cực E, phương tích k ≡ E P · EQ, ta có

I : P 7→ Q, X 7→ A, Y 7→ B, Z 7→ C, T 7→ D.

Hơn nữa theo chứng minh trên thì X , Y, Z, T đồng viên nên A, B, C, D đồng
viên hoặc thẳng hàng. Tuy nhiên A, B, C, D không thể thẳng hàng nên
chúng phải đồng viên và ta có.

Bài 6. Tam giác ABC nội tiếp trong đường tròn (O) có BC > CA > AB, I là
tâm đường tròn nội tiếp tam giác ABC, AI ∩ (O) ≡ K, OK ∩ BC = M và N là
điểm đối xứng của I qua M , K N ∩(O) ≡ L. Chứng minh rằng LB = LA+ LC.

Lời giải.
Xét phép nghịch đảo I có cực K, phương tích k := K B 2 = K C 2 . Ta có

I : B 7→ B, C 7→ C, (O) 7→ BC, A 7→ D ≡ AI ∩ BC, L 7→ E ≡ K L ∩ BC.

Hướng tới kỷ niệm 20 năm thành lập trường PTNK, ĐHQG TP HCM
Chuyên đề Toán học số 10 79

A
Chuyên đề Toán học Phổ thông năng khiếu TP.HCM số 10

M
B C
D E

Ta có
k · EB k · ED k · EC
LB = , LC = , LA = .
KE · KB KE · KD K E · KC
Vậy nên LB = LA + LC sẽ tương đương với
k · EB k · ED k · EC
= + ,
KE · KB K E · K D K E · KC
hay là
EB E D EC EB − EC ED
= + ⇔ = .
KB K D KC KB KD
Nhưng EB − EC = 2E M nên đẳng thức trên tương đương với
ED KD BD CD BC
= = = = ,
2E M KB BA CA AB + AC
hay
ED 2a
= .
EM b+c
Ta có
EB SK BE S SK BE − SN BE
= = N BE =
EC SKC E SN C E SKC E − SN C E
SK BN K B · BN · sin ∠N BK
= =
SKC N KC · C N · sin ∠N C K
I C sin ∠N BK
= · .
I B sin ∠N C K

Hướng tới kỷ niệm 20 năm thành lập trường PTNK, ĐHQG TP HCM
80 Chuyên đề Toán học số 10

Hơn nữa
Chuyên đề Toán học Phổ thông năng khiếu TP.HCM số 10

1
∠N BK = ∠K BC − ∠N BC = (∠BAC − ∠AC B) ,
2

1
∠N C K = ∠KC B − ∠N C B = (∠BAC − ∠ABC) ,
2

IC sin ∠I BC sin B2 cos A+C
2
= = C
= A+B
.
IB sin ∠I C B sin 2 cos 2
Nên

EB I C sin ∠N BK cos A+C A−C


2 sin 2 sin A − sin C a−c
= · = A+B A−B
= = .
EC I B sin ∠N C K cos 2 sin 2 sin A − sin B a−b

Suy ra

EB EC EB + EC a a (a − c)
= = = ⇒ EB = .
a−c a−b a−c+b−c 2a − b − c 2a − b − c

Suy ra tiếp

a (a − c) a a (b − c)
E M = EB − M B = − = ,
2a − b − c 2 2 (2a − b − c)

a (a − c) ac a2 (b − c)
E D = EB − BD = − = .
2a − b − c b + c (b + c) (2a − b − c)
Do đó
ED a2 (b − c) 2 (2a − b − c) 2a
= · = .
EM (b + c) (2a − b − c) a (b − c) b+c
ED 2a
Vậy đẳng thức = là đúng và ta có điều phải chứng minh.
EM b+c

Bài 7. Cho tam giác ABC cân tại A. Đường tròn (O) tiếp xúc với AB, AC
tại B, C. Q là một điểm tùy ý nằm trong góc ∠BAC, OP⊥AQ tại P. Đặt
K ≡ OP ∩ (BPQ) và L ≡ OP ∩ (C PQ). Chứng minh rằng OK = OL.

Lời giải.
Xét phép nghịch đảo sau

I1 A, k1 = AP · AQ : P 7→ Q, B 7→ B 0 ≡ AB ∩ (BPQ) , C 7→ C 0 ≡ AC ∩ (C PQ) .

Hướng tới kỷ niệm 20 năm thành lập trường PTNK, ĐHQG TP HCM
Chuyên đề Toán học số 10 81
Chuyên đề Toán học Phổ thông năng khiếu TP.HCM số 10

K B0
R
I

P
Q
M0 N0
M 0
O O
N
J A

L
C0

Ta có AB = AC nên AB 0 = AC 0 , suy ra B 0 C 0 k BC. Hơn nữa A, B, C, P đồng


viên nên B 0 , Q, C 0 thẳng hàng. Ký hiệu M ≡ QO ∩ (BPQ), N ≡ QO ∩ (C PQ),
I ≡ QB ∩ (O), J ≡ QC ∩ (O) và R ≡ BC ∩ (C PQ). Ta có CRQC 0 là hình
thang và nội tiếp trong đường tròn (C PQ) nên CRQC 0 là hình thang cân,
dẫn đến RC 0 = CQ. Từ đó

(P C, PQ) ≡ (BC, BA) ≡ (CA, C B) ≡ (I C, IQ) (mod π).

Suy ra P, N , L, C 0 , Q đồng viên và tương tự thì 0


 P, M , K, B , Q cũng đồng viên.
Lại xét phép nghịch đảo I2 Q, k2 = PQ/(O) . Ta có I2 : B 7→ I, C 7→ J nên

I2 : (BPQ) 7→ C I, (C PQ) 7→ BJ, O 7→ O0 , P 7→ P ≡ BI∩C I, M 7→ M 0 , N 7→ N 0 .

Theo tính chất của phép nghịch đảo thì vì OP⊥AQ nên P 0 O0 ⊥M 0 N 0 . Từ đó,
0 0 QM 0
N 0 = QN 0 . Do đó
bằng biến đổi tỷ lệ diện tích và định lý sin, ta thu được OO0M

k2 · O0 N 0 k2 · O0 M 0
ON = = = OM .
QO0 · QN 0 QO0 · QM 0

Mặt khác (K M , K P) ≡ (C M , C P) ≡ (LN , LP) (mod π) nên K M k LN . Vậy


nên M K N L là hình bình hành.
Do đó, trung điểm O của M N cũng là trung điểm của K L, tức là OK = OL.
Ta có đpcm.

Hướng tới kỷ niệm 20 năm thành lập trường PTNK, ĐHQG TP HCM
82 Chuyên đề Toán học số 10

Bài 8. Về phía ngoài của tam giác ABC ta lấy các điểm P, Q, R thỏa mãn các
Chuyên đề Toán học Phổ thông năng khiếu TP.HCM số 10

điều kiện
v
AR t3
∠PAB = ∠BQC = 45◦ , ∠ABP = ∠QBC = 75◦ , ∠CAR = 105◦ , = .
AC 2
RQ
Tính tỷ số .
CM
Lời giải.
Giả sử tam giác ABC có hướng dương
P

A
B
R

I
N

Gọi R là phép quay vector góc +75◦ . Ta có


€−→Š €−→ − → − →Š
R QR = R QB + BA + AR
€−→Š €−
→Š €−
→Š
= R QB + R BA + R AR
QB −→ AB −→ AR −→
= · CB + · BP + · CA.
BC BP AC
Mặt khác v
QB AB sin 75◦ t 3 AR
= = = = .
BC BP sin 60◦ 2 AC
Suy ra
v v
€−→Š t 3 €−→ −→ −→Š t 3 €−→ −→Š p −−→
R QR = C B + BP + CA = C P + CA = 6 · C M .
2 2
RQ p
Vậy = 6.
CM

Hướng tới kỷ niệm 20 năm thành lập trường PTNK, ĐHQG TP HCM
Chuyên đề Toán học số 10 83

Bài 9. Đường tròn (I) nội tiếp tam giác ABC, tiếp xúc với các cạnh BC, CA, AB
tại X , Y, Z. X 0 , Y 0 , Z 0 lần lượt là các điểm trên các tia I X , I Y, I Z sao cho
Chuyên đề Toán học Phổ thông năng khiếu TP.HCM số 10

I X 0 = I Y 0 = I Z 0 . Chứng minh rằng AX 0 , BY 0 , C Z 0 đồng quy.


Lời giải.
Xét phép đối xứng trục AI biến Z 7→ Y , I Z = I Y, I Z 0 = I Y 0 nên Z 0 7→ Y 0 ,
suy ra ∠ZAZ 0 = ∠Y AY 0 .
Tương tự ta cũng có ∠Z BZ 0 = ∠X BX 0 và ∠X C X 0 = ∠Y C Y 0 . Từ đó suy ra
∠ABX 0 = ∠C BZ 0 , ∠CAZ 0 = ∠BAY 0 và ∠BC Y 0 = ∠AC X 0 . Do đó

sin ∠X 0 AB R X 0 AC R X 0 AB R X 0 AB
= :
sin ∠X 0 AC sin ∠X 0 AC sin ∠X 0 AB R X 0 AC
AX 0 AX 0 X 0C sin ∠ABX 0 sin X 0 BC
= : · = · .
sin ∠AC X 0 sin ABX 0 X 0 B sin ∠AC X 0 sin X 0 C B

Y0
A

Z0 Y
Z

B X C

X0

0 0
Lý luận tương tự với các tỉ số sin ∠Y BC sin ∠Z CA
sin ∠Y 0 BA , sin ∠Z 0 C B . Nhân các tỉ số trên lại với
nhau suy ra
sin ∠X 0 AB sin ∠Y 0 BC sin ∠Z 0 CA
= 1.
sin ∠X 0 AC sin ∠Y 0 BA sin ∠Z 0 C B
Vậy theo định lý Ceva thì AX 0 , BY 0 , C Z 0 cũng đồng quy.

Hướng tới kỷ niệm 20 năm thành lập trường PTNK, ĐHQG TP HCM
84 Chuyên đề Toán học số 10

Bài 10. Lục giác ABC DE F có độ dài các cạnh bằng nhau đồng thời
Chuyên đề Toán học Phổ thông năng khiếu TP.HCM số 10

∠A + ∠C + ∠E = ∠B + ∠D + ∠F.

Chứng minh rằng ∠A = ∠D, ∠B = ∠E, ∠C = ∠F .

Lời giải. € €−→ −→ŠŠ


Xét phép quay R C, α = C B, C D : B 7→ D, A 7→ K. Ta có

360◦ = ∠AF E + ∠ABC + ∠C DE = ∠E DK + ∠C DK + ∠C DE.

Mà ∠C DK = ∠ABC nên ∠AF E = ∠E FA.


A B

E D

Mặt khác EK = EA, CA = C K suy ra DE = DC = DK. Từ đó

∠BC D = ∠AC K = 2∠EC K = ∠E DK = ∠AF E, tức là ∠C = ∠F.

Chứng minh tương tự ta có ∠A = ∠D và ∠B = ∠E.

Bài 11. Đường tròn (O1 ) và (O2 ) cắt nhau tại hai điểm P, Q. Một đường tròn
(O) đi qua P và cắt PQ, (O1 ) , (O2 ) tại A, B, C. Giả sử X , Y ∈ (O) sao cho
BX k C Y k AP, P X , PY cắt (O2 ) , (O1 ) lần nữa tại Z, T. Chứng minh rằng
X Y k ZT.

Lời giải.
Xét phép đối xứng trục O1 O2 biến

B 7→ U ∈ (O1 ) , C 7→ V ∈ (O2 ) .

Hướng tới kỷ niệm 20 năm thành lập trường PTNK, ĐHQG TP HCM
Chuyên đề Toán học số 10 85

Do phép đối xứng có trục là trung trực của AP biến X 7→ B và Y 7→ C nên


−→ −→ −→
Chuyên đề Toán học Phổ thông năng khiếu TP.HCM số 10

X U = AQ = Y V , suy ra U V k X Y .
Ký hiệu

Z ≡ U V ∩ (O2 ) , T ≡ U V ∩ (O1 ) , M ≡ P X ∩ QU, N ≡ PY ∩ QV.

Ta có
(QM , QN ) ≡ (AX , AY ) ≡ (P M , P N ) (mod π).
Suy ra M , N , P, Q đồng viên, từ đó

(P M , PQ) ≡ (N M , NQ) ≡ (V Z, V Q) ≡ (P Z, PQ) (mod π).

V
Z
Q

N
M
O1
O2

B C

O
X

Suy ra P, M , Z thẳng hàng và tương tự thì P, N , T thẳng hàng. Vậy các điểm
Z, T ở đây chính là các điểm Z, T được nói đến trong bài.

Bài 12. Về phía ngoài tam giác ABC lấy các điểm X , Y sao cho các tam giác
ABX , AC Y đồng dạng ngược hướng. Lấy các điểm Z, T sao cho các tam giác
BZ C, BX A, T X Y đồng dạng cùng hướng. Chứng minh rằng các tam giác
BZ C, T X Y có cùng chung tâm đường tròn ngoại tiếp.

Hướng tới kỷ niệm 20 năm thành lập trường PTNK, ĐHQG TP HCM
86 Chuyên đề Toán học số 10

Lời giải.
XZ
= BX
Chuyên đề Toán học Phổ thông năng khiếu TP.HCM số 10

Từ giả thiết ta thấy rằng 4BZ X ∼ 4BAC, suy ra AC BA , suy ra tiếp


BX = AB . Lại có
XZ AC

BT BX BT AY
4X BT ∼ 4X AY suy ra = ⇒ = .
AY AX BX AX
CY AC AY
4ABX ∼ 4AC Y ⇒ = = .
BX AB AX
Từ đó suy ra
XZ AC AY BT CY
= = = = ⇒ X Z = BT = C Y.
BX AB AX BX BX

Z
X

Y
O
B

Mặt khác, gọi O là tâm của phép đồng dạng thuận H : X 7→ Z, T 7→


B, Y 7→ C thì ta có

4OZ X ∼ 4OBT ∼ 4OC Y
⇒ 4OZ X = 4OBT = 4OC.
X Z = BT = C Y

Suy ra OZ = OB = OC và OX = OY = OT. Vậy (BZ C) và (T X Y ) có cùng


tâm đường tròn ngoại tiếp là O.

Hướng tới kỷ niệm 20 năm thành lập trường PTNK, ĐHQG TP HCM
Chuyên đề Toán học Phổ thông năng khiếu TP.HCM số 10

Sử dụng tính liên tục của chu kỳ


để chứng minh hàm hằng

Phạm Tuấn Huy


(Lớp Chuyên Toán khóa 2011 − 2014)

Có nhiều bài toán phương trình hàm có nghiệm làm hàm hằng, dù có hình thức
rõ ràng là liên quan đến hàm hằng (như việc có một tập các chu kỳ tuần hoàn T)
nhưng để chứng minh hàm hằng lại là công việc không đơn giản. Chúng tôi xin
phép bắt đầu vời bài toán như sau:

Bài 1. Tìm tất cả các hàm số f : R → R thỏa mãn điều kiện

f (x + y 2 + f ( y)) = f (x) với mọi x, y ∈ R.

Lời giải. 
Đặt A = y 2 + f ( y)| y ∈ R và B = {a1 − a2 |a1 , a2 ∈ A} thì rõ ràng f là
hàm tuần hoàn với chu kỳ t với mọi t ∈ A, t ∈ B.
Trong trường hợp f ( y) = − y 2 với mọi y ∈ R thì điều kiện đã cho là đúng.
Ngược lại, f sẽ có chu kỳ tuần sự a 6= 0, ta cần dựa vào đó để xây dựng
các chu kỳ khác.
Ta sẽ sử dụng B ở đây: b = y12 + f ( y1 ) − y22 − f ( y2 ).
Vì chưa có thông tin gì về f nên ta sẽ chọn y1 , y2 để f ( y1 ) = f ( y2 ), chẳng
hạn y1 = y2 + a. Khi đó, ta có

b = y12 − y22 = ( y2 + a)2 − y22 = 2 y2 a + a2 .

Vì a 6= 0 nên ở đây, ta có một hàm bậc nhất theo y2 ∈ R và do đó B = R.


Suy ra, với mọi b ∈ R thì f (x + b) = f (x) nên f (x) = c với c là hằng số
nào đó.
Rõ ràng các hàm số f (x) = −x 2 và f (x) = c thỏa mãn đề bài.

87
88 Chuyên đề Toán học số 10

Ta sẽ tiếp tục mở rộng bài toán trên:


Chuyên đề Toán học Phổ thông năng khiếu TP.HCM số 10

Bài 2. Tìm tất cả các hàm số f : R → R thỏa mãn

f (x + g( y) + f ( y)) = f (x) với mọi x, y ∈ R

trong đó, g là hàm đa thức và deg g > 2.

Lời giải.
Với các định nghĩa và phương pháp tương tự trên, trong trường hợp f ( y) 6=
−g( y), ta có

b = g( y + a) + f ( y + a) − g( y) − f ( y) = g( y + a) − g( y)

là một chu kỳ tuần hoàn của f .


Vì deg g ≥ 2 và a 6= 0 nên g( y + a) − g( y) là một đa thức h có deg h =
deg g − 1 ≥ 1.
Ở đây, ta không có H = {h(x)|x ∈ R} = R, chẳng hạn khi deg h là số chẵn,
nhưng ta vẫn có H chứa một miền [t; +∞) nào đó với t đủ lớn.
Từ đây, ta có thể thu được f hằng, chẳng hạn

f (x) = f (min{x, y} − t) = f ( y) với x, y ∈ R.

Bài toán kết thúc.

Trong bài viết, ta chỉ xét |A| > 1, vì với |A| = 1, chẳng hạn g( y) + f ( y) =
const thì ta đã có thể dễ xác định được hàm số f .
Ở cả hai ví dụ trên, ta đều thu được chu kỳ là một khoảng hay đoạn liên
tục trên R. Có nhiều trường hợp mà tập chu kỳ không như vậy, ta chỉ cần
chứng minh một khoảng hay một đoạn nhỏ bất kỳ là đủ. Ta xét bài toán,
cũng là bổ đề quan trọng sau.

Bài 3. Cho hàm số f : R → R sao cho f tuần hoàn chu kỳ t với mọi
t ∈ [a, b] và a < b. Chứng minh rằng f là hàm hằng.

Lời giải.
Xét một số x ∈ [a, b] bất kỳ, do f tuần hoàn chu kỳ x nên

f (x) = f (x − x) = f (0).

Ta có được một khoảng có độ dài b − a cùng bằng f (0).

Hướng tới kỷ niệm 20 năm thành lập trường PTNK, ĐHQG TP HCM
Chuyên đề Toán học số 10 89

Chú ý rằng {u + v|u, v ∈ [a, b]} = [2a, 2b] nên


Chuyên đề Toán học Phổ thông năng khiếu TP.HCM số 10

f (x) = f (0) với mọi x ∈ [2a, 2b].

Tương tự, ta cũng có được f (x) = f (0) với mọi x ∈ [ka, kb] với k ∈ Z+ .
Tồn tại k đủ lớn sao cho kb − ka > a, ta sẽ chứng minh rằng f hằng trên
[ka, +∞).
Thật vậy,
Với mọi t ∈ [kb, kb + b − a] thì

f (x) = f (kb) = f (0) ⇒ f (x) = f (0), t ∈ [ka, kb + b − a].

Tương tự, ta có thể thu được f (x) = f (0) với mọi x ∈ [ka, kb + l(b − a)]
với l ∈ Z+ .
Do lim l(b − a) = +∞ nên f (x) = f (0) với mọi x ∈ [ka, +∞).
Tiếp theo, ta sẽ tìm cách "giảm" miền xác định ở trên. Tiếp tục xét x ∈
[ka − (b − a); ka] thì
f (x) = f (ka) = f (0).
Một cách tương tự, ta cũng có f (x) = f (0), x ∈ [ka − l(b − a); ka]. Do
lim l(b − a) = +∞ nên f (x) = f (0) với mọi x ∈ (−∞; ka].
Vậy f là hàm hằng.

Ứng dụng kết quả trên khá hiệu quả trong một số bài toán mà hàm số tuần
hoàn với một tập chu kỳ từ giá trị x nào đó đủ lớn. Chẳng hạn bài toán
khá khó sau đây.

Bài 4. Tìm tất cả các hàm số f : R → R sao cho

f (x 2 + y + f ( y)) = f 2 (x) với mọi x, y ∈ R

Lời giải.
Đặt A = { y + f ( y)| y ∈ R} và B = {a1 − a2 |a1 , a2 ∈ A} thì

f 2 (x) = f (x 2 + a1 ) = f (x 2 + a2 ).

Suy ra với mọi x ≥ min{a1 , a2 } thì f tuần hoàn với chu kỳ |a1 − a2 | . Tuy
nhiên, việc triển khai tiếp tục như các ví dụ trên lại không thành công vì
y + a − y = a.

Hướng tới kỷ niệm 20 năm thành lập trường PTNK, ĐHQG TP HCM
90 Chuyên đề Toán học số 10

Do đó, ta phải chú ý vào x 2 và f 2 (x). Ta vẫn chọn y1 , y2 để f ( y1 ) = f ( y2 )


Chuyên đề Toán học Phổ thông năng khiếu TP.HCM số 10

nhưng y1 , y2 cần phải đặt trong bình phương để hiễu của nó là một khoảng
liên tục. Chẳng hạn
2
y1 = (x 2 + a1 ) + a2
2
y2 = (x 2 + a2 ) + a2

Khi đó f ( y1 ) = f 2 x 2 + a1 = f 4 (x) = f ( y2 ) nên
2 2
y1 + f ( y1 ) − y2 − f ( y2 ) = y1 − y2 = (x 2 + a1 ) − (x 2 + a2 )
= 2(a1 − a2 )x 2 + a12 − a22 , ∀x ∈ R.

Giả sử |A| 6= 1 (trường hợp |A| = 1 thì ta có thể thay vào kiểm tra trực tiếp
và thấy không có hàm nào thỏa mãn), ta chọn được a1 > a2 trong A. Khi
đó, B sẽ chứa một đoạn liên tục trong R và từ giá trị x ≥ x 0 nào đó thì f
là hàm tuần hoàn chu kỳ t thuộc đoạn trên.
Theo chứng minh kết quả trên thì f hằng với mọi x ≥ x 1 .
Đặt f (x) = c với mọi x ≥ x 1 . Lấy α sao cho x 2 + α ≥ x 1 , ∀x ≥ x 1 thì ta có
c 2 = c nên c ∈ {0; 1}.
Ta xét các trường hợp:

1. Nếu c = 0 thì rõ ràng A ⊇ {x ≥ x 1 }, suy ra ∀x ∈ R thì f 2 (x) =


f (x 2 + x 1 ) = 0 do x 2 + x 1 ≥ x 1 .

Suy ra f (x) = 0, ∀x ∈ R.

2. Nếu c = 1 thì A ⊇ [x 1 + 1, +∞), suy ra ∀x ∈ R thì

f 2 (x) = f (x 2 + x 1 + 1) = 1.

Suy ra f (x) = ±1, ∀x ∈ R. Do đó, tồn tại a ∈ A, a < m, ∀m ∈ R mà


y < m − 1 và f ( y) + y ≤ y + 1 < m. Điều này dẫn đến ∀x ∈ R, tồn
tại t, y sao cho x = t 2 + y + f ( y) và f (x) = f 2 (t) ≥ 0.

Từ đó, ta có f (x) = 1, ∀x ∈ R.

Vậy f (x) ≡ 0 hoặc f (x) ≡ 1 với mọi x ∈ R.

Bài toán tiếp theo cũng có nhiều yếu tố giống với bài toán trên

Bài 5. Tìm f : R → R thỏa mãn

f (x 2 + y 2 + 2 f (x y)) = f 2 (x + y) với mọi x, y ∈ R.

Hướng tới kỷ niệm 20 năm thành lập trường PTNK, ĐHQG TP HCM
Chuyên đề Toán học số 10 91

Lời giải.
Chuyên đề Toán học Phổ thông năng khiếu TP.HCM số 10

Ta viết lại đẳng thức ở dạng

f (x + y)2 + 2 f (x y) − 2x y = f 2 (x + y).


Chú ý rằng các giá trị x + y, x y khá độc lập và "hầu như không phụ thuộc
vào nhau".
Đặt a = x + y, b = x y và g(x) = 2 f (x) − 2x. Khi đó

f (a2 + g(b)) = f 2 (a)


a2
với mọi b ≤ 4 (khi b ≤ 0 thì a ∈ R).
Đặt A = {g(b)|b ∈ R} và B = {g(b1 ) − g(b2 )|b1 , b2 ∈ R}.
Vậy với mọi x đủ lớn, f tuần hoàn với chu kỳ t ∈ B. Ta có
€ 2
Š € 2
Š
g (x 2 + a1 ) + a2 − g (x 2 + a2 ) + a2
” € 2
Š € 2
Š 2 2
—
= 2 f (x 2 + a1 ) + a2 − f (x 2 + a2 ) + a2 + (x 2 + a2 ) − (x 2 + a1 )
 
= 2 f 4 (x) − f 4 (x) + 2(a2 − a1 )x 2 + a22 − a12
= 4(a2 − a1 )x 2 + a22 − a12 ∈ B

Giả sử tồn tại hai số a1 , a2 ∈ A sao cho a2 > a1 . Suy ra B chứa một đoạn
liên tục trong R và với mọi x đủ lớn thì f tuần hoàn theo chu kỳ tùy ý
trong đoạn này.
Theo kết quả trên, ta có f là hàm hằng với mọi x ≥ x 1 , tức là

f (x) = c, ∀x ≥ x 1 .

Chọn a đủ lớn để a ≥ x 1 , x 2 + a ≥ x 1 thì c 2 = c ⇒ c ∈ {0, 1}. Ta cũng xét


hai trường hợp:

1. Nếu c = 0 thì f ( y 2 + g(x)) = f 2 ( y) = f (− y)2 . Do đó, với mọi y


thỏa mãn y ≤ −x 1 hoặc y ≥ x 1 thì f ( y) = 0.

Suy ra ∀ y ≤ −x 1 : g( y) = −2 y. Với mọi x ∈ R, tồn tại y ≤ −x 1 để


x 2 − 2 y > x1.

Từ đó, f 2 (x) = f (x 2 + 2 y) = 0 ⇒ f (x) = 0, ∀x ∈ R.

2. Nếu c = 1 thì ∀ y ≤ −x 1 , ta có f ( y) = ±1, g( y) = ±2 − 2 y, suy ra

∀x ∈ R, ∃ y ≤ −x 1 : x 2 ± 2 − 2 y > x 1

Hướng tới kỷ niệm 20 năm thành lập trường PTNK, ĐHQG TP HCM
92 Chuyên đề Toán học số 10


f 2 (x) = f (x 2 ± 2 − 2 y) = 1 ⇒ f (x) = ±1, ∀x ∈ R.
Chuyên đề Toán học Phổ thông năng khiếu TP.HCM số 10

Trong trường hợp f không đồng nhất với 1, tồn tại x 0 sao cho
f (x 0 ) = −1.
x2
Suy ra x 0 6= x 2 + g( y)| y ≤ 4 . Xét y ≤ 0, x ∈ R thì

x 0 < g( y), ∀ y ≤ 0.

Mặt khác, g(0) ≤ 2 nên x 0 < 2, suy ra f (x) = 1, ∀x ≥ 2 và g(x) =


2 − 2x, ∀x ≥ 2.

Ta sẽ chứng minh rằng x 0 < 0. Thật vậy,

Giả sử x 0 ≥ 0, khi đó g(x 0 ) = −2 − 2x 0 và ∀a2 ≥ 4x 0 thì

f (a2 − 2 − 2x 0 ) = f 2 (a) ≥ 0 ⇒ f (t) = 1, ∀t ≥ 2x 0 − 2.

Suy ra x 0 < 2x 0 − 2 ⇔ x 0 > 2, mâu thuẫn, tức là phải có x 0 < 0.

Với g(x 0 ) = −2 − 2x 0 và f (a2 − 2 − 2x 0 ) = f 2 (a) ≥ 0, suy ra

2
f (t) = 1, ∀t ≥ −2 − 2x 0 ⇒ x 0 < −2 − 2x 0 ⇒ x 0 < − .
3

Do đó, f (x) = 1, ∀x ≥ − 23 . Ta sẽ chứng minh rằng x 2 + g( y) với


x 2 ≥ 4 y sẽ không thể nhận giá trị bé hơn − 23 .

Thật vậy, nếu y < − 23 thì g( y) = ±2 − 2 y và x 2 + g( y) ≥ −2 − 2 y ≥


− 23 ; còn nếu y ≥ − 23 : g( y) = 2 − 2 y thì ta có hai khả năng:

• Nếu y ≤ 0 : x 2 + g( y) = x 2 + 2 − 2 y ≥ 2 − 43 > 0.
• Nếu y > 0 : x 2 + g( y) ≥ 4 y + 2 − 2 y = 2 y + 2 > 0. Theo nhận
xét trên, ta có họ hàm f là

2
 f (x) = 1, ∀x ≥ −

3
2
 f (x) = ±1, ∀x < −

3

thỏa mãn.

Từ đó dễ dàng có kết luận cho bài toán.

Hướng tới kỷ niệm 20 năm thành lập trường PTNK, ĐHQG TP HCM
Chuyên đề Toán học số 10 93

Trong bài toán trên, tuy cần nhiều xử lý phức tạp nhưng phần quan trọng
Chuyên đề Toán học Phổ thông năng khiếu TP.HCM số 10

nhất vẫn là việc chứng minh tính liên tục của tập chu kỳ.
Chú ý rằng ở đây với hai biến x, y, khi ta cố định x + y thì x y vẫn nhận
giá trị trong một khoảng (−∞; t] nào đó, ta dựa vào đây để xét các chu
kỳ của hàm số.
Bài toán sau đây khá giống với bài toán trên theo phương diện đó:
Bài 6. Tìm tất cả các hàm số f : R → R thỏa mãn

f f (x 2 + y 2 ) + 2x y = f 2 (x + y) với mọi x, y ∈ R.
Lời giải.

Biểu thức trên có thể đưa về dạng



f x 2 + g( y) = f 2 (x) với g( y) = f ( y) − y, y ≥ 2x 2

f f (x 2 + y 2 ) − x 2 − y 2 + (x + y)2 = f 2 (x + y).


Với mọi x, y ∈ R, ta sẽ chứng minh rằng


f 2 (x) = f 2 ( y) ⇔ f (x 2 + g(a)) = f ( y 2 + g(b)).
Ta sẽ chứng minh tồn tại a, b > max{x 2 , y 2 } sao cho g(a)− g(b) = y 2 − x 2 .
Thật vậy,
Nếu tồn tại a, b mà g(a) − g(b) = y 2 − x 2 thì
g(t 2 + g(b)) − g(t 2 + g(a)) = f (t 2 + g(b)) − f (t 2 + g(a)) + g(a) − g(b)
= g(a) − g(b)
Khi đó, ta sẽ chọn t để t 2 + g(a), t 2 + g(b) > 2 max{x 2 , y 2 }. Xét biểu thức
2 2 2 2
g((t 2 + g(a)) + g(a)) − g((t 2 + g(b)) + g(b)) = (t 2 + g(b)) − (t 2 + g(a))
= 2[g(b) − g(a)]t 2 + g 2 (b) − g 2 (a)
Giả sử ∃g(b) > g(a) thì từ biểu thức trên, suy ra
∀r ≥ g 2 (b) − g 2 (a) : ∃a0 , b0 : g(a0 ) − g(b0 ) = r.
Với mọi x, y ∈ R, chọn z sao cho z 2 − y 2 > g 2 (b) − g 2 (a), z 2 − x 2 >
g 2 (b) − g 2 (a), suy ra
f (z) = f 2 ( y)
 2
⇒ f 2 (x) = f 2 ( y), ∀x, y ∈ R.
f 2 (z) = f 2 (x)
Vậy | f (x)| = c, ∀x ∈ R nên c = c 2 ⇒ c ∈ {0; 1}. Điều này dẫn đến g( y) ∈
{−1 − y; − y; 1 − y}. Ta xét các trường hợp:

Hướng tới kỷ niệm 20 năm thành lập trường PTNK, ĐHQG TP HCM
94 Chuyên đề Toán học số 10

1. Nếu c = 0 thì f (x) = 0 với mọi x ∈ R.


Chuyên đề Toán học Phổ thông năng khiếu TP.HCM số 10

2. Nếu c = 1 thì với mọi t ∈ R, ta sẽ chứng minh tồn tại x, y để:

t = 2x y + f (x 2 + y 2 ).

Thật vậy, cố định x 2 + y 2 = a ≥ t + 1, ta xét hệ

x 1 + y12 = x 22 + y22 = a
 2


t −1

x 1 y1 =

2
t + 1


 x 2 y2 =

2

Dễ dàng chứng minh được hệ có nghiệm. Suy ra f (x) = 1, t =


2x 1 y1 + f (x 12 + y12 ) và f (x) = −1, t = 2x 2 y2 + f (x 22 + y22 ).

Từ đây ta được

∀t ∈ R, ∃x, y : f (t) = f 2 (x + y) ≥ 0

hay f (t) = 1. Do đó f (x) ≡ 1, ∀x ∈ R.

Vậy có hai hàm số thỏa mãn là f (x) ≡ 0 hoặc f (x) ≡ 1 với x ∈ R.

Ngoài ra, ta còn có thể mở rộng bài toán theo hướng f (P(x) + g( y)) =
P ( f (x)) hay xây dưng các bài toán tương tự với các biểu thức đối xứng
nhiều biến.
Một số bài toán tương tự:

Bài 7. Tìm tất cả các hàm số f : R → R thỏa mãn



f f 2 (x + y) − x 2 − y 2 = f (2x y) với mọi x, y ∈ R

Bài 8. Tìm tất cả các hàm số f : R → R thỏa mãn



f f 2 (x + y) − x 2 − y 2 = 2 f (x y) với mọi x, y ∈ R

Hướng tới kỷ niệm 20 năm thành lập trường PTNK, ĐHQG TP HCM
Chuyên đề Toán học Phổ thông năng khiếu TP.HCM số 10

Ứng dụng của


tỉ số đoạn thẳng và tỉ số lượng giác

Hồ Quốc Đăng Hưng


(Lớp Chuyên Toán khóa 2012 − 2015)

Từ trước đến nay, đã có nhiều phương pháp sử dụng công cụ đại số để giải quyết
các bài toán hình học như tọa độ, số phức, vector,... Tuy nhiên, hầu hết các phương
pháp này đều không được ưa chuộng bởi quá nặng về tính toán, làm mất đi vẻ
thẩm mỹ và nét đẹp đặc trưng của hình học. Thực tế, rất nhiều kiến thức hình
học hay sử dụng trong các kì thi toán Olympic là kết quả của các phép toán đại
số, chẳng hạn như hàng điểm điều hòa, trục đẳng phương, phép nghịch đảo,...

Hình học từ lâu đã là một chủ đề quan trọng trong các kì thi toán trong và ngoài
nước, thế nhưng với sự giới hạn về thời gian cũng như ảnh hưởng tâm lí, việc tìm
ra lời giải "đẹp" và thuần túy hình học ngay trong bối cảnh phòng thi thực sự là
điều không đơn giản. Những lời giải xuất sắc không chỉ cần sự tinh tế trong quan
sát, phân tích mà còn đòi hỏi kinh nghiệm và đôi khi là người giải đã biết trước
một bổ đề hay kết quả nào đấy liên quan đến bài toán.

Trong bài viết này, người viết muốn chia sẻ một hướng tiếp cận với hình học mà
nhìn sơ qua thì cũng khá nặng về tính toán và thiếu tính thẩm mỹ: tỉ số đoạn
thẳng và lượng giác.

Tuy nhiên, nếu ta bỏ qua vẻ ngoài cồng kềnh đó mà chú ý hơn vào các phân tích,
đánh giá để đưa đến các hệ thức cần chứng minh và cách xử lí chúng thì ta sẽ
thấy được tất cả các yếu tố tính toán đều là kết quả của những suy nghĩ tự nhiên
và đậm chất hình học. Tư tưởng chung của các bài toán sau là đánh giá các dữ
kiện có sẵn và dùng tỉ số cạnh-lượng giác để đơn giản hóa bài toán. Bản thân
người viết là một người yêu thích hình học và cũng đã không ít lần thành công
với phương pháp này trong các cuộc thi toán Olympic.

95
96 Chuyên đề Toán học số 10

1. Một số kiến thức và ví dụ cơ bản


Chuyên đề Toán học Phổ thông năng khiếu TP.HCM số 10

Trước hết, hãy cùng tìm hiểu ví dụ sau:


Bài 1. Cho 4ABC nội tiếp (O) có L là giao điểm của hai tiếp tuyến kẻ từ B
và C của (O). Chứng minh rằng B
Ô AL = CAM
Ö.

Chúng ta cùng xem xét hai lời giải sau đây.

A K

M
B C

J
N

Lời giải 1. Đường thẳng qua A song song BC cắt (O) tại K. AM cắt (O) tại
J. Ta có A(BC M K) là chùm điều hòa nên tứ giác BKC J điều hòa, từ đó
suy ra K J đi qua L. Do đó L cũng là giao điểm của K J và trung trực BC.
Mặt khác ta có A và K đối xứng nhau qua trung trực BC nên AL và K J
cũng đối xứng nhau qua trung trực BC, vậy nếu gọi giao điểm của AL với
(O) là N thì ta suy ra được N đối xứng với J qua trung trực BC. Đến đây
ta suy ra N J KA là hình thang cân, và N J k AK k BC, cho nên N J C B cũng
là hình thang cân. Cuối cùng, ta kết luận cung BN và cung C J của (O) có
độ lớn bằng nhau. Vậy B Ô AL = CAM
Ö. ƒ

Hướng tới kỷ niệm 20 năm thành lập trường PTNK, ĐHQG TP HCM
Chuyên đề Toán học số 10 97

Lời giải 2. Chú ý rằng BÔAL = CAM


Ö khi và chỉ khi sin B
Ô AL = sin CAM
Ö vì tổng
hai góc này không thể bằng 180 . Tuy nhiên, để ý rằng với mọi α < π, hàm

Chuyên đề Toán học Phổ thông năng khiếu TP.HCM số 10

sin x
f (x) = sin(α−x) tăng thực sự trên khoảng (0; α), nên thay vì chứng minh
sin B
Ô AL = sin CAMÖ , ta có thể chuyển về chứng minh:

sin B
Ô AL sin CAM
Ö
= . (∗)
sin B
Ô AL sin B
Ö AM

Theo hệ thức liên hệ giữa tỉ số cạnh và tỉ số diện tích và chú ý rằng SAM C =
SAM B , ta có:

AC · sin CAM
Ö SAM C
=
AB · sin B
Ö AM SAM B
sin B
Ö AM AC
= . (1)
sin CAM
Ö AB

Bây giờ sử dụng công thức liên hệ giữa tỉ số cạnh và tỉ số lượng giác trong
tam giác, ta có:

sin B
Ô AL BL sin CAL
Ô CL
= và = .
sin A
ÔBL AL sin AC
ÔL AL

Chú ý rằng A
ÔB L = 180◦ − AC
Õ ÔL = 180◦ − ABC
B, AC Õ và LB = LC. Cho nên:

sin B
Ô AL sin A
ÔBL sin AC
Õ B AB
= = = . (2)
sin CAL
Ô sin AC
ÔL Õ AC
sin ABC

Từ (1) và (2) ta suy ra hệ thức ở (*), và kết luận B


Ô AL = CAM
Ö. ƒ

Nhận xét. Nhìn vào hai lời giải trên, chắc hẳn nhiều bạn sẽ cảm thấy ưng
ý với lời giải 1 hơn. Tuy nhiên việc kẻ thêm đường phụ và ứng dụng tính
chất của hàng điểm điều hòa không phải là điều đơn giản và đòi hỏi ít
nhiều yếu tố kinh nghiệm của người giải. Trong khi đó, lời giải 2 dù trông
qua khá cồng kềnh và nặng tính toán nhưng thực chất lại là kết quả của
một hướng suy nghĩ, phân tích rất tự nhiên. Khi phân tích bài toán, ta thấy
rằng trong 4AB L , B ÔAL, A
Ô LB, cùng với cạnh AL là những yếu tố rất khó
đánh giá. Song, nếu ta khéo léo đưa điều cần chứng minh về tỉ lệ lượng
giác, ta sẽ khai thác được tính đối xứng của hình vẽ, ở đây là B
Ô AL và CAL,
Ô
4AB L và 4AC L.Qua hình vẽ, ta thấy được rằng B L, C L, A Ô ÔL có liên
B L, AC
hệ trực tiếp với các cạnh và góc của 4ABC, và yếu tố gây khó chịu là AL
lại có thể được xử lí khi chia tỉ lệ, do đó ngay cả khi chưa tính toán cụ

Hướng tới kỷ niệm 20 năm thành lập trường PTNK, ĐHQG TP HCM
98 Chuyên đề Toán học số 10

thể, ta hoàn toàn có thể tự tin vào hướng tiếp cận này. Như vậy, dù lời giải
Chuyên đề Toán học Phổ thông năng khiếu TP.HCM số 10

hoàn chỉnh hơi cồng kềnh, nhưng ngay từ những bước đầu tiên hướng đi
đã được định hình rất rõ ràng, và công việc tính toán dù qua một số bước
nhưng cũng không gây khó khăn.
Tóm lại, không giống như tính toán dựa trên các phương pháp tọa độ, số
phức, sử dụng tỉ số đoạn thẳng, lượng giác không làm mất đi vẻ đẹp của
hình học mà ngược lại, thể hiện khả năng quan sát, bao quát hình vẽ của
người giải. Những lời giải các bài hình học dựa phương pháp này đòi hỏi
sự khéo léo trong việc đánh giá, tìm mối liên quan giữa các dữ kiện có sẵn
và điều cần chứng minh. Các hệ thức lượng giác chính là chiếc cầu nối
tuyệt vời liên hệ các yếu tố cơ bản của hình học là cạnh và góc.

MỘT SỐ HỆ THỨC VÀ TÍNH CHẤT QUAN TRỌNG

1. Trong 4ABC với bán kính đường tròn ngoại tiếp R, ta có:

BC AB AC
= = = 2R.
sin A sin C sin B

b, B
2. Với bốn góc nhọn A b, C,
b D b+ B
b bất kì mà A b = Cb + D
b , ta có:

sin A sin C
= b = Cb và B
⇔A b=D
b.
sin B sin D

3. Cho 4ABC và điểm D thuộc cạnh BC, khi đó:

AB · sin BAD BD
= .
AC · sin CAD CD

4. Cho AB và C D là hai dây cung của đường tròn (O). Khi đó với hai
điểm X , Y bất kì thuộc (O) (X , Y không nhất thiết phân biệt), ta
có hệ thức:
sin A
ÕXB AB
=
sin AY
Ô B CD

5. Giả sử trên đoạn thẳng AB có hai điểm C, D thỏa mãn

CA DA
= .
CB DB
Khi đó C trùng D.

Hướng tới kỷ niệm 20 năm thành lập trường PTNK, ĐHQG TP HCM
Chuyên đề Toán học số 10 99

Tất cả những hệ thức trên đều có chứng minh tương đối đơn giản, xin để
Chuyên đề Toán học Phổ thông năng khiếu TP.HCM số 10

lại cho bạn đọc. Chú ý rằng từ hệ thức 3 ta có thể suy ra hệ quả quan trọng
sau:
Hệ quả 1.1. Trong 4ABC, tia Ax đi qua trung điểm BC khi và chỉ khi:
sin B
Ô Ax AC
= .
sin CAx
Ô AB
Bài 2. Cho 4ABC nhọn có AD là phân giác trong. Đường thẳng qua D
vuông góc DA cắt đường tròn (B; BA) và (C; CA) lần lượt tại M , N . Chứng
minh rằng BM song song C N .

B
D C

Lời giải.
Bài toán phát biểu đơn giản nhưng lại khá khó chịu vì cách xác định hai
điểm M và N không mang lại nhiều gợi ý cho việc chứng minh song song.
Tuy nhiên, ta có thể suy ra một số kết quả đơn giản như BM = BA, C N =
CA và ×B DM = CÖ DN = 90◦ − A Õ DB. Đến đây, hệ thức 1 sẽ giúp ta đưa các
yếu tố liên quan đến M , N về các đại lượng "dễ chịu" là AB, AC, DB, DC.
Cụ thể:
sin ×
DM B BD BD
= = .
sin ×
B DM BM BA
Hoàn toàn tương tự, ta cũng có:
sin DN
Ö C CD
= .
sin C
Ö DN CA

Hướng tới kỷ niệm 20 năm thành lập trường PTNK, ĐHQG TP HCM
100 Chuyên đề Toán học số 10

BD CD
B DM = C
Mặt khác, do × Ö DN và = DM B =
nên ta suy ra ngay sin ×
BA CA
Chuyên đề Toán học Phổ thông năng khiếu TP.HCM số 10

sin DN
Ö C, hay BM k C N .

Bài 3. Đường tròn nội tiếp (I) của 4ABC tiếp xúc với các cạnh BC, CA, AB
lần lượt tại D, E, F . Gọi K là giao điểm của I D và E F . Chứng minh AK đi
qua trung điểm BC.

E
K
F

B D M C

Lời giải.
Để ý rằng từ hệ quả 2.2, để chứng minh AK đi qua trung điểm BC, ta chỉ
cần chứng minh:
sin Õ
B AK AC
= . (∗)
sin CAK
Õ AB
Áp dụng công thức 3 cho 4AE F và 4DE F , ta có:

KF AF · sin Õ
B AK sin Õ
B AK
 KE = = . (1)


AE · sin CAK
Õ sin CAK
Õ
 KF DF · sin IÔDF sin D
Õ E F · sin IÔ
DF
= = . (2)


KE DE · sin IÔDE sin D
Õ F E · sin IÔ
DE

Từ (*), (1) và (2) suy ra ta cần chứng minh

AC sin D
Õ E F · sin IÔ
DF
= . (3)
AB sin D
Õ F E · sin IÔ
DE

Hướng tới kỷ niệm 20 năm thành lập trường PTNK, ĐHQG TP HCM
Chuyên đề Toán học số 10 101

B
b
Õ=B
Để cho tiện, đặt ABC b, AC
Õ B = C,
b ta có D
Õ E F = 90◦ − ,D
Õ F E = 90◦ −
Chuyên đề Toán học Phổ thông năng khiếu TP.HCM số 10

2
Cb B
b Cb
DF = , IÔ
, IÔ DE = . Như vậy (3) tương đương với hệ thức sau:
2 2 2

AC sin(90◦ − B̂2 ) · sin B̂2


= .
AB sin(90◦ − Ĉ2 ) · sin Ĉ2

hay có thể viết lại thành:

sin B̂ 2 cos B̂2 sin B̂2


= .
sin Ĉ 2 cos Ĉ2 sin Ĉ2

Hệ thức cuối hiển nhiên đúng, do vậy ta suy ra (*) đúng, hay AK đi qua
trung điểm BC.

Bài 4 (Định lí con bướm). Cho PQ là một dây cung của đường tròn (ω) và
M là trung điểm PQ. AB và C D là hai dây cung bất kì của (ω) cắt nhau tại
M . Gọi giao điểm của AD, BC với PQ lần lượt là X , Y . Khi đó, chứng minh
M cũng là trung điểm X Y .

A
C

M Q
P
X Y

Hướng tới kỷ niệm 20 năm thành lập trường PTNK, ĐHQG TP HCM
102 Chuyên đề Toán học số 10

Lời giải.
Hướng suy nghĩ tự nhiên nhất là ta sẽ chứng minh M Y = M X hoặc Y Q =
Chuyên đề Toán học Phổ thông năng khiếu TP.HCM số 10

X P do đã có M P = MQ. Tuy nhiên, thực tế ta không cần chứng minh hai


đoạn thẳng nào bằng nhau cả, mà có thể sử dụng tính chất 2 cho trường
hợp cạnh (bạn đọc hãy tự phát biểu kết quả này!) . Cụ thể, ta chỉ cần
chứng minh hệ thức sau:
YM XM
= .
YP XP
Lần lượt áp dụng kết quả 3 cho 4C MQ và 4AM P, ta có:

YM C M · sin Ö
BC D
=


 .
 YQ

CQ · sin BCQ
Õ

AM · sin Õ

 XM B AD
=


 .
XP AP · sin Õ
DAP

Đến đây, áp dụng kết quả 4, ta lại suy ra:


 YM C M · BD
 = .
 YQ CQ · BQ

(∗)
 X M = AM · BD .


XP AP · DP
DP MP
Dễ thấy 4C MQ ∼ 4P M D nên = . Tương tự, do 4AM P ∼
CQ MC
AP MA
4QM B nên = . Từ hai hệ thức này và M P = MQ ta suy ra
BQ MQ

CM AM
= . (∗∗)
CQ · BQ AP · DP

Từ (*) và (**) ta suy ra điều phải chứng minh.

Bài 5. Cho tam giác ABC và điểm M thỏa mãn:

A
Ö M B − AM
Ö C = AC
Õ B − ABC
Õ

Chứng minh rằng M thuộc đường tròn Apollonius ứng với đỉnh A của 4ABC.

Lời giải.
Ta cần chứng minh MB
MC = AB
AC . Gọi N là giao điểm thứ hai của AM và đường

Hướng tới kỷ niệm 20 năm thành lập trường PTNK, ĐHQG TP HCM
Chuyên đề Toán học số 10 103

bM N = B
tròn ngoại tiếp tam giác M BC. Khi đó ta có B b C N và Cb M N = Cb BN .
Chuyên đề Toán học Phổ thông năng khiếu TP.HCM số 10

Từ đây áp dụng giả thiết:

A
Ö M B − AM
Ö C = AC
Õ B − ABC
Õ
−(B
× MN − C
× M N ) = AC
Õ B − ABC
Õ
BC
Ö N + AC
Õ B=C
Ö BN + ABC
Õ
A
ÕBN = AC
Õ N.

Áp dụng hệ thức 1 cho 4ABN và 4AC N , ta có:



sin A
bN B AB
= .


AN

 sin A
bBN

sin A
bN C AC


= .


sin A
bC N AN
Mặt khác, áp dụng hệ thức 4 cho đường tròn ngoại tiếp 4M BC, ta có:

MB sin A
ÕNB
= .
MC sin AN
Õ C
Từ những hệ thức trên suy ra:

AB bC N · sin A
sin A bN B
=
AC bBN · sin A
sin A bN C
MB
= bBN = A
. (do A bC N )
MC
Ta kết luận rằng M thuộc đường tròn Apollonius ứng với đỉnh A của 4ABC.

Bài 6. Cho tam giác ABC nhọn nội tiếp (O) có L là giao điểm 2 tiếp tuyến
tại B và C của (O). Gọi X là điểm đối xứng với A qua BC và K là giao điểm
của LX và tiếp tuyến tại A của (O). Chứng minh K thuộc đường thẳng Euler
của tam giác ABC.
Lời giải.
Gọi giao điểm của OK với AX là J, ta sẽ chứng minh J là trực tâm của
4ABC. Ta hoàn toàn có thể đưa điều cần chứng minh này về chứng minh
hai tỉ lệ đoạn thẳng bằng nhau qua kết quả 5. Gọi giao điểm của OL với
AK là I, theo định lý Thales ta có:
JA KJ JX
= = .
OI KO OL

Hướng tới kỷ niệm 20 năm thành lập trường PTNK, ĐHQG TP HCM
104 Chuyên đề Toán học số 10

JA OI
và từ đây suy ra được = .
JX OL
Chuyên đề Toán học Phổ thông năng khiếu TP.HCM số 10

O
H

M
B D C
K P

Gọi H là trực tâm của tam giác ABC và P là giao của AH và (O) thì do tính
HA JA
đối xứng nên AP = H X . Ta cần chứng minh = , cũng tương đương
HX JX
với:
AH OI
= . (1)
AP OL

Như vậy, từ điều cần chứng minh ban đầu là K thuộc đường thẳng Euler
của 4ABC, bằng việc khéo léo xác định điểm J và tính toán qua các bước
trung gian, ta đã đưa được bài toán về việc chứng minh hệ thức (1), một
hệ thức không liên quan gì đến K, J mà chỉ phụ thuộc các yếu tố rất dễ
Ô = 90◦ − OAH
chịu là AD, AP, OL, OI. Từ đây chú ý thêm OIA Õ = AC Õ P = α,
hướng giải quyết của ta đã rõ ràng hơn, ta có:

OA OC OI cos Õ
BAC
OI = , OL = , = . (2)
sin α cos Õ
BAC OL sin α
Mặt khác:

AH cos Õ
BAC
AH = 2R cos Õ
BAC, AP = 2R sin α, = . (3)
AP sin α

Hướng tới kỷ niệm 20 năm thành lập trường PTNK, ĐHQG TP HCM
Chuyên đề Toán học số 10 105

HA JA
Từ (2) và (3) suy ra hệ thức ở (1) đúng, do vậy = . Điều này
HX JX
Chuyên đề Toán học Phổ thông năng khiếu TP.HCM số 10

có nghĩa là H trùng J, suy ra K thuộc đường thẳng Euler của tam giác
ABC.

2. Một số định lí hình học cổ điển hay sử dụng


Ngoài những kết quả đã trình bày ở phần 2 thì ta cũng không thể bỏ qua
những tính chất và định lí quan trọng liên quan đến tính toán, chẳng hạn
như định lí Thales, tam giác đồng dạng, hệ thức lượng trong tam giác,
định lí Pythagores,... Sau đây, người viết xin giới thiệu một số định lí hình
học nâng cao hơn và rất hữu ích khi kết hợp để tính toán với tỉ số cạnh-
lượng giác. Thực chất, những định lí sau đây đều có thể được chứng minh
trực tiếp từ các hệ thức ở phần 2, và xin dành lại cho bạn đọc.

MỘT SỐ ĐỊNH LÍ CỔ ĐIỂN HAY SỬ DỤNG

1. (Định lí Menelaus) Cho 4ABC và các điểm D, E, F lần lượt


thuộc các đường thẳng BC, CA, AB sao cho có một số chẵn điểm
thuộc cạnh của 4ABC. Khi đó D, E, F thẳng hàng khi và chỉ khi:

DB EC FA
· · = 1.
DC EA F B

2. (Định lí Ceva) Cho 4ABC và các điểm D, E, F lần lượt thuộc các
đường thẳng BC, CA, AB sao cho có một số lẻ điểm thuộc cạnh
của 4ABC. Khi đó D, E, F thẳng hàng khi và chỉ khi:

DB EC FA
· · = 1.
DC EA F B

3. (Định lí Ceva-sin) Trong 4ABC, các tia Ax, B y, Cz đồng quy khi
và chỉ khi:
xAC sin Ô
sin Õ y BA sin zC
Ô B
· · .
sin xÔ y BC sin zCA
AB sin Õ Ô

4. (Định lí Ptolemy) Với mọi tứ giác ABC D nội tiếp, ta có hệ thức:

AB · C D + AD · BC = AC · BD.

Từ định lí Ceva-sin và hệ thức 4 ở phần 2, ta có một hệ quả quan trọng sau


đây.

Hướng tới kỷ niệm 20 năm thành lập trường PTNK, ĐHQG TP HCM
106 Chuyên đề Toán học số 10

Hệ quả 2.1. Cho tam giác ABC nội tiếp (O) và các điểm X , Y, Z thuộc (O).
Chuyên đề Toán học Phổ thông năng khiếu TP.HCM số 10

Khi đó AX , BY, C Z đồng quy khi và chỉ khi:

X B Y C ZA
· · = 1.
X C YA ZB

Bài 7. Đường tròn nội tiếp (I) của 4ABC tiếp xúc với các cạnh BC, CA, AB
lần lượt tại D, E, F . Các điểm X , Y, Z thuộc (I). Chứng minh rằng AX , BY, C Z
đồng quy khi và chỉ khi DX , BY, C Z đồng quy.

B D C

Lời giải.
Hướng đi tự nhiên nhất của ta là sử dụng định lí Ceva-sin. Cụ thể, ta sẽ
cần chứng minh hai hệ thức sau tương đương nhau:

Õ sin A
sin CAX Õ BY sin BC
Õ Z
= 1.



 · ·
 sin Õ
B AX sin C
Õ BY sin AC
Õ Z

sin Ö
F DX sin Ö
D EY sin E FY

 Õ
· · = 1.



sin EÕDX sin FÕ
EY sin ÖDF Y

Tính đối xứng của ba điểm A, B, C cũng như D, E, F giúp ta không phải
tính toán quá nhiều. Chú ý rằng EÕFX = ÕX EA và X
Õ EF = ÕX FA, áp dụng

Hướng tới kỷ niệm 20 năm thành lập trường PTNK, ĐHQG TP HCM
Chuyên đề Toán học số 10 107

định lí Ceva-sin cho 4AE F , ta có:


Chuyên đề Toán học Phổ thông năng khiếu TP.HCM số 10

sin CAX
Õ sin E
Õ F X sin Õ
X EA
= ·
sin Õ
B AX sin Õ
X FA sin XÕ EF

sin E F X
=
sin2 X
Õ EF
2 Õ
sin E DX
= .
sin2 Ö
F DX
Đến đây tính toán tương tự đối với hai đỉnh B, C, ta suy ra hệ thức:
 2
sin CAX
Õ sin A Õ BY sin BC
Õ Z sin Ö
F DX sin Ö
D EY sin E
Õ FY
· · = · · .
sin Õ
B AX sin C
Õ BY sin AC
Õ Z sin EÕDX sin FÕ
EY sin ÖDF Y

Từ hệ thức trên ta suy ra ngay điều phải chứng minh.

Bài 8 (#8 PTNK 2013). Cho 4ABC nhọn có trực tâm H và điểm P di chuyển
bên trong tam giác sao cho BH
Ö C = BP
Õ C. Đường thẳng qua B vuông góc AB
cắt P C tại M , đường thẳng qua C vuông góc AC cắt P B tại N . Chứng minh
trung điểm I của M N luôn thuộc một đường thẳng cố định.

A
N

P
H
I

B C

M
K

Lời giải.
Gọi (O) là đường tròn ngoại tiếp 4ABC và AD là đường kính của (O). Gọi

Hướng tới kỷ niệm 20 năm thành lập trường PTNK, ĐHQG TP HCM
108 Chuyên đề Toán học số 10

K là hình chiếu của B lên DC và L là hình chiếu của C lên DB, ta sẽ chứng
C = BDC
Õ = 180◦ − Õ
Chuyên đề Toán học Phổ thông năng khiếu TP.HCM số 10

minh I thuộc K L cố định. Chú ý là BP Õ BAC, cho nên


bốn điểm P, N , M , D đồng viên. Để chứng minh K L đi qua trung điểm M N ,
ta sẽ áp dụng định lí Menelaus cho 4DE F để đưa bài toán về chứng minh:
K D LM I N
· · = 1,
K N LD I M
hay
KD LD
= . (1)
KN LM
Ta có: 
KD BD · sin K
Õ BD
=


 .
 KN

BN · sin N
Ö BK
(∗)
LD C D · sin LC D

 Õ
= .


LM

C M · sin M
Ö CL
Mặt khác, áp dụng tính chất P N M D và BK LC nội tiếp, ta có N Ö BK =
90 − K N B = 90 − C M L = M C L và K BD = LC D. Từ đây, kết hợp với (∗)
◦ Ö ◦ Ö Ö Õ Õ
ta có thể đưa (1) về chứng minh hệ thức sau:
BD CD
= .
BN CM
Chú ý đến hai tam giác BDN và C DM , ta có:

BD sin BÖND
=


 .
 BM

sin BÖDN

CD sin ×
CMB


= .



CM sin C
× DM
BD CD
Do B
Ö ND = ×
C M B và B
Ö DN + C
× DM = 180◦ nên ta suy ra = . Kết
BM CM
luận, hệ thức (1) đúng, nên theo định lí Menelaus, I thuộc K L cố định.
Nhận xét. Định lý Menelaus là một phương pháp rất hiệu quả có thể giúp
chuyển nhiều bài toán cần chứng minh một đường thẳng đi qua trung
điểm một cạnh nào đó về việc chứng minh hai tỉ số cạnh bằng nhau.
Bài 9. Cho tam giác ABC nhọn có ba đường cao AD, BE, C F .
a) Chứng minh rằng ba đường thẳng Euler của 4AE F, 4BDF, 4C DE
đồng quy tại một điểm P.

Hướng tới kỷ niệm 20 năm thành lập trường PTNK, ĐHQG TP HCM
Chuyên đề Toán học số 10 109

b) Chứng minh rằng trong ba đoạn thẳng P D, P E, P F có môt đoạn có độ


Chuyên đề Toán học Phổ thông năng khiếu TP.HCM số 10

dài bằng tổng độ dài hai đoạn còn lại.


A

Ha
E
P
Oa

Hc
O
F
H
Ob
Oc

Hb
B D M
C

Lời giải.
Giả sử các điểm như hình vẽ và AB ≤ AC, những trường hợp khác ta chứng
minh tương tự.
a) Gọi da , d b , dc lần lượt là đường thẳng Euler của 4AE F, 4BDF, 4C DE.
Gọi O, Oa , Ob , Oc lần lượt là tâm đường tròn ngoại tiếp 4ABC, 4AE F,
4BDF, 4C DE. Ta có Oa là trung điểm AH, Ob là trung điểm BH, Oc là
trung điểm C H. Gọi H, H a , H b , H c lần lượt là trực tâm 4ABC, 4AE F,
4BDF, 4C DE và P là giao điểm của Ob H b với Oc H c .
Dễ thấy 4DBF ∼ 4DEC nên ta cũng có 4DOb H b ∼ 4DOc H c (tính
chất của phép đồng dạng). Từ đây suy ra Ú DOb H b = Ù
DOc H c . Như
vậy POb DOc nội tiếp, hay P thuộc đường tròn Euler của 4ABC. Vậy
d b và dc cắt nhau trên đường tròn Euler của 4ABC (giờ ta gọi là
đường tròn (ω)). Lí luận tương tự ta cũng có da , d b , (ω) đồng quy và
da , dc , (ω) đồng quy.
Đến đây ta vẫn chưa thể kết luận ba đường thẳng da , d b , dc đồng quy
(vì vẫn có khả năng chúng tạo thành một tam giác có ba đỉnh đều
thuộc (ω)). Bây giờ để ý rằng Oc ∈ (ω) nên P là giao điểm thứ hai
của Oc H c với (ω). Do vậy cả Ob H b và Oa H a đều cắt Oc H c tại giao
điểm thứ hai của Oc H c với (ω), cho nên ta suy ra ba đường thẳng
Euler của 4AE F, 4BDF, 4C DE đồng quy tại điểm P ∈ (ω).

b) Giả sử P thuộc cung E F không chứa D của (ω) như hình vẽ, ta sẽ
chứng minh P D = P E + P F .

Hướng tới kỷ niệm 20 năm thành lập trường PTNK, ĐHQG TP HCM
110 Chuyên đề Toán học số 10

BC
Gọi M là trung điểm BC thì M E = M F = . theo định lý Ptolemy
2
Chuyên đề Toán học Phổ thông năng khiếu TP.HCM số 10

ta có:

P M · E F = M F · P E + M E · P F = M E(P E + P F ).

Như vậy ta cần chứng minh:

PM · EF = M E · PD (∗)

Ta sẽ chứng minh 4AHO ∼ 4P DM .


Ta có 4ABC ∼ 4DEC nên 4AOH ∼ 4DOc H c , suy ra:
Õ=Ù
AOH DOc H c = ×
DM P (1)

Tiếp tục, ta có:

MPD = ×
× MFD
=M
Ö FB − D
Õ FB
= ABC
Õ − AC
Õ B (2)


Õ =Õ
HAO BAC − B
Õ AH − CAO
Õ

BAC − 2(90◦ − ABC)
Õ
= ABC
Õ − AC
Õ B (3)

Từ (2), (3) ta suy ra HAO


Õ =×D P M . Đến đây kết hợp với (1) thì có
được 4AHO ∼ 4P DM . Cho nên:
PD AH
=
PM AO
= 2 cos Õ
BAC
2E F
=
BC
EF
=
ME
Hệ thức trên tương đương với (∗). Vậy P D = P E + P F .

Nhận xét. Định lý Ptolemy rất hiệu quả trong việc xử lí các biểu thức liên
quan đến tổng đoạn thẳng trên đường tròn.

Hướng tới kỷ niệm 20 năm thành lập trường PTNK, ĐHQG TP HCM
Chuyên đề Toán học số 10 111

3. Các bài toán nâng cao


Chuyên đề Toán học Phổ thông năng khiếu TP.HCM số 10

3.1. Các bài toán có lời giải


Ở phần cuối cùng này, chúng ta sẽ cùng đến với những bài toán khó, ngoài
khả năng vận dụng các hệ thức và kết quả ở hai phần trên thì còn đòi hỏi
ở người giải khả năng phân tích và quan sát hình học ở một mức độ cao
hơn. Ngoài ra, cũng cần phải lưu ý là không nên quá lạm dụng tính toán
dựa trên tỉ số cạnh-lượng giác trong hình học. Đôi khi, tính toán chỉ đóng
vai trò dứt điểm bài toán nhanh hơn, còn các mấu chốt của bài toán thì
chỉ có thể được xử lí bằng công cụ hình học thuần túy hoặc đôi khi là kẻ
thêm yếu tố phụ.

Bài 1. Cho tam giác ABC có ba đường cao AD, BE, C F . Gọi I, I a , I b , I c lần
lượt là tâm đường tròn nội tiếp của 4ABC, 4AE F, 4BDF, 4C DE. Chứng
minh rằng I là trực tâm 4I a I b I c .

Ia
E
N

I
F
Ic

Ib

D M C
B

Lời giải.
Gọi M , N , P lần lượt là hình chiếu của I lên BC, CA, AB. Ta sẽ chứng minh
kết quả sau:
*I a , I b , I c lần lượt là trực tâm 4AN P, 4BP M , 4C M N :
Do tính đối xứng của hình vẽ, ta chỉ cần chứng minh đại diện I c là trực

Hướng tới kỷ niệm 20 năm thành lập trường PTNK, ĐHQG TP HCM
112 Chuyên đề Toán học số 10

tâm 4C M N , mà do 4C M N cân tại C và I c nằm trên đường phân giác góc


Chuyên đề Toán học Phổ thông năng khiếu TP.HCM số 10

C (cũng là đường cao từ C) nên ta chỉ cần chứng minh M I c ⊥ AC.


Để ý rằng nếu điều đó xảy ra thì ta sẽ có C × M I c = 90◦ − AC
Õ M C Ic =
B, và do ×
AC B
Õ
nên ba góc của 4M C I c hoàn toàn phụ thuộc vào AC Õ B. Nhưng đây là
2
điều ta chưa có và cần chứng minh. Quan sát thấy trong 4M C I c , các yếu
tố cạnh C I c , C M và góc xen giữa ×
M C I c là tương đối "dễ chịu", nên đến đây
ta có thể nghĩ đến ý tưởng chứng minh 4M C I c đồng dạng với một tam
giác có ba góc tương ứng theo trường hợp cạnh-góc-cạnh. Trong hình vẽ
ta khó có thể tìm được một tam giác như thế, nhưng ta có thể tạo ra một
AC
Õ B AC
Õ B
tam giác ảo X Y Z lần lượt có Xb = b90◦ − AC Õ B, Yb = b = 90◦ +
,Z và
2 2
chứng minh 4M C I c ∼ 4X Y Z. Ta có:
 
◦AC
Õ B AC B
sin 90 −
Õ
2 cos
XY sin Zb 2 . (1)
= = =
YZ sin Xb sin(90◦ − AC
Õ B) cos AC
Õ B
Mặt khác, chú ý rằng 4C DE ∼ 4CAB và có tỉ số đồng dạng là cos AC Õ B, ta
có:
MC AC
Õ B
cos
MC CI 2 . (2)
= =
C Ic C Ic cos AC
Õ B
CI
Từ (1), (2) ta suy ra 4M C I c ∼ 4X Y Z (c.g.c), do đó có luôn M I c ⊥ AC.
Vậy I a , I b , I c lần lượt là trực tâm 4AN P, 4BP M , 4C M N .
Hơn nữa, M I c ⊥ AC và M I b ⊥ AB nên:

b M I c = 180 − BAC = 180 − ( I b DB + I c DC) = I b DI c .


◦ ◦
IÖ Õ Ö Ö Ö

Vậy I b , D, M , I c đồng viên. Tiếp tục:


I c I b = 180◦ − M
IÕ × Ic I b − ×
M Ic C
= 180◦ − Ö
I b DB − ×
M Ic C
BAC
Õ AC
Õ B
= 180◦ − − (90◦ + )
2 2
ABC
Õ
= . (3)
2
Chứng minh tương tự, ta cũng sẽ có được
BAC
Õ AC
Õ B
I b Ia =
IÕ và I b Ic =
IÕ . (4)
2 2

Hướng tới kỷ niệm 20 năm thành lập trường PTNK, ĐHQG TP HCM
Chuyên đề Toán học số 10 113

Từ (3), (4) ta suy ra I I c ⊥ I a I b . Chứng minh tương tự ta cũng có I I a ⊥ I b I c .


Chuyên đề Toán học Phổ thông năng khiếu TP.HCM số 10

Vậy I là trực tâm 4I a I b I c .

Bài 2 (Đề kiểm tra đội tuyển IMO Việt Nam 2014). Cho tam giác ABC
nhọn nội tiếp (O) có H là trực tâm. E F là một dây cung của (O) song song
với BC sao cho E và B nằm cùng phía đối với trung trực BC. Đường thẳng
qua O song song AF cắt AB tại G. Gọi D là trung điểm H E, chứng minh
C
Ö DG = 90◦ .

G
K

N
M
S
L O
H

D
C
B

F
E

Lời giải.
Không mất tính tổng quát, giả sử các điểm như hình vẽ. Gọi giao điểm của
C H với AB và (O) lần lượt là L và N . Ta sẽ chứng minh C, L, G, D đồng
viên.
Kẻ đường kính C K của (O). Gọi M là trung điểm AB thì M cũng là trung
điểm H K nên M D song song và bằng nửa K E. Gọi S là giao điểm của K E
và C N . Ta có OM song song C S và M D song song K E nên OM × D=C Õ LE
suy ra D M B = N C E. Đến đây để ý rằng C, L, G, D đồng viên khi và chỉ khi
× Ö

Hướng tới kỷ niệm 20 năm thành lập trường PTNK, ĐHQG TP HCM
114 Chuyên đề Toán học số 10

× = DC
DGM ÕL, và lại có
Chuyên đề Toán học Phổ thông năng khiếu TP.HCM số 10

× + DM
DGM × G=×
DM B
=N
Ö CE
= DC
ÕL + DC
ÕE,

×=
nên ta nghĩ ngay đến kết quả 2 ở phần 2. Cụ thể, để chứng minh DGM
ÕL, ta sẽ chứng minh
DC

sin DGM
× sin DC
ÕL
= .
sin DM
× G sin DC
ÕE
hay tương đương với
MD CE
= . (1)
MG CH
Ta có GO song song AF và OM song song C N nên GOM × bằng với góc nhọn
tao bởi 2 dây cung AF và C N . Gọi góc này là α thì
1 _ _
α = (AN +sđ C F )
2
1 _ _
= (sđ BK +sđ BE) (do E F C B là hình thang cân nên sđ(BE) = sđ(C F ))
2
1 _ _ _ _
= (sđ AN +sđ BE) (OC
Õ B=N ÕCA vì cùng phụ với ÕBAC nên sđ BK= sđ AN )
2
= KC
Õ E.

Đến đây suy ra suy ra hai tam giác vuông OM G và C EK đồng dạng, và dễ
thấy là K E = 2M D cũng như C H = 2M O, cho nên:
MG MO
=
KE CE
MG CH
⇒ =
2M D 2C E
MG CH
⇒ = .
MD CE
× = DC
Vậy (1) đúng, có nghĩa là DGM ÕL, suy ra C, L, G, D đồng viên. Tóm
lại, C
Ö DG = 90◦ .
Bài 3. Cho tam giác ABC nhọn khác tam giác đều, đường tròn nội tiếp (I)
tiếp xúc các cạnh BC, CA, AB lần lượt tại D, E, F . Gọi H là trực tâm tam giác
DE F . Đường thẳng qua H vuông góc I H cắt BC, CA, AB lần lượt tại X , Y, Z.
Gọi A0 là điểm đối xứng của X qua D, B 0 đối xứng với Y qua E, C 0 đối xứng
với Z qua F . Chứng minh rằng A0 , B 0 , C 0 cùng thuộc 1 tiếp tuyến của (I).

Hướng tới kỷ niệm 20 năm thành lập trường PTNK, ĐHQG TP HCM
Chuyên đề Toán học số 10 115

C0
Chuyên đề Toán học Phổ thông năng khiếu TP.HCM số 10

Y
E

F B0
H

C
B X D A0

Lời giải.
Do vai trò như nhau của A0 , B 0 , C 0 nên ta chỉ cần chứng minh A0 B 0 tiếp xúc
(I). Ta đã có B 0 Y và A0 X là 2 tiếp tuyến của (I), do đó B 0 A0 là tiếp tuyến
của (I) cũng tương đương với:

E B0 I = A0 B 0 I.
(∗)
Õ Ö
IÕA0 D = B
Ö 0 A0 I.

Do X Y vuông góc I H nên dễ thấy I EY H và I DX H là các tứ giác nội tiếp. Từ


YE =Õ
đây ta có IÔ I H E, mà B 0 và Y đối xứng qua E nên E Õ B 0 I = IÕ
Y B0 = Õ I H E.
Hoàn toàn tương tự ta cũng có I A D = I X D = I H D.
Õ 0 Õ Õ
Đến đây đây ta lại thấy bài toán có nét giống với bài toán 4.2 ở trên,
vì nếu chứng minh được B Ö 0 A0 I + ÖI B 0 A0 = IÕA0 D + E
Õ B 0 I thì ta lại có thể áp
sin BÖ 0 A0 I A0 D
sin IÕ
dụng kết quả 2 phần 2, đưa (*) về chứng minh = . Ta
sin ÖI B 0 A0 sin EÕB0 I
sin BÖ 0 A0 I I B0 IY
có thể khai thác từ kết quả = 0 = và đưa về chứng minh
sin ÖI B 0 A0 IA IX

Hướng tới kỷ niệm 20 năm thành lập trường PTNK, ĐHQG TP HCM
116 Chuyên đề Toán học số 10

IY A0 D
sin IÕ
= , hay tương đương với
Chuyên đề Toán học Phổ thông năng khiếu TP.HCM số 10

IX sin EÕB0 I

IY sin IÕHD
= . (∗∗)
IX sin ÕIHE

Như vậy, việc cần làm lúc này là chứng minh B


Ö 0 A0 I + Ö
I B 0 A0 = IÕ
A0 D + E
Õ B 0 I,
cũng tương đương với việc chứng minh

A0 D + E
IÕ Õ B0 I + A
Ö0 I B 0 = 180◦ .

Với các dữ kiện vuông góc và tứ giác nội tiếp cùng với tính đối xứng thì
đây là một biểu thức không quá khó khăn. Ta có:

A
Ö0 I B0 = E
Ô ID − E
Õ I B0 − D
Õ IA0
=E
Ô ID − E
Ô IY − Õ
DI X
=E
Ô ID − Ö
EHY − D
Ö HX
=E
Ô I D − (180◦ − Ö
D H E)
= 2D
Õ FE − D
Õ FE
=D
Õ F E. (1)

Mặt khác,

A0 D + E
IÕ Õ B0 I = Õ
I X D + IÔ
YE
= IÕ
HD + Õ
IHE

D H E. (2)

Kết hợp (1) và (2) ta có ngay IÕ A0 D + E Õ B0 I + A


Ö0 I B0 = D
Õ FE + Ö
D H E = 180◦ ,
từ đó suy ra ta chỉ cần chứng minh (**). Điều này hiển nhiên vì I Y =
H D và I X = I D. sin Õ
I E. sin IÕ I H E mà I D = I E. Như vậy ta đã chứng minh
được (*), suy ra A0 B 0 tiếp xúc (I). Lập luận tương tự ta cũng có B 0 C 0 và
A0 C 0 tiếp xúc (I). Giả sử 3 điểm A0 , B 0 , C 0 không thẳng hàng, từ A0 ta đã có
2 tiếp tuyến tới (I) là A0 B 0 và A0 D, suy ra C 0 thuộc A0 D, tương tự suy ra C 0
thuộc B 0 E, cho nên C 0 trùng C (vô lí vì C 0 thuộc AB). Vậy A0 , B 0 , C 0 cùng
thuộc 1 tiếp tuyến của (I).

Nhận xét. Ta biết rằng một tam giác bất kì sẽ được xác định một cách duy
nhất nếu ta biết được độ dài hai cạnh và góc xen giữa chúng. Qua hai bài
toán trên, ta thấy ở 4M DG trong bài toán 4.2 và 4IA0 B 0 trong bài toán

Hướng tới kỷ niệm 20 năm thành lập trường PTNK, ĐHQG TP HCM
Chuyên đề Toán học số 10 117

4.3, ta đều có thể tính được một góc ( DM


× G và A
Ö0 I B 0 ), và 2 cạnh tạo thành
Chuyên đề Toán học Phổ thông năng khiếu TP.HCM số 10

góc đó đều liên hệ được với các yếu tố của giả thiết, sử dụng tương quan
giữa tỉ số cạnh và tỉ số lượng giác, ta đã tính được cả hai góc còn lại.

Bài 4 (#5 Vietnam TST 2009). Cho đường tròn (O) đường kính AB, M là
một điểm nằm trong (O). Phân giác trong A
bM B cắt (O) tại N (N khác phía
M đối với AB). Phân giác ngoài A
bM B cắt N A, N B lần lượt tại P, Q. AM cắt
đường tròn đường kính NQ tại điểm thứ 2R. BM cắt đường tròn đường kính
N P tại điểm thứ 2S. Chứng minh trung tuyến kẻ từ N của tam giác N SR
luôn đi qua một điểm cố định.

S
Q
M
P

L
A B
O

Lời giải.
Qua hình vẽ ta có thể xác định được ngay điểm cố định chính là O. Thay
vì chứng minh trực tiếp trung tuyến kẻ từ N của 4N SR đi qua O, ta có thể
chứng minh N O đi qua trung điểm của SR. Nhưng cách xác định điểm S
và R gây khá nhiều khó khăn và ta gần như không thể tìm được mối liên
hệ nào giữa trung điểm SR với các yếu tố còn lại. Để giải quyết khó khăn
này, ta nên chuyển hướng sang chứng minh

sin RN
Ö O NS
= , (∗)
sin SN
Ö O NR

Ö = RMQ,
một hệ thức không liên quan gì đến trung điểm SR. Ta có BMQ Ö
Ö = RNQ
mà RMQ Õ do R, Q, N , M cùng thuộc đường tròn đường kính NQ,

Hướng tới kỷ niệm 20 năm thành lập trường PTNK, ĐHQG TP HCM
118 Chuyên đề Toán học số 10

Ö = RNQ.
nên BMQ Õ Hoàn toàn tương tự, ta cũng có A
Ö MP = P
Õ N S, mà
M P = RMQ,
Chuyên đề Toán học Phổ thông năng khiếu TP.HCM số 10

A
Ö Ö cho nên ta suy ra được

Ö = BMQ
RMQ Ö = RNQ
Õ=P NS = Ö
Õ PMA = P
Ö M S = α.

Mặt khác,
RN
Ö O = BN
Ö O−B
Õ N R = OBN
Ö − α. (1)
Đến đây để ý rằng: M
× BN = BMQ
Ö + BQM Ö+Ö
Ö hay OBN M BA = α + BQM
Ö,
cho nên
Ö − α = BQM
OBN Ö −ÖM BA. (2)
Từ (1), (2), suy ra
RN
Ö O = BQM
Ö −ÖM BA. (3)
Đến đây, bài toán vẫn chưa tiến triển nhiều, ta phải tìm cách tính RN
Ö O
tường minh hơn nữa bằng cách vẽ thêm yếu tố phụ. Từ hệ thức (3) ta thấy
M LA = MQB
rằng nếu trên đoạn AB ta lấy điểm L sao cho Ö Ö thì ta sẽ có

B
Ö ML = Ö M BA = BQM
M LA − Ö Ö −ÖM BA = RN
Ö O. (∗∗)

Hơn nữa, với cách dựng điểm L như thế thì tứ giác MQB L nội tiếp. Đến
đây để ý rằng ÖM LA = MQB
Ö =× M N A, ta có thêm tính chất M , L, N , A đồng
viên. Tiếp tục, ta có: Ta có

SN
Õ R=A
ÕNB − P
Õ Õ = 90◦ − 2α.
N S − RNQ

NB = A
LÕ ÕNB − A
ÕNL
= 90◦ − (180◦ − Ö
L M A)
= 90◦ − ( Ö
P M A + BMQ
Ö +BÖM L)
= 90◦ − 2α − B
Ö ML
= SN
Õ R−B
Ö ML
= SN
Õ R − RN
Ö O (từ (**) ta có B
Ö M L = RN
Ö O)
= SN
Ö O.

N B = SN
Vậy ta có LÕ Ö LQB =
O. Đến đây do tứ giác MQB L nội tiếp ta có Õ
B M L = RN O. Lúc này, chú ý đến tam giác LNQ, ta có:
Ö Ö

sin RN
Ö O sin Õ
LQB LN
= = .
sin SN
Ö O sin LÕ NB LQ

Hướng tới kỷ niệm 20 năm thành lập trường PTNK, ĐHQG TP HCM
Chuyên đề Toán học số 10 119

LN NS
Theo (*), ta chỉ cần chứng minh = nữa là xong. Mà do 2 tam giác
LQ NR
Chuyên đề Toán học Phổ thông năng khiếu TP.HCM số 10

NS NP
vuông PSN và QRN đồng dạng (cùng có một góc nhọn α) nên = .
N R NQ
NP MN
Điểm P không có nhiều liên hệ với L, ta sẽ thay bởi . Vậy ta đưa
NQ MQ
hệ thức (*) ban đầu về chứng minh

LN MN
= . (∗ ∗ ∗)
LQ MQ

Ta hoàn toàn có thể làm cho hệ thức này đơn giản hơn nữa bằng cách lợi
dụng hai tứ giác nội tiếp MQB L và AM LN . Áp dụng kết quả 4, phần 2 cho
đường tròn (AM LN ):

LN sin B
Õ AN
= . (4)
MN sin ×
M AN
Và cũng tương tự cho đường tròn (M LBQ):

LQ sin Õ
LBQ sin A
ÕBN
= = . (5)
MQ sin MÖ BQ sin M× BN

Từ (*), (***), (4), (5), ta suy ra bài toán sẽ được giải quyết nếu ta chứng
minh được
sin B
Õ AN sin A
ÕBN
= .
sin ×
M AN sin M
× BN
hay tương đương với
NB sin ×
M AN
= ,
N A sin M× BN
một hệ thức chỉ phụ thuộc vào các điểm M , N , A, B ban đầu. Những thao
tác còn lại tương đối đơn giản, ta có:

NA MN


 = .
 sin A
× MN sin ×
M AN
(6)
 NB MN
= .



sin B
× MN sin M
× BN

Từ (4),(5),(6), kết hợp với A


× MN = B
× M N , ta suy ra hai hệ thức ở (***) và
(*) đúng, suy ra trung tuyến từ N của 4N SR đi qua O cố định.

Hướng tới kỷ niệm 20 năm thành lập trường PTNK, ĐHQG TP HCM
120 Chuyên đề Toán học số 10

Bài 5. Cho tam giác ABC nội tiếp (O) có ba đường cao AD, BE, C F . E F cắt
lại (O) tại A1 , A2 . Tương tự xác định các điểm B1 , B2 , C1 , C2 . Chứng minh
Chuyên đề Toán học Phổ thông năng khiếu TP.HCM số 10

rằng tâm đẳng phương của ba đường tròn (DA1 A2 ), (EB1 B2 ), (F C1 C2 ) chính
là trực tâm 4ABC.

C2 L
A2
A

B2
E

F S

H
A1

B D C

C1
B1

Lời giải.
Ta chỉ cần chứng minh H thuộc trục đẳng phương của (F C1 C2 ) và (EB1 B2 )
rồi áp dụng chứng minh tương tự cho các cặp đường tròn còn lại.
Để ý rằng ta có DB1 · DB2 = DC1 · DC2 nên D thuộc trục đẳng phương của
(F C1 C2 ) và (EB1 B2 ). Như vậy thì H thuộc trục đẳng phương của (F C1 C2 )
và (EB1 B2 ) khi và chỉ khi DH là trục đẳng phương của hai đường tròn đó.
Để chứng minh điều này ta sẽ tìm một điểm khác D trên đường thẳng DH
và chứng minh điểm đó có cùng phương tích với (F C1 C2 ) và (EB1 B2 ). Ta
chọn điểm S là giao của E F và AD.
Gọi K, L lần lượt là giao điểm của E F với (EB1 B2 ) và (F C1 C2 ). Ta cần
chứng minh:
SE · SK = SF · S L,
hay cũng tương đương với
EL FK
= . (1)
ES FS

Hướng tới kỷ niệm 20 năm thành lập trường PTNK, ĐHQG TP HCM
Chuyên đề Toán học số 10 121

Ta có:
Chuyên đề Toán học Phổ thông năng khiếu TP.HCM số 10

FA · F B = F B1 · F B2 = F E · F K.
§
EA · EC = EC1 · EC2 = E F · E L.
Từ hai hệ thức trên suy ra K, B, E, A đồng viên và L, C, F, A đồng viên. Do
đó:
4BK F ∼ 4EAF ∼ 4BDF.
Từ đây suy ra 4BK F = 4BDF (hai tam giác đồng dạng có một cạnh tương
ứng chung). Hoàn toàn tương tự ta cũng có được 4C LE = 4C DE. Vậy:
F K = F D.
§
E L = E D.

Do đó, kết hợp với dữ kiện DS là phân giác của E


Õ DF , ta có thể suy ra
EL DE SE
= = . (2)
FK DF SF
Từ (1), (2) suy ra SE · SK = SF · S L, hay S thuộc trục đẳng phương của
(F C1 C2 ) và (EB1 B2 ). Vậy DH là trục đẳng phương của (F C1 C2 ) và (EB1 B2 ).
Chứng minh tương tự, ta cũng sẽ có EH, F H lần lượt là các trục đẳng
phương của các cặp đường tròn (DA1 A2 ) và (F C1 C2 ), (DA1 A2 ) và (EB1 B2 ).
Ta suy ra điều cần chứng minh.
Bài 6 (G8 IMO Shortlist 2009). Cho tứ giác ngoại tiếp ABC D. Một đường
thẳng g qua A cắt đoạn BC và đường thẳng C D lần lượt tại M , N . Gọi
I1 , I2 , I3 lần lượt là tâm đường tròn nội tiếp 4ABM , 4C M N , 4ADN . Chứng
minh rằng trực tâm 4I1 I2 I3 thuộc đường thẳng g.

B
A
I1

K
M

I3 L I2
N

C
T
S

Hướng tới kỷ niệm 20 năm thành lập trường PTNK, ĐHQG TP HCM
122 Chuyên đề Toán học số 10

Lời giải.
Chuyên đề Toán học Phổ thông năng khiếu TP.HCM số 10

Ta có N C và N A đối xứng nhau qua N I2 , BC và BA đối xứng nhau qua BI1 ,


như vậy ta phát hiện ra ngay g chính là đường thẳng Steiner của điểm C
đối với 4I1 I2 I3 . Do đó, ta chỉ cần chứng minh C thuộc đường tròn ngoại
tiếp 4I1 I2 I3 thì sẽ suy ra ngay trực tâm 4I1 I2 I3 thuộc g. Gần như không
có cách nào liên hệ các góc liên quan đến C của tứ giác I1 I2 C I3 , nên ngay
cả việc chọn ra hợp lí hai góc nào đó để chứng minh chúng bằng nhau thôi
đã là không đơn giản. Thay vì chọn hai góc mà trong đó ta có thể dễ dàng
tính được một góc còn góc còn lại không thể tính được, ta có thể chọn cả
hai góc mà ta đều không tính trực tiếp được, đó là C Ö I1 I2 và C ÖI3 I2 . Lúc này
để ý rằng trong tam giác C M I1 , hai cạnh M C và M I1 ta có thể ít nhiều
khai thác được, còn C × M I1 cũng có thể tính được dễ dàng. Mặt khác, nếu
ta gọi L là tâm bàng tiếp góc N của 4N M C thì ta sẽ có LM I2 C nội tiếp,
C LI3 = C
tức là Ö × M I1 . Vậy để chứng minh C ÖI1 I2 = C
Ö I3 I2 , ta chỉ cần chứng
minh 4C LI3 ∼ 4C M I1 bằng cách chứng minh

LI3 M I1
= .
LC MC

Để cho tiện, đặt M × NC = N


Ò, N
× MC = M
Ò . Gọi S, T lần lượt là hình chiếu của
I3 , L lên C D. Ta có

ST
LI3 =
cos Õ
LN S
ST
= .
N
Ò
cos
2
Mặt khác

ST = N S − N T
N D + N A − AD N M + M C + C N
= −
2 2
M A − M C + C D − AD
= .
2
Suy ra
M A − M C + C D − AD
LI3 = . (1)
N
Ò
2 cos
2
N
Ò
Ò LC = 180◦ − M
Tiếp theo, xét 4LM C, do M Ò I2 C = 90◦ − và LM
Ö C =
2

Hướng tới kỷ niệm 20 năm thành lập trường PTNK, ĐHQG TP HCM
Chuyên đề Toán học số 10 123

M
Ò
90◦ − nên
Chuyên đề Toán học Phổ thông năng khiếu TP.HCM số 10

2
N
Ò
cos
MC sin M LC
Ö 2 . (2)
= =
LC sin LM C
Ö M
Ò
cos
2
Gọi hình chiếu của I1 lên AM là K thì
MK M B + M A − AD
M I1 = = . (3)
cos K
× M I1 M
Ò
2 cos
2
Từ (1), (2), (3) và C D −AD = BC −AD = M B + M C −AD (do tứ giác ABC D
ngoại tiếp), ta suy ra
LI3 .M C M A − M C + C D − AD
= = 1.
M I1 .LC M B + M A − AD

Vậy 4C LI3 ∼ 4C M I1 , suy ra C ÖI1 I2 = C


Ö I3 I2 , do đó C thuộc đường tròn
ngoại tiếp tam giác I1 I2 I3 . Mà điểm đối xứng với C qua I3 I2 thuộc g (do
C
Ö N I2 = Ø M N I2 ) và tương tự điểm đối xứng với C qua I1 I3 cũng thuộc g,
do A Õ BI1 = Ö C BI1 , suy ra g là đường thẳng Steiner của C ứng với tam giác
I1 I2 I3 , nên g đi qua trực tâm 4I1 I2 I3 .
Nhận xét. Ta có thể chứng minh chiều đảo của bài toán trên bằng cách
tương tự.

3.2. Các bài toán rèn luyện


Bài 7. Cho 4ABC vuông tại C. Đường tròn nội tiếp (I) của 4ABC tiếp xúc
AB, AC tại P, Q. Đường cao C H cắt PQ tại N . Gọi E là trung điểm BC và M
là giao điểm của E I và H C. Chứng minh M N song song BI.
Bài 8 (#4 IMO 2010). Cho P là một điểm nằm trong tam giác ABC (CA 6=
C B). Các đường thẳng AP, BP, C P lần lượt cắt đường tròn ngoại tiếp tam giác
ABC (kí hiệu là w) tại K, L, M . Tiếp tuyến tại C của w cắt AB tại S. Chứng
minh rằng nếu SC = SP thì M K = M L.
Bài 9 (#4a Vietnam TST 2014). Cho 4ABC có đường cao AD và P di động
trên AD. Các đường thẳng P C và AB cắt nhau ở F , các đường thẳng P B và
AC cắt nhau ở E. Giả sử tứ giác AE DF nội tiếp, chứng minh rằng:
PA A
= (tan B + tan C) · cot .
PD 2

Hướng tới kỷ niệm 20 năm thành lập trường PTNK, ĐHQG TP HCM
124 Chuyên đề Toán học số 10

Bài 10. Cho 4ABC nội tiếp (O) có B, C cố định, A thay đổi. Gọi AH, AK lần
Chuyên đề Toán học Phổ thông năng khiếu TP.HCM số 10

lượt là đường cao và phân giác trong của 4ABC. Gọi I, M lần lượt là trung
điểm AH, AK. Điểm N thuộc AK sao cho ACÖ M = BC
Ö N . Chứng minh I N luôn
đi qua một điểm cố định.

Bài 11. Cho 4ABC có ba đường cao AD, BE, C F . Đường thẳng DE cắt đường
tròn đường kính C H lần thứ hai tại K, đường thẳng DF cắt đường tròn
đường kính BH lần thứ hai tại L. Chứng minh K L vuông góc với đường
thẳng Euler của 4ABC.

Bài 12. Cho 4ABC nhọn nội tiếp (O). Trên tia BA lấy điểm D sao cho
BD = CA. Trên tia BA, CA lần lượt lấy M , N sao cho BM = C N . Gọi K
là giao điểm của BN và C M , E là giao điểm của K D và AC. Chứng minh
AD = AE.

Bài 13. Cho 4ABC nội tiếp (O) có B, C cố định, A thay đổi trên cung lớn
BC. Gọi BD, C F lần lượt là các phân giác trong và BE, C G lần lượt là các
phân giác ngoài của 4ABC. Gọi M là giao điểm của E F với BD và N là giao
điểm của GD với C F . Chứng minh phân giác góc ×
M AN luôn đi qua một điểm
cố định.

Bài 14. Cho 4ABC nhọn. Dựng ra phía ngoài của 4ABC các tam giác đều
ABF và AC E. Gọi T là giao điểm của BE và C F và S là điểm đẳng giác của
T trong 4ABC. Chứng minh rằng S là giao điểm chung của ba đường tròn
Apollonius trong 4ABC.

Bài 15 (#4 Day 5 Romania TST 2006). Cho 4ABC có đường cao AD. Gọi
(O1 ) là đường tròn tiếp xúc với DA, DB và tiếp xúc trong với (O), (O2 ) là
đường tròn tiếp xúc với DA, DC và tiếp xúc trong với (O). Tiếp tuyến chung
trong khác AD của (O1 ) và (O2 ) cắt BC tại M . Chứng minh M là trung điểm
BC khi và chỉ khi AB + AC = 2BC.

Bài 16 (#4b Trường Đông Toán học miền Nam 2013). Cho tam giác ABC
có AB < BC < CA nội tiếp (O) và ngoại tiếp (I). Trên các tia AB, AC lần lượt
lấy D, E sao cho AD = AE = BC. Đường tròn ngoại tiếp 4ADE cắt (O) tại
F . Dựng hình bình hành BI C J và gọi K là trung điểm cung BC không chứa
A của (O). Chứng minh F, J, K thẳng hàng.

Bài 17 (#3 USA TST 2013). Cho tam giác ABC vuông tại C có D là chân
đường cao hạ từ C. Gọi X là một điểm thuộc đoạn C D. K thuộc đoạn AX sao
cho BK = BC, L thuộc đoạn BX sao cho AL = AC. Đường tròn ngoại tiếp tam
giác DK L cắt AB tại điểm thứ hai T (T khác D). Chứng minh AC
Õ T = BC
Õ T.

Hướng tới kỷ niệm 20 năm thành lập trường PTNK, ĐHQG TP HCM
Chuyên đề Toán học số 10 125

Bài 18 (#2 Vietnam TST 2008). Cho tam giác ABC nội tiếp (O) có phân
Chuyên đề Toán học Phổ thông năng khiếu TP.HCM số 10

giác trong AD, BE, C F . Các điểm K, M , N lần lượt thuộc các đoạn AD, BE, C F
KA MB NC
sao cho = = = k. Kí hiệu (X ) là đường tròn qua K, A và tiếp
KD ME NF
xúc với OA. Các đường tròn (Y ), (Z) xác định tương tự.
1
a) Với k = , chứng minh rằng (X ), (Y ), (Z) có 2 điểm chung và trọng
2
tâm G của tam giác ABC thuộc đường nối 2 điểm chung này.

b) Tìm mọi k sao cho (X ), (Y ), (Z) có 2 điểm chung.

Bài 19 (#3 IMO 2013). Cho tam giác ABC nội tiếp (O) có A1 là tiếp điểm
của đường tròn bàng tiếp góc A lên cạnh BC. B1 , C1 được xác định tương tự.
Chứng minh nếu tâm đường tròn ngoại tiếp tam giác A1 B1 C1 thuộc (O) thì
4ABC vuông.

Bài 20. Cho 4ABC nội tiếp (O) có BC cố định, A thay đổi. Điểm M di
chuyển trên trung trực BC. Gọi I, J lần lượt là tâm đường tròn nội tiếp
4AM B, 4AM C. Chứng minh đường tròn ngoại tiếp 4AI J luôn đi qua một
điểm cố định khi A, M di chuyển.

Hướng tới kỷ niệm 20 năm thành lập trường PTNK, ĐHQG TP HCM
126 Chuyên đề Toán học số 10
Chuyên đề Toán học Phổ thông năng khiếu TP.HCM số 10

Hướng tới kỷ niệm 20 năm thành lập trường PTNK, ĐHQG TP HCM
Chuyên đề Toán học Phổ thông năng khiếu TP.HCM số 10

Giới thiệu về Toán mô hình

Cấn Trần Thành Trung


(Lớp Chuyên Toán khóa 2010 − 2013)
Lê Việt Hải
(Lớp Chuyên Toán khóa 2009 − 2012)

Toán mô hình giống với một công cụ hơn là một môn học, nắm được những ý
tưởng cơ bản của công cụ này các em có thể sử dụng nó vào những lĩnh vực khác
như: vật lý, hóa học, sinh học, hay các vấn đề và hiện tượng xã hội.

1. Giới thiệu
Toán là một ngôn ngữ gọn gàng với những quy tắc và phép tính được định
nghĩa rõ ràng, chặt chẽ, có thể được sử dụng để mô tả các vấn đề thực tế
một cách cụ thể, dễ hình dung hơn. Toán lý thuyết đã được nghiên cứu và
xây dựng cả ngàn năm nay nên cực kì “giàu” và “đẹp,” cung những công
cụ hữu ích để giải quyết các bài toán. Với sự hỗ trợ của máy tính, cái phép
tính toán phức tạp hay một khối lượng lớn các phép tính có thể được xử lí
dễ dàng.

Nhờ áp dụng công cụ Toán học vào giải quyết các vấn đề kinh tế, xã hội,
nhiều nhà khoa học đã đạt những thành tựu đáng kể. Tiêu biểu là hai
giải Nobel dựa trên hai bài viết, “Equilibrium points in n-person games”
của John Nash năm 1994, và “Stable allocations and the practice of market
design” của Lloyd Shapley năm 2012.

Qua bài viết bày, chúng ta sẽ tìm hiểu khái niệm, đặc điểm, tính chất của
Toán mô hình và những ví dụ cụ thể trong thực tế.

127
128 Chuyên đề Toán học số 10
Chuyên đề Toán học Phổ thông năng khiếu TP.HCM số 10

Hình 10.1: “Hệ” bao gồm các đối tượng trong thực tế và mối liên quan
giữa các đối tượng này.

2. Khái niệm
Hiểu một cách tổng quan, Toán Mô Hình là sử dụng các biến, công thức,
mệnh đề, công cụ toán học để mô tả một “hệ” và tìm câu trả lời cho những
câu hỏi, bài toán liên quan đến “hệ” đó.
Các ví dụ về đối tượng (và các mối liên hệ) để mô hình có thể kể đến là

Ví dụ 2.1.

• Người dân trong thành phố (tính tuổi thọ trung bình),

• Các học sinh và mentors của PiMA (sắp xếp lịch phỏng vấn phù hợp),

• Các tài khoản trên Facebook (tìm ra xu hướng sử dụng của giới trẻ),

• Các bãi biển ở Việt Nam (xây dựng tiêu chí để xếp hạng).

Có một câu hỏi được đặt ra: Tại sao không giải quyết thẳng ngay trên “hệ”
đó mà cần phải thông qua mô hình toán của nó?

Ví dụ 2.2. Tính tuổi thọ trung bình chỉ cần thống kê được số người và tuổi
từng người. Mỗi ngườiPđược đại diện bằng một biến x i (tuổi). Như vậy
x
tuổi trung bình là x̄ = n i , với n là tổng số người.

Ví dụ 2.3. Tìm đường đi bằng Google Map. Nếu 2 người khác nhau ở cùng
một vị trí thì sẽ được xem là cùng một điểm trên Google Maps. Các đường
đi sẽ được biểu diễn thành các đường trắng.

Hướng tới kỷ niệm 20 năm thành lập trường PTNK, ĐHQG TP HCM
Chuyên đề Toán học số 10 129
Chuyên đề Toán học Phổ thông năng khiếu TP.HCM số 10

Hình 10.2: Mô hình hóa việc tìm đường.

Nhưng đó vẫn chưa phải là mô hình toán cuối cùng. Đơn giản hơn nữa,
ngôn ngữ mà cả máy tính và con người đều có thể hiểu và làm việc được:
mô hình đồ thị. Như vậy, mỗi địa điểm sẽ được biểu diễn thành một đỉnh
của đồ thị, các đường đi sẽ thành các cạnh của đồ thị.

Hình 10.3: Tối giản hóa bằng ngôn ngữ đồ thị.

Chúng ta sẽ được tìm hiểu về đồ thị và cả thuật toán liên quan Google sử
dụng để tìm đường đi trên Google Maps.

Câu hỏi: Chúng ta đã từng mô hình hóa người bằng 1 điểm chưa? Tại sao
không giải quyết thẳng ngay trên “hệ” đó mà cần phải thông qua mô hình
toán của nó?

Các “hệ” trong thực tế rất phức tạp và có những yếu tố không liên quan
hoặc không quan trọng đến những câu hỏi cần trả lời. Mô hình toán làm
cho các đối tượng trong “hệ” trở nên đơn giản hơn, nhưng vẫn đủ thông
tin và đủ phức tạp để giải quyết các câu hỏi không tầm thường. Ví dụ là ở
bài toán tìm đường ở trên khi chuyển qua ngôn ngữ đồ thị ta không quan
tâm đến hình dạng của đường mà chỉ gán mỗi đường bởi một đường thẳng
có giá trị riêng thể hiện độ dài.

Ví dụ 2.4. (Google Page Rank) Thuật toán xếp hạng các trang web của
Google để tìm kiếm nhanh.

Hướng tới kỷ niệm 20 năm thành lập trường PTNK, ĐHQG TP HCM
130 Chuyên đề Toán học số 10
Chuyên đề Toán học Phổ thông năng khiếu TP.HCM số 10

Hình 10.4: Sơ đồ tóm tắt Toán mô hình.

Ví dụ 2.5. Lên kế hoạch vận chuyển sữa hộp từ các kho đến các đại lý một
cách tối ưu nhất.

Câu hỏi: “Hệ” ở đây là gì? Vấn đề cần giải quyết? Yếu tố quan trọng của
đối tượng mình có thể sử dụng được để trả lời câu hỏi?

Hình 10.5: Google PageRank.

Fun Fact: “PageRank” là kết hợp giữa “Page” (người sáng lập Google, không
phải trang giấy) + “Rank” (xếp hạng).

Ví dụ của Google PageRank

• Thuật toán đầu tiên của Google: dựa trên đường links giữa các
trang web này để tính xem trang web nào là quan trọng nhất.

Hướng tới kỷ niệm 20 năm thành lập trường PTNK, ĐHQG TP HCM
Chuyên đề Toán học số 10 131
Chuyên đề Toán học Phổ thông năng khiếu TP.HCM số 10

• Có hai giả thiết quan trọng:

1) Trang web sẽ càng quan trọng nếu có càng nhiều trang


web dẫn đến nó và các trang web ấy cũng được nhiều
trang web dẫn đến.
2) Một người đang ở một web bất kì sẽ phải ngẫu nhiên đến
một web khác có liên kết từ web này với xác suất như
nhau.

• Biểu diễn mối liên hệ giữa các web bằng đồ thị có hướng (sẽ
được đề cập sau).

• Giải quyết bài toán này dựa trên xác suất: mô hình Markov
Chain (sẽ được đề cập sau)

Liên hệ: Sử dụng mô hình trên cho vấn đề gì khác được nữa không? Người
ta còn thường sử dụng mô hình trên để xếp hạng độ quan trọng của các
điểm trong một mạng lưới. Có những mô hình, bài toán đã được xây dựng
sẵn và chúng ta có thế áp dụng nó cho những trường hợp tương tự mà
mình gặp phải.

Qua các ví dụ trên, ta đưa ra định nghĩa sau


Định nghĩa 2.6. Một mô hình toán học là một bộ ba (S, Q, M ) trong đó S
là một “hệ”, Q là một/các câu hỏi liên quan đến S, và M là tập hợp các đối
tượng và mệnh đề dùng để mô tả S và trả lời Q.
Hoạt động: Hãy xác định S, Q, M trong các ví dụ trên.

3. Các ví dụ cụ thể
Ví dụ 3.1. Thiết kế lon thiếc đựng thực phẩm và Bất Đẳng Thức

Hình 10.6: Các lon thiếc.

Hướng tới kỷ niệm 20 năm thành lập trường PTNK, ĐHQG TP HCM
132 Chuyên đề Toán học số 10

Các yếu tố quan trọng cần lưu ý


Chuyên đề Toán học Phổ thông năng khiếu TP.HCM số 10

• Thể tích của lon: sẽ cố định, cho người tiêu dùng tham khảo và định
giá.

• Hình dáng: hình trụ do vẫn đứng chắc được và không có góc nhọn
có thể gây thương tích. Cũng là dạng thiết kế phổ biến nhất hiện
nay.

• Do lon mỏng nên phí tỉ lệ với diện tích bề mặt.

Như vậy vấn đề còn lại là thiết kế chiều cao và đáy như thế nào để diện
tích bề mặt ít nhất nhằm tiết kiệm nguyên vật liệu.

Ta đưa ra một phương án như sau. Gọi h, r lần lượt là chiều cao và bán
kính đáy, V là thể tích không đổi:

• Tìm mối liên hệ giữa các biến trên?

• Hàm cần tìm tối ưu là gì?

Hình 10.7: Tham số hóa lon thiếc.

Nhận xét:

• Hàm số và bất đẳng thức là một công cụ hiệu quả trong các bài toán
tối ưu hóa. Chúng ta có một cách nhìn khác ở môn bất đẳng thức
theo khía cạnh hình học này.

• Các bước cơ bản là:

1. Xác định các biến.


2. Xác định hàm cần tối ưu.

Hướng tới kỷ niệm 20 năm thành lập trường PTNK, ĐHQG TP HCM
Chuyên đề Toán học số 10 133

3. Xây dựng các điều kiện ràng buộc, mối liên quan giữa các biến
Chuyên đề Toán học Phổ thông năng khiếu TP.HCM số 10

để tìm giá trị tối ưu và dấu bằng.

Ngoài ra công cụ hàm số còn hiệu quả trong những bài toán cần tính toán
hay đo đạt sự thay đổi, như ví dụ sau.

Ví dụ 3.2. Tính toán số lượng cá thể trong một quần thể theo thời gian và
đạo hàm (Logistics Model)

Hình 10.8: Đồ thị tiêu biểu của một hàm logistic.

Giả thiết P là số lượng cá thể phụ thuộc vào thời gian: P = P(t) trở thành
một hàm liên tục ở thời điểm t. Tỉ lệ gia tăng dân số ở thời điểm t là r(t).
Đạo hàm P 0 (t) đo tốc độ tăng của hàm số. Như vậy, theo các định nghĩa
trên, ta có
P 0 (t)
= r(t) hay P 0 (t) = r(t)P(t).
P(t)
Trong điều kiện lý tưởng, tài nguyên khoáng sản là không giới hạn thì
r(t) = r0 là một hằng số, nhờ vào lý thuyết phương trình sai phân, giải
phương trình trên ta được

P(t) = P0 e r0 t với P0 là số lượng cá thể ban đầu lúc t = 0.

Hướng tới kỷ niệm 20 năm thành lập trường PTNK, ĐHQG TP HCM
134 Chuyên đề Toán học số 10

Có thể thấy là dân số sẽ gia tăng rất nhanh theo hàm mũ, khá hợp lý
Chuyên đề Toán học Phổ thông năng khiếu TP.HCM số 10

trong trường hợp có nguồng tài nguyên vô hạn. Tất nhiên trong thực tế,
tài nguyên chỉ là hữu hạn, do đó tỉ lệ tăng dân số sẽ tỉ lệ nghịch với mật
độ (các cá thể sẽ tranh giành với nhau, có thể dẫn đến là đào thải lẫn nhau)

P(t)
Mật độ được tính bằng công thức K trong đó K là số lượng cá thể tối đa
để tài nguyên có thể đáp ứng được. Người ta đã kiểm tra được giả thiết này.

Như vậy r(t) tỉ lệ thuận với (1 − P(t)/K) và gần bằng r0 khi (1 − P(t)/K)
P(t)

gần bằng 1. Do đó, r(t) = r0 1 − K , trong đó r0 là tỉ lệ gia tăng dân số
trong điều kiện lý tưởng. Như vậy,
 ‹
P(t)
P (t) = r0 P(t) 1 −
0
.
K
Phương trình này được gọi là phương trình logistic, hay còn có tên là
phương trình Verhulst, một nhà toán học người Bỉ. Năm 1840, ông dùng
phương pháp này cũng số liệu những năm gần đó để dự đoán dân số của
Mĩ vào năm 1940 và chỉ sai số 1%.

Nhận xét:
• Cách dùng đạo hàm và thêm các giả thiết lý tưởng hóa để xây dựng
mối quan hệ rất phổ biến trong vật lý.

• Để giải được các phương trình vi phân cần đến kiến thức toán cao
cấp.

4. Các thành phần quan trọng

4.1. Phát biểu của bài toán


Mô tả ngắn gọn về S và những kết quả (output) mà mô hình sẽ cho ra
được để trả lời cho Q.
Ví dụ 4.1. Một cách phát biểu của các bài toán ở trên:
• Tìm ra đường đi ngắn nhất từ hai vị trí trên bản đồ và phương tiện
đi lại, cũng như ước tính thời gian cần để di chuyển.

• Xếp hạng các trang web dựa trên liên kết giữa chúng.

• Tính ra số lượng dân số trong tương lai dựa trên số dân trong một
số năm cũ.

Hướng tới kỷ niệm 20 năm thành lập trường PTNK, ĐHQG TP HCM
Chuyên đề Toán học số 10 135

Có thể kết hợp thêm mô tả ngắn gọn về input hay các mối liên hệ quan
Chuyên đề Toán học Phổ thông năng khiếu TP.HCM số 10

trọng khác.

4.2. Các giả thiết, giả sử


Những điều kiện/giả thiết không đúng 100% với thực tế nhưng có lý để
làm tiền đề cho việc xây dựng mô hình toán. Chúng thường được dùng khi
cần bỏ qua những chi tiết không quá quan trọng, những yếu tố khó kiểm
soát được hoặc không có dữ liệu để kiểm chứng. Ngoài ra, những giả thiết
ấy còn đơn giản hóa được các vấn đề nhưng vẫn đủ phức tạp và không
quá xa rời thực tế.

Ví dụ 4.2.

• Google Maps giả sử đường xá luôn ở trạng thái lưu thông bình
thường, tốc độ trung bình của mọi người là như nhau.

• Xác định các giả thiết, giả sử trong một ví dụ ở trên.

4.3. Mô hình toán, bao gồm các biến (input) và công thức
liên quan thu được sau khi đã có các giả thiết/giả sử
Ví dụ 4.3. Xác định mô hình toán trong các ví dụ trên.

4.4. Giải quyết mô hình toán để tìm nghiệm/ đáp số (out-


put)
Tùy theo mô hình để giải quyết ra đúng đáp số hoặc đôi khi là gần đúng
đáp số.

4.5. Nghiên cứu nghiệm/đáp số (thường là so sánh với


thực tiễn) và đánh giá mô hình để rút ra kết luận
hoặc cải thiện mô hình
Ví dụ 4.4. Các kinh nghiệm sau được suy ra từ kết quả của các mô hình
trên:

• Thuật toán PageRank được cải thiện sau một thời gian, sau khi đã có
thêm dữ liệu về số người truy cập và các thông tin khác trong thực
tế. Nhưng đó vẫn chưa là thuật toán cuối cùng để ra kết quả khi

Hướng tới kỷ niệm 20 năm thành lập trường PTNK, ĐHQG TP HCM
136 Chuyên đề Toán học số 10

search google do vẫn còn những yếu tố khác như lịch sử tìm kiếm
Chuyên đề Toán học Phổ thông năng khiếu TP.HCM số 10

(Search History) của mỗi người.

• Mô hình tính toán dân số của Verhulst sai lệch đi nhiều sau 1940,
nguyên nhân có thể là do chiến tranh thế giới lần thứ 2 và mô hình
giả sử dân số là một hàm liên tục ổn định theo thời gian. Như vậy, ta
cần cải thiện để mô hình điều chỉnh phù hợp với những biến cố/sự
kiện.

Đây là một phần vô cùng quan trọng và rất có ích trong việc hiểu thêm về
mô hình, về đối tượng. Phần này còn có thể cho thêm ý tưởng để cải thiện
mô hình cũ hay xây dựng các mô hình mới khác, hay thậm chí là hiểu rõ
hơn về vấn đề trong thực tế, vì sao nó không đúng như mô hình dự đoán.

5. Hai tính chất quan trọng của M

5.1. Input/ Output


Hiểu nôm na, mọi mô hình toán đều hoạt động theo nguyên lý nhập/xuất:
nhận vào những tham số, thông tin (input) và cho ra đáp số, câu trả lời
(output).
Ví dụ 5.1 Xác định input/output trong các ví dụ trên.
Input sẽ là các tham số, các thông tin, mệnh đề độc lập với nhau còn
output sẽ phụ thuộc vào input.

5.2. Open-ended (tạm hiểu là có thể xây dựng nhiều M


cho cùng một S và Q):
Ý nghĩa quan trọng nhất của toán mô hình không dừng lại ở việc tìm ra
được đáp số hay giải quyết một vấn đề mà là qua đó giúp chúng ta hiểu
thêm về các đối tượng và mối liên hệ giữa chúng trong “hệ”.

Trong nhiều vấn đề thực tế, không có một lời giải, mô hình duy nhất cho
một bài toán mà tùy thuộc vào cách tiếp cận, cách thiết lập các giả thiết
hay giả định, và cách định nghĩa những yếu tố “tốt nhất” hay “hiệu quả
nhất”.

Ví dụ 5.2. Chi phí vs thời gian: Trang tìm vé máy bay nổi tiếng Skiplagged
sẽ tìm ra đường bay “tốt” nhất theo nghĩa là rẻ nhất, nhưng cũng có thể
đường bay “tốt” nhất là đường bay tiết kiệm được nhiều thời gian nhất

Hướng tới kỷ niệm 20 năm thành lập trường PTNK, ĐHQG TP HCM
Chuyên đề Toán học số 10 137

trong một khoảng thời gian nhất định.


Chuyên đề Toán học Phổ thông năng khiếu TP.HCM số 10

Ví dụ 5.3 Mở tiệm Starbucks (SIMC 2010): http://www.nushigh.edu.sg/


qql/slot/u90/file/simc/mathmodel/SIMC2010_ChallengeQuestion.pdf
Ta có nhận xét:
• Rất khó để tìm ra một giải pháp hay lời giải hoàn hảo về mọi mặt,
mỗi cách giải hay mô hình có những được và mất riêng (trade-off),
ta cần chọn ra những yếu tố mình muốn tối ưu hóa, ghi lại những
yếu tố mà mô hình chưa đánh giá và tìm cách cải thiện nó.
• Đôi khi những mô hình toán tạo nên các bài toán mở và chưa có
những công cụ trong lý thuyết để mô tả và giải dễ dàng.
• Nếu không giải quyết được toàn bộ bài toán, hãy giải quyết một phần
của nó và lưu ý những phần còn lại.

5.3. Interdisciplinary (tạm hiểu là có thể dùng một M cho


nhiều S)
Một mô hình toán M có thể được sử dụng nhiều vấn đề khác nhau (S, Q
thay đổi) nhưng gần tương tự về bản chất hay mô tả.

Ví dụ 5.4. Tìm vé máy bay giá rẻ Skiplagged. Dùng lại lại thuật toán tìm
đường đi ngắn nhất của Google Maps, thay độ dài của quãng đường bằng
số tiền. Fun fact: Skiplagged đã từng bị kiện do làm ảnh hưởng đến doanh
thu của các hang máy bay.

Ví dụ 5.5. Mô hình Pagerank của Google có bản chất là mô hình chuỗi


Markov: được ứng dụng rộng rãi trong việc tính toán sự phân bố sau một
thời gian trong một hệ gồm nhiều trạng thái khép kín như sự phân bố dân
số, sự phân bố thị trường v.v.

Hình 10.9: Mô hình cơ bản của một chuỗi Markov.

Nhận xét:

Hướng tới kỷ niệm 20 năm thành lập trường PTNK, ĐHQG TP HCM
138 Chuyên đề Toán học số 10

• Trong nhiều trường hợp, người ta áp dụng cùng một M (hoặc gần
Chuyên đề Toán học Phổ thông năng khiếu TP.HCM số 10

tương tự M) cho những S khác nhau nhưng có cùng Q.

• Cần phải thay đổi cách phát biểu bài toán, các giả thiết hoặc in-
put/output cho phù hợp.

6. Phân loại
Dựa trên S, Q, M:

• Hệ (S)

– Các vấn đề tự nhiên (Vật lý, Hóa học, Sinh học. . . )


– Các vấn đề trong xã hội (Kinh tế, Dân số, . . . )
– Đối tượng vật lý cụ thể vs Đối tượng trừu tượng
– Có yếu tỗ ngẫu nhiên vs không ngẫu nhiên
– Vấn đề liên tục (sự thay đổi theo thời gian,. . . )
– Vấn đề rời rạc (yếu tố thời gian ko quan trọng. . . )

• Câu hỏi (Q)

– Lên kế hoạch
– Thiết kế
– Tính toán, dự đoán
– Tìm mối liên hệ

• Mô hình (M)

– Tuyến tính/Không tuyến tính


– Liên tục/Rời rạc
– Phương trình đại số/Phương trình vi phân
– Xác suất/Quy hoạch tuyến tính/Đồ thị

Hoạt động: Hãy thử phân loại một ví dụ ở trên.

Nhìn chung, một mô hình toán học có thể bao gồm nhiều hơn một loại
mô hình nhất định, là một tổ hợp của một số loại mô hình ở trên. Trong
chương trình lần này, chúng ta sẽ tập trung vào phần lên kế hoạch, thiết
kế tối ưu và được dạy về các công cụ rời rạc.

Hướng tới kỷ niệm 20 năm thành lập trường PTNK, ĐHQG TP HCM
Chuyên đề Toán học số 10 139
Chuyên đề Toán học Phổ thông năng khiếu TP.HCM số 10

Hình 10.10: Phân loại mô hình.

Hướng tới kỷ niệm 20 năm thành lập trường PTNK, ĐHQG TP HCM
140 Chuyên đề Toán học số 10
Chuyên đề Toán học Phổ thông năng khiếu TP.HCM số 10

Hướng tới kỷ niệm 20 năm thành lập trường PTNK, ĐHQG TP HCM
Tài liệu tham khảo
Chuyên đề Toán học Phổ thông năng khiếu TP.HCM số 10

[1] http://www.bowlandmaths.org.uk/materials/pd/online/pd_03/
pdf/pd_03_handbook_full.pdf

[2] http://www.indiana.edu/~hmathmod/modelmodel.html

[3] http://www.maths.bris.ac.uk/~madjl/course_text.pdf

141
142 Chuyên đề Toán học số 10
Chuyên đề Toán học Phổ thông năng khiếu TP.HCM số 10

Hướng tới kỷ niệm 20 năm thành lập trường PTNK, ĐHQG TP HCM
Chuyên đề Toán học Phổ thông năng khiếu TP.HCM số 10

Quy hoạch tuyến tính và Ứng dụng

Trần Hoàng Bảo Linh 1


(Lớp Chuyên Toán khóa 2010 − 2013)

1. Ví dụ mở đầu
Cũng như bất phương trình đối với phương trình, khái niệm quy hoạch
tuyến tính là mở rộng của khái niệm hệ phương trình tuyến tính. Ở đây,
nghiệm của hệ phương trình tương ứng với khái niệm miền ràng buộc
trong quy hoạch tuyến tính. Mục tiêu của một bài toán quy hoạch tuyến
tính là tìm một nghiệm trong miền ràng buộc đó để một hàm tuyến tính
đạt cực trị tại nghiệm đó.
Quy hoạch tuyến tính tuy là một khái niệm chỉ có mặt ở bậc Đại học,
những nền tảng cở bản lại khá dễ hiểu với kiến thức bậc trung học phổ
thông. Cộng với khả năng áp dụng đa dạng cho rất nhiều lớp bài toán tối
ưu, quy hoạch tuyến tính được xem như một khái niệm nền tảng trong
Toán ứng dụng. Sau đâu sẽ là một ví dụ quen thuộc.

Ví dụ 1.1. (PiMA Math Research Camp 2016 Application Round) Công


ty PIMA cần tạo ra một bữa ăn đạt chuẩn từ thịt gà và thịt bò, đảm bảo
cung cấp đầy đủ các dưỡng chất: protein, canxi và sắt, với chi phí nguyên
liệu thấp nhất có thể. Các nhà khoa học của PIMA nghiên cứu và phát hiện
ra: Một bữa ăn đạt chuẩn cần ít nhất 200g protein, 50g canxi và 30g sắt.
Mỗi kg thịt gà cung cấp 20g protein, 7g canxi và 5g sắt.
Mỗi kg thịt bò cung cấp 25g protein, 5g canxi và 10g sắt.
Thịt gà có giá 10 đồng/kg, thịt bò có giá 15 đồng/kg.
Hãy tìm cách để tạo được bữa ăn đạt chuẩn với giá thành nhỏ nhất có thể.

Mô hình. Gọi x và y lần lượt là lượng thịt gà và thịt bò trong một bữa ăn
tính bằng kg.
Ta có:
1
Bài viết trích từ tài liệu của cuộc thi Toán ứng dụng PIMA 2016

143
144 Chuyên đề Toán học số 10

Tổng số protein trong một bữa ăn: 20x + 25 y.


Tổng số canxi trong một bữa ăn: 7x + 5 y.
Chuyên đề Toán học Phổ thông năng khiếu TP.HCM số 10

Tổng số sắt trong một bữa ăn: 5x + 10 y.


Tổng giá tiền của một bữa ăn: 10x + 15 y.
Theo đề bài x và y phải thỏa mãn:

20x + 25 y

≥ 200
7x + 5 y ≥ 50
5x + 10 y

≥ 30

Ta cần tìm x và y để 10x + 15 y lớn nhất.

2. Khái niệm

2.1. Quy hoạch tuyến tính thực


Định nghĩa 2.1. Một hàm tuyến tính của n biến x 1 , . . . , x n là hàm có dạng:
f (x 1 , x 2 , . . . , x n ) = a1 x 1 + a2 x 2 + . . . + an x n , trong đó a1 , a2 , . . . an là những
hằng số thực.

Định nghĩa 2.2. Một phương trình/bất phương trình tuyến tính của n ẩn
là phương trình/bất phương trình có dạng: f (x 1 , x 2 , . . . , x n ) ≤ g(x 1 , x 2 , . . . , x n ),
trong đó f và g là những hàm tuyến tính n biến.

Định nghĩa 2.3. Một bài toán quy hoạch tuyến tính là một bài toán tối ưu
hóa có dạng như sau:
Tìm giá trị lớn nhất/nhỏ nhất của hàm f (x 1 , . . . , x n ), trong đó f là một
hàm tuyến tính n biến và các biến x 1 , x 2 , . . . , x n phải thỏa mãn một số
phương trình/hệ phương trình cho trước.

Một bài toán quy hoạch tuyến tính ở dạng chuẩn bao gồm những phần
sau:
(a) Các ẩn số không âm (decision variables): x 1 , x 2 , . . . , x n ≥ 0.
(b) Các phương trình điều kiện (constraints):


 a11 x 1 + a12 x 2 + . . . + a1n x n = b1

a21 x 1 + a22 x 2 + . . . + a2n x n = b2


...
a x + a x + . . . + a x

= bm
m1 1 m2 2 mn n

Hướng tới kỷ niệm 20 năm thành lập trường PTNK, ĐHQG TP HCM
Chuyên đề Toán học số 10 145

Trong đó a11 , a12 , . . . , a1n , . . . , amn và b1 , b2 , . . . , bm (n ≥ m) là những hằng


Chuyên đề Toán học Phổ thông năng khiếu TP.HCM số 10

số thực và bi ≥ 0 ∀i.
(c) Hàm tuyến tính cần được tối đa hóa (hàm mục tiêu - objective
function)
f (x 1 , x 2 , . . . , x n ) = c1 x 1 + c2 x 2 + . . . + cn x n

Với c1 , c2 , . . . , cn là những hằng số thực.

Ví dụ 2.4. Tìm giá trị lớn nhất của 2x + 1.5 y + 4z với điều kiện:


 x, y, z ≥0

x + 2 y − 3z ≤3


 4x − y + 0.5z ≤1
2.5x + 3 y + 2z ≤ 10

Lưu ý. (Dạng ma trận của bài toán quy hoạch tuyến tính) Một bài toán
quy hoạch tuyến tính có thể được phát biểu dưới dạng ma trận như sau:
1. Vector ẩn số: x = (x 1 x 2 . . . x n ) T , x i ≥ 0 ∀i
2. Điều kiện: Ax ≤ b
3. Tìm giá trị lớn nhất của hàm: F(x) = c T x

Dưới đây là dạng ma trận của ví dụ 1.1:

Ví dụ 2.5. Tìm giá trị lớn nhất của: f (x) = (2 1.5 4) T x với điều kiện:

x ≥0
   
1 2 −3 3
 4 −1 0.5 x ≤  1 
2.5 3 2 10

2.2. Biến thể: Quy hoạch tuyến tính nguyên


Một bài toán quy hoạch tuyến tính nguyên là một bài toán quy hoạch
tuyến tính có thêm điều kiện các ẩn là số nguyên. (Lưu ý: các hằng số vẫn
là số thực).
Những bài toán trong đó các ẩn số cần tìm biểu thị số lượng của những
vật thể đếm được (người, nhà, xe) thường được mô hình bằng quy hoạch
tuyến tính nguyên. Ngoài ra, những ẩn số biểu thị một quyết định có hoặc
không cũng có thể được mô hình bằng một ẩn số nguyên chỉ nhận giá trị
0 và 1.

Hướng tới kỷ niệm 20 năm thành lập trường PTNK, ĐHQG TP HCM
146 Chuyên đề Toán học số 10

Vì tính chất rời rạc của các số nguyên mà trong nhiều trường hợp, giải
Chuyên đề Toán học Phổ thông năng khiếu TP.HCM số 10

thuật của bài toán quy hoạch tuyến tính thực không thể áp dụng được cho
ẩn số nguyên. Tuy nhiên, cũng vì lí do đó mà bài toán quy hoạch tuyến
tính nguyên có những giải thuật đặc biệt không thể áp dụng được cho
trường hợp ẩn thực.

Ví dụ 2.6. (PiMA Math Research Camp 2016 Interview Round) Một tên
cướp đang ở một tiệm đá quý. Tiệm đá quý có 3 loại: kim cương, pha lê và
ngọc trai với trọng lượng và giá trị khác nhau. Mỗi viên kim cương có giá
12 triệu và nặng 5kg, mỗi viên ngọc trai có giá 8 triệu và nặng 4kg, mỗi
viên ngọc trai có giá 6 triệu và nặng 3kg. Trong tiệm có tổng cộng 2 viên
kim cương, 5 viên pha lê và 4 viên ngọc trai. Tên cướp chỉ mang được tối đa
20kg. Hãy tìm cách để tên cướp kiếm được nhiều tiền nhất.

Mô hình. Gọi x, y, z lần lượt là số viên kim cương, pha lê và ngọc trai tên
cướp lấy, vậy x, y, z là những số nguyên không âm. Ngoài ra, ta có:


 x ≤2

y ≤5


 z ≤4
5x + 4 y + 3z ≤ 20

Ta cần tìm số tiền lớn nhất tên cướp mang được, tức là tìm giá trị lớn nhất
của biểu thức sau:
F (x, y, z) = 12x + 8 y + 6z (11.1)
Vậy là ta đã tìm được mô hình thích hợp cho bài toán trên.

3. Sơ lược về cách giải

3.1. Quy hoạch tuyến tính thực


Trường hợp 2 ẩn: phương pháp đồ thị

Phương pháp đồ thị (graphic method) thường được áp dụng cho bài toán
quy hoạch tuyến tính 2 ẩn. Phương pháp này sử dụng tính chất sau của hệ
trục tọa độ Ox y: Miền nghiệm của bất phương trình tuyến tính ax + b y ≤
c là một trong hai nửa mặt phẳng tạo ra bởi đường thẳng ax + b y = c.
Sử dụng tính chất này, ta có thể xác định miền nghiệm chung của toàn bộ
những bất phương trình điều kiện trong bài toán (gọi là miền ràng buộc).

Hướng tới kỷ niệm 20 năm thành lập trường PTNK, ĐHQG TP HCM
Chuyên đề Toán học số 10 147

Miền này sẽ có hình một đa giác lồi nếu bị chặn, khi đó hàm mục tiêu chắc
Chuyên đề Toán học Phổ thông năng khiếu TP.HCM số 10

chắn đạt được giá trị lớn nhất tại một trong các đỉnh của đa giác này. Khi
đó, chúng ta có thể so sánh giá trị của hàm mục tiêu tại các đỉnh của đa
giác để tìm vị trí đạt giá trị lớn nhất.
Chúng ta sẽ không đi sâu vào chứng minh những tính chất trên mà sẽ tìm
hiểu ứng dụng của phương pháp đồ thị qua những ví dụ sau.
Ví dụ 3.1. Trung mở một quán trà sữa và mời Hưng đến uống ủng hộ. Trung
muốn làm một ly trà sữa thật ngon để tiếp đãi bạn. Trung biết rằng: Hưng
thích uống trà sữa với lượng sữa ít nhất gấp rưỡi lượng trà, trọng lượng ly
phải không quá 500g và có ít nhất 100g trà trong đó.
Giá 100g trà là 20.000 đồng và 100g sữa là 25.000 đồng. Hỏi Trung phải
làm thế nào để pha được ly trà sữa theo ý Hưng và tốn ít chi phí nhất?
Mô hình. Gọi x và y là lượng trà và sữa Trung cần dùng tính theo 100g.
Các điều kiện trong đề bài được viết lại như sau:
x, y ≥ 0



 1.5x − y ≤ 0

 x+y≤ 5

x ≥ 100
Hàm mục tiêu cần đạt giá trị nhỏ nhất: f (x, y) = 20000x + 25000 y.
Lời giải. Dựa vào những bất phương trình trên mô hình, ta tìm được miền
ràng buộc, có dạng một tam giác như trong hình 11.1. Tiếp theo, chỉ cần

Hình 11.1: Đồ thị biểu diễn miền nghiệm trong ví dụ 3.1

so sánh giá trị của hàm mục tiêu tại 3 đỉnh của tam giác này. Sau khi so
sánh, ta kết luận f (x, y) = 20000x + 25000 y đạt giá trị nhỏ nhất khi
(x, y) = (1, 1.5).
Do đó Trung sẽ tiết kiệm được chi phí nhất với 1 · 100 = 100g trà và
1.5 · 100 = 150g sữa.

Hướng tới kỷ niệm 20 năm thành lập trường PTNK, ĐHQG TP HCM
148 Chuyên đề Toán học số 10

Phương pháp đơn hình


Chuyên đề Toán học Phổ thông năng khiếu TP.HCM số 10

Đây là phương pháp tổng quát của phương pháp đồ thị ở trên, dùng cho
trường hợp n ẩn. Phương pháp này áp dụng 2 nguyên lý cơ bản:

Định lý 3.2. Nếu một bài toán quy hoạch tuyến tính có nghiệm tối ưu thì
nó cũng có nghiệm tối ưu nằm trên một đỉnh của miền ràng buộc.

Định lý 3.3. Nếu một nghiệm của bài toán là cực trị địa phương của hàm
mục tiêu thì nó cũng là một cực trị tuyệt đối, nghĩa là một nghiệm tối ưu.

Ý tưởng chung của phương pháp đơn hình là bắt đầu từ một nghiệm ban
đầu, nếu nó chưa phải tối ưu thì đi theo một cạnh của miền ràng buộc để
tới được một nghiệm tối ưu hơn. Khi thuật toán kết thúc thì nghiệm tìm
được chính là nghiệm tối ưu.
Để giải một bài toán quy hoạch tuyến tính bằng phương pháp đơn hình,
ta thực hiện các bước sau:
Bước 0. (Đưa bài toán về dạng chuẩn)
Ta áp dụng những biến đổi sau:

1. Nếu có biến x i ≤ 0 thì đặt yi = −x i và thay vào bài toán.

2. Nếu có biến x i ≥ c > 0 thì đặt yi = x i − c và thay vào bài toán.

3. Nếu có điều kiện dạng f (x 1 , . . . , x n ) + C1 ≤ g(x 1 , . . . , x n ) + C2 thì


chuyển vế để đưa về dạng h(x 1 , . . . , x n ) ≤ C

4. Nếu có điều kiện dạng h(x 1 , . . . , x n ) ≤ C với C ≥ 0 thì đưa thêm biến
s ≥ 0 vào để tạo thành dạng h(x 1 , . . . , x n ) + s = C.

5. Nếu có điều kiện dạng h(x 1 , . . . , x n ) ≤ C với C < 0 thì đưa thêm biến
t ≥ 0 vào để tạo thành dạng −t − h(x 1 , . . . , x n ) = −C.

6. Nếu có biến x i không có điều kiện ràng buộc nào (biến tự do) thì
viết x i = yi − zi với yi , zi ≥ 0 và thay vào bài toán.

Sau khi kết thúc, ta được một bài toán tương đương với bài toán ban đầu
và ở dạng chuẩn. Lưu ý rằng những biến đổi trên đảm bảo rằng số phương
trình không vượt quá số biến.
Bước 1. (Lập bảng đơn hình)
Ta có:
Hàm mục tiêu: F (X ) = c1 x 1 + c2 x 2 + . . . + cn x n .

Hướng tới kỷ niệm 20 năm thành lập trường PTNK, ĐHQG TP HCM
Chuyên đề Toán học số 10 149

Các phương trình ràng buộc:


Chuyên đề Toán học Phổ thông năng khiếu TP.HCM số 10



 a11 x 1 + a12 x 2 + . . . + a1n x n = b1

a21 x 1 + a22 x 2 + . . . + a2n x n = b2


 ...
a x + a x + . . . + a x

= bm
m1 1 m2 2 mn n

Ta lập bảng đơn hình như sau:

1 −c1 −c2 ... −cn 0


0 a11 a12 ... a1n b1
0 a21 a22 ... a2n b2
... ...
0 am1 am2 ... amn bn

Dùng biến đổi Gauss, ta có thể đưa bảng trên về dạng chuẩn:

1 −c1 −c2 ... −cm −cm+1 ... −cn 0


0 1 0 ... 0 a1(m+1) ... a1n b1
0 0 1 ... 0 a2(m+1) ... a2n b2
... ...
0 0 0 ... 1 am(m+1) ... amn bn

Ở dạng này, các biến tương ứng với các cột của ma trận đơn vị được gọi là
biến cơ bản, các biến còn lại là biến tự do. Dễ thấy khi các biến tự do bằng
0 thì ta có 1 nghiệm của miền ràng buộc: (b1 , b2 , . . . , bn , 0, . . . , 0). Đây sẽ
là nghiệm xuất phát cho thuật toán.
Ở các bước sau, ta chọn 1 biến tự do (gọi là biến vào) và 1 biến cơ bản
(biến ra) để hoán đổi vị trí của chúng sao cho bộ nghiệm mới có giá trị
hàm mục tiêu lớn hơn bộ cũ.
Bước 2. (Chọn biến vào)
Nếu mọi hệ số −ci với i > m đều không âm thì nghiệm hiện tại là nghiệm
tối ưu (hàm mục tiêu đạt giá trị lớn nhất).
Ngược lại, chọn i > m sao cho −ci < 0. Đặt x i là biến vào. Cột i gọi là trục
xoay.
Bước 3 (Chọn biến ra)
Sau khi đã chọn biến vào và trục xoay, ta xem xét các hệ số trên trục xoay.
Nếu các hệ số này đều không dương, hàm mục tiêu không bị chặn trên.
Thuật toán kết thúc.
Ngược lại, ta chọn r sao cho b r /a r i là nhỏ nhất trong {r|a r i > 0}. x r sẽ là

Hướng tới kỷ niệm 20 năm thành lập trường PTNK, ĐHQG TP HCM
150 Chuyên đề Toán học số 10

biến ra.
Chuyên đề Toán học Phổ thông năng khiếu TP.HCM số 10

Bước 4 (Biến đổi xoay vòng)


Đầu tiên, ta nhân hàng r với 1/a r i để biến a r i thành 1.
Sau đó ở hàng j khác r, ta biến đổi: R j = R j − a ji R r để biến a ji thành 0.
Bây giờ trục xoay đã trở thành cột thứ r của ma trận đơn vị.
Cuối cùng ta đổi thứ tự C r ↔ Ci và biến x i sẽ thay x r trở thành một biến
cơ bản. Bảng đơn hình mới cũng thuộc dạng chuẩn. Hơn nữa, giá trị của
hàm mục tiêu tăng lên.
Bước 5 Quay lại bước 2.

Định lý 3.4. Nếu bài toán quy hoạch tuyến tính có nghiệm tối ưu, thuật
toán đơn hình luôn kết thúc với một trong những nghiệm tối ưu.

Điều này có thể hiểu được dựa trên nguyên tắc rằng nghiệm tiếp theo được
sinh ra luôn có giá trị hàm mục tiêu lớn hơn hẳn so với nghiệm trước. Mặt
khác, số nghiệm được xét là hữu hạn (chỉ có hữu hạn cách chọn bộ các
biến cơ bản), do vậy thuật toán sẽ kết thúc tại 1 nghiệm nào đó. Theo định
lý 3.3, nghiệm này phải là nghiệm tối ưu.

Ví dụ 3.5. Tìm giá trị lớn nhất của: F (x) = 2x + 3 y + 4z biết rằng:

 x, y, z ≥ 0
3x + 2 y + z ≤ 10
2x + 5 y + 3z ≤ 15

Lời giải. Sau khi thêm các biến bù u và v, ta được bảng đơn hình sau:

1 −2 −3 −4 0 0 0
0 3 2 1 1 0 10
0 2 5 3 0 1 15

Biến cơ bản: {u, v}.


Ta chọn cột 1 làm trục xoay, tức là x là biến vào. Khi đó vì 10/3 < 15/2
nên hàng 1 được chọn, tức là u sẽ là biến ra. Bảng đơn hình mới như sau:

1 0 −5/3 −10/3 2/3 0 20/3


0 1 2/3 1/3 1/3 0 10/3
0 0 11/3 7/3 −2/3 1 25/3

Biến cơ bản: {x, v}.


Tiếp theo, chọn cột 3 làm trục xoay (z là biến vào). Khi đó vì 25/7 < 10/1
nên v là biến ra. Bảng đơn hình mới như sau:

Hướng tới kỷ niệm 20 năm thành lập trường PTNK, ĐHQG TP HCM
Chuyên đề Toán học số 10 151

1 0 25/7 0 −2/7 10/7 130/7


0 1 1/7 0 3/7 −9/7 15/7
Chuyên đề Toán học Phổ thông năng khiếu TP.HCM số 10

0 0 11/7 1 −2/7 3/7 25/7

Biến cơ bản: {x, z}.


Chọn cột 4 làm trục xoay. Khi đó x là biến ra. Ta được:
1 2/3 11/3 0 0 4/7 20
0 7/3 1/3 0 1 −3 5
0 2/3 5/3 1 0 −3/7 5

Biến cơ bản: {z, u}.


Đến đây các hệ số ở hàng 0 đều không âm nên thuật toán kết thúc. Giá
trị lớn nhất cần tìm là 20, đạt được tại (x, y, z, u, v) = (0, 0, 5, 5, 0) hay
(x, y, z) = (0, 0, 5).

3.2. Bài tập


Giải những bài toán sau (không giới hạn phương pháp):

3.2.1. Tìm giá trị nhỏ nhất của 5x + 7 y biết:




 x, y ≥ 0
 2x + 3 y ≥ 6



3x − y ≤ 15
−x + y ≤ 4





 2x + 5 y ≤ 27

3.2.2. Tìm giá trị nhỏ nhất của 2x + 3 y + 3z biết:

x, y, z ≥ 0


x + y −z ≥ 2



 x + 2 y − 3z ≥ 3
−x + 4 y + 5z ≥ 6

4. Mô hình các vấn đề bằng quy hoạch tuyến


tính
Trong một bài toán tối ưu hóa, đề bài thường yêu cầu lập một kế hoạch
bằng cách kết hợp nhiều phương án có sẵn với một số điều kiện ràng buộc.

Hướng tới kỷ niệm 20 năm thành lập trường PTNK, ĐHQG TP HCM
152 Chuyên đề Toán học số 10

Ví dụ: trong một kế hoạch ăn uống, những phương án sẽ là ăn thịt gà, thịt
Chuyên đề Toán học Phổ thông năng khiếu TP.HCM số 10

bò và thịt lợn. Lượng thịt mỗi loại sẽ là hệ số của phương án tương ứng
trong kế hoạch đó.
Những điều kiện ràng buộc thường là lượng tài nguyên dành cho những
phương án bị giới hạn hoặc một số phương án không thể thực hiện cùng
lúc. Mỗi phương án cũng tiêu thụ một lượng tài nguyên nhất định, gọi
là chi phí của phương án đó. Chi phí của một kế hoạch đối với một tài
nguyên sẽ là tống chi phí của những phương án thành phần với tài nguyên
đó.
Mỗi phương án mang lại một lượng lợi ích nhất định, gọi là giá trị của
phương án đó. Giá trị của một kế hoạch sẽ là tổng giá trị của những
phương án thành phần trong đó.
Mục tiêu của bài toán thường là tìm hệ số cho mỗi phương án sao cho giá
trị của kế hoạch lập được là cao nhất.

Để biến một bài toán thực tế thành một bài toán quy hoạch tuyến tính
cần thực hiện 3 bước:
1. Xác định các ẩn số. Như đã đề cập ở trên, mỗi kế hoạch là sự kết hợp
của những phương án thành phần mà mỗi phương án có một hệ số. Như
vậy, để xác định kế hoạch tối ưu (kế hoạch có giá trị cao nhất), chỉ cần xác
định hệ số của mỗi phương án thành phần. Do đó, các ẩn số chính là các
hệ số của những phương án tương ứng.

2. Xác định các bất phương trình điều kiện. Đầu tiên cần phải xác
định những tài nguyên được đề bài đưa ra, sau đó tìm chi phí của mỗi
phương án đối với từng tài nguyên. Với mỗi tài nguyên, lập biểu thức tính
chi phí của kế hoạch đối với tài nguyên đó (tức là tính tổng chi phí của
những phương án thành phần). Biểu thức lập được chính là vế trái của bất
phương trình điều kiện. Vế phải đơn giản là lượng tài nguyên tối đa mà đề
bài cho sẵn.

3. Xác định hàm mục tiêu. Cần phải xác định giá trị cần tối đa hóa mà đề
bài cho, sau đó xác định giá trị của những phương án có thể. Lập biểu thức
cho tổng giá trị kế hoạch dựa vào những giá trị thành phần với các hệ số
(chính là các ẩn số ở bước 1). Biểu thức này chính là hàm mục tiêu cần tìm.

Sau đâu chúng ta sẽ xem xét một số ví dụ của việc lập một mô hình quy
hoạch tuyến tính cho những bài toán khác nhau.

Hướng tới kỷ niệm 20 năm thành lập trường PTNK, ĐHQG TP HCM
Chuyên đề Toán học số 10 153

4.1. Quy hoạch tuyến tính thực


Chuyên đề Toán học Phổ thông năng khiếu TP.HCM số 10

Ví dụ 4.1. Một bác nông dân có tổng cộng 100 hecta đất để trồng lúa và
ngô. Mỗi hecta lúa bán được 40 triệu đồng và cần 3 tấn phân bón và 1.5 tấn
thuốc trừ sâu. Mỗi hecta ngô bán được 70 triệu đồng và cần 6 tấn phân bón
và 2 tấn thuốc trừ sâu.
Bác nông dân có tổng cộng 400 tấn phân bón và 250 tấn thuốc trừ sâu. Hỏi
bác phải làm thế nào để thu được lợi nhuận cao nhất?

Mô hình. Trong bài toán này, dễ thấy kế hoạch cần tìm là sự kết hợp giữa
2 phương án: trồng lúa và trồng ngô. Vậy tương ứng có 2 ẩn số: x là số
hecta trồng lúa và y là số hecta trồng ngô.
Các điều kiện được đề cập là giới hạn về diện tích đất, số phân bón và
thuốc trừ sâu. (x, y ≥ 0) Chi phí về đất cho 1 kế hoạch là: x + y.
Chi phí phân bón cho 1 kế hoạch là: 3x + 6 y.
Chi phí thuốc trừ sâu cho 1 kế hoạch là: 1.5x + 2 y.
Vậy ta có 3 phương trình điều kiện:

x + y ≤ 100


3x + 6 y ≤ 400
1.5x + 2 y ≤ 250

Sau cùng, lợi nhuận đem lại của 1 kế hoạch là: 40x + 70 y.
Vậy f (x, y) = 40x + 70 y là hàm mục tiêu mà ta cần tìm giá trị lớn nhất.
Bài toán này có thể giải được bằng phương pháp đồ thị.

Ví dụ 4.2. (Bài toán vận tải) Một công ty bán lẻ có m chi nhánh, được
đánh số từ 1 tới m. Lượng hàng ở chi nhánh thứ i được ký hiệu Si . Công ty
có n khách hàng, được đánh số từ 1 tới n.
Lượng hàng khách hàng thứ j muốn mua là D j . Chi phí vận chuyển mỗi
đơn vị hàng từ chi nhánh i tới khách hàng j là Ci j . Hãy lập kế hoạch vận
chuyển cho công ty sao cho mỗi khách hàng đều được thỏa mãn và chi phí
vận chuyển là nhỏ nhất.

Mô hình. Ở đây, mỗi phương án sẽ là một cuộc vận chuyển từ chi nhánh
i tới khách hàng j.
Gọi x i j là số hàng được chuyển từ chi nhánh i tới khách hàng j. Như vậy
có tổng cộng m · n ẩn số: x 11 , x 12 , . . . , x 1n , . . . , x m1 , . . . , x mn .

Có 2 loại điều kiện trong bài:

1) Giới hạn lượng hàng trong mỗi chi nhánh.

Hướng tới kỷ niệm 20 năm thành lập trường PTNK, ĐHQG TP HCM
154 Chuyên đề Toán học số 10

Tổng lượng hàng được chuyển từ chi nhánh i là: x i1 + x i2 + . . . + x in .


Chuyên đề Toán học Phổ thông năng khiếu TP.HCM số 10

Tổng lượng hàng có trong chi nhánh i là: Si .


Vậy ta có: x i1 + x i2 + . . . + x in ≤ Si ∀i = 1, . . . , n.

2) Lượng hàng mỗi khách hàng đặt.


Tổng lượng hàng khách hàng j nhận được: x 1 j + x 2 j + . . . + x m j .
Tổng lượng hàng khách hàng j đặt mua: D j .
Vậy ta có: x 1 j + x 2 j + . . . + x m j ≥ D j ∀ j = 1, . . . , m.

Trong bài toán này, đại lượng cần tối ưu hóa là chi phí vận chuyển, nhưng
mục tiêu là tìm giá trị nhỏ nhất chứ không phải lớn nhất. Tuy nhiên, đại
lượng T đạt giá trị nhỏ nhất cũng có nghĩa là đại lượng −T lớn nhất. Ta
đặt giá trị của phương án vận chuyển từ chi nhánh i tới khách hàng j là
−Ci j. Khi đó hàm mục tiêu cần tối đa hóa là:
F (x 11 , x 12 , . . . , x mn ) = −C11 x 11 − C12 x 12 − .... − Cmn x mn
Ví dụ 4.3. (Xếp lịch sản xuất) Công ty Triple H (đặt tên theo 3 nhà sáng
lập: Hải, Hưng và Hoàng) sản xuất nhựa tái chế từ nhựa phế liệu. Công ty
sử dụng 2 loại nguyên liệu chính là chai nhựa cũ và bao ni lông để sản xuất.
Chai nhựa rỗng cần 2 giờ để nấu chảy và mỗi tấn cho ra 0.8 tấn nhựa từ 0.5
tấn chất xúc tác. Bao ni lông cần 1 giờ để nấu chảy và cho ra 0.3 tấn nhựa
từ 0.1 tấn chất xúc tác. Mỗi ngày công ty thu thập được 8 tấn chai nhựa và
5 tấn bao ni lông. Công ty cần sản xuất ra ít nhất 6 tấn nhựa mỗi ngày. Để
đảm bảo an toàn về điện, không được đưa quá 4 tấn nguyên liệu vào lò tại
bất kì lúc nào. Hãy tìm giải pháp sản xuất tối ưu cho công ty.
Mô hình. Giải pháp tối ưu của công ty sẽ là giảm tối đa lượng chất xúc tác
sử dụng. Gọi x i và yi lần lượt là lượng chai nhựa và bao ni lông được đưa
vào nấu chảy từ tiếng thứ i (i = 1, 2, . . . , 24). Ta lần lượt có các phương
trình sau:
• Giới hạn nguyên liệu:
x 1 + x 2 + . . . + x 24 ≤ 8
y1 + y2 + . . . + y24 ≤ 5

• Giới hạn của lò đốt:


x i + x i−1 + yi ≤ 4 i = 1, 2, . . . , 24 (Xem x 0 = x 24 )

• Tiêu chuẩn nhựa hàng ngày:


0.8(x 1 + x 2 + . . . + x 24 ) + 0.3( y1 + y2 + . . . + y24 ) ≥ 6
Hàm mục tiêu cần tối thiểu hóa:
F (x 1 , x 2 , . . . , x 24 , y1 , . . . , y24 ) = 0.5(x 1 +x 2 +. . .+x 24 )+0.1( y1 + y2 +. . .+ y24 )

Hướng tới kỷ niệm 20 năm thành lập trường PTNK, ĐHQG TP HCM
Chuyên đề Toán học số 10 155

4.2. Quy hoạch tuyến tính nguyên


Chuyên đề Toán học Phổ thông năng khiếu TP.HCM số 10

Ví dụ 4.4. (Chia kẹo) Trong túi mẹ có 3 loại kẹo: 4 viên kẹo me, 3 viên
kẹo bạc hà và 6 viên kẹo sữa. Mẹ muốn chia kẹo cho Trung và Liên sao cho
tổng mức độ hài lòng của hai con là nhiều nhất. Biết rằng mức độ thích của
Trung với kẹo me, bạc hà và sữa lần lượt là a1 , a2 , a3 và Liên tương ứng là
b1 , b2 , b3 . Hãy tìm cách giúp mẹ chia kẹo.

Mô hình. Gọi x 1 , x 2 , x 3 lần lượt là số kẹo me, bạc hà và sữa Trung được
nhận và y1 , y2 , y3 là tương ứng của Liên. Khi đó x i , yi ∈ Z≥0 với i = 1, 2, 3.
Ta có các điều kiện sau:
 x 1 + y1 ≤ 4

x 2 + y2 ≤ 3
x 3 + y3

≤ 6

Hàm mục tiêu cần đạt giá trị lớn nhất:

F (x 1 , x 2 , x 3 , y1 , y2 , y3 ) = a1 x 1 + a2 x 2 + a3 x 3 + b1 y1 + b2 y2 + b3 y3

Ví dụ 4.5. (Cặp ghép lớn nhất) Một dịch vụ tìm bạn đời tổ chức một buổi
gặp gỡ cho m chàng trai và n cô gái độc thân. Mức độ tình cảm giữa chàng
trai i và cô gái j là L i j (có thể xem là mức độ hạnh phúc khi cả hai ở bên
nhau) (i = 1, 2, . . . , m và j = 1, 2, . . . , n). Hãy tìm cách ghép cặp mỗi chàng
trai với không quá một cô gái và mỗi cô gái với không quá một chàng trai
sao cho tổng mức độ hạnh phúc là nhiều nhất.

Mô hình. Trong bài toán này, một kế hoạch ghép cặp sẽ bao gồm m · n
quyết định: ghép hoặc không ghép chàng trai i và cô gái j. Mỗi quyết định
sẽ được biểu thị bằng một ẩn số nguyên x i j nhận 2 giá trị 0 hoặc 1. Ta có
x i j = 1 khi 2 người được ghép với nhau và x i j = 0 nếu ngược lại.
Điều kiện đầu tiên do vậy sẽ là:

0 ≤ x i j ≤ 1 ∀i, j

Tiếp theo, do mỗi chàng trai chỉ được ghép với không quá một cô gái:

x i1 + x i2 + . . . + x in ≤ 1 ∀i = 1, . . . , m

Lưu ý bất phương trình trên có nghĩa là trong các ẩn x i1 , x i2 , . . . , x in chỉ


có không quá một biến nhận giá trị 1, tức là có tối đa một cặp ghép chứa
chàng trai i. Tương tự với mỗi cô gái j:

x 1 j + x 2 j + . . . + x m j ≤ 1 ∀ j = 1, . . . , n

Hướng tới kỷ niệm 20 năm thành lập trường PTNK, ĐHQG TP HCM
156 Chuyên đề Toán học số 10

Cuối cùng, hàm mục tiêu sẽ là tổng mức độ hạnh phúc của những cặp
Chuyên đề Toán học Phổ thông năng khiếu TP.HCM số 10

được ghép với nhau. Lưu ý rằng với cặp i, j không được ghép với nhau,
x i j = 0 nên không làm thay đổi giá trị của tổng:

F (x 11 , x 12 , . . . , x mn ) = L11 x 11 + L12 x 12 + . . . + L mn x mn

Ví dụ 4.6. Trường tiểu học PiMA có 600 học sinh và 3 phòng máy tính, mỗi
phòng có sức chứa 40 học sinh. Mỗi giờ học Tin học kéo dài 2 tiếng liên tiếp
và diễn ra vào buổi sáng (7 giờ - 11 giờ). Chi phí hoạt động mỗi tiếng của
một phòng máy là 1 triệu đồng. Hãy sắp xếp giờ học Tin học sao cho mỗi
học sinh được học Tin học đúng 1 lần mỗi tuần và giảm tối đa chi phí phòng
máy.

Mô hình. Một giờ Tin học có thể bắt đầu vào 7, 8 hoặc 9 giờ sáng. Một
tuần học có 5 ngày, do đó có tổng cộng 3 · 5 = 15 giờ học tối đa. Gọi si j
là số học sinh học vào giờ học thứ i trong ngày thứ j (đánh số các giờ học
theo thứ tự thời gian: i = 7, 8, 9, j = 2, 3, 4, 5, 6). Trong mỗi ngày j, số học
sinh học Tin mỗi giờ như sau:

• Từ 7-8 giờ: s7 j học sinh.

• Từ 8-9 giờ: s7 j + s8 j học sinh.

• Từ 9-10 giờ: s8 j + s9 j học sinh.

• Từ 10-11 giờ: s9 j học sinh.

Bây giờ, gọi số phòng máy cần dùng trong khoảng i − (i + 1) giờ là ui j , ta
có:

• s7 j = 40u7 j − v7 j .

• s7 j + s8 j = 40u8 j − v8 j .

• s8 j + s9 j = 40u9 j − v9 j .

• s9 j = 40u10 j − v10 j .

Trong đó 0 ≤ vi j ≤ 39 ∀i. Từ đó suy ra:

• s7 j = 40u7 j − v7 j .

• s8 j = 40u8 j − v8 j − 40u7 j + v7 j .

• s8 j = 40u9 j − v9 j − 40u10 j + v10 j .

Hướng tới kỷ niệm 20 năm thành lập trường PTNK, ĐHQG TP HCM
Chuyên đề Toán học số 10 157

• s9 j = 40u10 j − v10 j .
Chuyên đề Toán học Phổ thông năng khiếu TP.HCM số 10

Vậy các ẩn của bài toán chính là si j , ui j và vi j . Ta có các phương trình ràng
buộc sau:

0 ≤ u7 j , u8 j , u9 j , u10 j ≤ 3



0 ≤ v7 j , v8 j , v9 j , v10 j ≤ 39




s7 j = 40u7 j − v7 j




s = 40u8 j − v8 j − 40u7 j + v7 j


 8j


s8 j = 40u9 j − v9 j − 40u10 j + v10 j , j = 2, 3, 4, 5, 6.


 s9 j = 40u10 j − v10 j

0 ≤ s7 j , s8 j , s9 j






 X6
(s7 j + s8 j + s9 j ) = 600




j=2

Hàm mục tiêu cần được tối thiểu hóa là:

F (u7,2 , u8,2 , . . . , u10,6 ) = u72 + u82 + . . . + u106

4.3. Bài tập


Lập mô hình cho các bài toán sau:

4.3.1. Một công ty dược phẩm pha chế một loại thuốc kháng sinh mới từ n
loại thành phần. 1g thành phần i có độ mạnh vi , giá thành ci và gây ra tác
dụng phụ có mức độ pi . Để được tiêu thụ trên thị trường, một viên thuốc
10g phải có độ mạnh ít nhất là V và gây tác dụng phụ không quá mức độ
P. Hãy tìm cách pha chế được viên thuốc đạt chuẩn và có giá thành thấp
nhất.

4.3.2. Trước trận chung kết giải vô địch Dota 2 thế giới, một nhà cái ước
tính xác suất xảy cho từng tỉ số và đưa ra tỉ lệ cược cho từng tỉ số. Mỗi tỉ
lệ cược có dạng “1 ăn x” với x là số thực dương. Hãy tìm cách để nhà cái
có thể thu được nhiều tiền nhất từ người đặt cược.

4.3.3. Tên trùm buôn lậu ma túy Donald Trung muốn vận chuyển 5 tấn
hàng từ North Carolina, Mỹ về TP.HCM trong vòng 3 ngày trước khi FBI
phát hiện. Có 3 đường bay từ Mỹ đến TP.HCM: thông qua Frankfurt (Đức),
Narita (Nhật) và Singapore. Giá vé máy bay và thời gian bay ở mỗi đường
như sau:

Hướng tới kỷ niệm 20 năm thành lập trường PTNK, ĐHQG TP HCM
158 Chuyên đề Toán học số 10

Đường bay Frankfurt Narita Singapore


Chuyên đề Toán học Phổ thông năng khiếu TP.HCM số 10

Giá vé (USD) 1000 1200 1400


Thời gian (giờ) 24 20 16

Để đảm bảo an toàn, ở mỗi đường bay, Trung chỉ bắt đầu chuyến vận
chuyển tiếp theo khi chuyến bay trước đã hạ cánh. Ngoài ra, mỗi ngày
không được vận chuyển quá 2 tấn ma túy. Hãy tìm cách để Trung chuyển
hàng thành công với chi phí tối thiểu.

4.3.4. Một công ty sử dụng m loại tài nguyên để sản xuất ra n loại sản
phẩm, mỗi loại sản phẩm tiêu tốn một lượng nhât định thuộc mỗi loại tài
nguyên. Biết số lượng mỗi loại tài nguyên có hạn, hãy lập kế hoạch sản
xuất cho công ty để đạt lợi nhuận cao nhất.

4.3.5. Có n ứng viên đến xin việc tại một công ty. Ứng viên i có trình độ
vi và đòi hỏi mức lương w i . Công ty muốn tuyển một nhóm làm việc cho
một dự án sao cho tổng trình độ trong nhóm phải ít nhất là V . Hãy lập
danh sách các thành viên trong nhóm dự trên các ứng viên xin việc sao
cho tổng mức lương là nhỏ nhất.

4.3.6. (Xếp lịch gieo trồng) Khang là một nông dân làm thuê cho Hưng,
một địa chủ giàu có. Hưng có tổng cộng 100 hecta đất và 2 giống lúa: lúa
ngắn ngày và lúa dài ngày. 1 hecta lúa ngắn ngày tiêu thụ 3 tấn phân bón
và 2 tấn thuốc trừ sâu, thu hoạch trong 3 tháng. 1 hecta lúa dài ngày tiêu
thụ 2 tấn phân bón và 1 tấn thuốc trừ sâu, thu hoạch trong 6 tháng. Sau
đây là sản lượng lúa tối thiểu mỗi tháng mà Khang phải thu hoạch:

Tháng 1 2 3 4 5 6 7 8 9 10 11 12
Sản lượng 40 45 60 50 55 65 45 55 50 60 45 50

Ban đầu, Hưng có 10 tấn phân bón và 8 tấn thuốc trừ sâu. Phân bón có giá
50 triệu/tấn và thuốc trừ sâu 30 triệu/tấn. Hãy lập kế hoạch gieo trồng
tối ưu cho Khang.

4.3.7. (Người bán hàng) Một người bán hàng phải di chuyển qua n thành
phố, mỗi thành phố đúng một lần để giao dịch. Khoảng cách từ thành phố
i tới j với i 6= j là ci j . Tìm đường đi cho người bán hàng để giảm tối đa
khoảng cách phải đi.

4.3.8. Một người đưa thư phải đi qua n địa chỉ và trở lại bưu điện. Khoảng
cách từ nhà i tới j với i 6= j là ci j . Khoảng cách từ bưu điện tới nhà i là di .
Hãy tìm cách để người đưa thư đi quãng đường ngắn nhất.

Hướng tới kỷ niệm 20 năm thành lập trường PTNK, ĐHQG TP HCM
Chuyên đề Toán học số 10 159

4.3.9. Một công ty xây dựng cần những thanh thép có độ dài l1 , l2 , . . . , l m .
Công ty cần bi thanh với độ dài l i với i = 1, 2, . . . , m. Các thanh thép dài
Chuyên đề Toán học Phổ thông năng khiếu TP.HCM số 10

có thể được cắt thành những thanh ngắn hơn. Trên thị trường hiện có các
loại thép với độ dài L1 , L2 , . . . , L n . Hãy tìm cách đặt mua các loại thép sao
cho công ty đáp ứng được nhu cầu và giảm tối đa chi phí.

4.3.10. (Bài toán phân công) Một công ty có n nhóm nhân viên và n
công việc cần thực hiện. Chất lượng của nhớm i khi làm công việc j là ci j .
Hãy tìm cách phân mỗi nhóm đúng 1 việc và mỗi việc đúng 1 nhóm sao
cho tổng chất lượng là cao nhất.

4.3.11. (Xếp lịch thi đấu) Một giải đấu bóng đá có 2n đội thi đấu theo
thể thức vòng tròn 1 lượt. Mỗi đội thi đấu 1 trận/tuần và mỗi cặp đấu
diễn ra đúng 1 lần. Theo ước tính của ban tổ chức, đội thứ k sẽ thu hút
được khoảng vk tiền lợi nhuận, bao gồm tiền vé xem trực tiếp và bản
quyền truyền hình. Giả sử trận đấu giữa đội thứ i và đội thứ k sẽ thu được
khoảng vi + vk vào thứ bảy và C(vi + vk ) vào Chủ Nhật với C > 1. Hãy tìm
cách sắp xếp để ban tổ chức thu được nhiều lợi nhuận nhất.

4.3.12. (Xếp lịch học) Một trung tâm dạy tiếng Anh mở m lớp, mỗi lớp
có một dung lượng học viên nhất định. Có n học viên đăng kí vào các khóa
học ở trung tâm, mỗi học viên cho biết mức độ ưa thích của mình đối với
từng khóa học (mức độ yêu thích 0 cũng đồng nghĩa với việc không đăng
ký). Hãy tìm cách xếp các học viên này vào các khóa học sao cho sĩ số mỗi
lớp không vượt quá dung lượng của lớp đó và mức độ ưa thích tổng cộng
cao nhất.

4.3.13. Liên là quản lí ở một nhà hàng. Nhà hàng của Liên hoạt động từ 9
giờ sáng đến 5 giờ chiều. Sau đây là số lượng nhân viên cần có mặt trong
nhà hàng vào mỗi giờ trong ngày:

Giờ 9-10 10-11 11-12 12-1 1-2 2-3 3-4 4-5


Nhân viên 4 3 4 6 5 6 8 8

Nhà hàng sử dụng 2 loại nhân viên: Nhân viên toàn thời gian (full-time)
làm từ 9am-5pm mỗi ngày, trừ 1 tiếng ăn trưa (có thể 12-1pm hoặc 1-
2pm), và nhận lương 50 nghìn/giờ. Nhân viên bán thời gian (part-time)
làm 4 tiếng liên tục, bắt đầu từ giờ nào cũng được và nhận lương 40
nghìn/giờ. Nhà hàng được quyền thuê tối đa 3 nhân viên part-time. Hãy
giúp Liên tìm cách thỏa mãn yêu cầu về số nhân viên với chi phí tối thiểu.

Hướng tới kỷ niệm 20 năm thành lập trường PTNK, ĐHQG TP HCM
160 Chuyên đề Toán học số 10
Chuyên đề Toán học Phổ thông năng khiếu TP.HCM số 10

Hướng tới kỷ niệm 20 năm thành lập trường PTNK, ĐHQG TP HCM
Chuyên đề Toán học Phổ thông năng khiếu TP.HCM số 10

Phép nghịch đảo

Nguyễn Huy Hoàng


(Lớp Chuyên Toán khóa 2012 − 2015)

Bài viết này xin được dành tặng cho thầy Lê Bá Khánh Trình và thầy Nguyễn Tăng
Vũ. Các thầy đã truyền cảm hứng cho tác giả qua những bài giảng cho đội tuyển
Phổ thông Năng khiếu dự thi VMO. Một số ý trong chuyên đề này chính là qua
các bài giảng đó được tác giả lưu lại.

I) Một số vấn đề liên quan đến phép nghịch đảo


Một số điều cần lưu ý khi đọc chuyên đề:

• Hãy làm thử bài toán ví dụ trước khi đọc lời giải.

• Hãy thử làm bài toán mà không sử dụng phép nghịch đảo.

1. Giới thiệu về phép nghịch đảo


Trong tất cả các phép biến hình, phép nghịch đảo là phép biến hình thú vị
nhất vì không những nó biến đường thẳng thành đường tròn và ngược lại,
mà còn là do đây là phép biến hình sơ cấp duy nhất có khả năng làm thay
đổi cấu trúc của hình vẽ, cho ta những hướng đi mới và kết quả mới trong
hình học.

1.1. Các tính chất đặc biệt của phép nghịch đảo
• Phép nghịch đảo bảo toàn phương tích.

• Phép nghịch đảo bảo toàn góc giữa hai đường thẳng, đường thẳng
và đường tròn, hai đường tròn.

161
162 Chuyên đề Toán học số 10

• Khi ta thay đổi phương tích nghịch đảo, ta sẽ ra một hình mới đồng
Chuyên đề Toán học Phổ thông năng khiếu TP.HCM số 10

dạng với hình cũ. Đồng thời, hợp của hai phép nghịch đảo sẽ là một
phép vị tự, do đó ta có thể nói phép nghịch đảo có yếu tố vị tự.

1.2. Hệ quả
Vì chỉ có hai yếu tố quyết định phép nghịch đảo là tâm và tỉ số phương
trích, mà việc thay đổi tỉ số phương tích vẫn giữ nguyên cấu trúc bài toán.
Do đó, cách chọn tâm nghịch đảo sẽ ảnh hưởng đến cấu hình.

2. Khởi động
Bài 1. Cho tam giác ABC nội tiếp (O). Hãy đếm có bao nhiêu điểm P thoả
mãn: Khi cho AP, BP, C P cắt (O) tại X , Y, Z tam giác X Y Z đều.

Gợi ý. Nghịch đảo tâm P, phương tích P/(O), ta sẽ thấy có 2 điểm P thoả
mãn tương ứng với hai giao điểm của 3 đường tròn Apollonius.

3. Phép nghịch đảo tâm nội tiếp

Hướng tới kỷ niệm 20 năm thành lập trường PTNK, ĐHQG TP HCM
Chuyên đề Toán học số 10 163

3.1. Giới thiệu và hệ quả


Chuyên đề Toán học Phổ thông năng khiếu TP.HCM số 10

Xét phép nghịch đảo tâm I, phương tích r 2 , ta có:


• Đường tròn nội tiếp được bảo toàn.

• Gọi X , Y, Z lần lượt là tiếp điểm của (I) trên BC, CA, AB ta có A →
A0 , B → B 0 , C → C 0 với A0 , B 0 , C 0 lần lượt là trung điểm Y Z, Z X , X Y.
Suy ra (ABC) → (A0 B 0 C 0 ) là đường tròn Euler của tam giác X Y Z.
Xét ví dụ rất quen thuộc sau:
Bài 2. Cho tam giác ABC nội tiếp (O), ngoại tiếp (I). Giả sử (I) tiếp xúc ba
cạnh tại X , Y, Z. Chứng minh OI là đường thẳng Euler của tam giác X Y Z.
Lời giải.
Xét phép nghịch đảo tâm I, phương tích r 2 , ta có đường tròn (ABC) sẽ
biến thành đường tròn Euler tam giác X Y Z. Do phép nghịch đảo có tính
chất bảo giác, nên đường thẳng nối tâm nghịch đảo và tâm đường tròn
thứ nhất (tạo thành một góc 90◦ ) cũng sẽ đi qua tâm đường tròn ảnh.

Như vậy, ta có OI đi qua tâm Euler tam giác X Y Z, suy ra OI chính là


đường thẳng Euler của tam giác này.
Bài 3. Cho tam giác ABC nội tiếp (O), ngoại tiếp (I). Giả sử (I) tiếp xúc ba
cạnh tại X , Y, Z. Đường tròn đường kính AI cắt (O) tại P và X H là đường
cao tam giác X Y Z. Chứng minh rằng

Hướng tới kỷ niệm 20 năm thành lập trường PTNK, ĐHQG TP HCM
164 Chuyên đề Toán học số 10

a) P, H, I thẳng hàng.
Chuyên đề Toán học Phổ thông năng khiếu TP.HCM số 10

b) P X là phân giác ∠BP C.

Lời giải.
a) Xét phép nghịch đảo tâm I, bán kính r 2 , ta có A biến thành A0 , P biến
thành P 0 , (AI) biến thành Y Z và (O) biến thành đường tròn Euler X Y Z. Vì
Y Z cắt đường tròn Euler tam giác X Y Z tại trung điểm và chân đường cao,
nên A0 , P 0 sẽ là trung điểm và chân đường cao của tam giác X Y Z.

Lại có I, A, A0 thẳng hàng, nên A0 chính là trung điểm Y Z, suy ra P 0 là chân


đường cao. Suy ra P, H, I thẳng hàng.
b) Gọi M là trung điểm cung BC. Qua phép nghịch đảo trên, ta có M 0
chính là giao điểm của trung trực Y Z với đường tròn Euler tam giác X Y Z
(khác A0 ).
Lại có X M 0 I H là hình thang cân, nên nó nội tiếp. Từ tính chất phép nghịch
đảo ta suy ra P X đi qua M .
Phép nghịch đảo tâm I còn biến tiếp điểm Mixtilinear Incircle với đường
tròn ngoại tiếp thành một điểm khác khá thú vị.

Bài 4. (Iran MO 1997) Cho tam giác ABC nội tiếp (O), ngoại tiếp (I). Giả
sử M , N là trung điểm cung nhỏ AC, AB. Cho P là một điểm di động trên
cung nhỏ BC, M P cắt C I tại T, N P cắt BI tại S. Chứng minh (P T S) đi qua
một điểm cố định trên (O).

Hướng tới kỷ niệm 20 năm thành lập trường PTNK, ĐHQG TP HCM
Chuyên đề Toán học số 10 165

Lời giải.
Chuyên đề Toán học Phổ thông năng khiếu TP.HCM số 10

Ngoài lời giải sử dụng đường tròn Apollonius và phép quay ra, ta có thể
tiếp cận bài toán này qua phép nghịch đảo.

Thật vậy, phép nghịch đảo tâm I biến M , N thành M 0 , N 0 là giao điểm của
trung trực Z X , X Y với đường tròn Euler tam giác X Y Z, (tam giác này định
nghĩa như trên), còn P biến thành P 0 là một điểm bất kì trên đường tròn
Euler của X Y Z.

Mặt khác P M , P N sẽ biến thành (M 0 I P 0 ) và (N 0 I P 0 ). Ta có T 0 , S 0 sẽ lần


lượt là giao của (M 0 I P 0 ) với C I, (N 0 I P 0 ) với BI.

Bằng biến đổi góc, ta biết được (P T S) sẽ đi qua I, nên ta chỉ cần chứng
minh T 0 S 0 đi qua điểm cố định. Qua phép nghịch đảo và lấy đối xứng qua
tâm đường tròn Euler, ta thu được bài toán mới sau đây:
Cho tam giác ABC có ba đường cao AD, BE, C F đồng quy tại H. Một điểm P
di động bất kì trên đường tròn Euler (Eu) của tam giác ABC. Gọi X là giao
của (F H P) và BH, Y là giao của (EH P) và C H. Chứng minh rằng X Y đi
qua P và một điểm cố định.

Hướng tới kỷ niệm 20 năm thành lập trường PTNK, ĐHQG TP HCM
166 Chuyên đề Toán học số 10
Chuyên đề Toán học Phổ thông năng khiếu TP.HCM số 10

Ta dễ dàng chứng minh bài toán này qua biến đổi góc. Điểm cố định chính
là trung điểm BC.

Biết được ảnh của tiếp điểm Mixtilinear Incircle, ta có thể biết được ảnh
của Mixtilinear Incircle.

Bài 5. (Định lý Lyness) Cho tam giác ABC nội tiếp (O) và ngoại tiếp (I).
Giả sử tồn tại đường tròn tiếp xúc AB, AC và tiếp xúc trong (O) lần lượt tại
P, Q, R. Chứng minh rằng PQ đi qua I.

Lời giải.
Qua I vẽ đường thẳng vuông góc phân giác trong ∠A sao cho nó cắt AB, AC
tại X , Y. Ta chứng minh đường tròn tiếp xúc AB, AC tại X , Y cũng tiếp xúc
với đường tròn (O).

Hướng tới kỷ niệm 20 năm thành lập trường PTNK, ĐHQG TP HCM
Chuyên đề Toán học số 10 167

Thật vậy, xét phép nghịch đảo tâm I, bài toán ban đầu sẽ trở thành:
Chuyên đề Toán học Phổ thông năng khiếu TP.HCM số 10

Cho tam giác ABC nội tiếp (O), qua O vẽ đường thẳng d song song với BC.
Gọi X , Y là hình chiếu của BC lên d. Ta sẽ có đường tròn tâm M trung điểm
BC, bán kinh M X = M Y sẽ tiếp xúc đường tròn Euler của tam giác ABC.

Chọn H là trực tâm ABC, H a là trung điểm HA, ta có

M X = M Y = M Ha = R

với R là bán kính của (O). Từ đây ta suy ra điều phải chứng minh.

Từ các bài toán trên, ta có các hệ quả sau:


Tiếp điểm của Mixtilinear Incircle và đường tròn ngoại tiếp sẽ biến thành
trung điểm nối trực tâm và đỉnh tam giác tương ứng. Kết quả này có thể suy
ra dựa trên tính chất tiếp điểm của Mixtilinear Incircle, tâm I và trung điểm
cung BAC thẳng hàng.
Tiếp theo sau đây là một bài toán khó hơn.

Bài 6. (Cosmin Pohoata, Taiwan TST 2014) Cho tam giác ABC nội tiếp (O),
ngoại tiếp (I) với các tiếp điểm là D, E, F, từ một điểm P di động trên (O) ta
vẽ các tiếp tuyến tới (I) sao cho nó cắt BC tại X , Y. Chứng minh (P X Y ) đi
qua điểm cố đinh.

Hướng tới kỷ niệm 20 năm thành lập trường PTNK, ĐHQG TP HCM
168 Chuyên đề Toán học số 10

Lời giải.
Chuyên đề Toán học Phổ thông năng khiếu TP.HCM số 10

Ấn tượng đầu tiên của tác giả khi giải bài toán này là rất khó khai thác
tính bất kì của điểm P và tính chất của điểm X , Y. Thêm nữa, các góc trong
tam giác P X Y không tính được theo các góc của tam giác ABC nên chứng
minh thông qua biến đổi góc là không thể. Một trong những các quy tắc
tiếp cận các bài toán hình học phẳng là phải khai thác tối đa các dữ kiện
đã cho, nếu không ta nên chuyển cấu hình ban đầu thành một cấu hình
khác dễ xử lí hơn.

Không khai thác được cặp đường tròn (O), (I), ta sẽ sử dụng phép nghịch
đảo tâm I để thu cặp đường tròn (O), (Eu). Gọi U, V là hai tiếp điểm trên
(I) của P X , PY. Phép nghịch đảo tâm I biến P → P 0 là một điểm trên
đường tròn Euler tam giác nội tiếp X → X 0 trung điểm DX , Y → Y 0 trung
điểm DY. Ta cần chứng minh (X 0 Y 0 P 0 ) đi qua một điểm cố định.
Gọi H là trực tâm DE F, H d là trung điểm DH. Ta chứng minh H d thuộc
(X 0 Y 0 P 0 ). Thật vậy, ta có
(H d X 0 , H d Y 0 ) = (H U, H V ).
Vì H là trực tâm DE F và P 0 thuộc (Eu), nên nếu ta lấy H 0 đối xứng với H
qua P 0 , ta vẫn có H 0 thuộc (Eu). Do đó, ta có
(H U, H V ) = (H 0 V, H 0 U) = (DV, DU) = (P 0 X 0 , P 0 Y 0 ).
Từ hai đẳng thức trên, ta có (H d X 0 , H d Y 0 ) = (P 0 X 0 , P 0 Y 0 ), suy ra điều phải
chứng minh.

Hướng tới kỷ niệm 20 năm thành lập trường PTNK, ĐHQG TP HCM
Chuyên đề Toán học số 10 169

Bài toán của Cosmin Pohoata có thể coi là một trong những ví dụ tiêu biểu
Chuyên đề Toán học Phổ thông năng khiếu TP.HCM số 10

nhất của phép nghịch đảo tâm I và cho ta thấy tại sao đây là một công cụ
khả dụng:

• Phép nghịch đảo hoán đổi cấu hình từ cặp đường tròn (O), (I) thành
(O), (Eu).

• Việc đưa về cấu hình (O), (Eu) khiến ta có thể biến đổi góc.

• Ta tìm được một cách để khai thác tính chất của tiếp điểm Mixtilinear
Incircle. Thật vậy, mặc dù ta có thể tính được tỉ lệ độ dài của hai dây
tạo bởi tiếp điểm Mixtilinear Incircle, nhưng ta không thể tính được
hai góc tạo theo các góc của tam giác ABC.

Do đó, Mixtilinear Incircle thông thường được khai thác dưới vai trò
là một tâm đồng dạng xoắn (ta có thể thấy điều này qua bài Iran
1997, với cách chứng minh phổ biến nhất là sử dụng đường tròn
Apollonius/chứng minh tỉ số cố định). Nhưng trong bài toán của
Cosmin, điều này gần như không thể do tính bất kì của P và cách
xác định của X , Y.

Ngoài ra, bài toán trên còn có một điểm đặc biệt nữa là cấu hình của nó
gắn liền với tính chất của Poncelet porism, một kết quả khá thú vị mà tác
giả sẽ giới thiệu trong phần tiếp theo.

4. Poncelet porism
4.1. Giới thiệu Poncelet porism
Đầu tiên, trước khi giới thiệu định lý này, người đọc hãy vẽ tam giác ABC
có đường tròn ngoại tiếp (O) và đường tròn nội tiếp (I). Sau đó, xoá tam
giác ABC đi và chọn điểm P bất kì trên đường tròn (O) và vẽ tiếp tuyến
P X , PY đến (I). Câu hỏi đặt ra là: X Y có luôn tiếp xúc (I)?
Đương nhiên là câu trả là có, và điều này luôn đúng với mọi P di động
trên (O). Nhưng khi đảo ngược vấn đề, ta biết rằng điều này không luôn
đúng với mọi cặp (O1 ), (O2 ) chứa nhau. Như vậy, điều này chỉ đúng với cặp
đường tròn nội tiếp - đường tròn ngoại tiếp của một tam giác nào đó. Vây,
hai đường tròn có mối quan hệ gì đặc biệt như vậy?
Đó chính là hệ thức Euler:

OI 2 = R2 − 2Rr.

Hướng tới kỷ niệm 20 năm thành lập trường PTNK, ĐHQG TP HCM
170 Chuyên đề Toán học số 10

Với những quan sát và nhận xét như vậy, nhà toán học Poncelet đã phát
Chuyên đề Toán học Phổ thông năng khiếu TP.HCM số 10

biểu một nhận xét sau:


Poncelet porism: Cho một cặp đường tròn chứa nhau mà tồn tại một đa
giác n cạnh nhận chúng lần lượt là đường tròn ngoại tiếp và đường tròn nội
tiếp, thì mọi điểm trên đường tròn nằm ngoài sẽ là một điểm của một đa giác
lưỡng tâm n cạnh.
Đa giác lưỡng tâm n cạnh là một phạm vi còn mới trong hình học sơ cấp,
nhưng ta có thể thấy Poncelet porism đúng với trường hợp n = 4. Trong
trường hợp này, hệ thức liên hệ bán kính hai đường tròn và khoảng cách
hai tâm là
1 1 1
+ = .
(R − x)2 (R + x)2 x2
Ở chuyên đề này, tác giả xin trình bày một chứng minh cho n = 3 bằng
phép nghịch đảo tâm I.
Lời giải.
Xét phép nghịch đảo tâm I, bán kính r 2 , ta có P biến thành P 0 là trung
điểm M N (M , N là tiếp điểm trên (I) của hai tiếp tuyến), P X , PY lần lượt
bị biến thành hai đường tròn đường kính I M , I N . Do X , Y là giao điểm
của P M , P N với (O), nên qua phép nghịch đảo, ảnh của nó (X 0 , Y 0 ) sẽ là
giao điểm thứ hai của đường tròn đường kính I M , I N với đường tròn ảnh
của (O).

Hướng tới kỷ niệm 20 năm thành lập trường PTNK, ĐHQG TP HCM
Chuyên đề Toán học số 10 171

Vì tồn tại một tam giác ABC nhận (O), (I) lần lượt là đường tròn ngoại tiếp
và đường tròn nội tiếp, do đó đường tròn ảnh của (O) sẽ là đường tròn
Chuyên đề Toán học Phổ thông năng khiếu TP.HCM số 10

Euler của tam giác tạo từ ba tiếp điểm của (I) với ABC. Suy ra, đường tròn
ảnh của (O) (gọi đường tròn này là (E)) có bán kính 2r .
Ta sẽ chứng minh, từ mọi điểm P 0 trên (E), kẻ dây M N của (I) vuông góc
I P 0 , lấy Q0 , R0 là giao điểm thứ hai của đường tròn đường kính I M , I N với
(E). Ta sẽ chứng minh MQ0 , N R0 cắt nhau trên (I).
Thật vậy,
Gọi H là tâm vị tự biến (E) thành (I) theo tỉ số 2. Rõ ràng là tồn tại H như
vậy do (E) có bán kính 2r . Lấy T 0 đối xứng với H qua P 0 , ta có T thuộc (I)
do H là tâm vị tự tỉ số 2 của (E) và (I). Lấy T đối xứng với T 0 qua I, ta
chứng minh H chính là trực tâm của tam giác M N T.
Ta chỉ cần chứng minh T H⊥M N . Thật vậy, ta có I P 0 là đường trung bình
tam giác H T T 0 , suy ra T H k I P 0 vuông góc M N . Do T M T 0 N là tứ giác nội
tiếp có T T 0 là đường kính, nên điểm H đối xứng với T 0 qua trung điểm
M N sẽ phải là trực tâm tâm giác M N T.
Có H là trực tâm, ta suy ra được (E) chính là đường tròn Euler tam giác
M N T. Từ đây ta có X 0 , Y 0 chính là trung điểm T M , T N .
Qua phép nghịch đảo ban đầu, mệnh đề X Y luôn tiếp xúc (I) tương đuong
với (I X 0 Y 0 ) tiếp xúc với (I), mà điều này đúng do X 0 , Y 0 là trung điểm
T M, T N.
Vậy Poncelet porism được chứng minh với n = 3.

4.2. IMO 2015 và bổ đề đường tròn Euler


Trước khi bắt đầu phần này, tác giả mời người đọc hãy thử làm bài toán số
3 của IMO 2015.
Ta biết rằng, phép nghịch đảo tâm I sẽ hoán đổi cấu hình từ cặp đường
tròn nội tiếp - ngoại tiếp thành ngoại tiếp - Euler. Nhìn dưới góc nhìn đại
số, đường tròn Euler có nhiều tính chất tương tự như đường tròn nội tiếp.
Thật vậy, chỉ cần lấy bất kì cặp đường tròn chứa nhau có bán kính R và
2R, ta đều có thể xây dựng một tam giác nhận hai đường tròn đó là đường
tròn ngoại tiếp và đường tròn Euler.
Qua phép chứng minh trên, ta thấy được rằng phép nghịch đảo tâm (I)
bắc một cây cầu chuyển Poncelet porism trường hợp n = 3 thành một
kết quả liên quan tới đường tròn Euler. Ta có các kết quả sau với ký hiệu
(O), (E), (I) lần lượt là các đường tròn ngoại tiếp, Euler, và nội tiếp của
cùng một tam giác nào đó.

Hướng tới kỷ niệm 20 năm thành lập trường PTNK, ĐHQG TP HCM
172 Chuyên đề Toán học số 10

1. Từ mọi điểm P trên (E), kẻ dây M N của (I) vuông góc I P, lấy Q, R là
giao điểm thứ hai của đường tròn đường kính I M , I N với (E). Ta sẽ
Chuyên đề Toán học Phổ thông năng khiếu TP.HCM số 10

chứng minh MQ, N R cắt nhau trên (I) tại T.

Vì (E) cắt một cạnh tại trung điểm và chân đường cao, nên ta cũng
có một kết quả tương tự cho chân đường cao.

2. Từ mọi điểm P trên (E), kẻ 2 dây M N , PQ của (I) không là đường


kính vuông góc nhau và cắt nhau trên (E) tại X . Ta có (E) chính là
đường tròn Euler của M N P hoặc M NQ.

Từ hai kết quả này, chúng ta cùng quay lại với bài toán IMO 2015.

Bài 7. (IMO 2015) Cho tam giác ABC có trực tâm H, đường cao AD. Chọn
P trên đường tròn (O) ngoại tiếp tam giác ABC sao cho AP H vuông. Chọn
Q trên đường tròn ngoại tiếp tam giác ABC sao cho PQH vuông. Gọi M là
trung điểm BC, chứng minh rằng (PQH) tiếp xúc với (M DQ).

Lời giải.
Đối với các bài toán chứng minh hai đường tròn tiếp xúc nhau, ta thường
có ba cách tiếp cận:

1. Nếu 2 đường tròn có ít nhất 1 giao điểm, chứng minh tồn tại 1 phép
vị tự biến đường tròn này thành đường tròn kia.

2. Nghịch đảo để biến 1 trong hai đường tròn thành đường thẳng.

3. Dùng định lý Casey hoặc các công cụ đại số.

Cấu hình đã cho có sự xuất hiện của điểm P, giao điểm thứ hai của (AH)
và (O), nên ta đều có thể tiếp cận theo phương án (1) hoặc (2) (nên nhớ
rằng P là tâm vị tự quay/vị tự của rất nhiều tam giác).
Tại kì thi IMO 2015, tác giả không tìm được lời giải theo cách tiếp cận thứ
nhất nên tác giả lựa chọn sử dụng nghịch đảo tâm H.
Áp dụng phép nghịch đảo tâm H, ta có

A → D, P → M , (O) → (Eu)

với (Eu) là đường tròn Euler tam giác ABC và Q → Q0 . Ta có

(PQH) → MQ0 và (M DQ) → (PAQ0 ).

Bài toán trở thành chứng minh MQ0 tiếp xúc (PAQ0 ).

Hướng tới kỷ niệm 20 năm thành lập trường PTNK, ĐHQG TP HCM
Chuyên đề Toán học số 10 173

Vì AP vuông góc với P M , MQ0 vuông góc với P M (do H P · H M = HQ · HQ0


suy ra ∠H MQ0 = ∠HQP = 90◦ ) do đó MQ0 song song với AP.
Chuyên đề Toán học Phổ thông năng khiếu TP.HCM số 10

Như vậy, (APQ0 ) tiếp xúc MQ0 khi và chỉ khi Q0 là trung điểm dây kẻ từ M
vuông góc với H M .

Và đây chính là kết quả 1 nêu trên. Ta có thể thấy sự hiện diện của bổ đề
trong bài toán này.

Nhưng qua bài toán này, ta có thể thấy phần nào cách tác giả bài hình học
IMO 2015 xây dựng bài toán. Nếu ta gọi giao điểm của (X H) với (O) với
H là trực tâm tam giác X Y Z nội tiếp (O) là điểm "đẹp" thì ta có một số
quan sát sau:

1. Trong bài toán IMO 2015, điểm Q chính là một điểm đẹp. Thật vậy,
cho MQ0 cắt (O) tại R, S, Q chính là điểm đẹp của tam giác PRS.
Thật ra, ta còn có thể có một kết quả tổng quát hơn:

2. Cho một đường (O) và một dây BC bất kì, lấy (O0 ) đối xứng với (O)
qua BC, M trung điểm BC, P thuộc (O0 ) sao cho P nằm trong (O).
Ta có nếu Q là giao của vector M P và (O) thì Q chính là điểm đẹp.

Tại sao ta lại quan tâm họ các điểm này (điểm đẹp). Một tiếp cận tự nhiên
khi giải các bài toán hình học này là cố gắng đưa về các cấu hình quen
thuộc. nhận diện cấu hình cũ trong bài toán mới để khai thác các tính chất

Hướng tới kỷ niệm 20 năm thành lập trường PTNK, ĐHQG TP HCM
174 Chuyên đề Toán học số 10

của yếu tố hình học đó. Việc ta nhận diện điểm Q là điểm đẹp giống như
Chuyên đề Toán học Phổ thông năng khiếu TP.HCM số 10

ta cho điểm Q thừa kế các tính chất của điểm đẹp và xử lí điểm Q dựa
trên các tính chất đó. Thật vậy, một tính chất của điểm đẹp là nó nằm trên
đường thẳng nối trung điểm một cạnh và trực tâm tam giác, nghĩa là ta có
thể suy ra Q0 là trung điểm RS bằng cách chứng minh Q là điểm đẹp.
Ngoài ra, việc xác định điểm đẹp còn cho ta thấy tính đại số của hình học
là tính tham số hoá. Nó có ý nghĩa là khi ta thay đổi một số điểm một
cách bất kì nhưng điểm đó vẫn nằm trên một quỹ đạo nhất định thì kết
quả không thay đổi. Một số ví dụ của tính tham số hoá là các bài toán tìm
điểm cố định khi điểm X di động trên đường tròn/đường thẳng cho trước,
...
Bài toán IMO 2015 cũng có tính tham số hoá, mặc dù tinh tế hơn: Tam
giác PRS và tam giác ABC tuy khác nhau nhưng vài trò như nhau. Tác giả
của bài IMO 2015 đã lợi dụng tính tham số hoá của họ điểm đẹp và qua
đó sáng tác ra bài toán hình học mới.
Với góc nhìn đại số/tham số hoá, ta có thể sử dụng linh hoạt hơn công cụ
hình học và đại số và có thể trừu tượng hoá các bài toán phức tạp. Trước
khi kết thúc bài viết, tác giả muốn chỉ ra mối quan hệ khăng khít của đại
số và hình học: Việc tham số hoá bài toán sẽ dẫn đến khả năng trừu tượng
hoá các vấn đề, và từ đó ta có thể nhận ra được quy luật/các tính chất
đơn giản bị che giấu. Những kết quả khai thác từ Poncelet porism trong
bài viết tuy còn đơn giản, nhưng cho thấy được sự thú vị trong cái bất kì,
cái bất biến trong cái thường biến.

5. Bài tập
Bài 1. (Nguyễn Huy Hoàng) Cho tam giác ABC nội tiếp (O), ngoại tiếp (I).
Giả sử (I) tiếp xúc ba cạnh BC, CA, AB lần lượt tại D, E, F. Vẽ đường tròn
(Wa ) đi qua B, C và tiếp xúc (I) tại X , D0 đối xứng với D qua AI. Xác định
Y, Z, E 0 , F 0 tương tự. Chứng minh D0 X , E 0 Y, F 0 Z đồng quy trên OI.

Bài 2. (Sharygin 2016, Final) Cho tam giác ABC nội tiếp (O), ngoại tiếp
(I). Gọi D, E, F là tiếp điểm của (I) trên ba cạnh BC, CA, AB tương ứng. Gọi
X , Y, Z lần lượt là chân đường phân giác trong của ∠A, ∠B, ∠C. Chứng minh
rằng tâm đẳng phương của (ADX ), (AEY ), (AF Z) thuộc OI.

Bài 3. (Sharygin 2016, Final) Cho tam giác ABC nội tiếp (O), ngoại tiếp
(I). Qua I vẽ đường thẳng vuông góc với IO cắt phân giác ngoại góc A và
BC lần lượt tại X và Y. Tính II YX .

Hướng tới kỷ niệm 20 năm thành lập trường PTNK, ĐHQG TP HCM
Chuyên đề Toán học số 10 175

Bài 4. (Sharygin 2016, Final) Cho tam giác ABC nội tiếp (O), ngoại tiếp
(I). Giả sử tồn tại một tam giác A0 B 0 C 0 khác ABC nhận (O), (I) là các đường
Chuyên đề Toán học Phổ thông năng khiếu TP.HCM số 10

tròn ngoại tiếp, nội tiếp. Chứng minh rằng với mọi điểm P nằm trong ABC
và A0 B 0 C, tổng khoảng cách từ P đến các cạnh của ABC bằng với tổng khoảng
cách từ P đến các cạnh của A0 B 0 C 0 .
Bài 5. (Sharygin 2016, First Round) Cho tam giác ABC nội tiếp (O), ngoại
tiếp (I). Gọi (A0 ) là đường tròn tiếp xúc trong (O) tại A và tiếp xúc ngoài (I)
tại A1 . Xác định (B 0 ), (C 0 ) tương tự. Chứng minh rằng
a) AA1 , BB1 , C C1 đồng quy tại P.

b) P thuộc OI.

II) Phép nghịch đảo đối xứng

1. Khởi động và nên đọc chuyên đề này thế nào?


Chúng ta bắt đầu bằng bài toán sau đây
Bài 1. Cho tam giác ABC và M là trung điểm BC. Gọi I1 , J1 là tâm đường
tròn nội tiếp và bàng tiếp góc M của ABM , xác định I2 , J2 tương tự. Chứng
minh rằng bốn điểm I1 , I2 , J1 , J2 đồng viên.
Lời giải.

Ta cần chứng minh M A · M B = M I1 · M J1 . Thật vậy, ta có bổ đề sau:


Cho tam giác ABC có đường tròn nội tiếp (I) và đường tròn bàng tiếp góc
A là (I a ), ta có
AB · AC = AI · AI a .

Hướng tới kỷ niệm 20 năm thành lập trường PTNK, ĐHQG TP HCM
176 Chuyên đề Toán học số 10
Chuyên đề Toán học Phổ thông năng khiếu TP.HCM số 10

Bổ để này có thể chứng minh dễ dàng bằng tam giác đồng dạng. Áp dụng
vào bài toán trên, ta sẽ thấy được

M I1 · M J1 = M A · M B = M A · M C = M I2 · M J2 .

Từ đó suy ra điều phải chứng minh.

Một số điều cần lưu ý khi đọc chuyên đề:

• Hãy làm thử bài toán ví dụ trước khi đọc lời giải.

• Hãy thử làm bài toán mà không sử dụng phép nghịch đảo.

• Sau mỗi ví dụ, nên phân tích ý tưởng của ví dụ đó vì chuyên đề này
bao hàm nhiều ý tưởng khác nhau của cùng một phương pháp.

2. Giới thiệu phép nghịch đảo đối xứng

2.1. Tính chất


Phép nghịch đảo đối xứng bao gồm hai phép biến hình:

1. Phép nghịch đảo tâm A, phương tích AB · AC.

2. Phép đối xứng phân giác ∠BAC.

Hướng tới kỷ niệm 20 năm thành lập trường PTNK, ĐHQG TP HCM
Chuyên đề Toán học số 10 177

Từ định nghĩa, ta có thể thấy được phép biến hình này cho ta những tính
Chuyên đề Toán học Phổ thông năng khiếu TP.HCM số 10

chất sau:

1. Hoán đổi B và C.

2. BC biến thành (ABC).

3. Vì AI · AI a = AB · AC với I và I a là tâm đường tròn nội tiếp và bàng


tiếp, nên I và I a cũng sẽ bị hoán đổi cho nhau qua phép nghịch đảo
đối xứng.

2.2. Hệ quả
• Do hợp với phép đối xứng trục, nếu B → B 0 và C → C 0 thì ta có BC
song song với B 0 C 0 (do tính đối song).

• Với mọi X thuộc BC, ta có X → X 0 thuộc (ABC) và AX , AX 0 đối song.


Lúc ấy ta có một phép đồng dạng xoắn tâm A biến ABX thành AX 0 C
và AC X thành AX 0 B.

3. Ví dụ
Phép nghịch đảo đối xứng là một công cụ rất mạnh. Chúng ta xem qua
một số ví dụ dưới đây.

Bài 2. Cho tam giác ABC nội tiếp (O), ngoại tiếp (I), T là tiếp điểm của
Mixtilinear Incircle và (O), N là tiếp điểm của đường tròn bàng tiếp tâm A
trên BC. Chứng minh AN và AT là 2 đường đẳng giác.

Lời giải.
Để ví dụ về phép biến hình nêu trên, đầu tiên ta sẽ giải bài toán này sử
dụng phép nghịch đảo đối xứng.
Xét phép nghịch đảo tâm A, phương tích AB · AC, lấy đối xứng qua phân
giác trong AI, ta có: B ↔ C, (ABC) ↔ BC, I ↔ I a .
Gọi D, E là tiếp điểm của đường tròn bàng tiếp (I a ) trên AC, AB, X , Y là
tiếp điểm của Mixtilinear Incircle trên AB, AC. Ta có X , I, Y thẳng hàng
theo định lý Lyness.
Qua phép nghịch đảo đối xứng, ta có X 0 , I a , Y 0 , A đồng viên, X 0 thuộc AC
và Y 0 thuộc AB. Do X Y ⊥AI tại I nên ta có X 0 , Y 0 là hình chiếu của I a trên
AC, AB, suy ra X 0 chính là D và Y 0 chính là E.

Hướng tới kỷ niệm 20 năm thành lập trường PTNK, ĐHQG TP HCM
178 Chuyên đề Toán học số 10
Chuyên đề Toán học Phổ thông năng khiếu TP.HCM số 10

Từ đây ta có (I a ) ↔ Mixtilinear Incircle qua phép nghịch đảo, suy ra


T ↔ N do (ABC) ↔ BC và đây là tiếp điểm tương ứng của Mixtilinear
Incircle và (I a ) trên (ABC) và BC.
Bài toán này còn có thể giải mà không sử dụng nghịch đảo. Để ý rằng ta
có thể tính được tỉ số CBTT và ta cũng có AB + N B = AC + N C, lại thêm cấu
hình đẳng giác nên ta có thể chứng minh được bài toán thông qua định lý
Ptoleme.
Vì có phép biến hình này đã bao gồm tính đối xứng, nên việc ứng dụng
phép nghịch đảo đối xứng vào chứng minh hai đường đẳng giác là điều
khá tự nhiên.
Bài 3. (Serbian MO 2013) Cho tam giác ABC nội tiếp (O) có đường tròn
Euler (E). Giả sử (E) cắt (BOC) tại hai điểm X , Y phân biệt. Chứng minh
rằng ∠BAX = ∠CAY.
Lời giải.
Đây là một bài toán khá thú vị. Trước hết, hai góc BAX và hai góc CAY
không thể tính được theo ba góc chuẩn A, B, C, cho nên việc chứng minh
trực tiếp bằng biến đổi góc gần như không thể. Thêm nữa là AX , AY cũng
rất khó tính bằng công cụ lượng giác, do đó cách tiếp cận lượng giác cũng
rất khó khăn. Khi đối mặt với các kiểu cấu hình khó chịu như vậy, sử dụng
phép biến hình để tìm ra các bước đi đột phá là một giải pháp.

Hướng tới kỷ niệm 20 năm thành lập trường PTNK, ĐHQG TP HCM
Chuyên đề Toán học số 10 179

Để ý rằng, nếu tồn tại một phép nghịch đảo đối xứng tâm A biến (E) thành
(BOC) và ngược lại thì giao điểm của hai đường tròn sẽ nằm trên hai tia
Chuyên đề Toán học Phổ thông năng khiếu TP.HCM số 10

đẳng giác.
Thật vậy, vì hai đường tròn là ảnh của nhau nên tập giao điểm {X ; Y } của
chúng sẽ bất động, tuy nhiên vì X khác Y và X , Y không thể đồng thời
nằm trên đường phân giác góc A nên X sẽ biến thành Y và ngược lại, suy
ra AX , AY đẳng giác.

Bây giờ ta sẽ chọn một phép nghịch đảo đối xứng thoả mãn điều này. Xét
phép nghịch đảo tâm A, phương tích AB · AC/2, đối xứng qua phân giác
trong góc A, ta có M ↔ C, N ↔ B với M , N là trung điểm AB, AC.
1
Đồng thời, lấy H là chân đường cao kẻ từ A. Vì AH, AO đẳng giác và AO = 2
đường kính AD của (O) nên
1 1
AH · AD = AH · AO = AB · AC.
3 2
Do đó H ↔ O. Kết hợp ba điều này, ta có (E) ↔ (BOC).

Một kĩ thuật thường thấy của phép nghịch đảo là chọn bán kính nghịch
đảo sao cho một số đường tròn được bảo toàn, dẫn tới một hệ quả là giao
điểm của các cặp đường tròn được bảo toàn. Một bài toán đơn giản cho ví
dụ này là:

Hướng tới kỷ niệm 20 năm thành lập trường PTNK, ĐHQG TP HCM
180 Chuyên đề Toán học số 10

(Olympic KHTN 2014) Cho tam giác ABC. Trên đoạn thẳng AC lấy điểm P
PA QP
Chuyên đề Toán học Phổ thông năng khiếu TP.HCM số 10

và trên đoạn thẳng P C lấy điểm Q sao cho = . Đường tròn ngoại
PC QC
tiếp tam giác ABQ cắt BC tại R khác B. Gọi S là giao điểm khác P của hai
đường tròn ngoại tiếp các tam giác PAB, PQR. Chứng minh rằng tam giác
C PS cân.
Sau đây, chúng ta sẽ đi sâu hơn vào đi tìm các ảnh của các điểm qua phép
nghịch đảo. Đầu tiên sẽ là một bài toán từ đề thi chọn đội tuyển Iran.
Bài 4. (Iran TST 2014) Cho tam giác ABC có tâm nội tiếp I, qua I kẻ đường
thẳng vuông góc với AI sao cho nó cắt AB, AC tại B 0 , C 0 tương ứng. Giả sử B 00
và C 00 là các điểm nằm trên tia BC và C B sao cho BB 00 = BA và C C 00 = CA,
ta lấy T là giao điểm của (AB 0 B 00 ) và (AC 0 C 00 ). Chứng minh rằng tâm của
đường tròn (AI T ) thuộc BC.
Lời giải.
Bài toán này có hai vấn đề cần suy nghĩ: Một là điểm T có tính chất gì, và
hai là tính chất tâm đường tròn ngoại tiếp tam giác AI T thuộc BC tương
đương với điều gì. Cả hai điều này có thể tiếp cận theo nhiều hướng khác
nhau, do vậy mà bài toán có ít nhất ba cách giải.
Đầu tiên ta sẽ suy luận về tính chất tâm (AI T ) thuộc BC, nếu ta nghĩ về
bản chất của vấn đề theo góc nhìn mối quan hệ của đường tròn này và
đường thẳng là gì thì ta có (AI T ) tạo một góc 90◦ với BC.
Nhắc lại góc của một đường tròn và một đường thẳng: Nếu một đường
thẳng có ít nhất một giao điểm với đường tròn thì góc giữa đường tròn và
đường thẳng được xác định bởi góc tạo bởi đường thẳng đó và tiếp tuyến
tại 1 giao điểm của đường thẳng và đường tròn.
Quay trở lại bài toán, quan hệ vuông góc của (AI T ) và BC gợi ta đến việc
biến đổi cấu hình này thành việc chứng minh một tính chất khác tương
đương như là hai đường thẳng vuông góc hoặc là một đường thẳng đi qua
tâm một đường tròn khác, và tính chất bảo giác của phép nghịch đảo cho
phép ta làm điều này.
Thật vậy, xét phép nghịch đảo tâm A, phương tích AB · AC và lấy đối xứng
qua phân giác trong góc A, ta có: B 0 ↔ Y, C 0 ↔ Z (với Y, Z là tiếp điểm
của đường tròn bàng tiếp góc A lên AC, AB tương ứng), I ↔ I a là tâm bàng
tiếp góc A, B 00 ↔ U và C 00 ↔ V. Điều chúng ta cần làm sẽ là xác định các
điểm B 00 và C 00 .
Từ hệ quả của phép nghịch đảo đối xứng, ta có U, V thuộc (O) và AB 00 đẳng
giác với AU, AC 00 đẳng giác với AV, suy ra ∠CAU = ∠BAB 00 = 90◦ − ∠B2 suy
ra S là trung điểm cung ABC, tương tự, ta có V là trung điểm cung AC B.

Hướng tới kỷ niệm 20 năm thành lập trường PTNK, ĐHQG TP HCM
Chuyên đề Toán học số 10 181
Chuyên đề Toán học Phổ thông năng khiếu TP.HCM số 10

Vậy (AB 0 B 00 ) ↔ U Y và (AC 0 C 00 ) ↔ V Z suy ra T ↔ T 0 là giao của U Y và


V Z. Vậy điều ta cần chứng minh tương đương với chứng minh O thuộc
I a T 0 . Điều này đúng do T 0 chính là tâm vị tự trong của (O) và (I a ).
Một lời giải thể hiện các thế mạnh của phép nghịch đảo đối xứng:tính bảo
giác và tính đẳng giác của các ảnh. Ngoài điểm B 00 và C 00 , ta có thể tìm
thấy ảnh của một số điểm đặc biệt trên đường tròn
• Chân tiếp tuyến tại A trên BC ↔ P sao cho AP k BC.

• Trung điểm cung BAC ↔ chân phân giác ngoài góc A. Giao điểm hai
tiếp tuyến tại B, C ↔ giao của đường tròn (H BC) và trung tuyến A
(H là trực tâm).

• Trung điểm cung nhỏ A ↔ P thuộc tia đối C B sao cho CA = C P.


Chân đường cao tại A ↔ điểm đối xứng với A qua O, và O ↔ điểm
đối xứng với A qua BC.
Cùng ý tưởng trên, phép nghịch đảo đối xứng biến trung điểm cung thành
các điểm dễ khai thác.

Hướng tới kỷ niệm 20 năm thành lập trường PTNK, ĐHQG TP HCM
182 Chuyên đề Toán học số 10

Bài 5. (Sharygin 2014, Final) Cho tam giác ABC thoả mãn AB +AC = 3BC.
Chuyên đề Toán học Phổ thông năng khiếu TP.HCM số 10

Giả sử M , N là trung điểm cung BCA và ABC. Chứng minh rằng M N đi qua
tâm đường tròn nội tiếp I của tam giác ABC.

Lời giải.
Xét phép nghịch đảo tâm A, phương tích AB · AC và lấy đối xứng qua phân
giác trong góc A, ta có M ↔ A M và N ↔ AN với A M , AN lần lượt là điểm
đối xứng với A qua C I và BI (theo kết quả của bài 3), suy ra

IA = IA M = IAN .

Gọi I a là tâm bàng tiếp góc A, T trung điểm cung nhỏ BC. Từ điều kiện
AB + AC = 3BC, ta suy ra được IA = I I a (tính chất quen thuộc, có thể
chứng minh bằng định lý Ptoleme).

Kết hợp hai kết quả trên, ta có IA = IA M = IAN = I IA, suy ra A, A M , AN , IA


nằm trên đường tròn đường kính AIA. Do đó, qua phép nghịch đảo ta suy
ra được M , I, N cùng nằm trên đường thẳng vuông góc với AI.

Hướng tới kỷ niệm 20 năm thành lập trường PTNK, ĐHQG TP HCM
Chuyên đề Toán học số 10 183

Sau đây sẽ là một bài Iran TST 2014 khác, nhưng cách xử lí sẽ cần phải
Chuyên đề Toán học Phổ thông năng khiếu TP.HCM số 10

tinh tế hơn nữa.

Bài 6. (Iran TST 2014) Cho tam giác ABC nội tiếp (O) và tâm nội tiếp I,
điểm D di động trên cạnh BC. Gọi J, K lần lượt là tâm đường tròn nội tiếp
tam giác ABD và AC D. Giả sử (AI J), (AI K) lần lượt cắt (O) tại P, Q. Chứng
minh PQ đi qua điểm cố định.

Lời giải.
Ta có P, Q là giao điểm của (O) với (AI J), (AI K) là hai đường tròn đi qua A
và I, điều này làm ta nghĩ đến phép nghịch đảo đối xứng vì nó sẽ sẽ biến
(AI J), (AI K) thành hai đường thẳng đi qua tâm bàng tiếp góc A.

Thật vậy, áp dụng phép nghịch đảo tâm A phương tích AB · AC, lấy đối
xứng qua phân giác AI, ta có

I ↔ I a , (ABC) ↔ BC, P ↔ P 0 , Q ↔ Q0 , (AI J) ↔ I a P 0 , (AI K) ↔ I a Q0 .

Hướng tới kỷ niệm 20 năm thành lập trường PTNK, ĐHQG TP HCM
184 Chuyên đề Toán học số 10

Vì P, Q thuộc (ABC) nên P 0 , Q0 thuộc BC. Bài toán của chúng ta trở thành
chứng minh (AP 0Q0 ) đi qua điểm S cố định khác A.
Chuyên đề Toán học Phổ thông năng khiếu TP.HCM số 10

Giả sử tồn tại điểm S cố định như vậy, nếu AS cắt BC tại T thì phương tích
T P 0 · TQ0 = TA · T S không đổi. Như vậy, hướng tiếp cận của bài toán là
chứng minh phương tích không đổi. Ta cần xác định mối quan hệ giữa P 0
và Q0 .
Để ý rằng ∠JAK = 21 ∠BAC, ta có ∠AJ I + ∠AK I = 90◦ . Suy ra hai đường
tròn (AI J) và (AI K) trực giao, do đó, khi qua phép nghịch đảo ta phải có
I a P 0 ⊥I a Q0 .
Gọi N là tiếp điểm của (I a ) và BC, ta có N P 0 · NQ0 = −I a2 không đổi. Chọn
S thuộc tia AN sao cho N A · N S = N P 0 · NQ0 , ta có S cố định và S thuộc
(AP 0Q0 ), suy ra (AP 0Q0 ) qua điểm cố định S khác A, suy ra PQ đi qua điểm
cố định khi D di động.

Bài toán trên thật ra còn có thể giải được không sử dụng phép nghịch đảo,
nếu ta biết cách xác định P, Q theo một cách khác. Thật vậy, P, Q chính là
tiếp điểm trên (O) của các đường tròn tiếp xúc AD, BC, tiếp xúc trong với
(O) và nằm cùng phía với A.
Một lần nữa, ta lại thấy tính bảo giác của phép nghịch đảo tỏ ra rất hữu
dụng. Nếu biết tận dụng tối đa tính bảo giác thì ta có thể chỉ ra được các
mối quan hệ hình học tưởng chừng như rất khó chứng minh, như trong
bài sau.

Bài 7. (IMO 2014) Cho tứ giác ABC D thỏa mãn tứ giác vuộng tại B và D.
Giả sử AH là đường cao tam giác ABD và S, T thuộc AB, AC thỏa mãn

∠BSH + ∠SC H = ∠DT H + ∠T C H = 90◦ .

Chứng minh đường tròn ngoại tiếp tam giác (HST ) tiếp xúc BD.

Lời giải.
Đầu tiên, ta cần xác định S và T theo cách khác: Vẽ (SC H) và (T C H) lần
lượt cắt AB, AD tại X , Y, ta có

∠X SH + ∠T X H = ∠BSH + ∠SC H = 90◦

suy ra (SC H) có SX là đường kính, do đó (SC H) có tâm thuộc AB. Như


vậy, S được xác định bởi đường tròn đi qua C H và có tâm trên AB. Tương
tự, T là giao điểm của đường tròn đi qua C H và có tâm trên AD.
Qua biến đổi góc, ta dự đoán rằng nếu C H⊥ST tại P và (BDP) tiếp xúc
S T thì ta có (HST ) tiếp xúc BD. Như vậy, ta sẽ chứng minh hai điều này.

Hướng tới kỷ niệm 20 năm thành lập trường PTNK, ĐHQG TP HCM
Chuyên đề Toán học số 10 185
Chuyên đề Toán học Phổ thông năng khiếu TP.HCM số 10

Nhận xét. C H vuông góc ST


Gọi S 0 , T 0 là giao điểm của (C HS), (C H T ) với AB, AD tương ứng. Ta sẽ
chứng minh ST k S 0 T 0 . Do C H vuông góc với đường nối tâm (C HS), (C H T )
nên điều đó sẽ dẫn đến ST, S 0 T 0 cùng vuông góc với C H.
Xét phép nghịch đảo tâm A, phương tích AB · AD và lấy đối xứng qua phân
giác ∠BAD, ta có, C ↔ H nên ảnh của (C HS) và (C H T ) sẽ là hai đường
tròn đi qua C H (do S, T khác A). Lại có, AB đi qua tâm (C HS) nên AC sẽ
đi qua tâm của ảnh của (C HS), tương tự, ta có AC đi qua tâm của ảnh
(C H T ).
Vì tồn tại duy nhất một đường tròn đi qua C H mà có tâm trên AB hoặc AC,
do đó ảnh của (C HS) phải là (C H T ) và ảnh của (C H T ) phải là (C HS).
Do đó, ta có (C HS) ↔ (C H T ) suy ra AS · AT 0 = AT · AS 0 tương đương với
AS 0 = AT 0 hay ST song song với S T , suy ra đường trung bình của hình
A AT 0 0

thang ST T 0 S 0 song song với ST.


Từ đây ta có ST ⊥C H do đường trung bình của hình thang chính là đường
nối tâm (C HS) và (C H T ) hiển nhiên sẽ vuông góc với C H.
Nhận xét. BP vuông góc C S và DP⊥C T

Hướng tới kỷ niệm 20 năm thành lập trường PTNK, ĐHQG TP HCM
186 Chuyên đề Toán học số 10

Đây là hệ quả trực tiếp của C H⊥ST. Thật vậy, ta có SP C B và T P C D nội


Chuyên đề Toán học Phổ thông năng khiếu TP.HCM số 10

tiếp nên qua biến đổi góc ta có

∠BSH + ∠SBP = ∠BSH + ∠SC H = 90◦ ,

suy ra BP⊥C S, tương tự ta có DP⊥C T.


Nhận xét. (BDP) tiếp xúc S T
Gọi P, Q là giao điểm của C H với ST và S 0 T 0 , ta có PQ vuông góc với ST
và S 0 T 0 .
Ta muốn chứng minh tâm của (BC P) nằm trên C H do C H⊥ST. Lại có do
vai trò của ST và S 0 T 0 là như nhau nên nếu (BC P) tiếp xúc ST thì ta cũng
có (BCQ) tiếp xúc S 0 T 0 .
Để ý rằng, nếu (BC P) tiếp xúc ST thì tâm (BC P) chính là trung điểm
C H, tương tự, ta có tâm (BCQ) là trung điểm C H, suy ra (BC P) trùng với
(BCQ). Như vậy, ta cần chứng minh tứ giác P BQC nội tiếp trong đường
tròn đường kính PQ.
Thật vậy, ta có SP C B và BCQS 0 nội tiếp nên theo giả thiết, ta có

∠BPQ + ∠BQP = ∠BS 0 C + ∠BSC = 90◦ ,

suy ra ∠P BQ vuông. Tương tự ta có ∠P DQ vuông, suy ra P BQC nội tiếp


đường tròn đường kính PQ.
Lại có PQ⊥ST nên ta suy ra (BC P) tiếp xúc ST.
Nhận xét. (SH T ) tiếp xúc BD
Ta có∠T SH = ∠T H D tương đương ∠SH P = ∠BDP hay ∠SC B = ∠SP B
(đúng do SP C B nội tiếp), suy ra (SH T ) tiếp xúc BD.
Ta có điều phải chứng minh.
Bài toán IMO 2014 có nhiều lời giải gọn gàng và tinh tế hơn, nhưng lời
giải đầy kĩ thuật này cũng có nét đẹp của nó. Lời giải không những chứng
minh (SH T ) tiếp xúc BD mà còn chứng minh các kết quả kèm theo của
cấu hình như là C H⊥ST, cũng là một kết quả rất đẹp. Sử dụng ý tưởng
về phép nghịch đảo đối xứng và do vai trò của bộ (S, T ) và (S 0 , T 0 ) là như
nhau, bộ C, H hoán đổi được cho nhau nên ta còn có thể suy ra nhiều kết
quả khác như: (ST C) tiếp xúc (ABC) hoặc (S 0 T 0 H) tiếp xúc BC.
Những kết quả này độc giả có thể tự chứng minh.
Trong cách sử dụng nghịch đảo trên, ta lại thấy tầm quan trọng của tính
bảo giác. Nếu biết sử dụng một cách tinh tế thì ta có thể xây dựng các mối
quan hệ hình học tưởng chừng rât khó chứng minh.

Hướng tới kỷ niệm 20 năm thành lập trường PTNK, ĐHQG TP HCM
Chuyên đề Toán học số 10 187

Tại sao lại sử dụng phép nghịch đảo đối xứng để chứng minh ST song
song S 0 T 0 ? Để ý rằng nếu X ↔ X 0 và Y ↔ Y 0 qua một phép nghịch đảo
Chuyên đề Toán học Phổ thông năng khiếu TP.HCM số 10

thông thường, ta có X Y đối song với X 0 Y 0 trong góc X OY với O là tâm


phép nghịch đảo. Kết hợp với phép đối xứng, ta sẽ có ST song song với
S 0 T 0 . Lại để ý rằng vai trò của (S, S 0 ) và (T, T 0 ) là như nhau nên chúng sẽ
là ảnh của nhau qua phép nghịch đảo.

Bài toán trên cũng là một gợi ý cho một cách sử dụng khác của phép
nghịch đảo đối xứng. Thay vì hướng đến tính đối song và tính đồng viên,
ta sẽ tìm đến tính vị tự và các phép đồng dạng thuận. Ví dụ như:

• Chân đường cao BE, C F sẽ biến thành giao điểm của BD, AC và
C D, AB với AD là đường kính (ABC).

• Nếu X ↔ Y thì tam giác ABX đồng dạng thuận với AY C.

Cuối cùng, để kết thúc bài viết này thì tác giả xin trình bày một ý tưởng
cuối cùng qua bài toán sau, dù bài toán sau không liên quan đến phép
nghịch đảo đối xứng, nhưng nó mang một ý tưởng khác liên quan đến
phép nghịch đảo.

Bài 8. (Lê Phúc Lữ) Cho tam giác ABC có I là tâm đường tròn nội tiếp,
BE, C F là các đường cao cắt nhau tại H. Đường tròn (W ) tiếp xúc với đường
tròn Euler tam giác ABC là (E) và hai cạnh AB, AC tại X , Y (biết rằng (W )
gần A hơn đường tròn Euler). Gọi K là tâm đường tròn nội tiếp AE F. Chứng
minh rằng AX KY là hình thoi.

Lời giải.
Xét phép nghịch đảo tâm A, phương tích A/(E), ta có đường tròn Euler
được bảo toàn, B, C sẽ biến thành B 0 , C 0 lần lượt là trung điểm AF, AE.

Để ý rằng nếu AX I Y là hình thoi thì (W ) chính là ảnh của đường tròn
Mixtilinear Incircle của AE F nhưng vị tự tâm A tỉ số 12 , ta có đường tròn
bàng tiếp góc A tam giác ABC tiếp xúc với AB, AC, và BC sẽ biến thành
đường tròn (V ) tiếp xúc với AB, AC và (AB 0 C 0 ), tức là đường tròn Mixtilin-
ear Incircle của AE F.

Ta cần chứng minh (V ) tiếp xúc với đường tròn Euler, từ đó suy ra (V )
trùng với (W ). Thật vậy, theo định lý Feuerbach, ta có đường tròn bàng
tiếp góc A tiếp xúc ngoài với (E), do đó qua phép nghịch đảo ta cũng có
(V ) tiếp xúc với (E), suy ra (V ) trùng (W ).

Hướng tới kỷ niệm 20 năm thành lập trường PTNK, ĐHQG TP HCM
188 Chuyên đề Toán học số 10
Chuyên đề Toán học Phổ thông năng khiếu TP.HCM số 10

Cấu hình Feuerbach là một trong những cấu hình kinh điển nhất về hai
đường tròn tiếp xúc nhau, hay theo định lý Casey thì là tồn tại đường tròn
tiếp xúc 4 đường tròn khác. Vì vậy, một ý tưởng để chứng minh hai đường
tròn tiếp xúc nhau là lợi dụng cấu hình Feuerbach

4. Bài tập
Bài 1. Cho tam giác ABC nội tiếp (O) và D, E lần lượt là giao điểm của phân
giác trong góc A với BC, (O) tương ứng. Đường tròn đường kính DE cắt (O)
tại X . Chứng minh rằng AX là đường đối trung.

Bài 2. Trên mặt phẳng cho góc xO y và đường tròn (I) cố định tiếp xúc hai
tia Ox, O y. Đường thẳng d di động luôn tiếp xúc (I) cắt Ox, O y tại A, B.
Chứng minh (OAB) tiếp xúc một đường tròn cố định.

Bài 3. (Vietnam TST 2014) Cho tam giác ABC có ∠A < ∠B < ∠C và nội
tiếp trong đường tròn (O). Trên cung nhỏ BC của (O) và không chứa điểm
A, lấy điểm D tùy ý. Giả sử C D cắt AB ở E và BD cắt AC ở F. Gọi O1 là tâm
đường tròn nằm trong tam giác EBD, tiếp xúc với EB, E D và tiếp xúc với
đường tròn (O). Gọi O2 là tâm đường tròn nằm trong tam giác F C D, tiếp
xúc với F C, F D và tiếp xúc với đường tròn (O).

Hướng tới kỷ niệm 20 năm thành lập trường PTNK, ĐHQG TP HCM
Chuyên đề Toán học số 10 189

a) Gọi M M là tiếp điểm của (O1 ) với BE và N là tiếp điểm của O2 với C F.
Chuyên đề Toán học Phổ thông năng khiếu TP.HCM số 10

Chứng minh rằng đường tròn đường kính M N luôn đi qua một điểm
cố định.
b) Đường thẳng qua M và song song với C E cắt AC ở P, đường thẳng qua
N và song song với BF cắt AB ở Q. Chứng minh rằng đường tròn ngoại
tiếp tam giác (AM P), (ANQ) cùng tiếp xúc với một đường tròn cố định.
Bài 4. (Vòng loại Olympic toán toàn Nga 2014) Cho hình thang ABC D với
AB, C D là hai cạnh bên. Vẽ đường tròn (W1 ) đi qua A, B, (W2 ) đi qua C, D
sao cho chúng tiếp xúc nhau và cung AB, C D tạo thành hai góc α, β với
đường tròn tương ứng. Dựng (W3 ) đi qua A, B, (W4 ) đi qua C, D sao cho
cung AB, C D tạo thành hai góc α, β với đường tròn tương ứng. Chứng minh
rằng (W3 ), (W4 ) tiếp xúc nhau.
Bài 5. (IMO Shortlist 2011)Cho tam giác ABC nội tiếp (O), M , N trung điểm
AB, AC, AD là đường cao tam giác ABC. Gọi (X ) là đường tròn đi qua M , N
và tiếp xúc (O) tại W khác A. Chứng minh rằng trọng tâm tam giác ABC
thuộc W D.
Bài 6. (USA TST 2016) Cho tam giác ABC nội tiếp (O). Phân giác trong góc
A cắt BC và (O) lần lượt tại E và F. Gọi D là tiếp điểm của đường tròn nội
tiếp tam giác ABC và BC. Đường tròn (DE F ) cắt (O) tại T và cắt đường
tròn bàng tiếp góc A tại S1 và S2 . Chứng minh rằng AT sẽ đi qua một trong
hai điểm S1 hoặc S2 .
Bài 7. (Iran TST 2014) Cho tam giác ABC có I làm tâm đường tròn nội tiếp
tiếp xúc BC tại D. Gọi K là một điểm trên đường tròn ngoại tiếp ABC sao
cho ∠BAD = ∠CAK. Giả sử Y, Z là hình chiếu của K lên BI, C I và P là trung
điểm Y Z. Chứng minh rằng BP = C P.
Bài 8. Cho tam giác ABC nội tiếp (O) cố định và A di động trên cung lớn
BC, với AD, BE, C F là ba đường cao. Gọi (Eu) là đường tròn Euler của tam
giác ABC, M là trung điểm BC. Giả sử AM cắt (Eu) tại X . Đường tròn đi
qua D và tiếp xúc với EX tại E cắt đường tròn đi qua D và tiếp xúc F X tại F
tại một điểm Y khác D. Chứng minh H Y đi qua điểm cố định khi A di động.
Bài 9. (Sharygin 2016, Final) Cho tứ giác nội tiếp ABC D có M là giao điểm
hai đường chéo. Đường tròn (W ) tiếp xúc đoạn M A, M D và (ABC D) lần lượt
tại P, Q, X . Chứng minh trục đẳng phương của (ACQ) và (BDP) đi qua X .
Bài 10. (IMO Shortlist 2014) Cho tam giác ABC nội tiếp (O), ngoại tiếp
(I). Đường thẳng qua I vuông góc C I cắt BC và (O) lần lượt tại U và V. Các
đường thẳng song song với AI qua U, V lần lượt cắt AV, AB tại X , Y. Lấy W, Z
lần lượt là trung điểm AX , BC. Chứng minh rằng I, X , Y thẳng hàng khi và
chỉ khi I, W, Z thẳng hàng.

Hướng tới kỷ niệm 20 năm thành lập trường PTNK, ĐHQG TP HCM
190 Chuyên đề Toán học số 10
Chuyên đề Toán học Phổ thông năng khiếu TP.HCM số 10

Hướng tới kỷ niệm 20 năm thành lập trường PTNK, ĐHQG TP HCM
Chuyên đề Toán học Phổ thông năng khiếu TP.HCM số 10

Định lý thặng dư Trung Hoa

Phạm Nguyễn Mạnh


(Lớp Chuyên Toán khóa 2014 − 2017)

Định lí thặng dư Trung Hoa là một định lí số học được phổ biến rất rộng rãi và
được áp dụng rất nhiều trong các kì thi học sinh giỏi quốc gia, quốc tế. Dù nổi
tiếng và được phát biểu rất đơn giản nhưng rất nhiều học sinh lại không áp dụng
hiệu quả định lí này, dẫn đến không ít những kết quả không như ý muốn (điển
hình như kì thi chọn đội tuyển PTNK 2015 − 2016). Bài viết này, tôi muốn giới
thiệu về định lí thặng dư Trung Hoa và những bài toán có thể sử dụng định lí này
một cách hợp lí.

1. Phát biểu định lí và chứng minh


Định lí được phát biểu như sau:
Định lý. Tồn tại duy nhất một nghiệm x (mod m1 m2 · · · mk ) thỏa mãn
x ≡ a1 (mod m1 )


 x ≡ a (mod m )

2 2

 ···
x ≡ ak (mod mk )

Với k ∈ Z+ và m1 , m2 , . . . , mk đôi một nguyên tố cùng nhau.


Chứng minh.
Hiển nhiên với k = 1 thì hệ phương trình có nghiệm duy nhất. Giả sử định
lí đúng với k = m ta sẽ chứng minh định lí đúng với k = m + 1.
0
Thật vậy gọi x là nghiệm (mod m1 m2 · · · mm ) của trường hợp k = m. Do
m1 m2 · · · mm , 2m1 m2 · · · mm , . . . , mm+1 m1 m2 · · · mm là hệ đầy đủ (mod mm+1 )
nên tồn tại a ∈ 1, mm+1 sao cho
am1 m2 · · · mm ≡ am+1 (mod mm+1 ),

191
192 Chuyên đề Toán học số 10

0
tương tự tồn tại b ∈ 1, m1 m2 · · · mm để bmm+1 ≡ x (mod m1 m2 · · · mm ), từ
Chuyên đề Toán học Phổ thông năng khiếu TP.HCM số 10

đó suy ra nghiệm

x ≡ am1 m2 · · · mm + bmm+1 (mod m1 m2 · · · mm ),

thỏa mãn yêu cầu định lí. Ta chứng minh quy nạp thành công.

2. Một số bài tập ứng dụng


Định lí thặng dư Trung Hoa thường được áp dụng trong những bài toán
liên quan đến hợp số, lũy thừa của một hợp số và số dư chính phương.
Những bài tập sau đây hi vọng sẽ minh họa cụ thể kĩ thuật sử dụng định
lí này vào bài toán.

Bài 1. Chứng minh rằng ∀n ∈ Z+ luôn tồn tại n số nguyên dương liên tiếp
là hợp số.

Lời giải.
Chúng ta đã biết có vô hạn số nguyên tố, vậy ta xét n số nguyên tố
p1 , p2 , . . . , pn phân biệt và xét số nguyên dương x thỏa mãn x > p1 p2 · · · pn

x ≡ −1 (mod p1 )


x ≡ −2 (mod p2 )

 ···
x ≡ −k (mod pk )

Vậy ta có x + 1, x + 2, . . . , x + n là hợp số, ta có điều phải chứng minh.

Bài 2. (USAMO 2008) Chứng minh rằng ∀ ∈ Z+ luôn tồn tại các số nguyên
dương đôi một phân biệt p1 , p2 , . . . , pn thỏa mãn p1 p2 · · · pn − 1 là tích của 2
số nguyên liên tiếp.

Lời giải.
Nếu ∃k ∈ Z+ thoả mãn p1 p2 · · · pn − 1 = k(k + 1) thì

4p1 p2 · · · pn = (2k + 1)2 + 3.

Nếu (2k + 1)2 + 3 có ít nhất n ước nguyên tố lẻ phân biệt thì ta sẽ có cách
phân hoạch p1 , p2 , . . . , pn như trên. Phương trình trên gợi ta chứng minh
có vô hạn số nguyên tố p lẻ thỏa

−3
 ‹
= 1 (∗)
p

Hướng tới kỷ niệm 20 năm thành lập trường PTNK, ĐHQG TP HCM
Chuyên đề Toán học số 10 193

Vì p lẻ nên sử dụng luật thuận nghịch, ta có


Chuyên đề Toán học Phổ thông năng khiếu TP.HCM số 10

3
 ‹  ‹
p p−1 3−1 p−1
· = (−1) 2 · 2 = (−1) 2 ,
p 3

tương đương

−1 3 −3
 ‹  ‹  ‹  ‹  ‹
p p−1 p−1 p
· · = (−1) · (−1) = 1 =
2 2 · . (∗∗)
p p 3 p 3

Vậy để (∗) đúng thì ta cần chứng minh có vô hạn số nguyên tố dạng 3k +1.
Từ (∗∗) ta cũng có p là ước của x 2 + 3 (x ∈ Z+ ) với x không chia hết cho
3 khi và chỉ khi p có dạng 3k + 1.
Giả sử có hữu hạn các số nguyên tố p1 , p2 , . . . , p t dạng 3k + 1, thì xét ước
nguyên tố của (p1 p2 · · · p t )2 + 3, ta suy ra có vô hạn số nguyên tố dạng
3k + 1.
Quay lại bài toán, xét p1 , p2 , . . . , pn là các số nguyên tố lẻ dạng 3k + 1, suy
ra ∃a1 , a2 , . . . , an thuộc Z+ thỏa mãn

a ≡ −3 (mod p1 )
 2
 12

a2 ≡ −3 (mod p2 )
 ···
a3 ≡ −3 (mod pk )
 2

Vậy ta xét các số nguyên dương x thoả

x ≡ 1 (mod 2)


x ≡ a1 (mod p1 )

 ···
x ≡ an (mod pn )

.
Theo định lí thặng dư Trung Hoa thì tồn tại x như vậy. Ngoài ra, x 2 + 3 .. 4
và có ít nhất n ước nguyên tố phân biệt. Vậy thế 2k + 1 bằng x ta sẽ có
điều phải chứng minh.

Chú ý. Với những đẳng thức là tích của 2 số nguyên liên tiếp ta có thể nhân
4 hai vế lên rồi cộng cho 1 để tạo một số chính phương.

Bài 3. (Romania TST 2004) Giả sử m, n là 2 số nguyên dương thỏa mãn với
.
mọi a nguyên dương nguyên tố cùng nhau với n thì a m − 1 .. n. Chứng minh
rằng n ≤ 4m(2m − 1). Dấu bằng xảy ra khi nào?

Hướng tới kỷ niệm 20 năm thành lập trường PTNK, ĐHQG TP HCM
194 Chuyên đề Toán học số 10

Lời giải.
.
Chuyên đề Toán học Phổ thông năng khiếu TP.HCM số 10

a) Nếu n lẻ, ta có (2, n) = 1 ⇒ 2m − 1 .. n ⇒ 4m(2m − 1) > 2m − 1 ≥ n.


b) Nếu n chẵn, đặt n = 2s · t(s, t ∈ Z+ ) với t lẻ. Theo định lí thặng dư
Trung Hoa tồn tại một số x lẻ thỏa mãn
. .
x ≡ 2 (mod t) ⇒ (x, n) = 1 ⇒ x m − 1 .. t ⇒ 2m − 1 .. t ⇒ 2m − 1 ≥ t (∗)

Nếu s ≤ 2 thì 4m ≥ 2s nên ta xét s ≥ 3.


. .
Ta có x m − 1 .. 2s và a2 − 1 .. 8 với mọi a lẻ nhưng a − 1 thì không.
Sử dụng bổ đề (a x −1, a y −1) = a(x, y) −1 (∀ a, x, y ∈ Z+ ) (tham khảo định
lí Bézoute) ta sẽ chứng minh được theo quy nạp rằng k = 2i−2 là số nhỏ
nhất thỏa mãn
.
x k − 1 .. 2i (∀ i ∈ Z+ , i ≥ 3) ⇒ m ≥ 2s−2 ⇒ 4m ≥ 2s .

Do đó, ta được
4m ≥ 2s ∀ s ∈ Z+ (∗∗)
Từ (∗) và (∗∗) ta có điều phải chứng minh. Dấu bằng xảy ra ⇔ m =
2s−2 , 2m − 1 = t. Đến đây tìm ra kết quả cụ thể sẽ là bài tập hay để người
đọc có thể luyện tập (hãy chứng minh với mọi ước nguyên tố p của t thì
m chia hết cho p − 1).
Bài 4. (PTNK 2015) Cho A = {a ∈ 1, 2016| (a, 2016) = 1}. Có bao nhiêu
phần tử a của A thỏa tồn tại số nguyên b sao cho a + 2016b là số chính
phương?
Lời giải.
Mệnh đề bài toán tương đương với mệnh đề sau: Có bao nhiêu số nguyên
x thỏa mãn x 2 ≡ a (mod 2016) ?
Ta có
2016 = 32 · 9 · 7


x ≡ 1, 9, 17, 25 (mod 32)
 2
 x 2 ≡ 1, 2, 4 (mod 7)
x ≡ 1, 4, 7 (mod 9)
 2

Vậy nếu a ∈ A là một số thỏa mãn điều kiện đề bài thì



 a ≡ 1, 9, 17, 25 (mod 32)
a ≡ 1, 2, 4 (mod 7)
 a ≡ 1, 4, 7 (mod 9)

Do 32, 7, 9 đôi một nguyên tố cùng nhau nên theo định lí thặng dư Trung
Hoa, với mỗi bước ta chọn 1 số dư mod 32, 9, 7 thỏa mãn hệ modulo trên

Hướng tới kỷ niệm 20 năm thành lập trường PTNK, ĐHQG TP HCM
Chuyên đề Toán học số 10 195

thì sẽ ra đúng 1 số thuộc A khác nhau, từ đó suy ra có 4 · 3 · 3 = 36 số thỏa


Chuyên đề Toán học Phổ thông năng khiếu TP.HCM số 10

hệ trên thuộc A. Ta sẽ chứng minh mỗi số thuộc A thỏa hệ trên đều thỏa
mãn yêu cầu đề bài, từ đó suy ra đáp số là 36.
Cho a là một số thuộc A thỏa mãn hệ modulo trên thì ∃x 1 , x 2 , x 3 ∈ Z+ sao
cho 
 x 12 ≡ a (mod 32)
x 2 ≡ a (mod 7)
 x 22 ≡ 1, 4, 7 (mod 9)
3

Vậy ta sẽ chọn x ∈ Z+ thoả mãn



 x ≡ x 1 (mod 32)
x ≡ x 2 (mod 7)
 x ≡ x (mod 9)
3

Tức x 2 ≡ a (mod 2016). Vậy có tổng cộng 36 số a ∈ A thoả mãn ∃b ∈ Z


để a + 2016b là số chính phương.

Bài 5. (Định lí thặng dư Trung Hoa mở rộng) Hệ phương trình đồng dư

x ≡ a1 (mod m1 )


x ≡ a2 (mod m2 )

 ···
x ≡ ak (mod mk )

.
có nghiệm duy nhất mod lcm(m1 , m2 , . . . , mk ) khi và chỉ khi ai −a j ..(mi , m j )
với mọi i, j ∈ 1, k.

Lời giải.
.
Dễ thấy nếu hệ modulo trên có nghiệm duy nhất thì ai − a j ..(mi , m j )∀ i, j ∈
1, k. Ta sẽ chứng minh chiều ngược lại.
.
Khi có a − a .. (m , m ) ∀ i, j ∈ 1, k ta thấy rằng nếu xét i, j sao cho
i j i j
(mi , m j ) > 1 thì 2 hệ mod lcm(mi , m j ) sao cho nếu hệ nào có nghiệm
thì nghiệm đó cũng là nghiệm hệ kia:

x ≡ ai (mod mi )
§
x ≡ a j (mod m j )


x ≡ ai (mod mi )

m
x ≡ a j (mod (mi ,mj j ) )

Hướng tới kỷ niệm 20 năm thành lập trường PTNK, ĐHQG TP HCM
196 Chuyên đề Toán học số 10

Do tính hữu hạn về số ước của m1 , m2 , . . . , mk nên sau hữu hạn lần thực
Chuyên đề Toán học Phổ thông năng khiếu TP.HCM số 10

hiện cách làm như trên, ta sẽ ra được hệ modulo

x ≡ a1 (mod m1 )


x ≡ a2 (mod m2 )

với m1 , m2 , . . . , mk nguyên tố cùng nhau.
 ···
x ≡ ak (mod mk )

Ngoài ra theo cách làm như trên, lcm(m1 , m2 , . . . , mk ) = lcm(m1 , . . . , mk )


và nghiệm hệ modulo trên cũng là nghiệm của hệ

x ≡ a1 (mod m1 )


x ≡ a2 (mod m2 )

 · ··
x ≡ ak (mod mk )

Mà nghiệm ở hệ trước đó theo định lí thặng dư Trung Hoa sẽ có nghiệm


duy nhất, từ đó suy ra nghiệm hệ đề bài cũng có nghiệm duy nhất, ta có
điều phải chứng minh.
Bài 6. Chứng minh rằng với mọi số p nguyên tố, luôn tồn tại vô hạn số
nguyên tố dạng pk + 1 (k ∈ Z+ ).
Lời giải.
.
Đặt pk +1 = q ⇒ q −1 .. p. Đến đây, từ ý tưởng cấp của số nguyên và định lí
.
Fermat, ta có ý tưởng chọn q sao cho a p −1 ..q với a là số nguyên dương nào
đó. Nếu p không là ước của q − 1 thì (p, q − 1) = 1 ⇒ (aq−1 − 1, a p − 1) =
. p
a − 1 ⇒ a − 1 .. q. Vậy ta chọn q là ước của a −1 và q 6= p, a 6= 1 (mod p) để
a−1
a p −1
q không là ước của a − 1 và a−1 không có ước nguyên tố khác p. Vậy tồn
tại số nguyên tố dạng pk + 1.
Giả sử có hữu hạn số nguyên tố p1 , p2 , . . . , pn dạng pk + 1 thì tất cả các số
nguyên tố đó đều lớn hơn p. Từ định lí Lagrange, đa thức P(x) = x p − 1
có tối đa p nghiệm (mod pi ) ∀ i ∈ 1, n, luôn tồn tại các số nguyên dương
p
x 1 , x 2 , . . . , x n sao cho x i 6= 1 (mod pi ) ∀ i ∈ 1, n.
Vậy ta chọn x nguyên dương thỏa mãn

x ≡ 0 (mod p)


x ≡ x 1 (mod p1 )

 ···
x ≡ x n (mod pn )

p
Như vậy, theo cách chọn ở trên x i 6= 1 (mod pi ) ∀ i ∈ 1, n có ước nguyên
tố dạng pk + 1 khác p1 , p2 , . . . , pn .

Hướng tới kỷ niệm 20 năm thành lập trường PTNK, ĐHQG TP HCM
Chuyên đề Toán học số 10 197

Nhận xét. Đây là trường hợp đặc biệt của định lí Dirichlet: Với mọi a, b ∈
Z, (a, b) = 1 tồn tại vô hạn số nguyên tố dạng ak + b. Tác giả nghĩ rằng tìm
Chuyên đề Toán học Phổ thông năng khiếu TP.HCM số 10

hiểu cách chứng minh định lí trên là không thực sự cần thiết, nhưng nó
rất hữu dụng đối với rất nhiều bài toán số học, thậm chí toán cao cấp, nên
xem qua. Ngoài ra, cách chứng minh bài toán trên tuy có thể không phải
cách hay nhất nhưng cách sử dụng những định lí để chứng minh vẫn đáng
để học hỏi.

3. Một số bài tập đề nghị


Bài 1. (Công thức Euler) Chứng minh rằng

1 1 1
 ‹ ‹  ‹
φ(n) = n 1 − 1− ··· 1 −
p1 p2 pn

trong đó p1 , p2 , . . . , pn là các ước nguyên tố phân biệt của n và φ(n) là số số


nguyên tố cùng nhau với n.

Bài 2. (IMO 2016) Gọi một tập các số nguyên dương là tập hương nếu mỗi
phần tử bất kì trong tập đều không nguyên tố cùng nhau với ít nhất một
phần tử khác trong tập. Ta đặt P(x) = x 2 + x + 1, tìm b ∈ Z+ nhỏ nhất để
tồn tại số nguyên không âm a sao cho

{P(a + 1), P(a + 2), . . . , P(a + b)},

là tập hương.

Bài 3. (Đài Loan TST 2002) Một điểm có tọa độ nguyên trong mặt phẳng
tọa độ được gọi là nhìn thấy được nếu đoạn thẳng nối điểm đó và gốc tọa độ
không đi qua bất kì điểm nguyên nào khác. Chứng minh rằng ∀ n ∈ Z+ tồn
tại một hình vuông n × n sao cho mọi điểm nguyên trên và trong hình vuông
đó đều không nhìn thấy được.

Bài 4. (Mở rộng Việt Nam MO 2008) Có bao nhiêu số n < 20072008 thoả
.
n(2n + 1)(5n + 1) .. 20072008 .
0 0 0
Bài 5. (Việt Nam MO 2013) Có bao nhiêu bộ nghiệm (a, b, c, a , b , c ) thoả
0 0

 ab + a b ≡ 1 (mod 15)
0 0
bc + b c ≡ 1 (mod 15)
0 0
ca + c a ≡ 1 (mod 15)

Hướng tới kỷ niệm 20 năm thành lập trường PTNK, ĐHQG TP HCM
198 Chuyên đề Toán học số 10

4. Lời kết
Chuyên đề Toán học Phổ thông năng khiếu TP.HCM số 10

Định lí thặng dư Trung Hoa như bao định lí toán học khác là một công
cụ hiểu quả để chứng minh các bài toán số học, nhất là những bài có liên
quan đến hợp số, lũy thừa của hợp số và số dư chính phương nhưng nên
nhớ tiềm năng của định lí này là vô hạn. Tác giả bài viết hi vọng rằng, qua
chuyên đề này bạn đọc sẽ hiểu rõ hơn về một công cụ hữu ích và tiếp tục
nuôi dưỡng niềm đam mê toán học của mình. Xin trân trọng cảm ơn.

Hướng tới kỷ niệm 20 năm thành lập trường PTNK, ĐHQG TP HCM
Tài liệu tham khảo
Chuyên đề Toán học Phổ thông năng khiếu TP.HCM số 10

[1] Trần Minh Hiền, Chuyên đề Định lí Thặng dư Trung Hoa.

[2] Phạm Hy Hiếu, Định lí Thặng dư Trung Hoa, Chuyên đề Toán học số 9
PTNK.

199
200 Chuyên đề Toán học số 10
Chuyên đề Toán học Phổ thông năng khiếu TP.HCM số 10

Hướng tới kỷ niệm 20 năm thành lập trường PTNK, ĐHQG TP HCM
Chuyên đề Toán học Phổ thông năng khiếu TP.HCM số 10

Một số phương pháp tiếp cận


bài toán hình học phẳng

Nguyễn Khắc Duy, Nguyễn Phan Quốc Khang


Huỳnh Phạm Minh Nguyên, Phạm Nguyễn Mạnh
(Lớp Chuyên Toán khóa 2014 − 2017)

Trong các kì thi học sinh giỏi, các bài toán hình học thường đòi hỏi học sinh độ
khéo léo và nhạy bén nhất định. Vì vậy, để có thể giải quyết bài toán trong một
khoảng thời gian cho trước, cần có được nhiều hường tiếp cận khác nhau. Các ý
tưởng đó hoàn toàn có thể được đúc kết trong quá trình luyện tập. Bài viết sau
đây xin nêu ra một số hướng tiếp cận với một số dạng bài toán, gồm có:

1. Chứng minh các đường đồng quy bằng vị tự.

2. Sử dụng bổ đề thuận đảo về hai đường tròn tiếp xúc để giải quyết bài toán
tiếp xúc.

3. Sử dụng công thức để tính toán nhờ mối liên hệ giữa các điểm.

Hi vọng qua các ví dụ trên, các tác giả có thể truyền tải đến các bạn một bài
hướng tiếp cận đối với các bài toán hình học trong các kì thi học sinh giỏi.

1. Chứng minh các đường đồng quy bằng vị tự


Mở đầu cho bài viết, chúng ta sẽ giới thiệu một bài toán rất đẹp và kinh
điển như sau

Bài 1. Cho 4ABC nội tiếp (O), ngoại tiếp I). Gọi D, E, F là tiếp điểm của
(I) với AB, BC, CA. Chứng minh OI là đường thẳng Euler của 4DE F .

Lời giải.

201
202 Chuyên đề Toán học số 10
Chuyên đề Toán học Phổ thông năng khiếu TP.HCM số 10

Gọi M , N , P là giao của AI, BI, C I của (O). Ta có

M I = M B = M C, P I = PA = P B ⇒ M P ⊥ BI

nên suy ra P k DF . Tương tự, M N k DE, N P k E F .


Xét hai tam giác M N P và 4DE F có các cặp cạnh tương ứng song song
nên suy ra M D, N E, BF đồng quy tại tâm vị tự S.
Xét phép vị tự S : 4M N P → 4DE F biến tâm ngoại tiếp O thành tâm
ngoại tiếp I, trực tâm I thành trực tâm H. Do đó OI là đường thẳng Euler
của 4DE F .
Nhận xét. Từ ví dụ trên ta có thể thấy nếu khéo léo phát hiện và sử dụng
phép vị tự, ta sẽ biến bài toán trở nên gọn nhẹ hơn.
Bài 2. (Vietnam TST 2003) Cho 4ABC có tâm ngoại tiếp O. Kí hiệu H, K, L
là chân các đường cao kẻ từ A, B, C. Gọi A0 , B0 , C0 lần lượt là trung điểm
AH, BK, C L. Đường tròn nội tiếp 4ABC có tâm I và tiếp xúc BC, CA, AB tại
D, E, F. Chứng minh A0 D, B0 E, C0 F đồng quy tại một điểm nằm trên IO.
Lời giải.

Ta thấy rằng nhờ việc phát hiện A0 D đi qua tâm bàng tiếp I a cũng như
4I a I b I c và 4DE F có các cặp cạnh tương ứng song song, ta có lời giải
bằng phép vị tự.

Hướng tới kỷ niệm 20 năm thành lập trường PTNK, ĐHQG TP HCM
Chuyên đề Toán học số 10 203
Chuyên đề Toán học Phổ thông năng khiếu TP.HCM số 10

Gọi I a , I b , I c lần lượt là tâm bàng tiếp góc A, B, C của 4ABC. Giả sử AI cắt
BC tại J. Ta có

(AJ I I a ) = −1 ⇒ D(AJ I I a ) = −1 ⇒ (DA, DH, DI, DI a ) = −1,


mà I D k AH nên DI a đi qua trung điểm AH. Suy ra D, I a , A0 thẳng hàng.
Tương tự, E, I b , B0 thẳng hàng; F, I c , C0 thẳng hàng.
Xét hai tam giác 4DE F và 4I a I b I c có I a I b k DE, I b I c k E F, I c I a k F D nên
DI a , E I b , F I c đồng quy tại một điểm G. Khi đó, G là tâm vị tự biến 4DE F
thành 4I a I b I c . Dễ thấy IO là đường thẳng Euler của 4I a I b I c nên IO đi
qua tâm đường tròn ngoại tiếp 4I a I b I c . Mà I là tâm ngoại tiếp 4DE F nên
G ∈ IO.
Vậy A0 D, B0 E, C0 F đồng quy trên IO.
Bài 3. Cho 4ABC có ba đường cao AA0 , BB 0 , C C 0 . Đường tròn tâm I nội tiếp
4ABC tiếp xúc BC, CA, AB lần lượt tại D, E, F. Gọi Ba , Ca là hình chiếu của
B, C trên AI. Gọi Ka là tâm đường tròn ngoại tiếp 4A0 Ba Ca . Các điểm K b , Kc
được xác định tương tự. Chứng minh DKa , EK b , F Kc đồng quy tại một điểm.

Hướng tới kỷ niệm 20 năm thành lập trường PTNK, ĐHQG TP HCM
204 Chuyên đề Toán học số 10

Lời giải.
Chuyên đề Toán học Phổ thông năng khiếu TP.HCM số 10

Gọi M , N , P là trung điểm BC, CA, AB.

Định nghĩa cặp điểm A b , C b tương tự cặp điểm Ba , Ca . Theo các kết quả
quen thuộc về đường tròn nội tiếp, ta có M , N , Ca , C b thẳng hàng.

a M = Ca AC = B ABa = Ba A M nên tứ giác A Ba M Ca nội tiếp. Kí


0 0
Ta có AC
Ø Ö Ö Ù
hiệu đường tròn ngoại tiếp tứ giác A0 Ba M Ca là (Ka ). Tương tự, ta cũng có
tứ giác B 0 C b N A b nội tiếp đường tròn (K b ).
Do tứ giác AA b Ba B nôi tiếp và M N k AB nên tứ giác A b Ba Ca C b nội tiếp nên
IA b · I C b = I Ba · I Ca . Tiếp tục, do tứ giác AB 0 A0 B nội tiếp và M N k AB nên
tứ giác B 0 N M A0 nội tiếp nên C N · C B 0 = C M · CA0 .
Xét phương tích của hai điểm I và C với hai đường tròn (Ka ) và (K b ). Vì
IA b · I C b = I Ba · I Ca và C N · C B 0 = C M · CA0 nên I và C cùng thuộc trục
đẳng phương của hai đường tròn trên. Do đó, C I ⊥ Ka K b ⇒ DE k Ka K b
(cùng vuông góc C I).
Tương tự, DF k Ka Kc , E F k Kc K b nên áp dụng bổ đề như trên, ta có đpcm.

Hướng tới kỷ niệm 20 năm thành lập trường PTNK, ĐHQG TP HCM
Chuyên đề Toán học số 10 205

Nhận xét. Tóm lại, để tiếp cận các bài toán đồng quy ta có các cách sau:
Chuyên đề Toán học Phổ thông năng khiếu TP.HCM số 10

• Dùng định lí Céva dạng lượng giác và tính toán.

• Chứng minh 3 đường thẳng cùng đi qua điểm cố định.

• Dùng các hệ quả của phép vị tự.

Từ đó ta thấy, nếu chọn cách tiếp cận thứ hai, ta nhận ra rằng điểm đồng
quy của 3 đường thẳng trong Bài 3 chính là điểm Feuerbach - tiếp điểm
của đường tròn nội tiếp 4ABC và đường tròn Euler của nó.

2. Sử dụng bổ đề thuận đảo về hai đường tròn


tiếp xúc để giải quyết bài toán tiếp xúc
Trước hết ta có bổ đề quen thuộc sau: Cho đường tròn (O) và một dây
cung C D. Một đường tròn tâm I tiếp xúc với dây cung CD tại T. Gọi M là
một giao điểm của (O) với (I) và P là trung điểm cung C D không chứa M
của (O). Khi đó, (O) tiếp xúc với I tại M khi và chỉ khi M T đi qua P.

Chúng ta sẽ áp dụng bổ đề trên cho các bài tập dưới đây.

Bài 4. (Iran 3rd round 2014) Cho 4ABC nội tiếp (O) có D di động trên BC.
Đường tròn (ω1 ) tiếp xúc AD, BD và tiếp xúc trong đường tròn (O), đường
tròn (ω2 ) tiếp xúc AD, C D và tiếp xúc trong đường tròn (O). Gọi X và Y lần
lượt là tiếp điểm của (ω1 ) và (ω2 ) với BC, điểm M là trung điểm X Y. Kí
hiệu I là tâm đường tròn nội tiếp 4ABC, điểm K là trung điểm I D. Chứng
minh đường thẳng M K đi qua điểm cố định.

Hướng tới kỷ niệm 20 năm thành lập trường PTNK, ĐHQG TP HCM
206 Chuyên đề Toán học số 10

Lời giải.
Đây là bài toán ứng dụng Bổ đề Sawayama: Cho 4ABC nội tiếp (O), D
Chuyên đề Toán học Phổ thông năng khiếu TP.HCM số 10

nằm trên cạnh BC, M là điểm chính giữa cung BC không chứa A. Đường
tròn (ω) tiếp xúc trong với (O) tại Z và tiếp xúc AD, BD theo thứ tự tại
X , Y. Khi đó X Y đi qua tâm nội tiếp của 4ABC, Y Z đi qua M .

Quay lại bài toán,

Giả sử (ω1 ) tiếp xúc AD và (O) tại E, G; (ω2 ) tiếp xúc AD và (O) tại F, H.
Gọi S là điểm chính giữa cung BC không chứa A.

Hướng tới kỷ niệm 20 năm thành lập trường PTNK, ĐHQG TP HCM
Chuyên đề Toán học số 10 207

Áp dụng các kết quả trên ta có GX , H Y đi qua S và


Chuyên đề Toán học Phổ thông năng khiếu TP.HCM số 10

SB 2 = SC 2 = SX · SG = SY · SH.

Gọi P, Q, N lần lượt là trung điểm X E, Y F, E F. Lại áp dụng các kết quả trên,
ta được X E, Y F đi qua I. Dễ thấy I P DQ là hình chữ nhật, mà K là trung
điểm I D nên K cũng là trung điểm PQ.
Theo tính chất đường trung bình của tam giác, ta có
XF
N P k MQ k X F, N P = MQ = ,
2
EY
NQ k P M k EY, NQ = P M = .
2
Do đó N P MQ là hình bình hành, mà K là trung điểm của PQ nên cũng là
trung điểm M N .
Xét phương tích của 3 điểm M , N , S với hai đường tròn (ω1 ) và (ω2 ). Ta
có SX · SQ = SY · SH, M X = M Y, N X = N Y nên M , N , S thẳng hàng (cùng
thuộc trục đẳng phương hai đường tròn (ω1 ) và (ω2 ). Vậy đường thẳng
K M đi qua điểm S cố định.
Bài 5. (IMO Shortlist 2002, G7) Cho 4ABC có đường tròn nội tiếp (I) tiếp
xúc BC tại K. Gọi AD là đường cao của 4ABC, M là trung điểm AD. Giả sử
K M cắt I tại N khác K. Chứng minh rằng đường tròn ngoại tiếp 4BC N tiếp
xúc với (I) tại N .
Lời giải.
Gọi J là tâm đường tròn bàng tiếp góc A của 4ABC, E là tiếp điểm của
(J) với BC, G là trung điểm KJ. Chứng minh tương tự Bài 2, ta có J, K, M
thẳng hàng.
Trước hết, dễ thấy rằng 4K GB = 4EGC (c.g.c) vì

BK = C E, GK = GE, BK
Õ G = GEC.
Õ

Do đó, GB = GC.
Gọi L là tiếp điểm của (I) với AB, P là trung điểm K L thì K L k BJ, suy ra

J BK = B
Õ Õ KL = Ö BJ K = Ö
L N K và Õ LK N ,
dẫn đến 4BK L = 4N LK(g.g), mà G, P lần lượt là trung điểm K J, K L
Õ = LÕ
nên GBK N P. Ta có BN là đường đối trung của 4LN K nên suy ra
B
Ö N K = LÕ
N P = GBK.
Õ Chứng minh tương tự, GC Ö K=C Ö N K nên
Õ = 180o − GBC
BGC Õ − GC
Õ B = 180o − B
Ö NK − C
Ö N K = 180◦ − BN
Ö C.

Hướng tới kỷ niệm 20 năm thành lập trường PTNK, ĐHQG TP HCM
208 Chuyên đề Toán học số 10
Chuyên đề Toán học Phổ thông năng khiếu TP.HCM số 10

Từ đó dẫn đến tứ giác BGC N nội tiếp, mà G là điểm chính giữa cung BC
của (BC N ) và G, K, N thẳng hàng nên (BC N ) tiếp xúc (I) tại N .

3. Sử dụng công thức để tính toán nhờ mối liên


hệ giữa các điểm
Đôi khi, nhận ra mối quan hệ giữa các điểm và sử dụng các phép tính giúp
chúng ta nhanh chóng giải quyết các bài toán.

Bài 6. Cho 4ABC nhọn nội tiếp (O). Tiếp tuyến tại B và C của (O) cắt nhau
ở L. Gọi X là điểm đối xứng với A qua BC và K là giao điểm của LX và tiếp
tuyến tại A của (O). Chứng minh K thuộc đường thẳng Euler của 4ABC.

Lời giải.
Gọi T là giao điểm của đường thẳng Euler của 4ABC với tiếp tuyến tại A
của (O). Ta sẽ chứng minh T, X , L thẳng hàng.
Gọi H là trực tâm 4ABC,giao điểm của AH với BC và (O) lần lượt là D, M ,
ta Có
TH S AH · sin T AH AH. sin AC M
= TAH =
Õ Ö
= .
TO S TAO AO · sin Õ
TAO AO

Hướng tới kỷ niệm 20 năm thành lập trường PTNK, ĐHQG TP HCM
Chuyên đề Toán học số 10 209

Dễ thấy D là trung điểm H M nên AM = H X . Ngoài ra,


Chuyên đề Toán học Phổ thông năng khiếu TP.HCM số 10

HX AM · sin(90◦ − Õ
BAC) AM · sin AC
Õ H
= = .
OL OB OA

Theo định lí sin thì


AH AC AC AM
= = = 2OA =
sin AC
Õ H sin AH
Õ C sin ABC
Õ sin AC
Ö M
Từ các điều trên, ta được TT HO = HX
OL . Kết hợp định lí Thales đảo ta có T, X , L
thẳng hàng hay T trùng K.
Vậy điểm K thuộc đường thẳng Euler của 4ABC.

Bài 7. (PTNK TST 2013) Cho 4ABC nhọn có trực tâm H. Gọi P là điểm di
động trong 4ABC sao cho BPÕ C = BP
Õ C. Đường thẳng qua B vuông góc với
AB cắt P C tại M , đường thẳng qua C vuông góc AC cắt P B tại N . Chứng
minh trung điểm I của M N luôn thuộc một đường thẳng cố định.

Lời giải.
Dựng hình bình hành ABDC. Kẻ BE ⊥ C D, C F ⊥ BD nên E, F cố định. Bây
giờ ta sẽ chứng minh I thuộc đường thẳng E F.

Hướng tới kỷ niệm 20 năm thành lập trường PTNK, ĐHQG TP HCM
210 Chuyên đề Toán học số 10
Chuyên đề Toán học Phổ thông năng khiếu TP.HCM số 10

Gọi T là giao điểm của BM và C N . Dễ thấy BT


Õ C = BH
Ö C = BP
Õ C nên tứ
giác MTPN nội tiếp. Ta có

FT BT · sin FÕBT sin B


Ö N T · sin FÕBT sin C
× M T · sin EC
Õ T
= = =
FN BN · sin Ö
F BN sin B
Ö T N · sin Ö
F BN sin C
× T M · sin Ö
EC N
C T · sin EC
Õ T ET
= =
C M · sin ×
EC M EM
F T EM IN
Suy ra · · = 1 dẫn đến I, E, F thẳng hàng.
FN ET I M

4. Phối hợp cả ba phương pháp


Sau cùng là một bài toán mà nhờ phối hợp tát cả các cách tiếp cận trên, ta
có một lời giải tự nhiên và nhẹ nhàng.

Hướng tới kỷ niệm 20 năm thành lập trường PTNK, ĐHQG TP HCM
Chuyên đề Toán học số 10 211

Bài 8. (PTNK TST 2014) 4ABC nội tiếp (O) sao cho B, C cố định, A thay
đổi trên (O). Kí hiêu (I) là đường tròn tâm I nội tiếp 4ABC. Gọi (O1 ) là
Chuyên đề Toán học Phổ thông năng khiếu TP.HCM số 10

đường tròn qua A, B và tiếp xúc (I) tại E, (O2 ) là đường tròn qua A, C và tiếp
xúc (I) tại F. Đường phân giác trong của A Ô EB cắt (O1 ) tại M , đường phân
giác trong của AF B cắt (O2 ) tại N . Gọi J là giao điểm của E M và F N . Chứng
Ô
minh đường thẳng I J luôn đi qua điểm cố định.

Lời giải.
Gọi X , Y, Z là tiếp điểm của (I) với BC, CA, AB.

Giả sử (O1 ) và (I) tiếp xúc nhau tại E, AB tiếp xúc (I) tại Z nên E Z đi
qua điểm giữa cung AB của (ABE), suy ra E, M , Z thẳng hàng. Tương tự,
F, Y, N thẳng hàng.

Dễ thấy M A2 = M Z · M E, N A2 = N Y · N F ⇒ MN là trục đẳng phương của


(I) và (A; 0) nên M N k Y Z.

Kí hiệu (O3 ) là đường tròn qua B, C và tiếp xúc với (I) tại D. Đường phân
giác trong góc BECÕ cắt (O3 ) tại P. Chứng minh như trên, ta có P M k
Z X , N P k X Y.

Hướng tới kỷ niệm 20 năm thành lập trường PTNK, ĐHQG TP HCM
212 Chuyên đề Toán học số 10

Một cách quen thuộc, ta có M Z, N Y, P X đồng quy. Mà E, M , Z thẳng hàng,


Chuyên đề Toán học Phổ thông năng khiếu TP.HCM số 10

F, Y, N thẳng hàng, P, D, X thẳng hàng nên M Z, N Y, P X đồng quy tại J. Khi


đó, J là tâm vị tự biến 4M N P thành 4X Y Z.
Ta có M A = M B, N A = N C nên OM ⊥ AB, ON ⊥ CA, mà M N ⊥ AI. Do đó
OM
× N =B
d AI = Ô
CAI = ON
× M ⇒ OM = ON .
Tương tự, OM = OP ⇒ OM = ON = OP, suy ra O là tâm đường tròn
ngoại tiếp 4M N P.
Vì I là tâm đường tròn ngoại tiếp 4X Y Z nên từ các điều trên, suy ra I J
luôn đi qua O cố định.
Cuối cùng, các tác giả xin gửi lời cảm ơn đến thầy Lê Bá Khánh Trình, thầy
Nguyễn Tăng Vũ , các anh Hồ Quốc Đăng Hưng (HCB IMO 2014), Nguyễn
Huy Hoàng (HCB IMO 2015) đã truyền đạt cho các tác giả những kinh
nghiệm quý báu này.

5. Bài tập đề nghị


Bài 1. (Brazil 2013) 4ABC nội tiếp (O), ngoại tiếp (I). Giả sử (I) tiếp xúc
BC, CA, AB tại D, E, F. Gọi P là giao điểm của AD và BE. Kí hiệu X , Y, Z theo
thứ tự là điểm đối xứng của P qua E F, F D, DE. Chứng minh AX , BY, C Z
đồng quy trên đường thẳng OI.
Bài 2. (China TST 2012) 4ABC không cân ngoại tiếp (I) và D, E, F là các
tiếp điểm của (I) với BC, CA, AB. Gọi L, M , N lần lượt là các điểm đối xứng
của D qua E F , E qua F D, F qua DE. Gọi P, Q, R lần lượt là giao điểm của
AL với BC, BM với CA, C N với AB. Chứng minh P, Q, R thẳng hàng.
Bài 3. Cho 4ABC nội tiếp (O), ngoại tiếp (I). Giả sử (I) tiếp xúc BC, CA, AB
tại D, E, F. Kí hiêu (ωa ) là đường tròn tiếp xúc BC tại D và tiếp xúc cung
BAC của (O) tại M . AM cắt BC tại X . Các điểm Y, Z được xác định tương
tự. Chứng minh X , Y, Z thẳng hàng.
Bài 4. Cho đường tròn (O) và dây cung BC cố định, A thay đổi trên (O). Các
đường cao AD, BE, C F của tam giác ABC đồng quy tại H. Gọi M là trung
điểm của BC. Đường thẳng H M cắt E F tại I. Chứng minh rằng đường thẳng
I D luôn đi qua điểm cố định khi A thay đổi trên (O).
Bài 5. Cho tam giác ABC nội tiếp đường tròn (O). Gọi I là tâm đường tròn
nội tiếp ABC. Gọi P, Q, R lần lượt là điểm giữa cung BAC, ABC, AC B của
đường tròn (O). Giả sử P I cắt BC tại D, QI cắt AC tại E và RI cắt AB tại E.
Chứng minh rằng AD, BE, C F đồng quy trên IO.

Hướng tới kỷ niệm 20 năm thành lập trường PTNK, ĐHQG TP HCM
Tài liệu tham khảo
Chuyên đề Toán học Phổ thông năng khiếu TP.HCM số 10

[1] Diễn đàn Art of Problem Solving.

[2] Hồ Quốc Đăng Hưng, Ứng dụng tỉ số đoạn thẳng – tỉ số lượng giác
trong hình học.

[3] Tập huấn đội tuyển Toán PTNK 2016.

213
214 Chuyên đề Toán học số 10
Chuyên đề Toán học Phổ thông năng khiếu TP.HCM số 10

Hướng tới kỷ niệm 20 năm thành lập trường PTNK, ĐHQG TP HCM
Chuyên đề Toán học Phổ thông năng khiếu TP.HCM số 10

Số phức trong hình học phẳng

Đỗ Trần Nguyên Huy


(Lớp Chuyên Toán khóa 2014 − 2017)

1. Các lý thuyết vận dụng giải toán

1.1. Kiến thức cần có


• Dạng đại số, lượng giác của số phức.

• Các phép cộng, trừ, nhân, chia số phức.

• Hai số phức bằng nhau.

• Số phức liên hợp.

• Module số phức.

• Công thức De Moivre.

1.2. Một số lưu ý


Cho các số phức z1 = a1 + b1 i, z2 = a2 + b2 i.

a) Tích thường

z1 z2 = z2 z1 = (a1 + b1 i)(a2 + b2 i) = (a1 a2 − b1 b2 ) + (a1 b2 + a2 b1 )i.

b) Tích thực
1
z1 ∗ z2 = (z1 z2 + z2 z1 ) ∈ R.
2
Tích thực có tính chất (bạn đọc tự chứng minh)

– z ∗ z = |z|2 .

215
216 Chuyên đề Toán học số 10

– Hoán vị : z1 ∗ z2 = z2 ∗ z1 .
Chuyên đề Toán học Phổ thông năng khiếu TP.HCM số 10

– Kết hợp : z1 ∗ (z2 + z3 ) = z1 ∗ z2 + z1 ∗ z3 .

c) Số phức liên hợp


n
X n
X n
Y n
Y
zk = zk , zk = zk .
k=1 k=1 k=1 k=1

d) Ta có z = z ⇔ Im(z) = 0 ⇔ z ∈ R, và z = −z ⇔ Re(z) = 0.

f) Bất đẳng thức về module số phức

|z1 | + |z2 | ≥ |z1 + z2 |.

Đẳng thức xảy ra khi và chỉ khi tồn tại k ∈ R thỏa z1 = kz2 .

2. Cơ sở phương pháp
y

b M

a
O x

Ta biết rằng mỗi số phức z = a + bi (a, b ∈ R) được biểu diễn duy nhất
bởi một điểm M (a, b) trong mặt phẳng tọa độ Ox y (do đó còn được gọi
là mặt phẳng phức).
Coi O là gốc tọa độ, ta có |z| cũng chính là độ dài đoạn thẳng OM và mỗi
−−→
điểm M trên mặt phẳng có thể được xác định duy nhất bởi vectơ OM . Điều
này gợi ý rằng có thể xem số phức là một vectơ trong mặt phẳng đang xét,
do đó, ta có thể áp dụng số phức để giải các bài toán hình học phẳng,


tức mỗi vectơ AB được đặc trưng bằng số phức b − a (a, b lần lượt là tọa
độ của A, B). Rõ ràng khi tìm tọa độ điểm thì phương pháp này tiện hơn
phương pháp đặt hệ trục tọa độ Ox y hay Ox yz vì số ẩn cần xác định ít
hơn.

Hướng tới kỷ niệm 20 năm thành lập trường PTNK, ĐHQG TP HCM
Chuyên đề Toán học số 10 217

3. Biễu diễn phức một số yếu tố hình học


Chuyên đề Toán học Phổ thông năng khiếu TP.HCM số 10

Trong phần này, từ hình học phẳng Ox y, chúng ta sẽ xây dựng cách biểu
diễn các yếu tố hình học trên mặt phẳng phức.

3.1. Khoảng cách giữa hai điểm


Cho trước các điểm A1 (a1 ), A2 (a2 ), A3 (a3 ). Khi đó khoảng cách giữa hai
điểm A1 , A2 là

A1 A2 = d(a1 , a2 ) = d(a2 , a1 ) =|a1 − a2 |.

Khoảng cách có tính chất

• d(a1 , a2 ) ≥ 0, ∀a1 , a2 ∈ C.

• d(a1 , a2 ) = 0 ⇔ a1 = a2 .

• Bất đẳng thức ba điểm: d(a1 , a2 ) ≤ d(a1 , a3 ) + d(a3 , a2 ).

Hệ quả. (Điểm thuộc đường thẳng) Cho hai điểm A(a), B(b) (A 6≡ B) và
M (m). Khi đó các mệnh đề sau tương đương

(i) Điểm M thuộc đường thẳng AB.


m−a
(ii) (Tiêu chuẩn kiểm tra sự thẳng hàng) b−a ∈ R.

(iii) (Phương trình tham số đường thẳng) ∃k ∈ R thoả z = (1 − k)a + kb.

 A(a), B(b) (với A 6≡ B). Khi đó


Hệ quả. (Toạ độ trung điểm) Cho hai điểm
trung điểm đoạn AB là điểm M m = a+b2 .

3.2. Đường thẳng


Phương trình chính tắc đường thẳng:

(d) : u · z + u · z + v = 0,

trong đó z là biến phức và u ∈ C, v 6∈ R là các hằng số (u 6= 0).


Phương trình chính tắc đường thẳng trong hình học Ox y

Ax + B y + C = 0.

trong đó A, B, C ∈ R là các hằng số và A2 + B 2 > 0.

Hướng tới kỷ niệm 20 năm thành lập trường PTNK, ĐHQG TP HCM
218 Chuyên đề Toán học số 10

Đặt z = x + y i ∈ C thì x = z+z 2 , y = 2i = − 2 i. Khi đó phương trình


z−z z−z
Chuyên đề Toán học Phổ thông năng khiếu TP.HCM số 10

chính tắc trong Ox y tương đương với

z+z z−z
 ‹  ‹
A − Bi + C = 0,
2 2
A + Bi A − Bi
·z+ · z + C = 0.
2 2
Đặt u = A−Bi
2 ∈ C, v = C ∈ R, ta thu được phương trình chính tắc trong số
2 2
phức. Chú ý rằng |u|2 = A +B
4 > 0 nên u là hằng số phức khác 0.
Tương tự, bạn đọc có thể thu được các dạng phương trình đường thẳng
khác (tham khảo [1]).

Hệ quả. Từ phương trình chính tắc đường thẳng, ta có một số hệ quả sau

• Hệ số góc của đường thẳng: Cho đường thẳng (d) : u·z +u·z + v = 0.
Khi đó hệ số góc của (d) là

A u+u
k=− = · i.
B u−u

• Quan hệ giữa hai đường thẳng: Cho hai đường thẳng

u1 z + u1 z + v1 = 0,
u2 z + u2 z + v2 = 0.

Khi đó
u1 u
d1 k d2 ⇔ = 2,
u1 u2
u1 u2
d1 ⊥ d2 ⇔ + = 0,
u1 u2
u1 u
d1 ∩ d2 6= ; ⇔ 6= 2 = 0.
u1 u2

Phương trình đường thẳng đi qua hai điểm: Cho hai điểm Z1 (z1 ), Z2 (z2 ).
Phương trình đường thẳng (Z1 Z2 ) thoả

z z 1
1 1
z2 z2 1 = 0,

z z 1

hay
(Z1 Z2 ) : (z1 − z2 ) · z − (z1 − z2 ) · z + (z1 z2 − z2 z1 ) = 0.

Hướng tới kỷ niệm 20 năm thành lập trường PTNK, ĐHQG TP HCM
Chuyên đề Toán học số 10 219

Phương trình đường thẳng qua một điểm và song song với một đường thẳng
Chuyên đề Toán học Phổ thông năng khiếu TP.HCM số 10

cho trước: Đường thẳng qua điểm P(p) và song song với đường thẳng
(d) : uz + uz + v = 0 có phương trình

u
z − p = − (z − p).
u

Toạ độ hình chiếu của một điểm xuống một đường thẳng: Hình chiếu H(h)
của điểm P(p) xuống đường thẳng (d) : uz + uz + v = 0 có toạ độ

up − up − v
h= .
2u

Hệ quả. Từ toạ độ hình chiếu, ta có một số hệ quả sau

• (Tiêu chuẩn vuông góc) Cho các điểm A(a), B(b), C(c), D(d). Khi đó

b−a

‹
AB ⊥ C D ⇔ Re = 0,
d−c
€ Š € Š
⇔ (b − a) ∗ (c − d) = 0 ⇔ (b − a) d − c + b − a (c − d) = 0.

• Cho các điểm A(a), B(b). Đường thẳng qua điểm P(b) và vuông góc
với đường thẳng AB có phương trình

(d) : (z − p) ∗ (b − a) = 0.

Khoảng cách từ một điểm đến một đường thẳng: Khoảng cách từ điểm P(p)
đến đường thẳng (d) : u · z + u · z + v = 0 là

|up + up + v|
d(P, (d)) = p .
2 uu

3.3. Diện tích tam giác


Cho các điểm A(a), B(b), C(c). Diện tích tam giác ABC (có thể suy biến
thành đoạn thẳng)
a a 1
1
SABC = · b b 1 .
4
c c 1

Hướng tới kỷ niệm 20 năm thành lập trường PTNK, ĐHQG TP HCM
220 Chuyên đề Toán học số 10

3.4. Góc
Chuyên đề Toán học Phổ thông năng khiếu TP.HCM số 10

Cho điểm Z(z = r(cos φ + i sin φ)) không trùng gốc tọa độ O. Dễ thấy φ
là góc hợp bởi tia Ox (theo chiều dương) và tia OZ hay

φ = (Ox, OZ) = arg(z).

Hệ quả. Theo định nghĩa góc, ta có một số hệ quả sau

• Cho hai điểm Z1 (z1 ), Z2 (z2 ). Khi đó

z2
 ‹
(OZ1 , OZ2 ) = arg .
z1

• Cho ba điểm A(a), B(b), C(c). Khi đó


c−a
(AB, AC) = arg .
b−a

• Cho bốn điểm A(a), B(b), C(c), D(d) và α là góc tạo bởi hai đường
thẳng AB, C D. Khi đó

b−a c−d
 ‹  ‹
α = arg ∨ α = arg .
c−d b−a

• (Tiêu chuẩn đồng dạng) Cho các điểm Ai (ai ) với i = 1, 6. Khi đó

a2 − a1 a5 − a4
4A1 A2 A3 ∼ 4A4 A5 A6 cùng hướng ⇔ = ,
a3 − a1 a6 − a4
a2 − a1 a5 − a4
 ‹
4A1 A2 A3 ∼ 4A4 A5 A6 ngược hướng ⇔ = .
a3 − a1 a6 − a4

• (Phép quay) Phép quay tâm A(a), góc φ biến điểm Z1 (z1 ) thành
Z2 (z2 ). Khi đó

z2 = a + (z1 − a) (cos φ + i sin φ) .

4. Ví dụ minh hoạ
Một số lưu ý

Hướng tới kỷ niệm 20 năm thành lập trường PTNK, ĐHQG TP HCM
Chuyên đề Toán học số 10 221

• Trong các bài toán tam giác, ta thường chọn tâm ngoại tiếp là gốc
tọa độ. Khi đó để ý rằng |a| = |b| = |c|, với a, b, c là tọa độ ba đỉnh
Chuyên đề Toán học Phổ thông năng khiếu TP.HCM số 10

tam giác.

• Việc tường minh hóa là việc cần làm, nhất là với các bài hình học
chứng minh “khi và chỉ khi” thì phương pháp này càng hiệu quả.

• Tọa độ điểm luôn được xác định là giao của hai yếu tố hình học
(đường thẳng, đường tròn, hệ thức vectơ, điểm và góc).

• Tọa độ một số điểm đặc biệt trong tam giác cần được ghi nhớ để vận
dụng nhanh hơn vào các bài toán khác.

4.1. Các bài toán về đường thẳng


Bài 1. Cho tam giác ABC, gốc tọa độ là tâm ngoại tiếp. Xác định tọa độ
trọng tâm G, trực tâm H theo tọa độ A, B, C và chứng minh O, G, H thẳng
hàng.
A

O
G
H

B M C

Lời giải.
Ta có O(0), đặt A(a), B(b), C(c).
 −→ −−→
. Toạ độ trọng tâm G(g). Gọi trung điểm BC là M b+c2 , do AG = 2 GM
 −→ −→ −→ −→
3  . Hoặc từ đẳng thức OA + OB + OC = 3OG, ta cũng suy
nên G g = a+b+c
ra G g = a+b+c
3 .
. Toạ độ trực tâm H(h). Do BH ⊥ AC, C H ⊥ AB nên

(h − b) ∗ (c − a) = 0,
(h − c) ∗ (b − a) = 0.
−→ −→ −→ −→
Suy ra h = a + b + c. Tổng quát hơn, từ đẳng thức HA + H B + H C = 2HO,
suy ra

(h − a) + (h − b) + (h − c) = 2 (h − o) ⇔ h = a + b + c − 2o.

Hướng tới kỷ niệm 20 năm thành lập trường PTNK, ĐHQG TP HCM
222 Chuyên đề Toán học số 10

Với o = 0, ta có h = a + b + c.
. Ta có
Chuyên đề Toán học Phổ thông năng khiếu TP.HCM số 10

h−o a + b + c − 3o
= a+b+c
= 3 ∈ R,
g−o 3 − o
nên O, G, H thẳng hàng.

→ −
→ −→
Chú ý. Bằng phương pháp tương tự, từ đẳng thức u IA + v I B + w I C = 0
(u, v, w lầnlượt là độ dài các cạnh BC, CA, AB), suy ra toạ độ tâm nội tiếp
I ua+v b+wc
u+v+w .

Bài 2. Cho tam giác ABC, tọa độ các đỉnh A(a), B(b), C(c). Các điểm D, E
DB
lần lượt thuộc các cạnh C B, CA sao cho DC = x, EC
EA = y. Xác định toạ độ
giao điểm của AD, BE.
C

E
M

A B

Lời giải.
Gọi M (m) là giao điểm AD, BE. Do M thuộc AD, BE nên m thoả hệ
(€ Š € Š
a − d · m − (a − d) · m + ad − ad = 0,
€ Š € Š
b − e · m − (b − e) · m + be − be = 0.
suy ra
(ad − ad)(b − e) + (be − be)(a − d)
⇒m=
(a − d)(b − e) − (a − d)(b − e)
Mặt khác, ta có
b + xc

( −→ −→ d =
DB = −x DC

1+ x

−→ → ⇒
− c + ya
EC = − y EA e =
1+ y
Thay vào biểu thức ở trên, ta được
x ya + b + xc
m= .
xy + x +1

Từ đó dẫn đến kết luận của bài toán.

Hướng tới kỷ niệm 20 năm thành lập trường PTNK, ĐHQG TP HCM
Chuyên đề Toán học số 10 223

Bài tương tự. (Định lí Céva) Cho tam giác ABC, lấy D, E, F lần lượt thuộc
Chuyên đề Toán học Phổ thông năng khiếu TP.HCM số 10

các cạnh BC, CA, AB thỏa AD, BE, C F đồng quy tại điểm M . Gọi t là tọa độ
DB
các điểm T . Chứng minh rằng tồn tại các số thực x, y, z thỏa DC = xy , EC
EA =
y FA x a+ y b+zc
z , FB = z
x và m = x+ y+z .

4.2. Các bài toán về góc


Bài 3. Cho các hình vuông bằng nhau ABC D, BE F C, EGH F xếp liền nhau
như hình vẽ. Tính ∠GDH + ∠E DH.
A B E G

D C F H

Lời giải.
Các hình vuông bằng nhau được xếp liền nhau như hình vẽ gợi ý xem độ
dài mỗi cạnh bằng một đơn vị trên trục tọa độ. Ta sẽ đặt gốc tọa độ tại D
(chú ý hai góc cần tính tổng có chung gốc tại D), trục hoành (trục thực)
là trục chứa DC, chiều dương D → C; trục tung (trục ảo) là trục chứa DA,
chiều dương D → A, đơn vị phức i và đơn vị thực 1 được biểu diễn qua độ
dài cạnh hình vuông. Tổng cần tìm chính là arg(e) + arg(g).
Dễ thấy e = 2 + i, g = 3 + i. Ta có
arg(e) + arg(g) = arg(2 + i) + arg(3 + i) = arg [(2 + i)(3 + i)] = arg(5 + 5i)
π π π
= arg(cos + i sin ) = .
4 4 4
Vậy ∠GDH + ∠E DH = 45◦ .
Bài 4. Cho hình vuông ABC D và điểm M là trung điểm cạnh C D. Điểm P
−→ −

thuộc đường chéo AC sao cho P C = 3AP. Chứng minh 4BP M vuông cân.
A B

D M C

Hướng tới kỷ niệm 20 năm thành lập trường PTNK, ĐHQG TP HCM
224 Chuyên đề Toán học số 10

Lời giải.
Chuyên đề Toán học Phổ thông năng khiếu TP.HCM số 10

Đặt gốc tọa độ tại D, trục hoành (trục thực) chứa DC, chiều dương D → C;
trục tung (trục ảo) chứa DA, chiều dương D → A, thỏa đơn vị phức i và
đơn vị thực 1 được biểu diễn qua cạnh hình vuông.
Dễ thấy d = 0, c = 1, a = i, b = 1+i. Do M trung điểm C D nên M m = 12 .

−→ −

Từ đẳng thức P C = 3AP, suy ra P p = 1+3i

4 .
Ta có
(1+i)− 1+3i
€ Š
b−p b−p
• m−p = 1 − 1+3i4 = 10i ⇒ Re m−p = 0 ⇒ P B ⊥ P M .
2 4

p
• P B = |b − p| = |3 + i| = 10 = | − 3i + 1| = |m − p| = P M .

Từ đó suy ra 4BP M vuông cân.

Bài 5. Cho tam giác ABN . Về phía ngoài tam giác, dựng các hình vuông
ABC D, BN M K. Gọi E là trung điểm cạnh AN , F là hình chiếu vuông góc của
B trên C K. Chứng minh ba điểm E, B, F thẳng hàng.

K F C

Lời giải.
Đặt gốc tọa độ tại F , trục thực F C, trục tung F E. Phép quay tâm B góc π2
biến C thành A nên a = b(1 − i) + ci. Tương tự, n = b(1 + i) − ki. Do E là
trung điểm AN nên e = a+n 2 = b + 2i , suy ra BE ⊥ C K, hay E, F, B thẳng
c−k

hàng.

4.3. Các bài toán có điều kiện


Bài 6. Cho tam giác ABC nội tiếp đường tròn (M , R) biết độ dài phân giác
trong và ngoài góc A bằng nhau. Chứng minh AB 2 + AC 2 = 4R2 .

Hướng tới kỷ niệm 20 năm thành lập trường PTNK, ĐHQG TP HCM
Chuyên đề Toán học số 10 225
Chuyên đề Toán học Phổ thông năng khiếu TP.HCM số 10

A M

E O B D C

Lời giải.
Gọi D, E là chân đường phân giác trong và ngoài góc A của 4ABC. Do
AD ⊥ AE và AD = AE nên 4ADE vuông cân tại A. chọn gốc tọa độ O là
trung điểm DE, OA là trục ảo, OC là trục thực, cho tương ứng i với OA, 1
với OD.
Ta có o = 0, a = i, e = −1, d = 1. Do AD là phân giác trong góc A nên

DB 2 AB 2 (b − 1)2 b2 + 1 1
= ⇔ = ⇒c= .
DC 2
AC 2
(c − 1)2
c +1
2 b

suy ra
‹2
1

AB + AC = b +
2 2
.
b
Đặt m = u + iv, (u, v ∈ R). Do M là tâm ngoại tiếp 4ABC nên

1 2
 ‹
2 2
M A = M B = M C ⇔ u + (v − 1) = (u − b) + v = u −
2 2 2 2
+ v2, 2
b

b2 +1
suy ra u = 2b , v = 1. Từ đó
‹2 
b +1 1 2
 2 ‹
4R = 4M A = 4
2 2
= b+ .
2b b

Từ các điều trên, ta suy ra điều phải chứng minh.

Bài tương tự. (Định lý bốn điểm) Cho tứ giác lồi ABC D. Chứng minh rằng
AC ⊥ BD khi và chỉ khi AB 2 + C D2 = AD2 + BC 2 .

Hướng tới kỷ niệm 20 năm thành lập trường PTNK, ĐHQG TP HCM
226 Chuyên đề Toán học số 10

5. Bài tập đề nghị


Chuyên đề Toán học Phổ thông năng khiếu TP.HCM số 10

Bài 1. Chứng minh lại các định lý quen thuộc : định lý Menelaus, đường
thẳng Simson, ...

Bài 2. Cho tam giác ABC có trọng tâm G và trung điểm các cạnh BC, CA, AB
lần lượt là D, E, F . Chứng minh rằng với mọi điểm M trong mặt phẳng ta có

M A2 + M B 2 + M C 2 + 9M G 2 = 4 M D2 + M E 2 + M F 2 .

Bài 3. (IMO Shortlist 1989) Cho hai tam giác đều OAB, OA0 B 0 cùng hướng.
Gọi S là tâm 4OAB và S 6= A0 , B 0 . Các điểm M , N lần lượt là trung điểm
cạnh A0 B, AB 0 . Chứng minh rằng 4SB 0 M ∼ 4SA0 N .

Bài 4. (Định lý Van Aubel) Cho tứ giác lồi ABC D. Trên các cạnh tứ giác, lần
lượt dựng các hình vuông về phía ngoài có tâm là O1 , O2 , O3 , O4 . Chứng minh
rằng O1 O3 ⊥ O2 O4 và O1 O3 = O2 O4 .

Bài 5. Cho tam giác ABC nhọn có trọng tâm G, tâm nội tiếp I và tâm đường
tròn Euler O9 . Đường tròn bàng tiếp góc A tiếp xúc cạnh BC tại điểm D, các
điểm E, F định nghĩa tương tự. Chứng minh rằng

a) (Điểm Nagel) Các đường thẳng AD, BE, C F đồng quy tại điểm N .

b) Ba điểm G, I, N thẳng hàng.


GO9
c) GO = 12 .

d) (Định lý Feuerbach) Đường tròn Euler tiếp xúc trong với đường tròn
nội tiếp và tiếp xúc ngoài với các đường tròn bàng tiếp.

Hướng tới kỷ niệm 20 năm thành lập trường PTNK, ĐHQG TP HCM
Tài liệu tham khảo
Chuyên đề Toán học Phổ thông năng khiếu TP.HCM số 10

[1] https://www.artofproblemsolving.com/community/
c76819h1274084_complex_numbers

[2] Diễn đàn https://www.artofproblemsolving.com

[3] Titu Andreescu, Dorin Andrica, Complex number from A to Z.

227
228 Chuyên đề Toán học số 10
Chuyên đề Toán học Phổ thông năng khiếu TP.HCM số 10

Hướng tới kỷ niệm 20 năm thành lập trường PTNK, ĐHQG TP HCM
Chuyên đề Toán học Phổ thông năng khiếu TP.HCM số 10

Tứ giác ngoại tiếp

Lương Văn Khải


(Lớp Chuyên Toán khóa 2013 − 2016)

Chúng ta đã quá quen với các bài toán liên quan đến tứ giác nội tiếp, nhưng
dường như ít người để ý tới người anh em của nó: tứ giác ngoại tiếp. Bài viết sau
đây cung cấp cho các bạn một cái nhìn tổng quan về tứ giác ngoại tiếp, các bài
toán liên quan tới nó và ứng dụng của nó trong giải các bài toán hình học.

1. Khái niệm
Định nghĩa 1. Tứ giác ngoại tiếp là tứ giác có cả bốn cạnh tiếp xúc với
một đường tròn. Đường tròn đó gọi là đường tròn nội tiếp tứ giác này.

B
A

D C

2. Tính chất
Cho tứ giác ABC D ngoại tiếp đường tròn (O). Gọi M , N , P, Q lần lượt là
tiếp điểm của các cạnh AB, BC, C D, DA. Trong phần này, xin giới thiệu một
số tính chất của tứ giác ABC D.

229
230 Chuyên đề Toán học số 10

Tính chất 1. (Định lý Pithot) Tứ giác ABC D ngoại tiếp khi và chỉ khi
AB + C D = AD + BC.
Chuyên đề Toán học Phổ thông năng khiếu TP.HCM số 10

Tính chất 2. ∠AOB + ∠COD = 180◦ .


Hai tính chất này dễ dàng suy ra từ tính chất của tiếp tuyến.

M
A

U
I
N
Q K
O H
X V

E
Y D P C

Tính chất 3. (Định lý Newton cho tứ giác ngoại tiếp)


(i) Các đường thẳng M N , PQ, AC đồng quy.

(ii) Điểm O nằm trên đường thẳng Gauss-Newton của tứ giác ABC D.
Lời giải.
(i) Gọi S là giao điểm M N và AC. Áp dụng định lí Menelaus cho cát tuyến
S M N trong 4ABC, ta có
SA N C M B
· · = 1.
SC N B M A
Kết hợp tính chất tiếp tuyến suy ra
SA P C SA P C QD
· = · · =1
SC QA SC P D QA
Do đó, theo đinh lý Menalaus S, P, Q thẳng hàng, hay M N , PQ, AC đồng
quy.
(ii) Gọi H, K là trung điểm AC, BD. Ta có
1 1
SHAB + SH C D = (SCAB + SAC D ) = SABC D ,
2 2
1 1
SKAB + SKC D = (S DAB + SBC D ) = SABC D .
2 2

Hướng tới kỷ niệm 20 năm thành lập trường PTNK, ĐHQG TP HCM
Chuyên đề Toán học số 10 231

Mặt khác
Chuyên đề Toán học Phổ thông năng khiếu TP.HCM số 10

1
SABC D = SOAB + SOBC + SOC D + SODA = · r · (AB + BC + C D + DA) ,
2
mà theo tính chất 1, suy ra
1
SOAB + SOC D = SABC D .
2
Gọi {E} = AB ∩ C D. Lấy X ∈ AB, Y ∈ C D sao cho EX = AB, EY = C D. Ta

SOAB + SOC D = SOEX + SOEY = SOX Y + S EX Y .
Từ đó SOX Y = SH X Y = SK X Y , suy ra O, H, K cùng thuộc đường thẳng song
song X Y , hay O nằm trên đường thẳng Gauss-Newton của ABC D.
Tính chất 4. Đường thẳng OS vuông góc với AC.
Lời giải.
Cách 1. Gọi {U} = OB ∩ M N , {V } = OD ∩ PQ. Theo định lý Pythagore và
phương tích, ta có

SB 2 − BM 2 = SU 2 − U M 2 = SM · SN = SP · SQ = SV 2 − V Q2 = SD2 − DQ2 .

Suy ra
SB 2 − SD2 = BM 2 − DQ2 = OB 2 − OD2 ,
nên theo định lý bốn điểm, ta có OS⊥BD.
Cách 2. Xét (O). Theo tính chất 2, S thuộc M N là đường đối cực của B,
nên theo định lý La Hire, B thuộc đường đối cực của S. Tương tự suy ra
BD là đường đối cực của S, hay OS⊥BD.
Tính chất 5. Các đường thẳng M P, NQ, AC, BD đồng quy.
Lời giải.
Gọi {I} = M P ∩ NQ, {S} = M N ∩ PQ. Áp dụng định lý Pascal cho sáu điểm
M , P, P, Q, N , N , ta có I, C, S thẳng hàng. Mà theo tính chất 2, S, A, C thẳng
hàng, suy ra I, A, C thẳng hàng. Tương tự, ta thu được M P, NQ, AC, BD
đồng quy.
Tính chất 6. (Diện tích tứ giác ngoại tiếp) Đặt AB = a, BC = b, C D =
c, DA = d. Khi đó
 ‹
A B C D
SABC D = r · (a + c) = r · (b + d) = r 2 cot + cot + cot + cot
2 2 2 2
p A+ C
= abcd · sin .
2

Hướng tới kỷ niệm 20 năm thành lập trường PTNK, ĐHQG TP HCM
232 Chuyên đề Toán học số 10

Lời giải.
Chuyên đề Toán học Phổ thông năng khiếu TP.HCM số 10

Ta có

1
SABC D = SOAB + SOBC + SOC D + SODA = · r · (a + b + c + d) .
2

mà theo tính chất 1 thì a + c = b + d nên

SABC D = r · (a + c) = r · (b + d) = r · (M A + M B + P C + P D)
.
 ‹
A B C D
= r cot + cot + cot + cot
2
2 2 2 2

Theo công thức Heron cho diện tích tứ giác, ta có


v
t A+ C
SABC D = ((p − a) (p − b) (p − c) (p − d) − abcd · cos2 ,
2

với p = (a + b + c + d) /2.

Mặt khác, theo tính chất 1 thì a + c = b + d, do dó

(p − a) (p − b) (p − c) (p − d) = abcd.

Suy ra
v
A+ C A+ C
t  ‹ p
SABC D = abcd 1 − cos 2 = abcd · sin .
2 2

Ta thu được điều phải chứng minh.

3. Bài tập

Bài 1. Cho giác ABC D ngoại tiếp đường tròn (O, r). Gọi M , N là tiếp điểm
của (O) với các cạnh AB, C D. Chứng minh các mệnh đề sau tương đương

(i) Tứ giác ABC D là hình thang có hai đáy AB, C D.

(ii) O, M , N thẳng hàng.

(iii) AC, BD, M N đồng quy.

Hướng tới kỷ niệm 20 năm thành lập trường PTNK, ĐHQG TP HCM
Chuyên đề Toán học số 10 233

A M B
Chuyên đề Toán học Phổ thông năng khiếu TP.HCM số 10

D N C

Lời giải.
Dễ thấy AB k C D khi và chỉ khi AB⊥ON , hay O, M , N thẳng hàng.
Mặt khác, gọi I là giao điểm AC, BD. Ta có AB k C D khi và chỉ khi

∠A ∠D
∠A = 180◦ − ∠D ⇔ = 90◦ − ⇔ ∠OAM = ∠DON ,
2 2
tương đương 4OAM ∼ 4DON hay M A· N C = r 2 . Tương tự M B · N D = r 2 .
B = N C . Từ đó, theo hệ quả của bổ đề E.R.I.Q thì I, M , N thẳng
MA ND
Suy ra M
hàng, hay AC, BD, M N đồng quy.

Bài 2. Cho tứ giác ABC D ngoại tiếp đường tròn (O, r). Gọi M , N , P, Q là tiếp
điểm của (O) với các cạnh AB, BC, C D, DA. Chứng minh các hệ thức sau
NQ
a) AB = 2 r 2 , BC =
· OA·OB MP
2 · OB·OC
r2 .

b) AB
CD = OA·OB AD
OC·OD , BC = OA·OD
OB·OC .

(14th German NMO 1974/75, 4th Round, Class 10)

OB 2 OD2 OA2 OC 2
c) AB·BC = C D·DA , DA·AB = BC·C D .

Lời giải.
a) Gọi {A0 } = QM ∩ OA, {B 0 } = M N ∩ OB. Do OA · OA0 = OB · OB 0 = r 2 nên
4AOB ∼ 4B 0 OA0 , suy ra

A0 B 0 OA0 NQ OA0 r2
= ⇒ A0 B 0 = = AB · = AB · ,
AB OB 2 OB OA · OB
NQ
hay AB = 2 · OA·OB
r 2 . Chứng minh tương tự ta được

M P OB · OC NQ OC · OD M P OD · OA
BC = · 2
, CD = · 2
, DA = · .
2 r 2 r 2 r2

Hướng tới kỷ niệm 20 năm thành lập trường PTNK, ĐHQG TP HCM
234 Chuyên đề Toán học số 10

b) Theo câu a), ta có


Chuyên đề Toán học Phổ thông năng khiếu TP.HCM số 10

NQ
AB 2 · OA·OB
r2 OA · OB
= NQ OC·OD
= .
CD 2 · r2
OC · OD
AD
Tương tự BC = OB·OC
OA·OD
.
c) Theo câu b), ta có
AB BC OA · OB OB · OC OB 2
· = · = ,
C D DA OC · OD OD · OA OD2
OB 2 OD2 OA2 OC 2
hay AB·BC = C D·DA . Tương tự, DA·AB = BC·C D .

Bài 3. Cho tứ giác ABC D và điểm P là giao điểm của hai đường chéo AC, BD.
Gọi R1 , R2 , R3 , R4 lần lượt là bán kính đường tròn ngoại tiếp các tam giác
PAB, P BC, P C D, P DA. Chứng minh rằng tứ giác ABC D ngoại tiếp khi và chỉ
khi R1 + R3 = R2 + R4 .
Lời giải.
Theo định lý Sin, ta có
R1 + R3 = R2 + R4
AB CD BC AD
⇔ + = + ,
sin ∠AP B sin ∠C P D sin ∠BP C sin ∠DPA
tương đương
AB + C D = AD + BC.
Theo tính chất 1, suy ra điều phải chứng minh.
Bài 4. Cho tứ giác ABC D ngoại tiếp đường tròn (O, r). Gọi M , N , P, Q là tiếp
điểm của (O) với các cạnh AB, BC, C D, DA và S là giao điểm của M N , PQ.
Gọi E, F là giao điểm của đường thẳng BD với đường tròn (O) . Chứng minh
rằng SE, SF là tiếp tuyến của đường tròn (O) .
S

B
M E
A

I
N
Q

D P C

Hướng tới kỷ niệm 20 năm thành lập trường PTNK, ĐHQG TP HCM
Chuyên đề Toán học số 10 235

Lời giải.
Chuyên đề Toán học Phổ thông năng khiếu TP.HCM số 10

Theo lời giải tính chất 4, ta có BD là đường đối cực của S nên theo tính
chất đường đối cực, rõ ràng SE, SF là tiếp tuyến của đường tròn (O) .

p Cho tứ giác ABCD ngoại tiếp đường tròn (O).


Bài 5. (IMO Shortlist 1986)
Chứng minh rằng SABC D ≤ abcd.

Lời giải.
Theo tính chất 6, ta có

p A+ C p
SABC D = abcd · sin ≤ abcd.
2

Đẳng thức xảy ra khi và chỉ khi

A+ C
sin = 1 ⇔ ∠A + ∠C = 180◦ ,
2

hay ABC D lưỡng tiếp (vừa nội tiếp, vừa ngoại tiếp).

Bài 6. (China TST 2003) Cho tứ giác ABCD ngoại tiếp đường tròn (O).
Chứng minh rằng
p
OA · OC + OB · OD = abcd.

Lời giải.
Theo tính chất 6, ta có

A+ C
 ‹ p
A B C D
SABC D = r cot + cot + cot + cot
2
= abcd · sin
2 2 2 2 2
A+C B+D
A+ C
 
sin 2 sin 2 p
⇔ r2 A C
+ B D
= abcd · sin
sin 2 · sin 2 sin 2 · sin 2 2
p
⇔ OA · OC + OB · OD = abcd.

Ta có điều phải chứng minh.

Bài 7. (Nguyễn Minh Hà) Cho tứ giác ABC D ngoại tiếp đường tròn (O) và
có một cặp cạnh đối không song song. Chứng minh điểm O là trọng tâm của
tứ giác ABC D khi và chỉ khi OA · OC = OB · OD.

Hướng tới kỷ niệm 20 năm thành lập trường PTNK, ĐHQG TP HCM
236 Chuyên đề Toán học số 10

E
Chuyên đề Toán học Phổ thông năng khiếu TP.HCM số 10

M
O N

D C

Lời giải.
Không mất tính tổng quát, giả sử AD và BC cắt nhau tại E. Do O là tâm
bàng tiếp 4EAB nên nếu gọi I là tâm nội tiếp 4EAB thì E, O, I thẳng hàng
và IAOB nội tiếp đường tròn (OI) .
Điều kiện cần. Giả sử O là trọng tâm ABC D. Gọi M , N là trung điểm của
AD, BC thì O là trung điểm của M N . Mà EO là phân giác ∠M ON nên
4E M N cân tại E. Suy ra EO⊥M N . Từ đó

∠AOM = 90◦ − ∠AOI = 90◦ − ∠ABI = ∠ABO = ∠OBN .

Suy ra 4M OA ∼ 4N BO. Tương tự 4M OD ∼ 4N CO. Từ đó


OA M A M D OD
= = = ,
OB ON ON OC
hay OA · OC = OB · OD.
Điều kiện đủ. Giả sử OA · OC = OB · OD. Đường thẳng qua O và vuông góc
EO cắt AD, BC tại M , N . Do EO vừa là phân giác, vừa là đường cao nên
4E M N cân tại E và O trung điểm M N . Chứng minh như trên, ta được
4M OA ∼ 4N BO và 4M OD ∼ 4N CO. Do đó
MA M A ON OA OC
= · = · = 1,
M D ON M D OB OD

nên M là trung điểm AD. Tương tự, N là trung điểm BC. Suy ra O là trọng
tâm ABC D.

Bài 8. (Sharygin 2014) Cho tứ giác ABC D ngoại tiếp đường tròn (O). Gọi
X là giao điểm hai tiếp tuyến tại A và C của đường tròn (AOC), Y là giao
điểm hai tiếp tuyến tại B và D của đường tròn (BOD). Chứng minh X , Y, O
thẳng hàng.

Hướng tới kỷ niệm 20 năm thành lập trường PTNK, ĐHQG TP HCM
Chuyên đề Toán học số 10 237

P
Chuyên đề Toán học Phổ thông năng khiếu TP.HCM số 10

A
B

D S O

X C

Y
R

Lời giải.
Gọi K là giao điểm thứ hai của (AOC) và (BOD). Lấy P ∈ OA, Q ∈ OB sao
cho ∠AKO = ∠AP K, ∠BKO = ∠BQK. Rõ ràng

OA · OP = OB · OQ = OK 2 . (1)

Gọi R = P K ∩ OC, S = QK ∩ OD. Do 4OAK ∼ 4OK P và OAKC nội tiếp


nên
∠OKR = 180◦ − ∠OK P = 180◦ − ∠OAK = ∠OC K.
Từ đó 4OC K ∼ 4OKR. Tương tự 4ODK ∼ 4OKS. Suy ra

OC · OR = OD · OS = OK 2 . (2)

Ta sẽ chứng minh PQRS là hình bình hành. Thật vậy, từ (1) và (2) suy ra
PABQ, QBCR, RC DS, SDAB nội tiếp. Từ đó

∠SPQ = ∠OPS + ∠OPQ = ∠ODA + ∠OBA =


= ∠ODC + ∠OBC = ∠ORS + ∠ORQ = ∠SRQ.

Tương tự, ∠PQR = ∠PSR. Do đó PQRS là hình bình hành, suy ra K là


trung điểm PR, QS.
Từ đó, do 4OAK ∼ 4OK P, 4OC K ∼ 4OKR nên
AK KO KO CK
= = = .
AO KP KR CO

Hướng tới kỷ niệm 20 năm thành lập trường PTNK, ĐHQG TP HCM
238 Chuyên đề Toán học số 10

Do đó AOC K điều hoà, hay X , O, K thẳng hàng. Tương tự, BODK điều hoà
Chuyên đề Toán học Phổ thông năng khiếu TP.HCM số 10

nên Y, O, K thẳng hàng. Suy ra X , Y, O thẳng hàng.

Hy vọng qua bài viết nhỏ này, bạn đọc sẽ tìm thấy sự thú vị riêng cũng
như nét đẹp của tứ giác ngoại tiếp và đóng góp thêm ý kiến cho chúng tôi.
Cuối cùng, xin gửi vài bài tập để bạn đọc thử sức.

Bài 9. (Darji Grinberg) Cho tứ giác ABC D ngoại tiếp đường tròn (O). Chứng
minh hệ thức sau

OA2 OB 2 OC 2 OD2
+ + + = 2.
DA · AB AB · BC BC · C D C D · DA

Bài 10. Cho tam giác 4ABC có M , N lần lượt là trung điểm AC, AB và trọng
tâm G. Chứng minh rằng tứ giác ADI E ngoại tiếp khi và chỉ khi 4ABC cân.

Bài 11. Cho tứ giác ABC D ngoại tiếp đường tròn (O). Các đường thẳng qua
đỉnh A và vuông góc với AB, AD lần lượt cắt OB, OD tại M , N . Chứng minh
rằng AC vuông góc với M N .

Bài 12. Cho tứ giác ABC D ngoại tiếp đường tròn (O) và M , N , P, Q là tiếp
điểm của (O) với các cạnh AB, BC, C D, DA. Gọi E, F, S, T lần lượt là giao
điểm của các cặp đường thẳng AD và BC, AB và C D, M N và PQ, MQ và
N P. Chứng minh rằng bốn điểm E, F, S, T thẳng hàng.

Bài 13. (Bulgaria Winter-MO 1996) Cho tứ giác ABC D ngoại tiếp đường
tròn (O). Các đường thẳng qua A, B, C, D và lần lượt vuông góc với OA, OB, OC, OD
tạo thành tứ giác X Y Z T . Chứng minh rằng X Y Z T nội tiếp và O là giao điểm
của hai đường thẳng X Z, Y T.

Bài 14. Đường tròn (O) nội tiếp hình vuông ABC D, tiếp xúc cạnh AB tại
điểm M . Một tiếp tuyến với đường tròn (O) cắt các cạnh C B, C D lần lượt ở
E, F. Chứng minh rằng

a) Các tam giác 4DF O và 4BOE đồng dạng.

b) M E song song với AF.

Bài 15. (Romanian TST 2007) Gọi M , N là tiếp điểm của đường tròn nội
tiếp tam giác 4ABD với các cạnh AB, AD và P, Q là tiếp điểm của đường tròn
nội tiếp tam giác 4C BD với các cạnh C B, C D. Giả sử hai đường tròn này
tiếp xúc với nhau. Chứng minh rằng tứ giác ABC D ngoại tiếp.

Bài 16. (Michael de Villiers) Chứng minh đường trung trực của các cạnh của
một tứ giác ngoại tiếp tạo thành một tứ giác ngoại tiếp khác.

Hướng tới kỷ niệm 20 năm thành lập trường PTNK, ĐHQG TP HCM
Tài liệu tham khảo
Chuyên đề Toán học Phổ thông năng khiếu TP.HCM số 10

[1] Lương Văn Khải, The works for Sharygin Olympiad 2014.

[2] Trần Lê Bách THPT chuyên Lê Quý Đôn Đà Nẵng, Tứ giác ngoại tiếp.

[3] http://vie.math.ac.vn/~mathclub/images/stories/fileupload/
Nguyen_Minh_Ha-hinhhocphang.pdf

[4] Các diễn đàn toán học: forum.mathscope.org, diendantoanhoc.


net, pitago.vn

239
240 Chuyên đề Toán học số 10
Chuyên đề Toán học Phổ thông năng khiếu TP.HCM số 10

Hướng tới kỷ niệm 20 năm thành lập trường PTNK, ĐHQG TP HCM
Chuyên đề Toán học Phổ thông năng khiếu TP.HCM số 10

Giới thiệu kỳ thi Sharygin


Olympic hình học toàn Nga mở rộng

Nguyễn Duy Tùng


(Lớp Chuyên Toán khóa 2013 − 2016)
Ngô Hoàng Anh
(Lớp Chuyên Toán khóa 2015 − 2018)

1. Giới thiệu về cuộc thi


Olympic hình học Sharygin là cuộc thi được tổ chức bởi MCCME (Moscow
Center for Continuous Mathematical Education) hàng năm ở Liên bang
Nga, bắt đầu từ 2005. Cuộc thi được đặt theo tên của I. F. Sharygin, nhà
sư phạm lỗi lạc, tác giả những bài toán hình học đặc sắc. Ban đầu, kì
thi này chỉ dành cho các học sinh ở Nga tham dự nhưng đến năm 2007,
với sức hút của nó, các học sinh nước ngoài cũng mong muốn tham dự
kì thi này và ban tổ chức đã tạo điều kiện cho họ. Cứ khoảng đầu tháng
1 hàng năm, ban tổ chức lại đăng tải đề thi cho năm đó trên trang web
geometry.ru/olimp/olimpsharygin.php bằng cả tiếng Anh lẫn tiếng Nga.
Học sinh trong độ tuổi cấp 2 và cấp 3 trên toàn thế giới có thể gửi bài tham
dự cuộc thi. Sau khoảng 2 - 3 tháng, các học sinh xuất sắc nhất sẽ được
chọn tham dự vòng Chung kết diễn ra ở Moscow. Ở vòng Chung kết. Các
thí sinh được phát một đề thi khác. Các thí sinh sẽ trình bày trực tiếp bài
giải của mình cho Hội đồng giám khảo nghe, sau đó Hội đồng giám khảo
sẽ chấm điểm và xét giải.
Các bài toán của Sharygin Olympiad không đòi hỏi nhiều kiến thức, nhưng
yêu cầu người giải phải nắm chắc hệ thống kiến thức cơ bản và vận dụng
nó khéo léo trong các mô hình lạ mắt. Điều quan trọng nhất với các thí
sinh là phải có cảm nhận tốt và tư duy trực quan nhanh nhạy, cũng như
một chút đột phá để tìm ra lời giải.

241
242 Chuyên đề Toán học số 10

Trong những năm gần đây, trường Phổ thông Năng khiếu liên tục có đại
Chuyên đề Toán học Phổ thông năng khiếu TP.HCM số 10

diện được chọn tham dự Chung kết, cụ thể như sau:


• 2013: Nguyễn Huy Hoàng, giải ba chung kết.

• 2014: Phạm Ngọc Quyền, có tham gia chung kết.


Lương Văn Khải, được mời vào vòng chung kết.
Nguyễn Minh Châu, được mời vào vòng chung kết.

• 2015: Lương Văn Khải, được mời vào vòng chung kết.
Nguyễn Xuân Sơn, giải khuyến khích chung kết.

2. Đề thi
2.1. Năm 2013
Bài 1. (8) Cho 4ABC cân tại B, điểm E nằm trên đoạn AB. D là chân
đường vuông góc kẻ từ E xuống BC. Cho biết AE = DE. Tính ∠DAC.
Bài 2. (8) 4ABC cân tại C nội tiếp (O) có ∠C = 20◦ . Các đường phân giác
trong góc A và B lần lượt cắt các cạnh đối diện tại A1 và B1 . Chứng minh
4A1 OB1 đều.
Bài 3. (8) Cho 4ABC vuông tại B. Đường tròn bàng tiếp góc A tiếp xúc
AB, AC lần lượt tại A1 và A2 . Các điểm C1 , C2 xác định tương tự. Chứng
minh rằng các đường vuông góc lần lượt kẻ từ A, B, C xuống C1 C2 , A1 C1 , A1 A2
đồng quy.
Bài 4. (8) Cho 4ABC không cân nội tiếp O. Gọi K là tâm của ω là đường
tròn ngoại tiếp 4BOC. Đường cao kẻ từ A cắt ω tại P. Đường thẳng P K
cắt (O) tại E và F. Chứng minh rằng PA = P E hoặc PA = P F .
Bài 5. (8) Có một điểm nằm trong một tứ giác lồi. Nối điểm đó với 4 đỉnh
của tứ giác để tạo thành 4 tam giác. Biết rằng 4 tam giác ấy bằng nhau,
vậy ta có thể kết luận tứ giác ban đầu là hình thoi được chưa?
Bài 6. (8 - 9) Hình thang ABC D có P là giao điểm hai đường chéo. Các
đường tròn ngoại tiếp tam giác ABP và C DP lần lượt cắt đường thẳng
AD tại điểm thứ hai là X vag Y . Gọi M là trung điểm X Y . Chứng minh
M B = M C.
Bài 7. (8 - 9) 4ABC có BD là phân giác trong. Gọi I a , I c lần lượt là tâm
đường tròn nội tiếp các tam giác ABD và C BD. Đường thẳng I a I c cắt AC ở
Q. Chứng minh rằng ∠DBQ = 90◦ .

Hướng tới kỷ niệm 20 năm thành lập trường PTNK, ĐHQG TP HCM
Chuyên đề Toán học số 10 243

Bài 8. (8 - 9) Cho một điểm X bất kì nằm trong đường tròn ngoại tiếp
4ABC. Các đường thẳng BX và C X lần lượt cắt (ABC) tại K và L. Đường
Chuyên đề Toán học Phổ thông năng khiếu TP.HCM số 10

thẳng K L lần lượt cắt AB và AC tại E và F . Tìm quỹ tích các điểm X sao
cho (AF K) và (AE L) tiếp xúc nhau.

Bài 9. (8 - 9) Gọi T1 , T2 lần lượt là tiếp điểm của các đường tròn bàng
tiếp 4ABC tiếp xúc với BC và CA. Biết rằng điểm đối xứng của tâm đường
tròn nội tiếp 4ABC qua trung điểm của AB nằm trên đường tròn ngoại
tiếp 4C T1 T2 . Tính ∠BCA.

Bài 10. (8 - 9) Đường tròn nội tiếp 4ABC tiếp xúc AB ở C 0 . Đường tròn nội
tiếp AC C 0 tiếp xúc AB, AC lần lượt ở C1 , B1 . Đường tròn nội tiếp BC C 0 tiếp
xúc AB, BC lần lượt ở C2 , A2 . Chứng minh các đường thẳng B1 C1 , A2 C2 , C C 0
đồng quy.

Bài 11. (8 - 9) Cho tứ giác lồi ABCD.

a) Gọi r1 , r2 , r3 , r4 là bán kính đường tròn nội tiếp ABC, BC D, C DA, DAB.
Biết r1 ≤ r2 ≤ r3 ≤ r4 . Hỏi bất đẳng thức r4 > 2r3 có luôn đúng
không?

b) Gọi E là giao điểm hai đường chéo của tứ giác ABC D. Gọi r1 , r2 , r3 , r4
là bán kính đường tròn nội tiếp các tam giác ABE, BC E, C DE, DAE.
Biết r1 ≤ r2 ≤ r3 ≤ r4 . Hỏi bất đẳng thức sau r2 > 2r1 có luôn đúng
hay không?

Bài 12. (8 - 11) Trên mỗi cạnh của 4ABC có hai điểm. Ta biết rằng những
điểm đó là chân các đường cao và đường phân giác của 4ABC, nhưng chưa
biết cụ thể điểm nào là chân đường nào.

a) Chỉ dùng một thước thẳng, hãy tìm ra đâu là các chân đường cao,
đâu là chân các đường phân giác. Thước thẳng ở đây được hiểu là
thước chỉ dùng để kẻ đường thẳng, không có vạch chia độ dài, không
có vạch chia góc, hình dạng bất kì - miễn là kẻ được đường thẳng.

b) Chỉ được vẽ thêm 3 đường thẳng, hãy giải quyết câu a.

Bài 13. (9 - 10) Gọi A1 và C1 lần lượt là tiếp điểm của tâm đường tròn nội
tiếp 4ABC với BC và AB. Gọi A0 và C 0 lần lượt là tiếp điểm của đường tròn
bàng tiếp góc ∠B với đường thẳng BC và AB. Chứng minh rằng trực tâm
H của 4ABC nằm trên đường thẳng A1 C1 khi và chỉ khi A0 C1 ⊥ AB.

Hướng tới kỷ niệm 20 năm thành lập trường PTNK, ĐHQG TP HCM
244 Chuyên đề Toán học số 10

Bài 14. (9 - 11) Gọi M , N là trung điểm hai đường chéo AC, BD của hình
thang vuông ABC D (∠A = ∠D = 90◦ ). Đường tròn ngoại tiếp các tam giác
Chuyên đề Toán học Phổ thông năng khiếu TP.HCM số 10

ABN và C DM cắt đường thẳng BC ở Q, R. Chứng minh rằng khoảng cách


từ Q, R tới trung điểm của M N bằng nhau.

Bài 15. (9 - 11)

a) Các tam giác A1 B1 C1 và A2 B2 C2 nội tiếp 4ABC sao cho

C1 A1 ⊥ BC, A1 B1 ⊥ CA, B1 C1 ⊥ AB, B2 A2 ⊥ BC, C2 B2 ⊥ CA, A2 C2 ⊥ AB.

Chứng minh hai tam giác A1 B1 C1 và A2 B2 C2 bằng nhau.

b) Các điểm A1 , B1 , C1 , A2 , B2 , C2 nằm trong tam giác ABC sao cho A1 ∈


AB1 , B1 ∈ BC1 , C1 ∈ CA1 , A2 ∈ AC2 , B2 ∈ BA2 , C2 ∈ C B2 và các góc

∠BAA1 , ∠C BB1 , ∠AC C1 , ∠CAA2 , ∠ABB2 , ∠BC C2

bằng nhau. Chứng minh hai tam giác A1 B1 C1 và A2 B2 C2 bằng nhau.

Bài 16. (9 - 11) Đường tròn nội tiếp 4ABC tiếp xúc BC, CA, AB lần lượt
tại A0 , B 0 , C 0 . Đường vuông góc kẻ từ tâm nội tiếp I đến trung tuyến kẻ từ
C cắt đường thẳng A0 B 0 tại K. Chứng minh C K k AB.

Bài 17. (9 - 11) Gọi φ là góc nhọn tạo bởi hai đường chéo của một tứ giác
nội tiếp. Chứng minh rằng các góc nhọn tạo bởi các tứ giác khác có cùng
độ dài cạnh với tứ giác nội tiếp đó đều nhỏ hơn φ.

Bài 18. (9 - 11) 4ABC có AD là đường phân giác trong. M và N lần lượt
là hình chiếu của B và C xuống AD. Đường tròn đường kính M N cắt BC
tại X và Y . Chứng minh ∠BAX = ∠CAY .

Bài 19. (10 - 11)

a) Đường tròn nội tiếp 4ABC tiếp xúc AC và AB lần lượt tại B0 và C0 .
Đường phân giác trong của ∠B và ∠C lần lượt cắt trung trực của
phân giác AL tại Q và P. Chứng minh rằng P C0 , QB0 , BC đồng quy.

b) Gọi O1 và O2 lần lượt là tâm đường tròn ngoại tiếp các tam giác
AB L và AC L. Gọi B1 và C1 lần lượt là hình chiếu của C và B xuống
đường phân giác trong góc B và C. Chứng minh các đường thẳng
O1 C1 , O2 B1 , BC đồng quy.

c) Chứng minh hai điểm đồng quy ở câu a và câu b trùng nhau.

Hướng tới kỷ niệm 20 năm thành lập trường PTNK, ĐHQG TP HCM
Chuyên đề Toán học số 10 245

Bài 20. (10 - 11) Cho 4ABC, điểm C1 trên cạnh AB. Trên tia BC và AC
lần lượt lấy A1 và B1 sao cho ∠AC1 B1 = ∠BC1 A1 = ∠AC B. Gọi C C2 là giao
Chuyên đề Toán học Phổ thông năng khiếu TP.HCM số 10

điểm của hai đường thẳng AA1 và BB1 . Chứng minh rằng đường thẳng
C1 C2 đi qua một điểm cố định.

Bài 21. (10 - 11) Cho đường tròn ω và một điểm A nằm bên trong đường
tròn đó. Qua A kẻ hai đường thẳng, một đường thẳng cắt ω tại B và C,
đường thẳng kia cắt ω tại D và E sao cho D nằm giữa A và E. Qua D kẻ
một đường thẳng song song với BC cắt ω tại F . Đường thẳng AF cắt ω tại
T . Gọi M là giao điểm của hai đường thẳng E T và BC, N là điểm đối xứng
của A qua M . Chứng minh rằng trung điểm của BC nằm trên đường tròn
ngoại tiếp 4DEN .

Bài 22. (10 - 11) Đường vuông góc chung cùa các cặp cạnh đối trong tứ
giác không nằm trên 1 mặt phẳng thì vuông góc lẫn nhau. Chứng minh
rằng các đường vuông góc chung đó đồng quy.

Bài 23. (10 - 11) Cho hai đa diện lồi A và B không có điểm chung. Đa diện
A có đúng 2012 mặt đối xứng.

a) Biết rằng B có 2012 mặt phẳng đối xứng, hỏi hình A∪ B có bao nhiêu
mặt phẳng đối xứng?

b) Biết rằng B có 2013 mặt phẳng đối xứng, hỏi hình A∪ B có bao nhiêu
mặt phẳng đối xứng?

a) Câu hỏi tương tự cho ý b), nhưng thay mặt phẳng đối xứng bởi trục
đối xứng.

2.2. Năm 2014


Bài 1. (8) Cho tam giác ABC vuông. Cạnh góc vuông AB là đáy tam giác
ADB đều nằm ngoài tam giác ABC, cạnh huyền AC là đáy tam giác AEC
đều nằm trong tam giác ABC. Giả sử DE và AB giao nhau tại M . Giả sử cả
hệ thống hình vẽ trên đều bị xóa trừ hai điểm A và B. Hãy dựng lại điểm
M.

Bài 2. (8) Cho một mảnh giấy vuông độ dài 2. Từ đây, ta có thể cắt thành
hình 12 cạnh có tất cả các cạnh đều bằng 1, và mỗi góc chia hết cho 45◦
không ?

Bài 3. (8) Cho 4ABC cân tại C. Đường thẳng l tiếp xúc với đường tròn
ngoại tiếp 4ABC tại B. Gọi D là chân đường vuông góc kẻ từ C tới l. Gọi
AE, BF là các đường cao của 4ABC. Chứng minh DE F thẳng hàng.

Hướng tới kỷ niệm 20 năm thành lập trường PTNK, ĐHQG TP HCM
246 Chuyên đề Toán học số 10

Bài 4. (8) Một hình vuông "nội tiếp" hình tam giác (Có một cạnh tam giác
Chuyên đề Toán học Phổ thông năng khiếu TP.HCM số 10

chứa 2 đỉnh và mỗi cạnh còn lại của tam giác chứa 1 đỉnh). Chứng minh
rằng tâm nội tiếp tam giác nằm trong hình vuông.

Bài 5. (8) Trong tam giác nhọn ABC, AM là trung tuyến, AL là phân giác
và AH là đường cao (H nằm giữa L và B). Biết rằng M L = LH = H B. Tìm
tỉ lệ các cạnh tam giác ABC.

Bài 6. (8 - 9) Cho một đường tròn tâm O và điểm P không năm trên
đường tròn đó. Cho X là điểm bất kì trên đường tròn và Y là giao điểm
của phân giác góc ∠POX và trung trực của P X . Tìm quỹ tích điểm Y khi
X di chuyển.

Bài 7. (8 - 9) Cho hình bình hành ABC D. Đường vuông góc từ C xuống
C D cắt đường vuông góc từ A xuống BD tại F , và đường vuông góc từ B
BC
xuống AB cắt đường trung trực của AC tại E. Tìm tỉ số .
EF

Bài 8. (8 - 9) Cho hình chữ nhật ABC D. Hai đường vuông góc qua điểm
B, đoạn đầu tiên cắt AD tại K và đoạn thứ hai cắt C D tại L. F là giao điểm
của K L và AC. Chứng minh BF ⊥ K L.

Bài 9. (8 - 9) Hai đường tròn (ω1 ) và (ω2 ) tiếp xúc ngoài với nhau tại
điểm L và "nằm gọn" trong ∠BAC sao cho (ω1 ) tiếp xúc tia AB tại E, (ω2 )
tiếp xúc tia AC tại M . Đoạn E F giao (ω2 ) lần thứ hai tại Q. Chứng minh
MQ k AL.

Bài 10. (8 - 9) Điểm K, L, M , N lần lượt trên các cạnh AB, BC, C D, DA của
hình vuông ABC D là các đỉnh của hình vuông khác. DK và N M giao nhau
tại E, và KC và LM giao nhau tại F . Chứng minh E F ⊥ AB.

Bài 11. (9 - 10) Hai đường tròn (ω1 ) và (ω2 ) giao nhau tại A và B. Lấy K1
và K2 lần lượt thuộc (ω1 ) và (ω2 ) sao cho K1 A tiếp xúc (ω2 ), K2 A tiếp xúc
(ω1 ). Đường tròn (K1 BK2 ) giao AK1 và AK2 lần thứ hai lần lượt tại L1 và
L2 . Chứng minh L1 và L2 cùng cách đều AB.

Bài 12. (9 - 10) Cho AC là một dây cung thay đổi của đường tròn (ω)
với tâm O. Điểm B di chuyển trên cung AC. Một điểm P thay đổi nằm
trên AC. Đường thẳng đi qua P và song song với AO cắt BA tại điểm A1 ;
đường thẳng qua C và song song với CO cắt BC tại điểm C1 . Chứng minh
rằng tâm đường tròn ngoại tiếp tam giác A1 BC1 di chuyển trên một đường
thẳng cố định.

Hướng tới kỷ niệm 20 năm thành lập trường PTNK, ĐHQG TP HCM
Chuyên đề Toán học số 10 247

Bài 13. (9 - 11) Trong một đường tròn cho trước, hãy dựng một "phần
Chuyên đề Toán học Phổ thông năng khiếu TP.HCM số 10

con" sao cho diện tích của nó bằng một nửa diện tích của đường tròn ban
đầu và diện tích phần giao của "phần con" đó với hình đối xứng của nó
qua một đường kính bất kì bằng diện tích của đường tròn ban đầu.
Bài 14. (9 - 11) Cho 4ABC không cân. Đường cao kẻ từ A, đường phân
giác kẻ từ B và đường trung tuyến kẻ từ C đồng quy tại điểm K.
a) Trong 3 cạnh của tam giác, cạnh nào có giá trị độ dài nằm giữa giá
trị độ dài của 2 cạnh còn lại?

b) Trong số đo độ dài của 3 đường AK, BK, C K, số đo độ dài của cạnh


nào nằm ở giữa?
Bài 15. (9 - 11) Cho tam giác ABC và một điểm bất kì D. Các đường
thẳng đi qua D và vuông góc với các đường thẳng DA, DB, DC cắt các
đường thẳng BC, CA, AB tại các điểm A1 , B1 , C1 tương ứng. Chứng minh
rằng trung điểm của các cạnh AA1 , BB1 , C C1 thẳng hàng.
Bài 16. (10 - 11) 4ABC vuông ở B. Đường phân giác BD được kẻ sẵn, và
các điểm chính giữa E và F của cung BD của các đường tròn ngoại tiếp
tam giác ADB và C DB được đánh dấu lại (các đường tròn đã được xóa
đi). Chỉ bằng thước thẳng, hãy dựng lại tâm của các đường tròn đó. Thước
thẳng được hiểu là thước chỉ dùng để kẻ đường thẳng, không có vạch chia
độ dài, hình dạng bất kì miễn là vẽ được đường thẳng.
Bài 17. (10 - 11) Cho tứ giác ABC D ngoại tiếp đường tròn tâm I. Tiếp
tuyến của đường tròn (AI C) tại các điểm A và C cắt nhau tại điểm X . Tiếp
tuyến của đường tròn (BI D) tại các điểm B và D gặp nhau tại điểm Y .
Chứng minh rằng X , I, Y thẳng hàng.
Bài 18. (10-11) Hai đường tròn (ω1 ) và (ω2 ) tiếp xúc ngoài với nhau tại
điểm P. Cho A là một điểm nằm trên (ω2 ) không nằm trên đường thẳng qua
hai tâm của hai đường tròn, và AB, AC là các tiếp tuyến với (ω1 ). Đường
thằng BP, C P cắt (ω2 ) lần thứ hai tại các điểm E và F . Chứng minh rằng
đường thẳng E F , tiếp tuyến với (ω2 ) tại điểm A và đường tiếp tuyến chung
tại P đồng quy
Bài 19. (10 - 11) Cho tứ giác K LM N . Một đường tròn với tâm O cắt
cạnh K L lại điểm A và A1 . Các điểm B, C, D, B1 , C1 , D1 được xác định tương
tự. Chứng minh nếu các đường tròn ngoại tiếp của K DA, LAB, M BC, N C D
đồng quy với nhau tại điểm N thì:
a) Đường tròn ngoại tiếp của các tam giác K D1 A1 , LA1 B1 , M B1 C1 , N C1 D1
cũng đồng quy với nhau tại một điểm Q nào đó.

Hướng tới kỷ niệm 20 năm thành lập trường PTNK, ĐHQG TP HCM
248 Chuyên đề Toán học số 10

b) Điểm O nằm trên đường trung trực của PQ.


Chuyên đề Toán học Phổ thông năng khiếu TP.HCM số 10

Bài 20. (10 - 11) Cho ABC D là một tứ giác ngoại tiếp. Đường tròn (ω)
nội tiếp nó tiếp xúc với các cạnh BC và DA tại các điểm E và F tương ứng.
Biết rằng các đường thẳng AB, F E và C D đồng quy. Các đường tròn ngoại
tiếp của tam giác AE D và BF C cắt (ω) lần thứ hai tại các điểm E1 và F1 .
Chứng minh rằng E F k E1 F1 .

Bài 21. (10 - 11) Có tồn tại hay không một khối đa diện sao cho nó có các
đường chéo mà mỗi đường chéo nhỏ hơn các cạnh của nó?

Bài 22. (11) Cho A, B, C và D là một bộ 4 điểm điều hòa, sao cho AB·C D =
AC · BD = AD · BC. Cho A1 là một điểm khác A sao cho bộ 4 điểm A1 , B, C
và D điều hòa. Các điểm B1 , C1 , D1 được định nghĩa tương tự. Chứng minh
rằng:

a) A, B, C1 , D1 cùng nằm trên một đường tròn.

b) Bộ 4 điểm A1 , B1 , C1 , D1 điều hòa.

Bài 23. (11) Cho hình chóp nội tiếp S.ABC D. Những cạnh đối nhau của
đáy hình chóp gặp nhau tại điểm P và Q sao cho A và B lần lượt nằm trên
đoạn P D và P C. Mặt cầu nội tiếp tiếp xúc mặt phẳng (ABS) và (BC S) lần
lượt tại K và L. CHứng minh nếu P K và QL đồng phẳng thì điểm tiếp xúc
của mặt cầu với đáy sẽ nằm trên BD.

2.3. Năm 2015


Bài 1. (8) Tanya cắt một đa giác lồi từ một tờ giấy, gắp nó nhiều lần và
thu được một hình 4 cạnh 2 lớp (Tức hình tạo bởi 2 hình tứ giác có 1 cạnh
chung). Liệu đa giác lồi lúc đầu đó có thể có 7 cạnh hay không?

Bài 2. (8) Cho O và H lần lượt là tâm ngoại tiếp và trực tâm tam giác ABC.
Đường thẳng qua trung điểm OH và song song với BC giao AB và AC lần
lượt tại D và E. Biết O cũng là tâm nội tiếp tam giác ADE. Tính ∠ABC.

Bài 3. (8) Cạnh AD của hình vuông ABC D là đáy của tam giác tù cân
AE D với điểm E nằm bên trong hình vuông. AF là đường kính của đường
tròn (ADE), và G thuộc đoạn C D sao cho C G = DF . Chứng minh rằng
1
∠BGE < ∠AE D.
2
Bài 4. (8) Cho hình bình hành ABC D có các đoạn chia ba lần lượt tại hai
góc A và B. Để O là giao của 2 đoạn chia ba trên mà gần AB nhất. Cho

Hướng tới kỷ niệm 20 năm thành lập trường PTNK, ĐHQG TP HCM
Chuyên đề Toán học số 10 249

AO giao với đường chia ba thứ hai của góc ∠B tại A1 , BO giao với đường
Chuyên đề Toán học Phổ thông năng khiếu TP.HCM số 10

chia ba thứ hai của góc A tại B1 , M là trung điểm A1 B1 . M O giao AB tại N .
Chứng minh A1 B1 N là tam giác đều.

Bài 5. (8 - 9) Cho 4ABC. Vẽ hai đường tròn cùng qua A và lần lượt tiếp xúc
BC lần lượt tại B và C và D là điểm giao khác A của hai đường tròn này (A
gần BC hơn D). Biết rằng BC = 2BD. Chứng minh rằng ∠DAB = 2∠ADB.

Bài 6. (8 - 9) 4ABC nhọn có AA0 , BB 0 vC C 0 là các đường cao. Các điểm


Ca , C b đối xứng với C 0 qua AA0 và BB 0 . Các điểm A b , Ac , Bc , Ba được xác
định tương tự. Chứng minh rằng các đoạn A b Ba , Bc C b và Ca Ac song song.

Bài 7. (8 - 9) Hai đường cao AA1 và C C1 của tam giác ABC giao nhau tại
H. Điểm HA đối xứng với H qua A. Đoạn HA C1 gặp BC tại C 0 ; điểm A0 được
xác định tương tự. Chứng minh AC k A0 C 0 .

Bài 8. (8 - 9) Đường chéo của hình thang cân ABC D (BC k AD) vuông góc
với nhau. Để E là hình chiếu vuông góc từ D xuống AB, F là hình chiếu
1
vuông góc từ C xuống DE. Chứng minh ∠DBF = ∠F C D.
2
Bài 9. (8 - 9) Cho ABC là tam giác nhọn. Hãy dựng A0 , B 0 , C 0 lên các cạnh
BC, CA, AB sao cho các điều kiện sau được thoả mãn:

a) AB k A0 B 0 .

b) C 0 Clà phân giác góc ∠A0 C 0 B 0 .

c) A0 C 0 + B 0 C 0 = AB.

Bài 10. (8 - 9) Các đường chéo của một tứ giác lồi chia nó thành 4 tam
giác đồng dạng với nhau. Chứng minh rằng đây là tứ giác ngoại tiếp.

Bài 11. (8 - 10) Gọi H là trực tâm tam giác nhọn ABC. Đường trung trực
BH cắt BA, BC lần lượt tại A0 , C0 . Chứng minh độ dài đường kính (A0 H C0 )
bằng AC với O là tâm ngoại tiếp 4ABC.

Bài 12. (8 - 11) Tìm số lượng lớn nhất các đường tròn cùng nằm trên một
mặt phẳng sao cho mỗi cặp đường tròn đều cắt nhau tại hai điểm phân
biệt và không có bộ ba đường tròn nào có hai điểm chung.

Bài 13. (9 - 10) 4ABC có AH1 , BH2 và C H3 là các đường cao. Gọi M là
trung điểm H2 H3 , AM cắt H2 H1 tại K. Chứng minh K nằm trên đường
trung bình song song với AC của 4ABC.

Hướng tới kỷ niệm 20 năm thành lập trường PTNK, ĐHQG TP HCM
250 Chuyên đề Toán học số 10

Bài 14. (9 - 11) Cho 4ABC nhọn, không cân. Điểm A1 và A2 lần lượt đối
Chuyên đề Toán học Phổ thông năng khiếu TP.HCM số 10

xứng qua đường phân giác trong và ngoài của góc ∠A qua trung điểm BC.
Gọi α là đường tròn đường kính A1 A2 . Đường tròn β và γ được xác định
tương tự. Chứng minh rằng cả 3 đường tròn này có chung 2 giao điểm.
Bài 15. (9 - 11) Độ dài các cạnh của tam giác ABC không lớn hơn 1.
Chứng minh rằng p(1 − 2Rr) ≤ 1, với p là nửa chu vi tam giác, r và R lần
lượt là bán kính đường tròn nội tiếp và ngoại tiếp 4ABC.
Bài 16. (9 - 11) Đường chéo của một tứ giác lồi chia hình tứ giác đó thành
4 hình tam giác. Hãy dựng lại hình tứ giác ban đầu, với điều kiện là ta chỉ
biết tâm ngoại tiếp của 2 tam giác chung cạnh trong 4 hình tam giác trên
và tâm nội tiếp của 2 tam giác đối nhau bất kỳ trong 4 hình tam giác trên.
Bài 17. (10 - 11) Gọi O là tâm ngoại tiếp 4ABC. Hình chiếu từ D và X
tới các cạnh của tam giác nằm trên đường thẳng l và L sao cho sao cho
l k X O. Chứng minh rằng các góc tạo bởi L và hai đường chéo tứ giác
ABC D bằng nhau.
Bài 18. (10 - 11) Cho ABC DE F là một lục giác nội tiếp. K, L, M , N lần
lượt là giao điểm của AB và C D, AD và BC, AF và DE, AE và DF . Chứng
minh rằng nếu 3 trong 4 điểm này thẳng hàng thì điểm thứ tư cũng nằm
trên đường thẳng tạo bởi 3 điểm đó.
Bài 19. (10 - 11) 4ABC có AL và AK lần lượt là phân giác trong và phân
giác ngoài. P là giao của 2 đường tiếp tuyến tại B và C của (ABC). Hạ
LQ ⊥ BC (Q ∈ AP). Chứng minh Q nằm trên đường trung bình của 4K LP.
Bài 20. (10 - 11) Cho một đường tròn và một elip "nằm gọn" trong đường
tròn đó và cùng có tâm C. Tìm quỹ tích tâm đường tròn ngoại tiếp ABC,
với AB là cung của đường tròn đó và tiếp xúc với elip.
Bài 21. (10 - 11) Tứ giác ABC D nội tiếp đường tròn ω tâm O. M1 và M2
lần lượt là trung điểm AB và C D. Để Ω là đường tròn ngoại tiếp 4M1 OM2 .
X 1 và X 2 là giao điểm của Ω và ω, Y1 và Y2 là giao điểm thứ hai của Ω với
đường tròn ngoại tiếp 4M1 C D và 4M2 AB. Chứng minh X 1 X 2 k Y1 Y2 .
Bài 22. (10 - 11) Các mặt của đa diện 20-mặt-tam-giác-đều được tô năm
màu sao cho hai mặt cùng màu không có chung cạnh và đỉnh. Chứng minh
rằng với một điểm bất kỳ nằm trong đa diện đó, khoảng cách từ điểm đó
tới mặt "xanh dương" bằng khoảng cách từ điểm đó tới mặt "đỏ".
Bài 23. (11) Cho tứ diện ABC D. Đường tròn nội tiếp 4ABC và đường tròn
nội tiếp 4ABD tiếp xúc AB tại T1 , T2 . Mặt phẳng πAB qua trung điểm T1 T2
và vuông góc với O1 O2 . Các mặt phẳng πAC , πBC , πAD , πBD , πC D được xác
định tương tự. Chứng minh 6 mặt phẳng này đều có điểm chung.

Hướng tới kỷ niệm 20 năm thành lập trường PTNK, ĐHQG TP HCM
Chuyên đề Toán học số 10 251

Bài 24. (11) Mặt cầu tâm O nội tiếp tứ diện ABC D tiếp xúc với các mặt tứ
Chuyên đề Toán học Phổ thông năng khiếu TP.HCM số 10

diện tại các điểm A1 , B1 , C1 và D1 .

a) Lấy Pa là điểm sao cho điểm đối xứng cúa nó qua các đường thẳng
OB, OC và OD đều thuộc mặt phẳng (BC D). Các điểm Pb , Pc và Pd
được xác định tương tự. Chứng minh rằng A1 Pa , B1 Pb , C1 Pc và D1 Pd
đồng quy tại điểm P nào đó.

b) Mặt cầu tâm I nội tiếp tứ diện A1 B1 C1 D1 và A2 là điểm giao của A1 I


với mặt phằng (B1 C1 D1 ). Các điểm B2 , C2 và D2 được xác định tương
tự. Chứng minh rằng P nằm trong tứ diện A2 B2 C2 D2 .

2.4. Năm 2016


Bài 1. (8) Hình thang cân ABC D với đáy AD và BC sao cho AB = BD. Gọi
M là trung điểm DC. Chứng minh ∠M BC = ∠BCA.

Bài 2. (8) Hãy dựng một tam giác sao cho nửa chu vi của nó bằng với
tổng độ dài hai đường trung tuyến nhỏ nhất của tam giác đó.

Bài 3. (8) Gọi AH1 , BH2 là các đường cao của 4ABC nhọn, D là hình chiếu
của H1 xuống AC, E là hình chiếu của D xuống AB, F là giao điểm của E D
và AH1 . Chứng minh H2 F ⊥ BC.

Bài 4. (8) Cho tứ giác ABC D có ∠B = ∠D = 90◦ , AC = BC + C D. Điểm


P thuộc tia BD sao cho BP = AD. Chứng minh C P song song với đường
phân giác ∠ABD.

Bài 5. (8) Cho tứ giác ABC D có AB = C D, M và K là trung điểm BC và


AD. Chứng minh rằng góc giữa hai đường thẳng M K và AC bằng một nửa
tổng của 2 góc ∠BAC và ∠DCA.

Bài 6. (8) 4ABC có M là trung điểm CA, M D và M E là 2 đường vuông


góc kẻ từ M tới lần lượt AB và BC. Chứng minh rằng khoảng cách giữa 2
AC
tâm của đường tròn ngoại tiếp tam giác ABE và BC D bằng .
4

Bài 7. (8 – 9) Coi mọi độ dài giữa 2 đỉnh bất kỳ của đa giác lồi n cạnh
(n > 3) là khác nhau.

a) Một đỉnh được coi là "không đặc biệt" nếu đỉnh “gần nhất theo độ
dài” của đỉnh đó là điểm kề. Tìm số nhỏ nhất các điểm "không đăc
biệt" theo n.

Hướng tới kỷ niệm 20 năm thành lập trường PTNK, ĐHQG TP HCM
252 Chuyên đề Toán học số 10

b) Một đỉnh được coi là "bất thường" nếu đỉnh "xa nhất theo độ dài"
Chuyên đề Toán học Phổ thông năng khiếu TP.HCM số 10

của đỉnh đó là điểm kề. Tìm số lớn nhất các điểm "bất thường" theo
n.

Bài 8. (8 - 9) Cho ngũ giác ABC DE nội tiếp có ∠B+∠E = ∠C +∠D. Chứng
π
minh ∠CAD < < ∠A.
3

Bài 9. (8 – 9) 4ABCvuông tại C có C H là đường cao. Các điểm O1 và O2


lần lượt là tâm nội tiếp tam giác AC H và BC H; P1 và P2 là điểm tiếp xúc
của 2 đưởng tròn nội tiếp tại AC và BC. Chứng minh giao điểm của O1 P1
và O2 P2 nằm trên AB.

Bài 10. (8 - 9) Điểm X di chuyển trên đoạn AB của 4ABC và điểm Y di


chuyển trên đường tròn ngoại tiếp của tam giác sao cho đường thẳng X Y
đi qua điểm chính giữa cung AB. Tìm quỹ tích các tâm đường tròn ngoại
tiếp của tam giác I X Y , trong đó I là tâm đường tròn nội tiếp tam giác
ABC.

Bài 11. (8 - 10) Dựng lại 4ABC với đỉnh B, trọng tâm và giao điểm của
đường đối trung kẻ từ B và đường tròn ngoại tiếp của tam giác.

Bài 12. (9 - 10) Cho BB1 là đường đối trung của tam giác nhọn không cân
ABC. Tia BB1 cắt đường tròn ngoại tiếp tam giác ABC lần thứ 2 tại điểm
M . AHA, BH B , C H C là các đường cao của tam giác ABC. Tia BH B cắt đường
tròn ngoại tiếp tam giác ABC lần thứ hai tại điểm T . Chứng minh rằng
HA, H C, T, L cùng nằm trên một đường tròn.

Bài 13. (9 - 10) Cho tam giác ABC và một đường thẳng l cắt BC, AC, AB
tại các điểm LA, L B , L C tương ứng. Đường vuông góc kẻ từ L B xuống BC
cắt AB và AC tại các điểm AB và AC tương ứng. Điểm Oa là tâm đường tròn
ngoại tiếp của tam giác AAB AC . Điểm Ob và Oc được xác định tương tự.
Chứng minh rằng Oa , Ob , Oc thẳng hàng.

Bài 14. Cho tam giác ABC. Xét đường tròn tiếp xúc với đường tròn ngoại
tiếp tam giác ABC tại A và tiếp xúc ngoài với đường tròn nội tiếp tam giác
ABC tại điểm A1 nào đó. Điểm B1 và C1 được định nghĩa tương tự.

a) (9 - 10) Chứng minh AA1 , BB1 , C C1 đồng quy.

b) (10 - 11) Cho A2 là điểm tiếp xúc của đường tròn nội tiếp tam giác
ABC với cạnh BC. Chứng minh rằng hai đường thẳng AA1 và AA2 đối
xứng với nhau qua đường phân giác của ∠A.

Hướng tới kỷ niệm 20 năm thành lập trường PTNK, ĐHQG TP HCM
Chuyên đề Toán học số 10 253

Bài 15. (9 - 11) Cho O, M , N là tâm đường tròn ngoại tiếp, trọng tâm và
Chuyên đề Toán học Phổ thông năng khiếu TP.HCM số 10

điểm Nagel của một tam giác. Chứng minh rằng tam giác M ON vuông khi
và chỉ khi một trong các góc của tam giác đó bằng 60◦ .

Bài 16. (9 - 11) Cho BB1 và C C1 là các đường cao của tam giác ABC. Tiếp
tuyến của đường tròn ngoại tiếp tam giác AB1 C1 cắt AB và AC tại M và N
tương ứng. Chứng minh rằng điểm chung của hai đường tròn (AM N ) và
(AB1 C1 ) khác A nằm trên đường thẳng Euler của tam giác ABC.

Bài 17. (9 - 11) Cho D là một điểm tùy ý trên cạnh BC của tam giác ABC.
Các đường tròn (ω1 ) và (ω2 ) đi qua A và D sao cho BA tiếp xúc với (ω1 )
và CA tiếp xúc với (ω2 ). Kẻ BX là tiếp tuyến thứ hai với (ω1 ) và C Y là tiếp
tuyến thứ hai với (ω2 ). Chứng minh rằng đường tròn ngoại tiếp tam giác
X DY tiếp xúc với BC.

Bài 18. (9 - 11) Cho ABC là tam giác vuông tại C và K, L là trung điểm của
những cung nhỏ AC, BC của đường tròn ngoại tiếp tam giác ABC. Đường
thẳng KC cắt AC tại điểm N . Tìm số đo ∠N I C trong đó I là tâm đường
tròn nội tiếp tam giác ABC.

Bài 19. (9 - 11) Cho lục giác đều ABC DE F . Điểm P và Q nằm trên các
tiếp tuyến với đường tròn ngoại tiếp lục giác kẻ từ A và D sao cho PQ tiếp
xúc với cung nhỏ E F của đường tròn. Tìm số đo góc tạo bởi 2 đường thẳng
P B và QC.

Bài 20. (10 - 11) Đường tròn nội tiếp (ω) của 4ABC tiếp xúc với BC, CA, AB
tại các điểm A0 , B0 , C0 tương ứng. Đường phân giác của 4B và 4C cắt
trung trực của đoạn thẳng AA0 lần lượt tại P và Q. Chứng minh rẳng giao
điểm của P C0 và QB0 nằm trên (ω).

Bài 21. (10 – 11) Diện tích hình chữ nhật P and Q bằng nhau, nhưng độ
dài đường chéo của P lớn hơn. Hình Q có thể được bao phủ bởi hai hình
bản sao của P. Chứng minh P có thể được bao phủ bởi hai hình bản sao
của Q.

Bài 22. (10 – 11) Gọi MA, MB , MC là trung điểm các cạnh BC, CA, AB của
tam giác không cân ABC. Các điểm HA, H B , H C nằm ở các cạnh đối và
phân biệt với MA, MB , MC sao cho MA H B = MA H C , MB HA = MB H C , MC HA =
MC H B . Chứng minh HA, H B , H C là các chân đường cao tam giác ABC.

Bài 23. (10 – 11) Cho mặt cầu nội tiếp tứ diện. Gọi a, b, c và d là các
đoạn tiếp tuyến từ các đỉnh tới mặt cầu. Có thể tạo được một tam giác từ
các đoạn nào đó trong đây không? (Không bắt buộc phải dùng hết. Có thể
hình thành tam giác chỉ từ 2 cạnh)

Hướng tới kỷ niệm 20 năm thành lập trường PTNK, ĐHQG TP HCM
254 Chuyên đề Toán học số 10

Bài 24. (11) Một mặt cầu nội tiếp hình lẳng trụ ABCA0 B 0 C 0 và tiếp xúc
với các mặt phẳng (BC C 0 B 0 ), (CAA0 C 0 ), (ABB 0 A0 ) lần lượt tại A0 , B0 , C0 . Biết
Chuyên đề Toán học Phổ thông năng khiếu TP.HCM số 10

rằng ∠A0 BB 0 = ∠B0 C C 0 = ∠C0 AA0 .

a) Tìm mọi giá trị có thể của các góc này.

b) Chứng minh các đường thẳng AA0 , BB0 , C C0 đồng quy.

c) Chứng minh rằng hình chiếu của tâm cầu xuống A0 B 0 , B 0 C 0 , C 0 A0 là


các đỉnh của 1 tam giác đều.

Hướng tới kỷ niệm 20 năm thành lập trường PTNK, ĐHQG TP HCM
Chuyên đề Toán học Phổ thông năng khiếu TP.HCM số 10

Giới thiệu đề thi


chọn đội tuyển PTNK qua các năm

Ban biên tập

Lời Ban biên tập: Trong Chuyên đề Toán học số 9, chúng tôi đã giới thiệu hai đề
thi chọn đội tuyển PTNK năm học 2008-2009 và 2009-2010. Để tiếp nối nội dung
này, trong Chuyên đề số 10, chúng tôi xin giới các đề 2010-2011, 2011-2012,
2012-2013, 2013-2014, 2014-2015 và 2015-2016. Ngoài ra, Ban biên tập cũng
giới thiệu lời giải cho hai đề gần nhất.

1. Đề thi chọn đội tuyển PTNK 2010 − 2011


Ngày thi thứ 1

5(x + y) 6(x + z)

+ =4


 x + y + 6x y x + z + 5xz



6(z + y) 4(x + y)
Bài 1. Giải hệ phương trình: + =5
 z + y + 4z y x + y + 6x y
4(x + z) 5( y + z)


+ =6


x + z + 5x y y + z + 4 yz

Bài 2. Tìm tất cả các hàm f : R → R thỏa mãn

f (|x| + y + f ( y + f ( y))) = 3 y + | f (x)| với mọi x, y ∈ R

Bài 3. Cho p là số nguyên tố lẻ và n = 2p + r với r ∈ {0, 1, 2, . . . , p − 1}. Đặt


X = {1, 2, . . . , n}. Ánh xạ f : X → X được gọi là có tính chất P nếu f không
phải là ánh xạ đồng nhất và f ( f . . . ( f ( f (k))) . . .) = k (ánh xạ hợp p lần)
với mọi k ∈ X . Đặt A f = {k ∈ X | f (k) = k}.

1. Chứng minh rằng nếu f có tính chất P thì |A f | ≡ r (mod p).

255
256 Chuyên đề Toán học số 10

2. Gọi d là số các ánh xạ f có tính chất P. Chứng minh rằng d không là


Chuyên đề Toán học Phổ thông năng khiếu TP.HCM số 10

ước số của n!.


Bài 4. Cho tam giác ABC nội tiếp đường tròn (O) có A cố định và B, C thay
đổi trên (O) sao cho BC luôn song song với một đường thẳng cố định. Các
tiếp tuyến của (O) tại B và C cắt nhau tại K. Gọi M là trung điểm BC, N là
giao điểm của AM với (O). Chứng minh rằng đường thẳng K N luôn đi qua
một điểm cố định.

Ngày thi thứ 2

Bài 5. Cho a, b, c là độ dài các cạnh của một tam giác. Chứng minh rằng
(2a + 2b − c)(2b + 2c − a)(2a + 2c − b) > 25abc.
p 2u2n + 5un + 5
Bài 6. Cho dãy số un thỏa mãn u1 = 2, un+1 = , n ≥ 1. Tính
2un + 4
u2n − 3un + 5
giới hạn sau lim 2 .
3n + 4n − 1
Bài 7. Xét số tự nhiên n > 1. Bắt đầu từ bộ số 1, 2, . . . , 2n − 1, 2n ta thực
hiện phép biến đổi sau: chọn hai số a, b bất kì sao a − b > 1, xóa hai số này
và thay bởi hai số a − 1 và b + 1. Với bộ số mới này ta lại tiếp tục thực hiện
phép biến đổi tương tự.
1. Chứng minh rằng ta sẽ đạt đến trạng thái dừng, tức là không thể thực
hiện phép biến đổi như vậy được nữa.

2. Gọi k là số lần phép biến đổi cần thực hiện để đạt đến trạng thái dừng.
Tìm giá trị nhỏ nhất và lớn nhất của k.
Bài 8. Cho đường tròn (γ1 ) đường kính AB và đường tròn (γ2 ) tâm A cắt (γ1 )
tại C và D. Điểm M thay đổi trên cung C D (nằm bên trong (γ1 )) của (γ2 ),
NC + ND
BM cắt (γ2 ) tại N (N khác M và B). Tìm giá trị nhỏ nhất của .
NM

2. Đề thi chọn đội tuyển PTNK 2011 − 2012


Ngày thi thứ 1

Bài 1. Cho các số a, b, c dương thỏa mãn ab + bc + ca = 1. Chứng minh bất


đẳng thức

1 1 1
+ + ≥ 1.
3 + 2(a2 − bc) 3 + 2(b2 − ca) 3 + 2(c 2 − ab)

Hướng tới kỷ niệm 20 năm thành lập trường PTNK, ĐHQG TP HCM
Chuyên đề Toán học số 10 257

Bài 2. Có bao nhiêu tập {x; y; z; t} ⊂ N thỏa 12 ≤ x < y < z < t và


x + y + z + t = 2011?
Chuyên đề Toán học Phổ thông năng khiếu TP.HCM số 10

Bài 3. Cho tam giác ABC nội tiếp đường tròn (O) bán kính R, (C1 ) là đường
tròn thay đổi và luôn qua B, C và lần lượt cắt các cạnh AB, AC tại M , N (M 6=
B, N 6= C).

1. Chứng minh (AM N ) luôn tiếp xúc với một đường cố định.

2. Cho B, C cố định và BC = 2R, còn A thay đổi trên (O). Đường thẳng
qua A vuông góc BC cắt (O) lần nữa tại D, cắt (C
p1 ) tại E, F . Chứng
EF 5
minh rằng nếu A và (C1 ) thay đổi sao cho = thì (AM N ) luôn
AD 2
tiếp xúc với một đường cố định.

Bài 4. Cho p là số nguyên tố lẻ và đa thức Q(x) ≡ (p − 1)x p − x − 1. Dãy


p−1
số (an ) thỏa: a0 = 2 , an = an−1 + Q(an−1 ) ∀n ∈ N∗. Chứng minh rằng với
mọi n nguyên dương ta có:

1. gcd(an , p) = 1.

2. Q(an ) ≡ 0 mod p n .

Ngày thi thứ 2


1 2
Bài 5. Cho dãy (un ) thỏa u1 = và un+1 = u2n + un , ∀n ∈ N∗. Tìm giới hạn
6 3
5u2n+1 − 2u2n un+1 + 5un un+1
sau lim .
3u2n + un un+1 (4 + u2n )

Bài 6. Cho hàm số f : N × N → N thỏa f (0, 0) = 0 và


(     .
f 2a , 2b nếu a + b .. 2
f (a, b) = . .
1 + f 2a , 2b nếu a + b − 1 .. 2
   

1. Có bao nhiêu số tự nhiên m ≤ 2011 sao cho f (2011, m) = 5?

2. Cho số lẻ p, n ∈ N (1 < p < 2n ) và A là tập hợp gồm p số tự nhiên


p2 − 1
không vượt quá 2n −1. Chứng minh rằng {a,b}⊂A f (a, b) ≤ n·
P
.
4
Bài 7. Cho tam giác ABC nội tiếp đường tròn (O) với B, C cố định còn A
thay đổi trên (O). Trung trực d của BC cắt AB, AC tại M , N . Gọi P, Q là các
điểm đối xứng của M , N qua O và K là giao điểm của BQ và C P.

Hướng tới kỷ niệm 20 năm thành lập trường PTNK, ĐHQG TP HCM
258 Chuyên đề Toán học số 10

1. Chứng minh K luôn thuộc một đường tròn cố định.


Chuyên đề Toán học Phổ thông năng khiếu TP.HCM số 10

2. Kết luận trên còn đúng không nếu thay d bằng đường thẳng Euler của
∆ABC?
Bài 8. Với mọi số nguyên dương n, đặt Sn = x n + y n + z n . Ta biết rằng
Sn ≡ Pn (s, t, p) với s = x + y + z, t = x y + yz + z x, p = x yz. Hãy tính tổng
các hệ số của các đơn thức chứa p trong P2011 (s, t, p).

3. Đề thi chọn đội tuyển PTNK 2012 − 2013


Ngày thi thứ 1
Bài 1. Giải hệ phương trình:


 x+y = 3;

xz + y t = 5;


 xz 2 + y t 2 = 41;
 xz 3 + y t 3

= 121.

Bài 2. Cho dãy (un ) giảm và có giới hạn bằng 0. Xét hai dãy số sau
vn = u1 + u2 + · · · + un − nun+1 ,
z n = u1 + u2 + · · · + u n .
Chứng minh nếu (vn ) bị chặn thì (zn ) hội tụ.
Bài 3. Cho tập X = {1; 2; 3; . . . ; 4n}. Hai tập con A và B của X được gọi là
không giống nhau nếu |A∆B| ≥ 2n + 1. Trong đó A∆B = (A \ B) ∪ (B \ A).
Cho {A1 ; A2 ; . . . ; Am } là tập con của X gồm m phần tử đôi một không giống
nhau.
1. Chứng minh m ≤ 2n.
4(n+1)
2. Chứng minh m ≤ 3 .

Bài 4. Cho 4ABC với M , N thuộc cạnh BC sao cho ∠BAM = ∠CAN = α
(M nằm giữa B, N ). Gọi I là trung điểm BC. Kẻ BH ⊥ AM , C K ⊥ AN lần
lượt tại H, K.
1. Chứng minh tâm đường tròn (I H K) luôn thuộc một đường thẳng cố
định.

2. Tính α theo ∠ABC và ∠AC B sao cho (I K H) tiếp xúc với đường tròn
đường kính AB hoặc đường tròn đường kính AC.

Hướng tới kỷ niệm 20 năm thành lập trường PTNK, ĐHQG TP HCM
Chuyên đề Toán học số 10 259

Ngày thi thứ 2


Chuyên đề Toán học Phổ thông năng khiếu TP.HCM số 10

4 1
Bài 5. 1. Chứng minh rằng với mọi x > 0 thì 2(x 3 + 4 + 1) ≥ 3(x + 1x ).
x3

1
2. Tìm số thực dương a nhỏ nhất sao cho 2(x a + xa + 1) ≥ 3(x + 1x ) với
mọi x > 0.
2 2
+n+1) +n+1)
Bài 6. Tìm n tự nhiên sao cho An = 1 + 320(n + 914(n là số nguyên
tố.

Bài 7. Tìm hàm số f : N ∗ → N ∗ thỏa mãn đồng thời hai điều kiện sau:

i) f (m f (n)) = n6 · f (mn).

ii) Với mọi m, n là các số nguyên tố cùng nhau thì f (m), f (n) cũng nguyên
tố cùng nhau.

Bài 8. Cho tam giác ABC có AB = AC. Gọi H là chân đường vuông góc hạ
từ A xuống BC. (C) là một đường tròn đi qua H có tâm I di động trên đoạn
AH. (C) cắt AB tại M , N và cắt AC tại P, Q sao cho M nằm giữa A và N , P
nằm giữa A và Q. BP và BQ cắt (C) tại E, F. Chứng minh rằng giao điểm D
của N E và M F là một điểm cố định.

4. Đề thi chọn đội tuyển PTNK 2013 − 2014


Ngày thi thứ 1

Bài 1. Tìm tất cả các hàm số f : R → R thoả mãn

f (x 3 + y + f ( y)) = 2 y + x 2 f (x), ∀x, y ∈ R.

Bài 2. Cho dãy {un } thoả mãn u1 = 2013, un+1 = u3n − 4u2n + 5un ∀n ∈ N∗ .
Tìm tất cả các số nguyên tố p là ước của (u2014 + 2009) và p ≡ 3 (mod 4).

Bài 3. Trong một hội nghị khoa học có 5000 đại biểu tham dự, mỗi một đại
biểu biết ít nhất một thứ tiếng. Một uỷ ban gồm một số đại biểu được gọi là
"uỷ ban làm việc" nếu tất cả thành viên trong uỷ ban đều biết chung một thứ
tiếng; gọi là "uỷ ban thách thức" nếu không có hai thành viên nào của uỷ
ban biết chung một thứ tiếng (uỷ ban có thể gồm 1 thành viên; uỷ ban này
gọi là làm việc hoặc thách thức đều được). Chứng minh rằng có thể chia các
đại biểu thành 100 uỷ ban rời nhau (mỗi đại biểu thuộc một uỷ ban) sao
cho các uỷ ban này hoặc là uỷ ban làm việc hoặc là uỷ ban thách thức.

Hướng tới kỷ niệm 20 năm thành lập trường PTNK, ĐHQG TP HCM
260 Chuyên đề Toán học số 10

Bài 4. Tam giác ABC có B, C cố định còn A di động sao cho AB = AC và


∠BAC > 60◦ . Đường thẳng đối xứng với BC qua AB cắt AC tại P. Trên đoạn
Chuyên đề Toán học Phổ thông năng khiếu TP.HCM số 10

P C lấy M sao cho P M = P B. Gọi N là giao điểm của AB với phân giác ngoài
góc BCA. Chứng minh M N luôn đi qua một điểm cố định.

Ngày thi thứ 2


P2014
Bài 5. Cho 2014 số thực x 1 , x 2 , . . . , x 2014 thỏa mãn điều kiện i=1 x i = 0
P2014
và i=1 x i2 = 2014. Tìm giá trị lớn nhất của biểu thức P = x 1 x 2 · · · x 2014 .
u
Bài 6. Cho dãy số un xác định bởi u1 = 1, un+1 = p 2 n p với mọi n ∈ N∗ .
un +1+ 2
un+1
Tìm giới hạn lim .
un

Bài 7. Cho n nguyên dương và A là tập con khác rỗng của X = {1, 2, . . . , n} .

1. Tính giá trị của tổng S (A) = E⊂X . (−1)| E A| ,trong đó E lấy trên tất
P T

cả các tập con của tập X (kể cả tập rỗng).

2. Cho m ∈ N∗ ,xét m tập con khác rỗng của X là A1 , A2 , . . . , Am và m số


a1 + a2 + · · · + am < 0.
nguyên khác không là a1 , a2 , . . . , am sao cho P
m
Chứng minh tồn tại tập con E của X sao cho i=1 (−1)|E∩A| ai > 0 (Kí
hiệu |A| chỉ số phần tử của tập A, số phần tử của tập rỗng là 0).

Bài 8. Cho tam giác ABC nhọn có H là trực tâm và P là điểm di động bên
trong tam giác ABC sao cho ∠BP C = ∠BH C. Đường thẳng qua B và vuông
góc với AB cắtP C tại M .Đường thẳng qua C và vuông góc với AC cắt P B tại
N . Chứng minh rằng trung điểm I của M N luôn thuộc một đường cố định.

5. Đề thi chọn đội tuyển PTNK 2014 − 2015


Ngày thi thứ 1

Bài 1. Cho a, b, c > 0 thỏa mãn điều kiện (a + 1)(b + 1)(c + 1) = 1 + 4abc.
Chứng mình rằng ta có bất đẳng thức a + b + c ≤ 1 + abc.

Bài 2. Cho tập hợp A = n3 − 4n + 15|n ∈ N . Tìm tất cả các phần tử a ∈ A
thỏa mãn đồng thời các điều kiện sau:

i. Số a là số chẵn.

ii. Nếu x, y là các ước lớn hơn 1 của a


2 thì gcd(x, y) > 1.

Hướng tới kỷ niệm 20 năm thành lập trường PTNK, ĐHQG TP HCM
Chuyên đề Toán học số 10 261

f (n)
 ‹
∗ ∗
Bài 3. Tìm tất cả f : N → N thỏa mãn f = n2 với mọi n ∈ N.
n
Chuyên đề Toán học Phổ thông năng khiếu TP.HCM số 10

Bài 4. Cho tam giác ABC nội tiếp (O), có B, C cố định và A thay đổi trên
(O). Ký hiệu (I) là đường tròn nội tiếp ABC. Gọi (O1 ) là đường tròn qua A, B
và tiếp xúc với đường tròn (I) tại E. Gọi (O2 ) là đường tròn qua A, C và tiếp
xúc với đường tròn (I) tại điểm F. Đường phân giác trong của góc AEB cắt
(O1 ) tại M và đường phân giác trong của góc AF C cắt (O2 ) tại N .
1. Chứng minh rằng tứ giác E F M N nội tiếp.
2. Gọi J là giao điểm của E M , F N . Chứng minh rằng đường thẳng I J
luôn qua điểm cố định.

Ngày thi thứ 2


Æ
Bài 5. Cho dãy số (x n ) xác định bởi x 0 = 1, x 1 = 2014 và x n+1 = 3 2
x n x n−1
với n ≥ 1.
1. Chứng minh rằng dãy số (x n ) có giới hạn và tìm giới hạn đó.
2. Với mỗi n ≥ 2, tìm số nguyên dương k nhỏ nhất sao cho a = (x n )k là
một số nguyên. Chứng minh rằng khi đó, a không thể viết được dưới
dạng tổng các lũy thừa bậc ba của hai số tự nhiên.
Bài 6. Cho X là tập hợp có 19 phần tử.
1. Chứng minh rằng tồn tại tập hợp F gồm ít nhất 2600 tập con 7 phần
tử của X sao cho với mọi A khác B thuộc F thì |A ∩ B| ≤ 5.
2. Xét một họ Ω gồm k tập con có 7 phần tử của X . Một tập A ⊂ X được
gọi là một cận trên của họ Ω nếu như |A| = 8 và tồn tại một tập con
F của họ Ω sao cho F ⊂ A. Gọi d là số tập con là cận trên của họ Ω.
Chứng minh rằng d ≥ 23 k.
Bài 7. Cho tam giác ABC không cân. Gọi I là trung điểm BC. Đường tròn
(I) đi qua A cắt AB, AC lần lượt tại M , N . Giả sử M I, N I lần lượt cắt (I) tại
P, Q. Gọi K là giao điểm của PQ với tiếp tuyến tại A của (I). Chứng minh
rằng K thuộc đường thẳng BC.
Bài 8. Tìm n nguyên dương nhỏ nhất thỏa mãn các điều kiện sau:
i. n không chia hết cho 3.
ii. Bảng vuông n × n không thể được phủ kín bằng quân tetramino 1 × 4
và các quân trimino 1 × 3. Trong phủ các quân tetramino và trimino
được phép quay dọc nhưng không được phép chườm lên nhau hoặc
nằm ra ngoài bảng vuông.

Hướng tới kỷ niệm 20 năm thành lập trường PTNK, ĐHQG TP HCM
262 Chuyên đề Toán học số 10

6. Đề thi chọn đội tuyển PTNK 2015 − 2016


Chuyên đề Toán học Phổ thông năng khiếu TP.HCM số 10

Ngày thi thứ 1


Bài 1. Cho tập hợp

A = {n ∈ N|1 ≤ n ≤ 2015, (n, 2016) = 1} .

Hỏi có bao nhiêu số nguyên a ∈ A sao cho tồn tại số nguyên b mà a + 2016b
là số chính phương?

Bài 2. Cho a, b, c, d là các số thực thỏa mãn điều kiện

a2 ≤ 1, a2 + b2 ≤ 5, a2 + b2 + c 2 ≤ 14, a2 + b2 + c 2 + d 2 ≤ 30.

1. Chứng minh rằng a + b + c + d ≤ 10.

2. Chứng minh rằng ad + bc ≤ 10.

Bài 3. Tìm tất cả các hàm số f : R → R thỏa mãn điều kiện

f (x − 2 f ( y)) = 5 f (x) − 4x − 2 f ( y)

với mọi x, y ∈ R.

Bài 4. Cho đường tròn k và các điểm B, C thuộc đường tròn, không phải là
đường kính; I là trung điểm BC. Điểm A di động trên cung lớn BC của k.
Gọi i1 là đường tròn qua I và tiếp xúc với AB tại B; i2 là đường tròn qua I
và tiếp xúc với AC tại C. Các đường tròn i1 , i2 cắt nhau tại D (khác I).

1. Chứng minh rằng đường tròn ngoại tiếp tam giác AI D luôn đi qua một
điểm cố định.

2. Gọi K là trung điểm AD, E là tâm đường tròn qua K và tiếp xúc với
AB tại A, F là tâm đường tròn qua K và tiếp xúc với AC tại A. Chứng
minh rằng góc EAF có số đo không đổi.

Ngày thi thứ 2


1
Bài 5. Dãy số (x n ) được xác định bởi công thức x n = với mọi n ≥ 1.
n cos 1n
Tính giới hạn sau
x 1 + x 3 + x 5 + · · · + x 2n−1
lim .
n→+∞ x 2 + x 4 + x 6 + · · · + x 2n

Hướng tới kỷ niệm 20 năm thành lập trường PTNK, ĐHQG TP HCM
Chuyên đề Toán học số 10 263

Bài 6. Tìm các giá trị của b sao cho tồn tại a để hệ phương trình sau có
nghiệm (x, y)
Chuyên đề Toán học Phổ thông năng khiếu TP.HCM số 10

(x − 1)2 + ( y + 1)2 = b


y = x 2 + (2a + 1)x + a2

Bài 7. Cho n là số nguyên dương, n ≥ 2 và X = {1, 2, 3, . . . , n}. Gọi A1 , A2 , . . . , Am


và B1 , B2 , . . . , Bm là hai dãy các tập con khác rỗng của X thỏa mãn điều kiện:
Với mỗi i, j ∈ {1, 2, 3, . . . , n}, Ai ∩ B j = ∅ nếu và chỉ nếu i = j.

1. Chứng minh rằng với mỗi hoán vị (x 1 , x 2 , . . . , x n ) của X , có không quá


một cặp tập hợp (Ai , Bi ) với i = 1, 2, 3, . . . , n sao cho nếu x k ∈ Ai và
x l ∈ Bi thì k < l.

2. Gọi ai , bi lần lượt là số phần tử của tập hợp Ai , Bi với i = 1, 2, 3, . . . , m.


Chứng minh rằng
m
X 1
ai ≤ 1.
i=1
C a i +b i

Bài 8. Cho tam giác ABC nhọn nội tiếp đường tròn tâm O. Đường tròn tâm
I đi qua B, C lần lượt cắt các tia BA, CA tại E, F.
1. Giả sử các tia BF, C E cắt nhau tại D và T là tâm đường tròn (AE F ).
Chứng minh rằng OT k I D.

2. Trên BF, C E lần lượt lấy các điểm G, H sao cho AG⊥C E, AH⊥BF. Các
đường tròn (ABF ), (AC E) cắt BC tại M , N (khác B, C) và cắt E F tại
P, Q (khác E, F ). Gọi K là giao điểm của M P, NQ. Chứng minh rằng
DK vuông góc với GH.

7. Lời giải đề chọn đội tuyển năm 2014 − 2015


Ngày thi thứ 1

Bài 1. Cho a, b, c > 0 thỏa mãn điều kiện (a + 1)(b + 1)(c + 1) = 1 + 4abc.
Chứng mình rằng ta có bất đẳng thức a + b + c ≤ 1 + abc.
Lời giải.
Điều kiện đã cho viết thành ab + bc + ca + a + b + c = 3abc. Chia hai vế
cho abc rồi đặt a = 1x , b = 1y , c = 1z , ta có x y + yz + z x + x + y + z = 3.
Bất đẳng thức đã cho có thể viết thành

x y + yz + z x − x yz ≤ 1 hay x + y + z + x yz ≥ 2.

Hướng tới kỷ niệm 20 năm thành lập trường PTNK, ĐHQG TP HCM
264 Chuyên đề Toán học số 10

Theo bất đẳng thức Schur thì


Chuyên đề Toán học Phổ thông năng khiếu TP.HCM số 10

(x + y + z)3 + 9x yz ≥ 4(x y + yz + z x)(x + y + z).

Đặt m = x + y + z, n = x y + yz + z x thì m + n = 3 và

4mn − m3
x yz ≥ .
9

Ta sẽ chứng minh rằng

4mn − m3
m+ ≥ 2 ⇔ m3 + 4m2 − 21m + 18 ≤ 0
9

hay (m − 2)(m2 + 6m − 9) ≤ 0. Chú ý rằng m2 ≥ 3n nên

m2 ≥ 3(3 − m) ⇔ m3 + 3m ≥ 9.

Do đó m2 + 6m − 9 ≥ 0. Ta xét các trường hợp

1. Nếu m > 2 thì x + y + z > 2 nên hiển nhiên bất đẳng thức cần chứng
minh là đúng.

2. Nếu m ≤ 2 thì m − 2 ≤ 0 nên ta cũng có (m − 2)(m2 + 6m − 9) ≤ 0.

Vậy trong mọi trường hợp, ta luôn có đpcm.



Bài 2. Cho tập hợp A = n3 − 4n + 15|n ∈ N . Tìm tất cả các phần tử a ∈ A
thỏa mãn đồng thời các điều kiện sau:

i. Số a là số chẵn.

ii. Nếu x, y là các ước lớn hơn 1 của a


2 thì gcd(x, y) > 1.

Lời giải.
Ta thấy rằng a = n3 −4n+15 chẵn nên n3 +15 chẵn hay n lẻ. Đặt n = 2k+1
với k ∈ N. Ta có

a = n3 − 4n + 15 = (n + 3)(n3 − 3n + 15)
= (2k + 4)(4k2 − 2k + 3)

nên 2a = (k + 2)(4k2 − 2k + 3). Điều kiện ii) cho thấy rằng 2a phải là lũy
thừa của một số nguyên tố, vì nếu nó có hai ước nguyên tố trở lên, đặt
là p, q thì chọn x = p, y = q, ta có x, y > 1 nhưng gcd(x, y) = 1, không
thỏa.

Hướng tới kỷ niệm 20 năm thành lập trường PTNK, ĐHQG TP HCM
Chuyên đề Toán học số 10 265

Vì (4k2 − 2k + 3) − (k + 2) = 4k2 − 3k + 1 > 0 với mọi k ∈ N. Do đó, ta phải


có k + 2|4k2 − 2k + 3. Suy ra
Chuyên đề Toán học Phổ thông năng khiếu TP.HCM số 10

4k2 − 2k + 3 23
= 4k − 10 + ∈ Z.
k+2 k+2
Do đó k + 2 ∈ {1, 23} vì k + 2 > 0. Ta xét các trường hợp

1. Nếu k + 2 = 1 thì k = −1 hay n = 2k + 1 = −1 < 0, không thỏa.

2. Nếu k + 2 = 23 thì k = 21 hay n = 43, tính được a


2 = 3 · 52 · 232 , cũng
không thỏa.

Vậy không tồn tại số a nào thỏa mãn.


f (n)
 ‹
∗ ∗
Bài 3. Tìm tất cả f : N → N thỏa mãn f = n2 với mọi n ∈ N.
n
Lời giải.
Với n ∈ N∗, ta thấy rằng nếu n = 1 thì f ( f (1)) = 1.
Nếu n > 1 thì gọi p là một ước nguyên tố bất kỳ của n.
f (n)
Vì n ∈ N∗ nên n| f (n). Đặt a = vp (n), b = vp ( f (n)) thì trước hết, ta có
a ≤ b.

f (n) f (n)
€ Š
Từ f n = n2 , ta suy ra rằng n n2 hay f (n)|n3 , tức là b ≤ 3a.
f (n)
Trong biểu thức đã cho, thay n → n thì
 € Š
f (n)
f n 
f (n) 2
‹  3 ‹ 
n f (n) 2
‹
f   = ⇔f =
f (n) n f (n) n
n

Do đó, ta phải có
‹2 3
f (n)

n ⇔ f 3 (n) n5 nên 3b ≤ 5a.

n f (n)

n5
Sau đó lại tiếp tục thay n trong biểu thức đã cho bởi f 3 (n) và cứ như thế,
ta xây dựng được hai dãy hệ số của a, b như sau

u0 = v0 = 1, u1 = 3, v1 = 1 và

uk+1 = 2uk−1 + vk , vk+1 = 2vk−1 + uk với k ≥ 1.

Hướng tới kỷ niệm 20 năm thành lập trường PTNK, ĐHQG TP HCM
266 Chuyên đề Toán học số 10

Khi đó
v2k b u2k+1
Chuyên đề Toán học Phổ thông năng khiếu TP.HCM số 10

≤ ≤ .
u2k a v2k+1
Biến đổi công thức của hai dãy, ta có un+2 = 5un − 4un−2 , vn+2 = 5vn − 4vn−2
và cả hai dãy đều có phương trình đặc trưng là t 2 − 5t + 4 = 0. Ngoài ra,
dãy chẵn và dãy lẻ trong mỗi dãy đều độc lập với nhau.
Ta có u0 = 1, u2 = 3, v0 = 1, v2 = 5 nên

13 + 2 · 16k 11 + 4 · 16k
u2k = , v2k = , k ≥ 1.
15 15
u2k+1
Từ đó, dễ dàng tính được lim v2k+1 = 2.
u2k
Một cách tương tự, ta tính được lim v2k = 21 . Do đó, số b
a bị kẹp ở giữa và
là số nguyên nên chỉ có thể là b
a = 2 ⇔ b = 2a.
Rõ ràng tập hợp ước nguyên tố của n và f (n) là giống nhau. Hơn nữa, với
một ước nguyên tố cụ thể thì số mũ trong f (n) gấp đôi số mũ trong n. Suy
ra f (n) = n2 , ∀n > 1.
Tiếp theo, giả sử f (1) = n > 1 thì ta có f ( f (1)) = 1 nên f (n) = 1, mâu
thuẫn. Vì thế nên chỉ có thể f (1) = 1.
Vậy tất cả các hàm thỏa mãn là f (n) = n2 , ∀n ∈ N∗.

Bài 4. Cho tam giác ABC nội tiếp (O), có B, C cố định và A thay đổi trên
(O). Ký hiệu (I) là đường tròn nội tiếp ABC. Gọi (O1 ) là đường tròn qua A, B
và tiếp xúc với đường tròn (I) tại E. Gọi (O2 ) là đường tròn qua A, C và tiếp
xúc với đường tròn (I) tại điểm F. Đường phân giác trong của góc AEB cắt
(O1 ) tại M và đường phân giác trong của góc AF C cắt (O2 ) tại N .

1. Chứng minh rằng tứ giác E F M N nội tiếp.

2. Gọi J là giao điểm của E M , F N . Chứng minh rằng đường thẳng I J


luôn qua điểm cố định.

Lời giải.
1) Trước hết, ta thấy rằng O1 , I, E thẳng hàng và O2 , I, F thẳng hàng.
Vì M là trung điểm cung AB của (O1 ) nên O1 M là trung trực của AB, suy
ra O ∈ O1 M . Tương tự, ta cũng có O ∈ O1 N .
Gọi P, Q lần lượt là tiếp điểm của (I) với AB, AC.

Hướng tới kỷ niệm 20 năm thành lập trường PTNK, ĐHQG TP HCM
Chuyên đề Toán học số 10 267
Chuyên đề Toán học Phổ thông năng khiếu TP.HCM số 10

Vì I P k O1 M (cùng vuông góc với AB) nên ∠M O1 E = ∠P I E. Hơn nữa, các


tam giác O1 M E, I P E đều cân với đỉnh là O1 , I nên suy ra chúng đồng dạng,
tức là ∠I E P = ∠O1 E M hay E, P, M thẳng hàng. Tương tự thì F, Q, N cũng
thẳng hàng.
Vì ta đã có E, F, P, Q cùng thuộc đường tròn (I) nên để có E, F, M , N cùng
thuộc một đường tròn thì ∠E M N = ∠E F N = ∠E PQ hay M N k PQ.
Mặt khác, AI⊥PQ nên ta cần có AI⊥M N . Thật vậy, sử dụng phương tích
với đường tròn (I) ta có

M A2 − N A2 = M P · M E − NQ · N F = M I 2 − N I 2

nên theo định lý bốn điểm thì AI⊥M N , từ đó ta có đpcm.


2) Vì PQ k M N , OM k I P nên dễ dàng có ∠I PQ = ∠OM N . Tương tự
∠I PQ = ∠ON M .
PQ
Do đó, hai tam giác I PQ, OM N đồng dạng với nhau, tức là IP
OM = MN .

Ngoài ra, JJMP = OMIP


, kết hợp với ∠J P I = ∠J M O, ta có hai tam giác
J P I, J M O đồng dạng, dẫn đến ∠P J I = ∠M JO.
Từ đây suy ra I, J, O thẳng hàng hay I J luôn đi qua điểm O cố định.

Hướng tới kỷ niệm 20 năm thành lập trường PTNK, ĐHQG TP HCM
268 Chuyên đề Toán học số 10

Ngày thi thứ 2


Chuyên đề Toán học Phổ thông năng khiếu TP.HCM số 10

Æ
Bài 5. Cho dãy số (x n ) xác định bởi x 0 = 1, x 1 = 2014 và x n+1 = 3 2
x n x n−1
với n ≥ 1.

1. Chứng minh rằng dãy số (x n ) có giới hạn và tìm giới hạn đó.

2. Với mỗi n ≥ 2, tìm số nguyên dương k nhỏ nhất sao cho a = (x n )k là


một số nguyên. Chứng minh rằng khi đó, a không thể viết được dưới
dạng tổng các lũy thừa bậc ba của hai số tự nhiên.

Lời giải.
1) Đặt un = log2014 (x n ) thì ta thu được dãy (un ) như sau

u0 = 0, u1 = 1
(
1 2
un+1 = un + un−1
3 3
Từ đó tìm được
3 3 −2 n
 ‹
un = − ·
5 5 3
3
Suy ra lim un = nên ta có được
n→+∞ 5

lim x n = lim (2014un ) = 20143/5


n→+∞ n→+∞

3k(3n − (−2)n )
2) Ta thấy rằng để có (x n )k là một số nguyên thì ∈ Z
5 · 3n
nguyên. Ta xét các trường hợp
3 + 2n n
 n ‹
..
1. Nếu n lẻ thì 3 − (−2) = 3 + 2 . 5. Vì gcd
n n n n
, 3 = 1 nên ta
5
được 3n | 3k nên k nhỏ nhất thỏa mãn điều này là k = 3n−1 .

2. Nếu n chẵn thì 3n − 2n ≡ (−2)n − 2n = 0 (mod 5) và tương tự, ta


cũng tìm được k = 3n−1 .

Do đó số k nhỏ nhất cần tìm là k = 3n−1 . Tiếp theo, ta sẽ chứng minh rằng
phương trình sau không có nghiệm tự nhiên

a3 + b3 = 2014n ⇔ (a + b)(a2 − ab + b2 ) = 2014n

Gọi n0 số nguyên dương nhỏ nhất sao cho tồn tại a, b ∈ Z+ để a3 + b3 =


2014n0 . Dễ thấy n0 = 1 không thỏa nên ta chỉ xét n0 ≥ 2. Ta xét các trường
hợp

Hướng tới kỷ niệm 20 năm thành lập trường PTNK, ĐHQG TP HCM
Chuyên đề Toán học số 10 269

1. Nếu gcd(a + b, a2 − ab + b2 ) = 1 thì dễ thấy (a − b)2 ≥ 1. Khi đó


Chuyên đề Toán học Phổ thông năng khiếu TP.HCM số 10

p
a2 − ab + b2 ≥ a + b > a2 − ab + b2 .

Vì 2014 = 2 · 19 · 53 nên chỉ có thể xảy ra

a + b = 19n0 , a2 − ab + b2 = 106n0 .

Ngoài ra (a + b)2 ≤ 4(a2 − ab + b2 ) nên ta phải có 361n0 ≤ 4 · 106n0 .


Đánh giá này sai khi n0 ≥ 2 nên trường hợp này không thỏa.

2. Nếu gcd(a + b, a2 − ab + b2 ) > 1 thì chẳng hạn

a + b = 2 x u, a2 − ab + b2 = 2 y v với gcd(u, 2) = gcd(v, 2) = 1.

Các trường hợp còn lại chứng minh tương tự. Ngoài ra

uv = 1007n0 , x + y = n0 .

Chú ý rằng (a + b)2 − (a2 − ab + b2 ) = 3ab nên 3ab cũng chẵn, tức
là cả hai số a, b đều chẵn (vì nếu không thì a3 + b3 lẻ).

Từ đây dễ dàng chứng minh được 3v2 (a) = 3v2 (b) = n0 , ta đưa về
3 3
x 0 + y 0 = 1007n0 . Cứ như thế, ta được 2014|a, 2014|b nên phương
trình sau cũng có nghiệm nguyên dương
 a 3  b ‹3
+ = 2014n0 −3 .
2014 2014

Điều này mâu thuẫn với các chọn n0 nên phương trình trên vô
nghiệm. Các trường hợp còn lại tương tự.

Ta có đpcm.

Bài 6. Cho X là tập hợp có 19 phần tử.

1. Chứng minh rằng tồn tại tập hợp F gồm ít nhất 2600 tập con 7 phần
tử của X sao cho với mọi A khác B thuộc F thì |A ∩ B| ≤ 5.

2. Xét một họ Ω gồm k tập con có 7 phần tử của X . Một tập A ⊂ X được
gọi là một cận trên của họ Ω nếu như |A| = 8 và tồn tại một tập con
F của họ Ω sao cho F ⊂ A. Gọi d là số tập con là cận trên của họ Ω.
Chứng minh rằng d ≥ 23 k.

Hướng tới kỷ niệm 20 năm thành lập trường PTNK, ĐHQG TP HCM
270 Chuyên đề Toán học số 10

Lời giải.
Chuyên đề Toán học Phổ thông năng khiếu TP.HCM số 10

1) Không mất tính tổng quát, ta có thể giả sử X là tập hợp 19 số nguyên
dương đầu tiên. Gọi X (k) là tập hợp tất cả các tập con có 7 phần tử của X
và tổng các phần tử của nó chia 19 dư k.
Khi đó, dễ thấy rằng |X (0)| + |X (1)| + · · · + |X (18)| chính là số tập con có
7
7 phần tử tùy ý của X và là C19 .
Ta thấy rằng hai tập hợp A, B ∈ X (k) tùy ý đều thỏa mãn đề bài.
Thật vậy,
Giả sử |A ∩ B| = 6 (không thể có |A ∩ B| = 7 vì khi đó hai tập hợp trùng
nhau). Đặt A = {a1 , a2 , a3 , a4 , a5 , a6 , x}, B = {a1 , a2 , a3 , a4 , a5 , a6 , y} thì
6
X 6
X
ai + x ≡ ai + y ≡ k(mod19)
i=1 i=1

nên x ≡ y(mod19). Suy ra x = y, mâu thuẫn. Đến đây, dễ thấy rằng


7
C19
max {|X (k)|} ≥ = 2652 > 2600.
0≤k≤18 19
Ta có đpcm.
2) Xét một tập hợp F thuộc họ Ω. Vì |X \F | = 19 − 7 = 12 nên có tất cả 12
tập hợp A ⊂ X với |A| = 8 và F ⊂ A.
Ngược lại, ứng với một tập hợp A là một cận trên của họ Ω, có không quá
8 tập F trong họ Ω sao cho F ⊂ A.
12
Do đó d ≥ 8 k hay d ≥ 32 k.
Đẳng thức có thể xảy ra khi họ Ω là tập hợp tất cả các tập con có 7 phần
tử của X .

Bài 7. Cho tam giác ABC không cân. Gọi I là trung điểm BC. Đường tròn
(I) đi qua A cắt AB, AC lần lượt tại M , N . Giả sử M I, N I lần lượt cắt (I) tại
P, Q. Gọi K là giao điểm của PQ với tiếp tuyến tại A của (I). Chứng minh
rằng K thuộc đường thẳng BC.

Lời giải.
Không mất tính tổng quát, giả sử AB < AC.
Kẻ đường kính AJ của đường tròn (I). Khi đó, dễ thấy tứ giác ABJ C và
AN JQ là các hình bình hành nên J B k AC, JQ k AN dẫn đến J, Q, B thẳng
hàng. Tương tự J, P, C thẳng hàng.

Hướng tới kỷ niệm 20 năm thành lập trường PTNK, ĐHQG TP HCM
Chuyên đề Toán học số 10 271

Gọi H là hình chiếu của A lên BC thì tứ giác AQBH nội tiếp.
Chuyên đề Toán học Phổ thông năng khiếu TP.HCM số 10

Suy ra
∠QH B = ∠QAB = ∠QAM = ∠QP M = ∠QP I
nên tứ giác PQH I cũng nội tiếp. Gọi (O) là đường tròn ngoại tiếp tam giác
ABC thì dễ thấy đường tròn (AH I) tiếp xúc với (O) tại A.

Xét ba đường tròn (O), (AH I), (PQH I) thì

• Trục đẳng phương của (O), (AH I) là tiếp tuyến của (O) tại A.

• Trục đẳng phương của (O), (PQH I) là PQ.

• Trục đẳng phương của (PQH I), (AH I) là H I.

Do đó, K chính là tâm đẳng phương của ba đường tròn nên K ∈ H I hay
K, B, C thẳng hàng.

Bài 8. Tìm n nguyên dương lớn nhất thỏa mãn các điều kiện sau:

i. n không chia hết cho 3.

ii. Bảng vuông n × n không thể được phủ kín bằng quân tetramino 1 × 4
và các quân trimino 1 × 3. Trong phủ các quân tetramino và trimino
được phép quay dọc nhưng không được phép chườm lên nhau hoặc
nằm ra ngoài bảng vuông.

Hướng tới kỷ niệm 20 năm thành lập trường PTNK, ĐHQG TP HCM
272 Chuyên đề Toán học số 10

Lời giải.
Ta sẽ chứng minh n = 5 là giá trị lớn nhất cần tìm.
Chuyên đề Toán học Phổ thông năng khiếu TP.HCM số 10

Ta nhận thấy rằng nếu n = 3k + 1, k ≥ 1 thì ta luôn phủ được bảng vuông
n × n bằng cách phủ hàng đầu tiên bằng 1 quân tetramino kích thước 1 × 4
(ta sẽ gọi tắt là tetramino) và k − 1 quân trimino kích thước 1 × 3 (ta sẽ
gọi tắt là trimino). Các cột còn lại có chiều dài 3k có thể phủ được bằng
các quân trimino (xoay dọc lại).
Ta chứng minh rằng nếu n = 3k + 2, k ≥ 2 thì bảng vuông n × n cũng phủ
được. Cách phủ với n = 8 được minh họa như sau

Dễ dàng thấy rằng với k ≥ 3 thì ta có thể thu được cách phủ cho bảng
vuông n × n bằng cách phủ phần hình vuông 8 × 8 ở góc trên bên trái như
trên, phần còn lại gồm 1 hình chữ nhật kích thước 3(k − 2) × (3k + 2) và
1 hình chữ nhật kích thước 8 × 3(k − 2) phủ được bằng các quân trimino.
Bây giờ ta chứng minh bảng vuông 5 × 5 không thể phủ được bằng 1 quân
tetramino và 7 quân trimino.
Trước hết ta chứng minh bổ đề: Nếu bảng vuông 5 × 5 có thể phủ được
bằng một hình vuông 1 × 1, ta gọi là unomino và 8 quân trimino thì quân
unomino 1 × 1 phải phủ ô trung tâm.
Thật vậy,
Ta đánh số các ô của bảng vuông 5 × 5 như hình vẽ

Hướng tới kỷ niệm 20 năm thành lập trường PTNK, ĐHQG TP HCM
Chuyên đề Toán học số 10 273

Ta thấy rằng một quân trimino luôn phủ đúng một ô mang số 1, một ô
Chuyên đề Toán học Phổ thông năng khiếu TP.HCM số 10

mang số 2 và một ô mang số 3. Vì số các số 2 bằng 9, còn số các số 1 và 3


bằng 8 nên nếu phép phủ ở đề bài thực hiện được thì quân unomino phải
phủ một ô mang số 2.
Mặt khác, ta có thể đánh số bảng vuông 5 × 5 bằng một cách khác

Các tính chất nói ở trên vẫn đúng cho cách đánh số này, tuy nhiên ở đây
số số 1 là 9, còn số số 2 và 3 là 8. Do đó, một lần nữa ta kết luận quân
unomino phải phủ một ô mang số 1.
Giao hai điều kiện cần nói trên lại, ta thấy với một cách phủ hợp lệ thì
quân unomino phải phủ ô trung tâm.
Quay trở lại với vấn đề phủ bảng vuông 5 × 5 bằng 1 quân tetramino và
7 quân trimino. Nếu tồn tại một cách phủ như thế thì cắt quân tetramino
thành 1 quân unomino và 1 quân trimino, ta thu được một phép phủ bảng
vuông 5 × 5 bằng 1 quân unomino và 8 quân trimino.
Theo bổ đề thì quân unomino phải nằm ở ô trung tâm, nghĩa là một đầu
của quân tetramino phải nằm ở ô trung tâm, mâu thuẫn (vì khi đó quân
tetramino sẽ bị lòi ra ngoài bảng vuông).
Với những lý luận ở trên, ta kết luận n = 5 là giá trị lớn nhất cần tìm.

8. Lời giải đề chọn đội tuyển năm 2015 − 2016


Ngày thi thứ 1
Bài 1. Cho tập hợp A = {n ∈ N|1 ≤ n ≤ 2015, (n, 2016) = 1}. Hỏi có bao
nhiêu số nguyên a ∈ A sao cho tồn tại số nguyên b mà a + 2016b là số chính
phương?

Lời giải.
Cho n là số nguyên dương lớn hơn 1, ta quy ước gọi một số nguyên dương
a được gọi là thặng dư chính phương theo modulo n nếu (a, n) = 1 và tồn

Hướng tới kỷ niệm 20 năm thành lập trường PTNK, ĐHQG TP HCM
274 Chuyên đề Toán học số 10

tại số nguyên x sao cho a ≡ x 2 (mod n). Trong bài này, để đơn giản, ta
Chuyên đề Toán học Phổ thông năng khiếu TP.HCM số 10

quy ước xét các thặng dư chính phương nhỏ hơn n.


Đặt s(n) là số các số nhỏ hơn n và là thặng dư chính phương theo modulo
n. Ta sẽ chứng minh hai bổ đề dưới đây:
Bổ đề 1: Cho p là số nguyên tố và k là số nguyên dương. Khi đó:

1. Nếu p = 2 thì s(2k ) = 2max(k−3,0) .

p k − p k−1
2. Nếu p > 2 thì s(p k ) = .
2
Bổ đề 2: s(n) là hàm nhân tính.
Thật vậy,
p−1
Trước hết, ta biết rằng s(p) = 2 với p là số nguyên tố lẻ. Ta sẽ tính s(p k )
với k ∈ Z+ .
Xét một thặng dư chính phương a của p, khi đó tồn tại x sao cho

a ≡ x 2 (modp).

Đặt a = x 2 + pq thì hiển nhiên

a ≡ x 2 + pq(modp k ) ⇔ a − pq ≡ x 2 (modp k )

và khi đó, ta có p k−1 cách chọn q để các số a − pq là các thặng dư chính


phương modp k .
Suy ra
p k − p k−1
s(p k ) = p k−1 s(p) = .
2
Xét số nguyên tố p = 2, với k = 1, 2, 3, dễ dàng kiểm tra được s(2k ) = 1.
Ta xét k ≥ 4, tương tự trên, ở bước chọn q, ta chỉ có 2 cách nên s(2k ) =
2s(2k−1 ). Từ đó bằng quy nạp, ta có được

s(2k ) = 2k−3 , k ≥ 4.

Tiếp theo, xét hai số a, b nguyên dương và (a, b) = 1. Gọi A là tập hợp các
thặng dư chính phương theo modulo ab và B là tập hợp các số là thặng
dư chính phương chung của a, b.
Nếu x ∈ A thì tồn tại y sao cho x ≡ y 2 (modab). Rõ ràng khi đó,

x ≡ y 2 (mod a), x ≡ y 2 (mod b)

Hướng tới kỷ niệm 20 năm thành lập trường PTNK, ĐHQG TP HCM
Chuyên đề Toán học số 10 275

(chú ý rằng nếu x > a, ta có thể chọn x 0 sao cho x 0 < a và x ≡ x 0 (mod a);
Chuyên đề Toán học Phổ thông năng khiếu TP.HCM số 10

tương tự với b). Do đó, x ∈ B, tức là x ∈ A ⇒ x ∈ B nên |A| ≤ |B|.


Tiếp theo, xét x ∈ B. Khi đó tồn tại r, s sao cho x ≡ r 2 (mod a), x ≡ s2
(mod b). Theo định lý thặng dư Trung Hoa, tồn tại số nguyên z sao cho

z ≡ r (mod a), z ≡ s (mod b).

Khi đó
x ≡ z 2 (mod a), x ≡ z 2 (mod b)
nên
.
x − z 2 ..ab hay x ≡ z 2 (modab).
Do đó: x ∈ A, tức là x ∈ B ⇒ x ∈ A nên |A| ≥ |B|.
Từ đây ta có
|A| = |B| hay s(a)s(b) = s(ab).
Vậy s(n) là hàm nhân tính.
Các bổ đề đều được chứng minh.
Trở lại bài toán, ta thấy rằng

2016 = 25 · 32 · 7.

Rõ ràng bài toán yêu cầu đếm số thặng dư chính phương theo modulo
2016. Theo bổ đề 2 thì

s(2016) = s(25 )s(32 )s(7).

Theo bổ đề 1 thì

32 − 3 7−1
s(25 ) = 22 = 4, s(32 ) = = 3, s(7) = = 3.
2 2

Do đó, số các số a cần tìm là 4 · 3 · 3 = 36.

Bài 2. Cho a, b, c, d là các số thực thỏa mãn điều kiện

a2 ≤ 1, a2 + b2 ≤ 5, a2 + b2 + c 2 ≤ 14, a2 + b2 + c 2 + d 2 ≤ 30.

1. Chứng minh rằng a + b + c + d ≤ 10.

2. Chứng minh rằng ad + bc ≤ 10.

Hướng tới kỷ niệm 20 năm thành lập trường PTNK, ĐHQG TP HCM
276 Chuyên đề Toán học số 10

Lời giải.
1) Dự đoán dấu bằng xảy ra khi a = 1, b = 2, c = 3, d = 4 nên ta có các
Chuyên đề Toán học Phổ thông năng khiếu TP.HCM số 10

đánh giá sau


a + 1 ≥ 2a
 2


 b2 + 4 ≥ 4b


 c 2 + 9 ≥ 6c

d + 16 ≥ 8d
 2

Do đó, ta có

24(a + b + c + d) ≤ 3(d 2 + 16) + 4(c 2 + 9) + 6(b2 + 4) + 12(a2 + 1)


= 3d 2 + 4c 2 + 6b2 + 12a2 + 120
= 3(a2 + b2 + c 2 + d 2 ) + (a2 + b2 + c 2 ) + 2(a2 + b2 ) + 6a2 + 120
≤ 3 · 30 + 14 + 2 · 5 + 6 · 1 + 120 = 240

Suy ra a + b + c + d ≤ 10.
2) Ta có:
16a2 + d 2 ≥ 8ad và 9b2 + 4c 2 ≥ 12bc.
Từ đó suy ra

24(ad + bc) ≤ 3(16a2 + d 2 ) + 2(9b2 + 4c 2 )


= 3(a2 + b2 + c 2 + d 2 ) + 5(a2 + b2 + c 2 ) + 10(a2 + b2 ) + 30a2
≤ 3 · 30 + 5 · 14 + 10 · 5 + 30 · 1 = 240

Suy ra ad + bc ≤ 10.

Bài 3. Tìm tất cả các hàm số f : R → R thỏa mãn điều kiện

f (x − 2 f ( y)) = 5 f (x) − 4x − 2 f ( y)

với mọi x, y ∈ R.

Lời giải.
Gọi (∗) là điều kiện đề bài cho. Trong (∗), thay x = y = 0, ta có

f (−2 f (0)) = 3 f (0).

Đặt f (0) = a thì f (−2a) = 3a. Trong (∗), thay x = 0 và y = −2a, ta có

f (−2 f (−2a)) = 5a − 2 f (−2a) ⇔ f (−6a) = −a.

Hướng tới kỷ niệm 20 năm thành lập trường PTNK, ĐHQG TP HCM
Chuyên đề Toán học số 10 277

Trong (∗), thay x = −2a, y = −6a, ta có


Chuyên đề Toán học Phổ thông năng khiếu TP.HCM số 10

f (−2a − 2 f (−6a)) = 5 f (−2a) − 4x − 2 f (−6a)


⇔ f (0) = 15a + 8a + 2a
⇔ a = 25a
⇔a=0

Do đó f (0) = 0.
Trong (∗), thay y = 0, ta có

f (x) = 5 f (x) − 4x ⇔ f (x) = x.

Thử lại ta thấy thỏa.


Vậy hàm số cần tìm chính là

f (x) = x, ∀x ∈ R.

Bài 4. Cho đường tròn k và các điểm B, C thuộc đường tròn, không phải là
đường kính; I là trung điểm BC. Điểm A di động trên cung lớn BC của k.
Gọi i1 là đường tròn qua I và tiếp xúc với AB tại B; i2 là đường tròn qua I
và tiếp xúc với AC tại C. Các đường tròn i1 , i2 cắt nhau tại D (khác I).

1. Chứng minh rằng đường tròn ngoại tiếp tam giác AI D luôn đi qua một
điểm cố định.

2. Gọi K là trung điểm AD, E là tâm đường tròn qua K và tiếp xúc với
AB tại A, F là tâm đường tròn qua K và tiếp xúc với AC tại A. Chứng
minh rằng góc EAF có số đo không đổi.

Lời giải.
1) Gọi O là tâm của đường tròn k. Không mất tính tổng quát, giả sử tia AD
nằm giữa hai tia AO, AB, các trường hợp còn lại tương tự.
Ta có:
∠I DB = ∠ABC, ∠I DC = ∠AC B

nên
∠BAC + ∠BDC = ∠BAC + ∠ABC + ∠AC B = 180◦ .

Hướng tới kỷ niệm 20 năm thành lập trường PTNK, ĐHQG TP HCM
278 Chuyên đề Toán học số 10

Do đó, tứ giác ABDC nội tiếp hay D ∈ (O). Ta thấy


Chuyên đề Toán học Phổ thông năng khiếu TP.HCM số 10

∠DAO + ∠OI D
= ∠BAC − (∠DAB + ∠OAC) + 360◦ − (90◦ + ∠DI C)
= ∠BAC − (∠I C D + 90◦ − ∠ABC) + 270◦ − ∠DI C
= ∠BAC + ∠ABC − (∠I C D + ∠DI C) + 180◦
= (180◦ − ∠AC B) − (180◦ − ∠I DC) + 180◦
= ∠I DC − ∠AC B + 180◦ = 180◦

Do đó, AOI D nội tiếp hay đường tròn (AI D) đi qua O cố định.
2) Ta có:
∠EAC = 90◦ − ∠BAC, ∠FAB = 90◦ − ∠BAC

nên
∠EAF = 180◦ − 2∠BAC + ∠BAC = 180◦ − ∠BAC.

Do đó, góc ∠EAF có số đo không đổi.

Hướng tới kỷ niệm 20 năm thành lập trường PTNK, ĐHQG TP HCM
Chuyên đề Toán học số 10 279

Ngày thi thứ 2


Chuyên đề Toán học Phổ thông năng khiếu TP.HCM số 10

1
Bài 5. Dãy số (x n ) được xác định bởi x n = n cos 1 với mọi n ≥ 1. Tính giới
n
hạn sau
x 1 + x 3 + x 5 + · · · + x 2n−1
lim .
x 2 + x 4 + x 6 + s + x 2n

Lời giải.
Trước hết, ta chứng minh bổ đề sau:
Giá trị của biểu thức
1 1 1 1
+ + + ··· +
1 2 3 n
tiến tới vô cực khi n → +∞. Thật vậy,
Xét hàm số f (x) = ln(1 + x) − x với x > 0. Ta có

1
f 0 (x) = −1<0
1+ x

nên đây là hàm nghịch biến, suy ra f (x) < f (0) = 0 hay ln(1 + x) <
x, ∀x > 0. Thay x bởi 1n , ta được

1 1 1
 ‹
ln 1 + < ⇔ > ln(1 + n) − ln n.
n n n

Do đó,

1 1 1 1
+ + + + > ln 2 − ln 1 + ln 3 − ln 2 + · · · + ln(n + 1) − ln n = ln(n + 1).
1 2 3 n
Vì ln(n + 1) → +∞ khi n → +∞ nên

1 1 1 1
+ + + · · · + → +∞.
1 2 3 n

Trở lại bài toán, đặt

x 1 + x 3 + x 5 + · · · + x 2n−1
yn =
x 2 + x 4 + x 6 + · · · + x 2n
1
∈ 0; π2 nên cos 1n > 0, suy ra

với n ≥ 1. Ta thấy vì n

1
xn = > 0, n ≥ 1.
n cos 1n

Hướng tới kỷ niệm 20 năm thành lập trường PTNK, ĐHQG TP HCM
280 Chuyên đề Toán học số 10

Xét hàm số f (t) = cost t với t ∈ 0; π2 thì f 0 (t) = coscos


t+t sin t

2t > 0 nên đây là
1 1
Chuyên đề Toán học Phổ thông năng khiếu TP.HCM số 10


hàm đồng biến. Chú ý rằng x n = f n , mà n là dãy giảm nên x n cũng là
dãy giảm.
Suy ra x 1 > x 2 , x 3 > x 4 , . . . , x 2n−1 > x 2n nên yn > 1.
Ngoài ra, ta cũng có x 3 < x 2 , x 5 < x 4 , . . . , x 2n−1 < x 2n−2 nên
x 1 + (x 2 + x 4 + · · · + x 2n−2 )
yn < =
x 2 + x 4 + · · · + x 2n
x 1 − x 2n x1
1− <1−
x 2 + x 4 + · · · + x 2n x 2 + x 4 + · · · + x 2n

Dễ thấy rằng
n n n
X 1 X 1 1X1
x 2 + x 4 + · · · + x 2n = ≥ = .
i=1 2i cos 2i1 i=1
2i 2 i=1 i
n
1
P
Theo bổ đề trên thì i tiến tới vô cực nên
i=1

lim x 2 + x 4 + · · · + x 2n = +∞.

Do đó
x1
 ‹
lim 1 − = 1 − 0 = 1.
x 2 + x 4 + · · · + x 2n
Theo nguyên lý kẹp, ta có lim x n = 1.
Bài 6. Tìm các giá trị của b sao cho tồn tại a để hệ phương trình sau đây có
nghiệm (x, y)
(x − 1)2 + ( y + 1)2 = b


y = x 2 + (2a + 1)x + a2
Lời giải.
Đặt X = x − 1, Y = y + 1, thay vào, ta có
X + Y2 = b
 2

Y − 1 = (X + 1)2 + (2a + 1)(X + 1) + a2


X + Y2 = b
 2

Y = X 2 + (2a + 3)X + a2 + 2a + 3

Ta đưa về tìm điều kiện của b để tồn tại a mà hệ trên có nghiệm (X , Y ).


Do

Y − (X + 2) = X 2 + 2(a + 1)X + (a + 1)2 = (X + a + 1)2 ≥ 0

Hướng tới kỷ niệm 20 năm thành lập trường PTNK, ĐHQG TP HCM
Chuyên đề Toán học số 10 281

nên Y ≥ X + 2. Suy ra Y − X ≥ 2 > 0, tức là (X − Y )2 ≥ 4. Ta có


Chuyên đề Toán học Phổ thông năng khiếu TP.HCM số 10

(X − Y )2 + (X + Y )2 (Y − X )2
b = X2 + Y2 = ≥ ≥ 2.
2 2

Mặt khác, với b ≥ 2, nếu chọn X = −(a + 1) thì có Y = X + 2 = 1 − a. Khi


đó, ta có
X 2 + Y 2 = (a + 1)2 + (a − 1)2 = 2(a2 + 1) = b.
Như thế, với a thỏa mãn 2(a2 + 1) = b thì hệ có nghiệm là

(X , Y ) = (−a − 1, 1 − a).

Dễ dàng thấy rằng do b ≥ 2 nên luôn tồn tại a như thế.


Vậy các giá trị cần tìm của b là b ≥ 2.

Bài 7. Cho n nguyên dương, n ≥ 2 và X = {1, 2, 3, . . . , n}. Gọi A1 , A2 , . . . , Am


và B1 , B2 , . . . , Bm là hai dãy các tập con khác rỗng của X thỏa mãn điều kiện:

Với mỗi i, j ∈ {1, 2, 3, . . . , n}, Ai ∩ B j = ∅ nếu và chỉ nếu i = j.

1. Chứng minh rằng với mỗi hoán vị (x 1 , x 2 , . . . , x n ) của X , có không quá


1 cặp (Ai , Bi ) với i = 1, 2, 3, . . . , n sao cho nếu x k ∈ Ai và x l ∈ Bi thì
k < l.

2. Gọi ai , bi lần lượt là số phần tử của các tập hợp Ai , Bi với i = 1, 2, . . . , m.


Chứng minh rằng
m
X 1
ai ≤ 1.
i=1
Cai +bi

Lời giải.
1) Giả sử ngược lại, tồn tại 2 cặp (Ai , Bi ) và (A j , B j ) thỏa mãn điều kiện đề
bài đã cho.
Vì i 6= j nên theo giả thiết,

A ∩ B ≥ 1, A ∩ B ≥ 1.
i j j i

Đặt x r ∈ Ai ∩ B j , x s ∈ A j ∩ Bi với 1 ≤ r, s ≤ n thì:

• Do x r ∈ B j nên với mọi x k ∈ A j , ta đều có k < r.

• Do x r ∈ Ai nên với mọi x k ∈ Bi , ta đều có k > r.

Hướng tới kỷ niệm 20 năm thành lập trường PTNK, ĐHQG TP HCM
282 Chuyên đề Toán học số 10

Từ đây suy ra
Chuyên đề Toán học Phổ thông năng khiếu TP.HCM số 10

A j ⊂ {x 1 , x 2 , . . . , x r−1 } , Bi ⊂ {x r+1 , x r+2 , . . . , x n } .

Điều này cho thấy A j ∩ Bi = ∅, mâu thuẫn với giả thiết.


Vậy tồn tại không quá 1 cặp (Ai , Bi ) thỏa mãn điều kiện đã cho.
2) Gọi T là tập hợp các cách chọn hai dãy

A1 , A2 , . . . , Am và B1 , B2 , . . . , Bm

thỏa mãn điều kiện là: với mỗi i, j ∈ {1, 2, 3, . . . , n}, Ai ∩ B j = ∅ nếu và chỉ
nếu i = j.
Gọi Ti ⊂ T là các cách chọn sao cho sao cho cặp (Ai , Bi ) thỏa mãn điều
kiện là: cặp (Ai , Bi ) với i = 1, 2, 3, . . . , n sao cho nếu x k ∈ Ai và x l ∈ Bi thì
x k < x l (ở đây ta xét thứ tự ban đầu của các phần tử của X ). (*)
Theo câu 1) thì Ti ∩ T j = ∅ với i 6= j nên ta có

|T1 | + |T2 | + · · · + |Tm | = |T1 ∪ T2 ∪ . . . ∪ Tm | ≤ T.

Tiếp theo, với 1 ≤ i ≤ m, xét một tập hợp S ⊂ X và |S| = ai + bi . Khi đó,
tương ứng với S, có đúng 1 cách chọn (Ai , Bi ) thỏa mãn tính chất (∗) – tức
là Ai sẽ nhận ai số nhỏ nhất trong tập S, Bi là lấy phần còn lại.
a
Trong khi đó, nếu không có điều kiện (∗), ta có thể chọn tùy ý Caii+bi phần
tử trong S và A và số còn lại cho B.
Do đó, ta có
|T |
|Ti | = a
Caii+bi
với i = 1, 2, . . . , m. Từ đây suy ra
m m
X |T | X 1
ai ≤ |T | ⇔ ai ≤1
i=1
Cai +bi i=1
Cai +bi

Ta có đpcm.

Bài 8. Cho tam giác ABC nhọn nội tiếp đường tròn tâm O. Đường tròn tâm
I đi qua B, C lần lượt cắt các tia BA, CA tại E, F.

1. Giả sử các tia BF, C E cắt nhau tại D và T là tâm đường tròn (AE F ).
Chứng minh rằng OT k I D.

Hướng tới kỷ niệm 20 năm thành lập trường PTNK, ĐHQG TP HCM
Chuyên đề Toán học số 10 283

2. Trên BF, C E lần lượt lấy các điểm G, H sao cho AG⊥C E, AH⊥BF. Các
đường tròn (ABF ), (AC E) cắt BC tại M , N (khác B, C) và cắt E F tại
Chuyên đề Toán học Phổ thông năng khiếu TP.HCM số 10

P, Q (khác E, F ). Gọi K là giao điểm của M P, NQ. Chứng minh rằng


DK vuông góc với GH.

Lời giải.
1) Giả sử E F cắt BC ở L và (T ), (O) cắt nhau tại J khác A. Suy ra AJ chính
là trục đẳng phương của (T ), (O). Do đó OT ⊥AJ.
Khi đó,
LB · LC = LE · LF

nên L thuộc trục đẳng phương của (T ), (O). Suy ra A, J, L thẳng hàng.
Theo định lý Brocard cho tứ giác BE F C nội tiếp trong đường tròn (I) thì I
chính là trực tâm của tam giác ADL.
Vì thế nên I D⊥AL, mà OT ⊥AJ nên I D k OT .

2) Dễ dàng thấy rằng D là trực tâm của tam giác AGH nên AD⊥GH. Ta sẽ
chứng minh rằng A, D, K thẳng hàng.
Ta có DB · DF = DE · DC nên D có cùng phương tích tới 2 đường tròn
(ABF ), (AEC). Suy ra AD chính là trục đẳng phương của 2 đường tròn này.

Hướng tới kỷ niệm 20 năm thành lập trường PTNK, ĐHQG TP HCM
284 Chuyên đề Toán học số 10
Chuyên đề Toán học Phổ thông năng khiếu TP.HCM số 10

Bằng biến đổi các góc nội tiếp, ta thấy rằng

∠M PQ = ∠M BF = ∠C E F = ∠C NQ.

Suy ra M N PQ nội tiếp, dẫn đến K M · K P = K N · KQ, tức là K cũng có cùng


phương tích tới 2 đường tròn (ABF ), (AEC).
Từ đó suy ra A, D, K thẳng hàng. Do đó, DK vuông góc với GH.

Các đề thi và lời giải được sử dụng từ nhiều nguồn, trong đó có

• Diễn đàn http://mathscope.org

• Diễn đàn http://diendantoanhoc.net

Xin cám ơn các thành viên của diễn đàn, các thầy cô, học sinh đã đóng
góp để xây dựng lời giải trên.

Hướng tới kỷ niệm 20 năm thành lập trường PTNK, ĐHQG TP HCM

You might also like